Superpage
Lymphoma & related disorders

Authors: Fouad Abdelhalim, M.D., Engy Abdellatif, M.B.B.Ch., M.D., Ph.D., Syeda F. Absar, M.D., M.P.H., Buthaina Al-Maashari, M.D., João Víctor Alves de Castro, M.D., Catalina Amador, M.D., Cade Arries, M.D., Sepideh Nikki Asadbeigi, M.D., Saja Asakrah, M.D., Ph.D., Jon C. Aster, M.D., Ph.D., Aaron Auerbach, M.D., M.P.H., Jayalakshmi Balakrishna, M.D., Carmen Bárcena, M.D., Puneet Bedi, M.D., Samuel Bidot, M.D., Patrick Bladek, M.D., Kyle Bradley, M.D., M.S., Tamar C. Brandler, M.D., M.S., Sanjay Bridgelall, M.D., Daniel Cassidy, M.D., Jennifer Chapman, M.D., Jinjun Cheng, M.D., Ph.D. , John Kim Choi, M.D., Ph.D., Elizabeth Courville, M.D., Genevieve M. Crane, M.D., Ph.D., Nicholas Joseph Dcunha, M.B.B.S., M.D., Laurence de Leval, M.D., Ph.D., Luiz Paulo de Lima Guido, M.D., Chelsey D. Deel, M.D., Tom Deng, M.D., Siba El Hussein, M.D., Mark G. Evans, M.D., Julie Feldstein, M.D., Maria C. Ferrufino-Schmidt, M.D., Abdallah Flaifel, M.D., Abhilasha Ghildyal, M.D., M.P.H., Kathryn Gibbons, M.D., Busra Bacik Goksu, M.D., Raul S. Gonzalez, M.D., Miguel Gonzalez-Mancera, M.D., Alejandro A. Gru, M.D., Xiaoling Guo, M.D., Ph.D., Jordan M. Hall, M.D., Emily M. Hartsough, B.S., Aubrey J. Hough, M.D., S. David Hudnall, M.D., Meghan Hupp, M.D., Elaine S. Jaffe, M.D., Yoon Kyung Jeon, M.D., Dragan Jevremovic, M.D., Ph.D., Béla Kajtár, M.D., Ph.D., Dia Kamel, M.D., Ph.D., Hatem Kaseb, M.D., Ph.D., M.P.H., Mahsa Khanlari, M.D., Pallavi Khattar, M.D., Joseph Khoury, M.D., Sumire Kitahara, M.D., Matthew M. Klairmont, M.D., Kerri-Ann Latchmansingh, M.D., Robert E. LeBlanc, M.D., Dragos C. Luca, M.D., Marie Therese Manipadam, M.B.B.S., M.D., Mario L. Marques-Piubelli, M.D., Emily Mason, M.D., Ph.D., L. Jeffrey Medeiros, M.D., Roberto N. Miranda, M.D., Pawel Mroz, M.D., Ph.D., Carlos A. Murga-Zamalloa, M.D., Cameron Neely, M.D., Pichayut Nithagon, M.D., Nicholas Nowacki, M.D., Chi Young Ok, M.D., Anna B. Owczarczyk, M.D., Ph.D., Neval Ozkaya, M.D., Christopher Y. Park, M.D., Ph.D., Sudhir Perincheri, M.B.B.S., Ph.D., Nat Pernick, M.D., Anamarija M. Perry, M.D., Julio Poveda, M.D., Annapurna Saksena, M.B.B.S., M.D., Christian Salib, M.D., Nikhil Sangle, M.D., John Schmieg, M.D., Ph.D., Roman Segura-Rivera, M.D., Allam Shawwa, M.D., Anna Shestakova, M.D., Ph.D., Min Shi, M.D., Ph.D., Alexa J. Siddon, M.D., Elanthenral Sigamani, M.B.B.S., M.D., Amrit P. Singh, M.D., Lauren Smith, M.D., Michael Stone, D.O., Mehrnoosh Tashakori, M.D., Ph.D., Beenu Thakral, M.D., Carlos A. Torres-Cabala, M.D., Patricia Tsang, M.D., M.B.A., Jayalakshmi Venkateswaran, M.D., Yanhua Wang, M.D., Nicholas Ward, M.D., Dietrich Werner, M.D., Reid Wilkins, M.D., Richard K. Wood, M.D., Martha Frances Wright, M.D., Samantha C. Wu, B.A., Guang "Geoff" Yang, M.D., Ph.D., Lena Young, D.O., Ling Zhang, M.D.
Resident / Fellow Advisory Board: Mario L. Marques-Piubelli, M.D.
Editorial Board Members: Elizabeth Courville, M.D., Genevieve M. Crane, M.D., Ph.D., Julie Feldstein, M.D., Mario L. Marques-Piubelli, M.D., Roberto N. Miranda, M.D., Anamarija M. Perry, M.D., Alexa J. Siddon, M.D., Patricia Tsang, M.D., M.B.A.
Deputy Editor-in-Chief: Genevieve M. Crane, M.D., Ph.D.
Editor-in-Chiefs: Patricia Tsang, M.D., M.B.A., Debra L. Zynger, M.D.

Copyright: 2001-2024, PathologyOutlines.com, Inc.

HemePath related: Jobs, Fellowships, Conferences, Cases, CME, Board Review, What's New

Related chapters: Bone marrow nonneoplastic, Bone marrow neoplastic, Hematology, Lymph nodes and spleen, nonlymphoma

Editorial Board oversight: Mario L. Marques-Piubelli, M.D. (last reviewed November 2023), Roberto N. Miranda, M.D. (last reviewed November 2023), Julie Feldstein, M.D. (last reviewed December 2022), Genevieve M. Crane, M.D., Ph.D. (last reviewed December 2022)
Page views in 2024 to date: 17

ALK+ LBCL
Definition / general
Essential features
  • Aggressive B cell lymphoma with monomorphic large immunoblast / plasmablast-like B cells, usually having plasma cell phenotype
  • Positive for ALK immunohistochemistry with granular cytoplasmic restriction and negative for CD20 and CD30
  • ALK gene is translocated and activated; most frequent cytogenetic abnormality is t(2;17) responsible for CLTC (clathrin)-ALK fusion protein
Terminology
  • Large B cell lymphoma expressing ALK kinase and lacking t(2;5) translocation (obsolete)
  • ALK positive plasmablastic B cell lymphoma (obsolete)
ICD coding
  • ICD-O: 9737/3 - ALK positive large B cell lymphoma
Epidemiology
  • It is a very rare lymphoma
  • Occurs in immunocompetent, HIV negative young men, although reported in a wide age range and both genders
  • M:F = 5:1
  • Age range of 9 - 85 years; median age of 43 years (J Clin Oncol 2009;27:4211)
Sites
Pathophysiology
  • Translocation of ALK gene on chromosome 2 with a fusion gene partner
  • ALK oncogenic protein overexpression and related activation of STAT3 pathway
  • MYC expression in the absence of MYC translocations or amplifications, probably due to transcriptional activation downstream of STAT3 (Mod Pathol 2013;26:1329)
  • No known association with immunosuppression
Etiology
  • Not specified
  • Normal counterpart: a postgerminal center B cell with plasmablastic differentiation
Clinical features
  • Most patients present with generalized lymphadenopathy
  • 60% of cases present in advanced stage (III / IV) and bone marrow infiltration is seen in 25% of cases (J Clin Oncol 2009;27:4211)
Diagnosis
  • Based on a combination of light microscopy and immunohistochemical markers
Prognostic factors
  • Generally, this lymphoma has a poor prognosis
  • However, factors for a good prognosis include localized disease (stage I and II) and lymphoma occurring in childhood
Case reports
Treatment
  • Most cases treated with chemotherapy regimens of CHOP or E-POCH / CHOEP; a few patients are also given additional radiotherapy (PLoS One 2017;12:e0178416)
  • Since most cases are negative for CD20 and CD30, rituximab and brentuximab vedotin are not given
  • There is a single case report of complete response to nivolumab (Clin Lymphoma Myeloma Leuk 2020;20:e113)
Microscopic (histologic) description
  • Tumor arranged as diffuse sheets, sinusoidal infiltrate or a mixture of diffuse and sinusoidal patterns
  • Tumor composed of monomorphic large immunoblast-like cells containing a large central nucleolus and abundant cytoplasm
  • Some cases show plasmablastic differentiation, which can be either an exclusive population of plasmablasts or a spectrum of morphology from plasmacytes to plasmablasts
  • Anaplastic / pleomorphic morphology has also been described in a few cases (Am J Surg Pathol 2017;41:25)
  • Atypical multinucleated tumor giant cells may be seen (Am J Surg Pathol 2017;41:25)
Microscopic (histologic) images

Contributed by Marie Therese Manipadam, M.B.B.S., M.D.
Tumor with diffuse involvement

Tumor with diffuse involvement

Large lymphoid tumor cells

Large lymphoid tumor cells

Immunoblast-like tumor cells and few plasmablasts

Immunoblast-like
tumor cells and
few plasmablasts

Interfollicular expansion by tumor cells

Interfollicular expansion by tumor cells

Intrasinusoidal growth of tumor

Intrasinusoidal growth of tumor

Cytoplasmic granular ALK positivity

Cytoplasmic granular ALK positivity


MUM1 positivity

CD138 positivity

CD138 positivity

MIB1 index of 80 - 90%

MIB1 index of 80 - 90%

CD30 negativity in tumor cells

CD30 negativity in tumor cells

CD3 negativity in tumor cells

CD3 negativity in tumor cells

CD20 negativity in tumor cells

CD20 negativity in tumor cells


Weak positivity for CD79a

Weak positivity for CD79a

Lymphoma involving bone marrow biopsy

Lymphoma involving bone marrow biopsy

ALK positivity in tumor cells

Granular ALK positivity in tumor cells

Granular ALK positivity in tumor cells

ALK positive large B cell lymphoma cells

ALK positive large B cell lymphoma cells

Cytology description
  • Only a few case reports are available in the literature and give a brief view of cytological findings
  • One case report states, "most of the neoplastic cells had pseudopodial cytoplasmic projections and some of them had eosinophilic pseudopodial cytoplasmic projections that resembled flaming plasma cells" (Int J Clin Exp Pathol 2013;6:2631)
  • A second case report states, "aspiration cytology revealed many clusters of cohesive, large and solitary cells that showed abundant cytoplasm and large round to oval nuclei with prominent nucleoli" (Diagn Cytopathol 2014;42:69)
Positive stains
  • ALK positivity is important for diagnosis and characteristically demonstrates a restricted granular cytoplasmic staining pattern, highly indicative of the expression of CLTC-ALK fusion protein or ALK-SEC31A translocation
  • Other patterns (diffuse cytoplasmic, nuclear or nucleolar) of ALK staining correspond to translocation of ALK with NPM1 [t(2;5)(p23;q35)], SQSTM1, RANBP2 or EML4, which are associated with a poorer prognosis (Int J Hematol 2016;103:399)
  • EMA (73%), CD138 (96%), PRDM1 (BLIMP1), MUM1 / IRF4 (100%), CD45 (96%), IgA (67%), BOB1 (100%), OCT2 (81%), PAX5 (28%) and CD79a (30%)
  • Tumor may be positive for CD4 (67%), CD57, CD43 and perforin
  • 83% of cases showed kappa or lambda light chain restriction by immunohistochemistry or in situ hybridization (Am J Surg Pathol 2017;41:25)
Negative stains
Molecular / cytogenetics description
  • Clonal rearrangement of IG genes
  • ALK is overexpressed due to fusion proteins generated by translocations
  • Most common translocation is t(2;17)(p23;q23), responsible for CLTC (clathrin)-ALK fusion protein
  • Few cases are associated with t(2;5) (Blood 2003;102:2642)
  • ALK may have other fusion partners (given above)
Sample pathology report
  • Right cervical lymph node, excisional biopsy:
    • ALK positive large B cell lymphoma (see comment)
    • Comment: Tumor cells are positive for ALK (cytoplasmic granular staining) and MUM1 with high MIB1 proliferation index. Tumor cells are negative for CD3, CD20, CD30, CD45, EBV-LMP1 and LANA. Few cells show weak positivity for CD79a.
    • Microscopic description: Section studied shows completely effaced lymph node architecture with diffuse infiltration by a tumor arranged in sheets and composed of monomorphic large immunoblast-like cells containing a round to oval nucleus, vesicular chromatin, large central nucleolus and abundant cytoplasm. Few cells show plasmablast-like morphology. An intrasinusoidal pattern is seen focally.
Differential diagnosis
Board review style question #1
Which of the following combinations of IHC markers is diagnostic of ALK positive large B cell lymphoma that comprises monomorphic large lymphoid cells?

  1. ALK positive, MUM1 positive and CD30 positive
  2. ALK positive, MUM1 positive and CD30 negative
  3. ALK positive, MUM1 negative and CD30 positive
  4. ALK positive, MUM1 negative and CD30 negative
Board review style answer #1
B. ALK positive, MUM1 positive and CD30 negative

Comment Here

Reference: ALK+ large B cell lymphoma
Board review style question #2

What is the most common pattern of ALK IHC positivity in ALK positive large B cell lymphoma?

  1. Granular cytoplasmic positivity
  2. Nuclear positivity
  3. Nuclear and cytoplasmic positivity
  4. Membranous positivity
Board review style answer #2
A. Granular cytoplasmic positivity

Comment Here

Reference: ALK+ large B cell lymphoma

ATLL
Definition / general
  • Aggressive T cell malignancy of mature CD4+ T cells causally associated with human T lymphotropic virus 1 (HTLV-1; also called human T cell leukemia virus) (Blood 2017;129:1071)
Essential features
  • Lymphoma cells show monoclonal integration of HTLV-1 (Blood 2017;129:1071)
  • Occurs in regions endemic for HTLV-1 (~ 2.5% in HTLV-1 carriers) (Int J Hematol 2002;76:240)
  • Affected individuals are usually exposed to HTLV-1 very early in life (Front Microbiol 2018;9:461)
  • Develops after a long latency (therefore occurs in adults)
  • Most often composed of highly pleomorphic small to medium sized lymphoid cells
  • Widely disseminated and clinically aggressive
  • Clinical spectrum of HTLV-1 associated diseases is wide and includes neoplastic and nonneoplastic disorders (Clin Microbiol Rev 2010;23:577)
Terminology
  • Abbreviation: adult T cell leukemia / lymphoma (ATLL)
  • Synonyms: adult T cell leukemia, adult T cell lymphoma, adult T cell leukemia / lymphoma (HTLV-1+)
ICD coding
  • ICD-O: 9827/3 - adult T cell leukemia / lymphoma (HTLV-1+)
  • ICD-10: C91.5 - adult T cell lymphoma / leukemia (HTLV-1 associated)
Epidemiology
  • HTLV-1 virus is endemic in Southwestern Japan, the Caribbean basin, sub-Saharan Africa, South America and parts of the Middle East and Australo-Melanesia (Front Microbiol 2012;3:388)
  • Third to ninth decade, mean 58 years
  • M:F = 1.5:1
  • Transmission: requires the presence of living HTLV-1 infected cells
    1. Mother to infant (mainly by lymphocytes in breast milk) (J Pediatric Infect Dis Soc 2018;7:350)
    2. Sexual fluids
    3. Blood and blood products (not transmitted in fresh frozen plasma)
Sites
  • Widespread lymph node involvement in most
  • Systemic: spleen and extranodal sites including the skin, lungs, liver, gastrointestinal tract and central nervous system
    • Main extranodal sites: skin (involved in > 50% of cases) and peripheral blood
Pathophysiology
  • Cell of origin is peripheral CD4+ T regulatory αβ T cell (CD4+ CD25+ FoxP3+ T regulatory cells are the closest normal counterpart)
  • ATLL patients suffer from profound immunodeficiency; the concept that the cell of origin of ATLL is a T regulatory T cell provides a biologic basis for disease associated immunodeficiency
Etiology
  • ATLL is uniformly causally associated with HTLV-1 infection
  • HTLV was the first human retrovirus to be identified, isolated from a patient derived T cell lymphoma cell line (Proc Natl Acad Sci USA 1980;77:7415)
  • HTLV-1 is a type C retrovirus, Deltaretrovirus genus
    • Single strand of RNA is converted into double strand of DNA in host via reverse transcriptase enzyme (Viruses 2010;2:2037)
    • Monoclonal, randomly integrates into host cell genome (Oncology 2015;89:7)
    • Infection alone is not sufficient to result in neoplastic transformation of infected cells
  • HTLV-1 can infect immature thymocytes and mature CD4+ T cells
    • Enters by cell to cell contact via 3 cellular molecules: neuropilin 1, heparan sulfate proteoglycan (HSPGs) and GLUT1 (J Virol 2006;80:6844)
  • Tax protein and HTLV-1 basic leucine zipper factor (HBZ): two major oncoproteins with key role in the development of ATLL in chronically infected individuals
    • Tax (p40): activates viral promoter, the cyclic AMP response element binding transcription factor (CREB) and nuclear factor κβ pathways (Retrovirology 2004;1:20)
    • HBZ: consistently expressed in all cases of ATLL; contributes to cellular proliferation and survival of the neoplastic clone (Retrovirology 2016;13)
  • Epigenetic alterations and hypermethylation are also involved in the development and disease progression (Nat Genet 2015;47:1304, Am J Pathol 2010;176:402)
Diagrams / tables

Contributed by Jennifer Chapman, M.D.

Differential diagnostic algorithm

Clinical features
  • 4 clinically defined variants in Shimoyama classification: acute, lymphomatous, chronic and smoldering (Br J Haematol 1991;79:428)
  • Acute variant: ~ 55 to 60% of cases, survival usually < 1 year
    • Leukocytosis (often greater than 100 x 109/L)
    • Peripheral blood involvement with numerous flower cells
    • Skin rash and lymphadenopathy
    • Hepatosplenomegaly
    • Hypercalcemia
    • Elevated serum LDH
    • Rapidly progressive course and frequent opportunistic infections
  • Lymphomatous variant: ~ 20% of cases, survival usually < 1 year
    • Lymphadenopathy
    • Skin lesions
    • No / minimal peripheral blood involvement
    • Hypercalcemia is less frequent (compared with acute variant)
  • Chronic variant: ~ 15 to 20% of cases, survival > 2 years but < 5 years due to progression to acute or lymphomatous ATLL
    • Lymphocytosis
    • Exfoliative skin lesions
    • Lymphocyte count is much lower than in the acute variant
    • Few atypical lymphocytes in peripheral smear review
    • Mild hepatosplenomegaly
    • Mild lymphadenopathy
    • No hypercalcemia
  • Smoldering variant: ~ 5% of cases, survival > 2 years but < 10 years due to progression to acute ATLL or infectious complications
    • Skin or lung lesions
    • More than 5% atypical lymphocytes in absence of leukocytosis
    • No hepatosplenomegaly, hypercalcemia or lymphadenopathy
  • Primary cutaneous tumoral type
    • No leukemic phase, lymphadenopathy, hepatosplenomegaly or hypercalcemia
    • Proposed as a fifth clinical type in the Shimoyama classification (Int J Dermatol 2010;49:1099)
  • Nonneoplastic HTLV-1 associated diseases


Comparison of clinical forms of adult T cell leukemia / lymphoma
Clinical manifestation Acute Lymphomatous Smoldering Chronic
Lymphocytosis Increased No No Mildly increased
Blood abnormal lymphocyte Increased No* > 5% Mildly increased
Increased LDH Yes No No Minimal
Hypercalcemia Yes Variable No No
Skin rash Variable: > 50% Variable: > 50% Erythematous rash Rash, papules
Lymphadenopathy Usually present Yes No Mild
Hepatosplenomegaly Usually present Often present No Mild
Bone marrow infiltration May be present No No No
Median survival < 1 year < 1 year > 2 years > 2 years
LDH = lactate dehydrogenase; *may develop later in the course of disease
Diagnostic criteria
  • Strongest support for a diagnosis of ATLL: demonstration of viral integration in the tumor cells
  • Clinical features favoring the diagnosis: appropriate patient demographic (patient derived from HTLV-1 endemic area), hypercalcemia, skin lesions and a leukemic phase
  • Histologic features favoring the diagnosis: pleomorphic T cell lymphoma, leukemic phase, CD4+ αβ type with T regulatory immunophenotype, expression of CD25
Laboratory
  • Evidence of HTLV-1 or 2 sequences at the molecular level
  • Seropositivity for anti-HTLV-1 antibody as surrogate for demonstration of monoclonal integration of virus
    • Only useful in areas with low prevalence of HTLV-1 infection
    • In areas of high HTLV-1 prevalence, viral integration must be demonstrated
  • Expression of CD25 by lymphoma cells
  • Other laboratory tests / results:
    • CBC: elevated leukocyte count and circulating neoplastic lymphocytes (leukemic phase)
    • Elevated serum LDH level reflects disease burden / activity
    • Hypercalcemia: more common in patients with acute variant; variably associated with lytic bone lesions
    • Eosinophilia and neutrophilia are common
    • Elevated soluble IL-2 receptor α chain levels in patients with aggressive ATLL
Radiology description
  • May have lytic bone lesions
    • Skull, pelvis, spine and long bones can be affected
    • Punched out lesions similar to those seen in plasma cell myeloma can be found
  • F-18 deoxyglucose PET / CT is usually positive in sites of disease activity
  • CT detects sites of nodal and extranodal disease
Prognostic factors
  • Clinical variant, age, performance status, serum calcium and LDH levels are major prognostic factors
  • Death often stems from opportunistic infections
  • Acute and lymphomatous variants:
    • Aggressive clinical course
    • Recurrent infection: parasitic (Strongyloides stercoralis) and viral infections (Parasite Immunol 2004;26:487)
    • p16 gene deletion and p53 mutation: more aggressive clinical course
  • Chronic and smoldering variants:
    • More protracted clinical course
    • Progression to an acute phase with an aggressive course in ~25%
    • p16 gene deletion and chromosomal deletion detected via comparative genomic hybridization (CGH) in the chronic phase is a negative prognostic factor (J Clin Oncol 1997;15:1778)
Case reports
Treatment
  • Chronic or smoldering ATLL: observation may be appropriate for patients who are asymptomatic
  • ATLL is resistant to most chemotherapy
  • No standard chemotherapy regimen
  • Intensive high dose combination chemotherapy and bone marrow transplantation have been used in limited numbers of patients (Blood 2010;116:1369)
  • Monoclonal antibody based therapies have been attempted directed against:
    • IL-2R (anti-Tac)
    • CCR4 (mogamulizumab)
    • CD52 (alemtuzumab)
  • Recent clinical trials use arsenic trioxide, interferon α and zidovudine (Adv Ther 2018;35:135)
Clinical images

Images hosted on other servers:

Nodules

Gross description
  • Skin lesions have been classified as erythema, papules or nodules
  • Rare cases show tumor-like lesions or erythroderma as seen in mycosis fungoides / Sézary syndrome
  • Enlarged and effaced lymph node
Microscopic (histologic) description
  • Bone marrow:
    • Degree of bone marrow infiltration is less than expected, given the marked lymphocytosis that is often present
    • Pattern of marrow involvement can be diffuse, interstitial or sinusoidal
    • Often evidence of bone resorption and osteoclastic activity
    • Bone trabeculae: may show remodeling
    • Lytic bone lesions can be present even in the absence of tumoral bone infiltration
  • Lymph nodes:
    • Involves paracortical T cell zones
    • Typically show diffuse architectural effacement
    • May be subdivided according to cell type and pattern into:
      • Pleomorphic small cell
      • Pleomorphic medium and large cell type / pattern (most common)
      • Anaplastic large cell-like [resembling anaplastic large cell lymphoma (ALCL)]
      • Angioimmunoblastic T cell lymphoma-like (AITL)
      • Hodgkin lymphoma-like: seen in early phase of some adult T cell lymphoma
      • Leukemic pattern of infiltration with preservation or dilation of lymph node sinuses that contain malignant cells
    • Size or shape of the neoplastic cells or identification / classification of the above patterns does not impact the clinical course
  • Skin lesions:
    • Epidermal infiltration with Pautrier-like micro-abscesses is common (Mod Pathol 2018;31:1046)
    • Hyperparakeratosis is variably present in the overlying epidermis
    • Dermal infiltration: mainly perivascular but larger tumor nodules with extension to subcutaneous fat may be observed
    • Erythematous lesions: composed of smaller cells in perivascular pattern in dermis
    • Papules and nodules: composed of larger cells that replace dermis
  • Other sites commonly involved by ATLL include: liver, spleen, lungs and central nervous system
  • Grading: there is no formal grading system for ATLL

Note: ATLL should be strongly considered in any T cell lymphoma developing in a patient from an endemic area regardless of histopathologic features and anti-HTLV-1 serology testing should be performed
Microscopic (histologic) images

Contributed by Jennifer Chapman, M.D.

ATLL mimicking angioimmunoblastic T cell lymphoma


ATLL with Hodgkin-like cells mimicking classical Hodgkin lymphoma

ATLL mimicking mycosis fungoides


ATLL, lymphomatous type, mimicking anaplastic large cell lymphoma

Cytology description
  • ATLL cells show variable appearances
  • Irregular / polylobulated nuclei, homogeneous, condensed chromatin, small nucleoli
  • Agranular basophilic cytoplasm
  • Tumor cells in lymph nodes are frequently pleomorphic and most cases have a mixture of small to large pleomorphic cells
Peripheral smear description
  • Medium to large sized lymphocytes with multilobulated nuclei (flower cells) in the peripheral blood of acute (leukemic) ATLL
  • Slightly larger than normal peripheral blood lymphocytes with moderately condensed chromatin, absent or small nucleoli, scant slightly basophilic cytoplasm and some with multilobulated nuclei in chronic ATL
Peripheral smear images

Contributed by Jennifer Chapman, M.D.

ATLL, acute variant,
mimicking
T prolymphocytic
lymphoma



Images hosted on other servers:

Flower cells

CLL-like morphology

Positive stains
Negative stains
  • CD7, TdT, TCL1 (rare cases positive), ALK1, B cell antigens, cytotoxic molecules (rare cases positive), myeloid associated antigens
  • Most cases are CD8 negative
Flow cytometry description
Flow cytometry images

Contributed by Jennifer Chapman, M.D.

CD45

CD4

CD56

CD7

CD25



Images hosted on other servers:

CD3+, CD4+, CD25+, CD7-, CD8-

Electron microscopy description
  • Viral particles, 80 to 120 nm, are present in both cytoplasm and extracellular space
Electron microscopy images

Images hosted on other servers:

ATLL cell

Molecular / cytogenetics description
  • Clonal integration of HTLV-1 genome can be demonstrated
  • PCR assays are positive for clonal T cell receptor gene rearrangement
  • Quantitative HTLV-1 levels
  • CCR4 mutations in ~25% of cases (Nat Genet 2015;47:1304)
  • RHOA mutations detected in ~15% of cases (Blood 2016;127:596)
  • Tumor suppressor genes are inactivated either by mutation or epigenetic silencing
  • Numerous complex chromosomal abnormalities are frequent, especially in acute and lymphomatous variants
  • Clonal chromosome abnormalities are frequent but not specific
Differential diagnosis
Board review style question #1
Which of the following is the most common site of extranodal involvement in adult T cell leukemia / lymphoma (ATLL)?

  1. Central nervous system
  2. Heart
  3. Liver
  4. Skin
Board review style answer #1
D. Skin

Comment here

Reference: ATLL
Board review style question #2


A 67 year old Haitian man with hepatitis B presents with fatigue and weakness. No other symptoms are present. Laboratory work up shows leukocytosis (55 x 109/L) and thrombocytopenia (100 x 109/L). Physical examination and imaging studies show massive splenomegaly with no lymphadenopathy. There was no hypercalcemia or increased in serum LDH. Peripheral smear review showed abnormal lymphocytosis with circulating lymphoma cells having mature nuclear features and highly lobulated nuclei. Flow cytometry performed in bone marrow aspirate is provided above. Serology for anti HTLV-1 antibody is positive. Which of the following is the most accurate statement?

  1. Dual positivity for CD4 and CD8 is uncommon in adult T cell leukemia / lymphoma
  2. Expression of CD25 is specific for ATLL among T cell lymphomas / leukemias
  3. Patient has acute myeloid leukemia with maturation (NK / myeloid type)
  4. Patient has T cell large granular lymphocytic leukemia (LGL)
Board review style answer #2
A. Dual positivity for CD4 and CD8 is uncommon in adult T cell leukemia / lymphoma

Comment here

Reference: ATLL

ATLL

Aggressive NK cell leukemia
Definition / general
  • Aggressive NK cell leukemia (ANKL) is a rare disease with a dismal prognosis and many diagnostic challenges
  • ANKL shares several overlapping features with other NK cell neoplasms
  • The clinical presentation and course can help distinguish from other NK cell neoplasms
  • ANKL can be subtle to detect morphologically and immunophenotypically
Essential features
  • ANKL overlaps morphologically and genetically with extranodal NK / T cell lymphoma; however, its clinical presentation is acute and outcome is poorer
  • As rare EBV negative ANKL have been described, EBV negative status cannot be used as the sole criterion to exclude a diagnosis of ANKL
  • Bone marrow involvement by ANKL can be prominent or subtle
  • Cytologic atypia can be notable, however, in a subset of cases; neoplastic cells are deceptively bland
Terminology
  • Aggressive NK cell leukemia
ICD coding
  • ICD-O: 9948/3 - aggressive NK cell leukemia
Epidemiology
Sites
  • Peripheral blood, bone marrow, liver, spleen
Pathophysiology
  • In EBV positive ANKL, EBV encoded small RNAs induce release of high amounts of IL10:
    • IL10 activates the JAK / STAT pathway
    • Stimulation of STAT3 phosphorylation
    • Downstream MYC activation leading to clonal expansion
  • In EBV negative ANKL, other unidentified mechanisms lead to
    • Stimulation of STAT3 phosphorylation
    • Downstream MYC activation leading to clonal expansion
  • Reference: Cell Res 2018;28:172
Etiology
Clinical features
  • Fever, B symptoms, hepatosplenomegaly, lymphadenopathy
Diagnosis
  • Bone marrow biopsy
  • Biopsy of organs suspected of involvement by CT scan
  • Peripheral blood smear
  • Flow cytometry analysis
Laboratory
  • Neutropenia, anemia, thrombocytopenia
  • Elevated serum lactate dehydrogenase (LDH) levels
  • Elevated liver functions tests
  • Disseminated intravascular coagulopathy
  • Hemophagocytic syndrome
  • Reference: Cancers (Basel) 2020;12:2900
Prognostic factors
  • Prognosis is very poor with a median survival of less than 2 months, despite intensive chemotherapy
Case reports
Treatment
  • Allogeneic hematopoietic stem cell transplantation (SCT) improves outcome in ANKL patients for a limited time
  • No consensus chemotherapeutic regimen has been established to manage patients with ANKL; chemotherapy regimens used to manage ANKL include:
    • SMILE (dexamethasone, methotrexate, ifosfamide, etoposide and L-asparaginase)
    • AspaMetDex (L-asparaginase, methotrexate and dexamethasone)
    • VIDL (etoposide, ifosfamide, dexamethasone and L-asparaginase)
    • Reference: Cancers (Basel) 2020;12:2900
Microscopic (histologic) description
  • Bone marrow involvement by ANKL can be prominent or subtle
  • Two main patterns of infiltration: interstitial and sinusoidal
  • Neoplastic cells are medium sized, with a moderate amount of cytoplasm, highly irregular nuclei with condensed chromatin and conspicuous nucleoli
  • In some cases, atypia is subtle and neoplastic cells can look deceptively bland
  • Apoptosis and focal necrosis are common findings
  • Geographic necrosis is typically not observed in ANKL, unlike cases of extranodal NK / T cell lymphoma (ENKTL)
  • Reference: Cancers (Basel) 2020;12:2900
Microscopic (histologic) images

Contributed by Siba El Hussein, M.D. and Joseph Khoury, M.D.
Neoplastic cells involving the bone marrow Neoplastic cells involving the bone marrow Neoplastic cells involving the bone marrow

Neoplastic cells involving the bone marrow

Neoplastic cells involving the bone marrow Neoplastic cells involving the bone marrow Neoplastic cells involving the bone marrow

Neoplastic cells involving the bone marrow


Immunohistochemical stains Immunohistochemical stains Immunohistochemical stains Immunohistochemical stains

Immunohistochemical stains

Virtual slides

Images hosted on other servers:

ANKL bone marrow biopsy

Cytology description
  • Moderate amounts of basophilic agranular cytoplasm
  • Punched out cytoplasmic vacuoles
  • Highly irregular nuclear contours and an open chromatin pattern with prominent nucleoli
  • Reference: Cancers (Basel) 2020;12:2900
Peripheral smear description
  • Cells are intermediate to large in size
  • Moderate amounts of basophilic agranular cytoplasm
  • Punched out cytoplasmic vacuoles
  • Highly irregular nuclear contours and an open chromatin pattern with prominent nucleoli
  • Reference: Cancers (Basel) 2020;12:2900
Positive stains
Negative stains
Flow cytometry description
Flow cytometry images

Contributed by Siba El Hussein, M.D. and Joseph Khoury, M.D.
Flow cytometry characteristic Flow cytometry characteristic

Flow cytometry characteristic

Molecular / cytogenetics description
  • Array based comparative genomic hybridization (aCGH) analyses can demonstrate nonspecific cytogenetic findings (Genes Chromosomes Cancer 2005;44:247):
    • Gains of 1q23.1-q23.2 and 1q31.3-q44
    • Losses of 7p15.1-q22.3 and 17p13.1
  • Whole genome and exome sequencing and next generation sequencing have shown mutations in the following pathways (Am J Surg Pathol 2020;44:1235, Nat Commun 2018;9:1567, Cell Res 2018;28:172):
    • JAK / STAT (STAT3, STAT5B, STAT5A, JAK2, JAK3, STAT6, SOCS1, SOCS3 and PTPN11)
    • RAS / MAPK
    • Epigenetic modifiers (TET2, CREBBP, KMT2D, BCOR, SET2D, GFI1)
    • RNA helicase (DDX3X)
    • Cell cycle regulation and DNA damage repair (TP53, ASXL1, ASXL2, BRINP3)
    • mRNA splicing (PRPF40B)
Sample pathology report
  • Bone marrow, posterior iliac crest, core biopsy, clot section, aspirate smears and touch imprint:
    • Aggressive NK cell leukemia (ANKL) (see comment)
    • Comment: Hypocellular bone marrow for age 20 - 30%, with residual trilineage hematopoiesis
    • Flow cytometry immunophenotypic studies are performed using bone marrow aspirate material of the specimen. A distinct population of aberrant NK cells is detected. The neoplastic cells account for 49% of all analyzed cells. The neoplastic cells are positive for CD2, cytoplasmic CD3, CD7 (bright), CD45 (bright), CD48 (partial), CD56 and CD158b. The neoplastic cells are negative for CD1a, surface CD3, CD4, CD5, CD10, CD8, CD13+CD33, CD19, CD25, CD30, CD34, CD117, CD123, CD158a, CD158e, TdT, TCRαβ, TCRγδ, myeloperoxidase and HLA-DR.
    • Bone marrow biopsy: quality - suboptimal, subcortical; cellularity - 20 - 30% in the most cellular areas; megakaryocytes - significantly decreased, with unremarkable morphology; infiltrate - numerous medium sized cells with dense chromatin and scant cytoplasm in interstitial distribution
    • Bone marrow clot section: quality - adequate, particles present; cellularity: 30 - 40%; megakaryocytes - present; infiltrate - numerous medium sized cells with dense chromatin and scant cytoplasm in interstitial distribution
    • Stains on clot: Immunohistochemical stains were performed using fixed, paraffin embedded tissue of clot specimen. The neoplastic cells are positive for CD2, CD56 and p53. In situ hybridization studies for Epstein-Barr virus encoded small RNA (EBER) using fixed, paraffin embedded tissue of clot specimen were performed. The neoplastic cells are strongly positive for EBER.
    • Bone marrow aspirate: quality / cellularity - adequate, particles present; granulocytes - complete maturation; erythrocytes - complete maturation; megakaryocytes - present; lymphocytes - markedly increased, predominantly medium size, with open nuclear chromatin, inconspicuous nucleoli and moderate amount of deep basophilic cytoplasm with vacuoles
Differential diagnosis
  • Extranodal NK / T cell lymphoma (ENKTL):
    • Strong association with EBV infection
    • Cytokine secretion function
    • Overlapping morphologic and genetic features with ANKL
    • Less aggressive clinical presentation and outcome
    • Predilection to involve upper aerodigestive tract (most commonly nasal cavity), gastrointestinal tract, skin, soft tissue and testes
  • Chronic lymphoproliferative disorder of NK cells (CLPD-NK):
    • Unknown stimulus, possibly viral
    • No EBV association
    • Cytotoxic function
    • Uniform CD8 positivity
    • Loss of CD2 expression
    • Less aggressive clinical course
    • Patients are mostly asymptomatic or have symptoms related to cytopenias (Eur J Haematol 2018;100:444)
Additional references
Board review style question #1

Which of the following statements is true?

  1. Aggressive NK cell leukemia (ANKL) and extranodal NK / T cell lymphoma (ENKTL) overlap morphologically and genetically
  2. ANKL harbors specific cytogenetic features
  3. ANKL is characteristically negative for sCD3 and cCD3 by flow cytometry analysis
  4. EBER negativity rules out the diagnosis of ANKL
Board review style answer #1
A. ANKL and ENKTL overlap morphologically and genetically, rendering these 2 diagnoses very challenging in certain cases to distinguish clinically. ANKL is characteristically negative for sCD3 but positive for cCD3 by flow cytometry analysis. No specific cytogenetic features characterize ANKL cases. EBER negative ANKL cases have been well established in literature, thus EBER negativity does not rule out ANKL.

Comment Here

Reference: Aggressive NK cell leukemia
Board review style question #2
Which of the following statements is true?

  1. Aggressive NK cell leukemia (ANKL) tumor burden in bone marrow sample is invariably high
  2. Cytologic atypia in ANKL is consistently prominent
  3. Prognosis of ANKL is remarkably ameliorated posttransplant and chemotherapy
  4. The most commonly involved genetic pathway in ANKL is the JAK / STAT pathway
Board review style answer #2
D. The most commonly involved genetic pathway in ANKL is the JAK / STAT pathway. ANKL tumor burden in bone marrow sample is variable and some cases have a very low bone marrow involvement, making such cases very challenging to diagnose morphologically and immunophenotypically. Cytomorphology of leukemic cells in ANKL can be deceptively bland in a prominent percentage of cases. Prognosis of ANKL remains dismal, even posttransplant and chemotherapy.

Comment Here

Reference: Aggressive NK cell leukemia

Anaplastic large cell lymphoma, ALK negative
Definition / general
Essential features
  • T cell lymphoma
  • Characteristic hallmark cells
  • Morphologically indistinguishable from ALCL, ALK positive
  • CD30 diffusely and strongly positive
  • ALK expression negative
  • ALK gene rearrangement negative
Terminology
  • ALK- ALCL
ICD coding
  • ICD-10:
    • C84 - mature T / NK cell lymphomas
    • C84.7 - anaplastic large cell lymphoma, ALK negative
    • C84.70 - unspecified site
    • C84.71 - lymph nodes of head, face and neck
    • C84.72 - intrathoracic lymph nodes
    • C84.73 - intra-abdominal lymph nodes
    • C84.74 - lymph nodes of axilla and upper limb
    • C84.75 - lymph nodes of inguinal region and lower limb
    • C84.76 - intrapelvic lymph nodes
    • C84.77 - spleen
    • C84.78 - lymph nodes of multiple sites
    • C84.79 - extranodal and solid organ sites
Epidemiology
Sites
  • Both lymph nodes and extranodal sites, including bone, soft tissue and skin (primary cutaneous ALCL needs to be distinguished)
Pathophysiology
Clinical features
  • Mostly presents at advanced stage (stage III - IV) and B symptoms
Diagnosis
Laboratory
Radiology description
  • Enlarged lymph nodes / mass lesions in extranodal sites
Prognostic factors
Case reports
Treatment
Gross description
  • Enlarged lymph nodes
  • Firm fleshy mass
Microscopic (histologic) description
  • Most of the time, morphologically indistinguishable from ALCL, ALK positive
  • Effaced architecture with solid, cohesive sheets of neoplastic cells
  • May show preserved lymph node architecture with neoplastic cells growing intrasinusoidally or only within the T cell areas
  • Large cells with round to indented nuclei, deeply staining cytoplasm, with prominent Golgi zone
  • Hallmark cells (cells with eccentric, horseshoe or kidney shaped nuclei)
  • Small cells, as described in the small cell or lymphohistiocytic variant of ALCL, ALK positive, are uncommon
  • Cases with DUSP22 rearrangements show smaller, monomorphic cells with central nuclear pseudoinclusions (doughnut cells) (Haematologica 2019;104:e158)
  • Cases positive for DUSP22 rearrangements lack a cytotoxic phenotype
  • Cases with ERBB4 and COL29A1 expression show Hodgkin-like morphology (Blood 2016;127:221)
Microscopic (histologic) images

Contributed by Jayalakshmi Balakrishna, M.D. and Elaine S. Jaffe, M.D.

Diffuse infiltrate

Invasion of sinus

Large atypical cells

Mitotic figures and apoptotic bodies

Large atypical cells


Large atypical cells

Hallmark cells

IHC stain, CD3

IHC stain, CD4

IHC stain, CD3

IHC stain, CD25


IHC stain, Ki67

IHC stain, CD2

IHC stain, perforin

IHC stain, CD30

IHC stain, CD43


IHC stain, CD45

IHC stain, EMA

IHC stain, granzyme



Contributed by Doan Minh Khuy, M.D.

23 year old man with a cervical lymphoid nodule, no B symptoms


CK-

EMA+

ALK-

CD15-

CD20-

CD5+

Virtual slides

Images hosted on other servers:

ALCL, ALK-, ALK1

ALCL, ALK-, CD30

ALCL, ALK-, H&E

ALCL, ALK+, ALK1

ALCL, ALK+, CD30

ALCL, ALK+, H&E

Cytology description
  • Large atypical neoplastic cells, multinucleation / multilobated nuclei, prominent Golgi zone, basophilic cytoplasm
  • Basophilic nucleoli, generally less prominent than seen in Hodgkin Reed-Sternberg cells
Cytology images

Contributed by Jayalakshmi Balakrishna, M.D.

Lymph node biopsy touch preparation

Positive stains
  • Strong and diffuse uniform CD30 staining in all tumor cells (membrane and Golgi zone pattern, also cytoplasmic)
  • Variable expression / loss of pan-T cell antigens: CD2+ and CD3+ more often than CD5+
  • Almost always CD43+ and CD45+; can be used to distinguish from carcinoma
  • Often CD4+, rarely CD8+
  • TIA1, granzyme B, perforin, clusterin, fascin, EMA (variable)
  • Cases with DUSP22 rearrangements tend to be less frequently positive for cytotoxic markers
Negative stains
Flow cytometry description
  • Flow cytometry shows the anaplastic large cell lymphoma cells with high forward and side light scatter properties (consistent with large and complex cells) and high density CD45 expression simulating monocytes
  • Tumor cells express CD30 and one or more of the T cell associated antigens (Arch Pathol Lab Med 2009;133:49)
Flow cytometry images

Contributed by Sanam Loghavi, M.D.

CD45 and side light scatter

CD3 and CD4

CD4 and CD8

CD5 and CD2


TCR A / B

CD3 and CD30

CD4 and CD30

Molecular / cytogenetics description
  • T cell receptor (TCR) clonal gene rearrangement in most cases, irrespective of T cell antigen expression
  • DUSP22-IRF4 locus on 6p25.3 (referred to as DUSP22 rearrangements); (6;7)(p25.3;q32.3) (Blood 2011;117:915)
  • Recurrent rearrangement involving the TP63 gene in a minority of cases (Blood 2012;120:2280)
  • Gains of the 1q41-qter and 6p21 are common cytogenetic aberrations (Br J Haematol 2008;140:516)
  • ERBB4 and COL29A1 are expressed in 24% of ALK- ALCL, mutually exclusive with TP63 rearrangement (Blood 2016;127:221)
  • STAT3 activating mutations in a subset of cases (Cancer Cell 2015;27:516)
Molecular / cytogenetics images

Contributed by Andrew Feldman, M.D.

Break apart probe FISH for chromosome region 6p25.3

Sample pathology report
  • Mass, left breast, excision:
    • Anaplastic large cell lymphoma, ALK negative (see synoptic report)
Differential diagnosis
  • ALCL, ALK positive:
    • ALK+, younger age, less aggressive
  • Classical Hodgkin lymphoma:
    • Characteristic Hodgkin Reed-Sternberg cells, CD30 and CD15 positive
    • Can be EBV positive, cytotoxic markers and T cell markers negative
  • Peripheral T cell lymphoma, NOS:
    • Difficult differential, WHO recommends conservative approach (diagnose ALCL, ALK- only if very similar to ALCL, ALK+, except for ALK expression)
    • Overall worse prognosis than ALCL, ALK-
  • Primary cutaneous ALCL:
    • Gene expression profiling shows that ALCL, ALK negative is closely related to primary cutaneous ALCL
    • Much better prognosis, clinical correlation with staging necessary
Board review style question #1
What is the morphologic characteristic of anaplastic large cell lymphoma, ALK negative with DUSP22-IRF4 rearrangement?

  1. Eosinophilia
  2. Hodgkin-like cells
  3. Prominent nucleoli and intranuclear inclusions (doughnut cells)
  4. Small cell morphology
Board review style answer #1
C. Prominent nucleoli and intranuclear inclusions (doughnut cells)

Comment Here

Reference: Anaplastic large cell lymphoma, ALK negative
Board review style question #2
What is a prognostic factor in anaplastic large cell lymphoma, ALK negative?

  1. Age of the patient
  2. Gender of the patient
  3. Mitotic count
  4. Tumor necrosis
Board review style answer #2

Anaplastic large cell lymphoma, ALK positive
Definition / general
Essential features
  • T cell lymphoma
  • Characteristic hallmark cells
  • CD30 diffusely and strongly positive
  • ALK positive
  • ALK gene rearrangement
Terminology
  • Also called Ki-1 (CD30+) lymphoma, ALCL (obsolete)
ICD coding
  • ICD-10: C84 - mature T / NK cell lymphomas
    • C84.6 - anaplastic large cell lymphoma, ALK positive
    • C84.60 - unspecified site
    • C84.61 - lymph nodes of head, face, and neck
    • C84.62 - intrathoracic lymph nodes
    • C84.63 - intra-abdominal lymph nodes
    • C84.64 - lymph nodes of axilla and upper limb
    • C84.65 - lymph nodes of inguinal region and lower limb
    • C84.66 - intrapelvic lymph nodes
    • C84.67 - spleen
    • C84.68 - lymph nodes of multiple sites
    • C84.69 - extranodal and solid organ sites
Epidemiology
Sites
  • Frequent involvement of lymph nodes or extranodal sites (J Clin Exp Hematop 2017;57:120)
  • Nodal involvement often not contiguous
  • Most common extranodal sites are skin, bone, soft tissues, lung and liver (Blood 2000;96:3681)
  • Compared to classic Hodgkin lymphoma, inguinal nodal involvement is more common and mediastinal disease is less common
  • Gastrointestinal involvement is rare
  • Marrow involvement detected in 10 - 20% of cases; CD30 is helpful in identifying bone marrow involvement (Histopathology 2013;63:13, Hum Pathol 2008;39:1331)
  • Central nervous system involvement is more commonly seen with bone marrow involvement and is a poor prognostic finding
  • Occasional leukemic presentation in peripheral blood with the small cell variant (Br J Haematol 2014;165:545)
Pathophysiology
Etiology
  • Etiology is unknown; the anaplastic lymphoma receptor tyrosine kinase ALK gene (2p23) is overexpressed due to t(2;5)(p23;q35) translocation (Orphanet: Anaplastic Large Cell Lymphoma [Accessed 27 January 2020])
  • Cases with the NPM-ALK translocation show both nuclear and cytoplasmic staining for ALK
  • Variant translocations involving gene partners other than NPM show mainly cytoplasmic staining
  • Not Epstein-Barr virus (EBV) related (Hum Pathol 2004;35:455)
  • Clinical presentation following insect bites has been described (Haematologica 2010;95:449)
    • This may be related to homing of neoplastic cells to sites of inflammation
Clinical features
Diagnosis
Laboratory
Radiology description
  • Enlarged lymph nodes / mass lesion in involved tissue
Prognostic factors
  • Moderately aggressive, may be curable; better prognosis than other peripheral T cell lymphomas and ALK- (80% versus 48%) (J Clin Exp Hematop 2017;57:120)
  • 5 year failure free survival (FSS) also better than ALK- tumors (60% versus 36%) (Blood 2008;111:5496)
  • International prognostic index (IPI) has limited prognostic value compared to other lymphomas
  • No prognostic difference between classic and variant translocations
  • Small cell or lymphohistiocytic variant morphology often present with disseminated disease at diagnosis (J Clin Oncol 2011;29:4669)
  • Circulating tumor cells are associated with poor prognosis (Pathology 2020;52:100)
  • 30% relapse, often remain sensitive to chemotherapy (J Clin Exp Hematop 2017;57:120)
Case reports
Treatment
Gross description
  • Enlarged lymph node
  • Firm, fleshy mass
Microscopic (histologic) description
  • Broad morphologic spectrum: small to medium sized to large anaplastic cells (Pathology 2020;52:100)
  • Infiltration of interfollicular T zones and subcapsular sinuses by neoplastic cells
  • Cells show cohesive growth pattern
  • Intrasinusoidal infiltration pattern (Blood 2015;126:17)
  • Anaplastic large cells with abundant cytoplasm, wreath-like or multiple nuclei, open chromatin, multiple nucleoli, perinuclear eosinophilic region (prominent Golgi zone) (Crit Rev Oncol Hematol 2012;83:293)
  • Cells with horseshoe / kidney / embryo shaped nuclei are referred to as hallmark cells (Cancers (Basel) 2018;10:E107)
  • Occasional nuclear pseudoinclusions (doughnut cells) (Adv Anat Pathol 2015;22:29)
  • Brisk mitotic activity
  • Sinusoidal involvement may mimic histiocytic neoplasms
  • In the lymphohistiocytic variant, neoplastic cells may be masked by abundant histiocytes
  • Peripheral distribution of neoplastic cells is common in lymph nodes and other sites
  • Bone marrow varies from extensive involvement by tumor cells to only scattered cells that may be overlooked
Morphologic variants
  • Lymphohistiocytic (10%) (Histopathology 1990;16:383, Virchows Arch 2008;452:599)
    • Numerous histiocytes and small lymphocytes, occasional erythrophagocytosis
    • Neoplastic cells tend to cluster around blood vessels
  • Small cell (5 - 10%) (Arch Pathol Lab Med 2010;134:1706)
    • 25% transform to classic anaplastic large cell lymphoma, usually associated with death within a year; necrosis may predict transformation (Am J Surg Pathol 1999;23:49)
    • Often systemic symptoms, often misdiagnosed as peripheral T cell lymphoma, not otherwise specified
    • Loss of CD3 is clue to diagnosis and distinction from peripheral T cell lymphoma
    • High incidence of marrow involvement but difficult to identify without immunostains
    • Leukemic presentation more common in this variant
    • Predominantly small cells with irregular nuclei, hallmark cells present (tend to be perivascular), occasional fried egg cells, rare signet ring cells (Arch Pathol Lab Med 2011;135:19)
  • Hodgkin-like (1 - 3%) (Am J Surg Pathol 2006;30:223)
    • Mimics nodular sclerosis classic Hodgkin lymphoma; capsular fibrosis and fibrotic bands forming cellular nodules
    • Mononuclear cells
    • Binucleate Reed-Sternberg-like cells and mummified cells may be present
    • Eosinophils and neutrophils less common than in classic Hodgkin lymphoma
  • Hypocellular (Am J Surg Pathol 2000;24:1537)
    • Small cell types may have hypocellular, granulation tissue-like appearance
    • Young patients with lymphadenopathy
    • Lymphoid cells separated by edematous or fibromyxoid stroma with myofibroblast-like neoplastic cells forming short, sweeping fascicles and histiocytes; occasional large cells with atypical nuclei noted; perivascular cuffing of large cells
  • Other patterns described
    • Sarcomatoid, giant cell rich and signet ring-like patterns
  • Composite pattern (10 - 20%)
    • More than one pattern may be seen in a single lymph node
    • Relapses may differ morphologically from the original tumor
Microscopic (histologic) images

Contributed by Jayalakshmi Balakrishna, M.D., Elaine S. Jaffe, M.D.

Diffuse infiltrate

Large pleomorphic cells

Large cells with pleomorphic nuclei

Large cells with pleomorphic nuclei


Large cells with pleomorphic nuclei

Mitoses and apoptosis

Intrasinusoidal infiltrate

Intrasinusoidal infiltrate


Focal infiltrate

Hallmark cells

Hallmark cells

Polymorphous background

CD3


CD3

CD30

ALK1

ALK1

ALK1

EMA


Granzyme B

CD45

Bone marrow aspirate smear

Bone marrow biopsy

Lymphohistiocytic variant


Small cell variant

Sarcomatoid variant

Virtual slides

Images hosted on other servers:

Anaplastic large cell lymphoma, ALK Positive

Cytology description
  • Deeply basophilic cytoplasm, prominent vacuoles, round or lobate nuclei, prominent nucleoli, clumped chromatin, multinucleation
Cytology images

Contributed by Jayalakshmi Balakrishna, M.D.

Large neoplastic cells

Large neoplastic cells in polymorphous background

Positive stains
  • ALK expression varies by translocation; use monoclonal antibodies (Adv Anat Pathol 2015;22:29)
    • Note: all postnatal human tissue is ALK- except rare cells in brain
    • ALK is name of gene and protein
    • ALK1 is a commonly used antibody for IHC
  • CD30 (cytoplasmic and Golgi staining, strongest in large cells), EMA (majority)
  • CD2, CD4, CD5 (one is positive in 70% of cases), also TIA1, granzyme B, perforin, CD45, CD45RO, CD61, CD25 (strong)
  • Often negative for CD3 (< 25%), CD43, B cell markers (10%)
  • Rare CD8CD15, clusterin, fascinfactor VIII, CD13 (Arch Pathol Lab Med 2000;124:1804)
  • Small cell and lymphohistiocytic variants are ALK1+ but cells are not large or anaplastic
  • ALK+ anaplastic large cell lymphoma (T or null) is considered a single entity since no other differences can be found and null cell phenotypes show genetic evidence for T cell lineage
Negative stains
Electron microscopy description
  • Gold immunolabeling: CD30+ particles in the Golgi complex and cell membranes
  • Long, slender, microvillus-like processes (anemone cell tumors, Ultrastruct Pathol 1984;7:143)
  • Not used in routine diagnosis
Molecular / cytogenetics description
  • Clonal T cell receptor (TCR) gene rearrangement seen in 90% (irrespective of T / null cell phenotype)
  • All translocations result in upregulation of ALK protein (Science 1994;263:1281)
  • ALK translocation results in activation of the downstream oncogenic transcription factor, STAT3 (Cancer Cell 2015;27:516)
  • STAT3 activation regulates availability of hypoxia inducible factors involved in tumor growth and metastasis (Cancer Res 2014;74:6094)
  • t(2;5)(p23;q35): ALK and NPM (75 - 85%); translocation of anaplastic lymphoma kinase on chromosome 2 and nucleophosmin gene on chromosome 5; gene product present in nucleus and cytoplasm, can also be detected by RT-PCR (Science 1994;263:1281)
  • t(1;2)(q25;p23): tropomyosin 3 (TPM3) and ALK (13%), diffuse cytoplasmic staining with peripheral intensification (Blood 1999;93:3088)
  • inv(2)(p23;q35): ALK and ATIC (1%), diffuse cytoplasmic staining (Blood 1998;92:2688)
  • t(2;3)(p23;q21): ALK and TRK fused gene (TFG) (< 1%), diffuse cytoplasmic staining (Blood 1999;94:3265)
  • t(2;17)(p23;q23): ALK and clathrin heavy chain-like (CLTCL) (< 1%), granular cytoplasmic staining
  • t(2;X)(p23;q11-12): ALK and moesin gene (MSN) (< 1%), membrane staining (Hum Pathol 2004;35:1038)
  • t(2;19)(p23;p13.1): ALK and nonmuscular tropomyosin gene (TPM4) (< 1%), diffuse cytoplasmic staining
  • t(2;22)(p23;q11.2): ALK and myosin heavy chain 9 gene (MYH9) (< 1%), diffuse cytoplasmic staining (Genes Chromosomes Cancer 2003;37:427)
  • t(2;17)(p23;q25): ALK and ALK lymphoma oligomerization partner (ALO17) (< 1%), diffuse cytoplasmic staining
  • Secondary genetic alterations: -4, del11q, del13q, +7, +17p, +17q (different from anaplastic large cell lymphoma, ALK negative, Br J Haematol 2008;140:516)
  • ALK immunohistochemistry, FISH and RT-PCR results are comparable (Am J Surg Pathol 1999;23:1386)
Sample pathology report
  • Lymph node, right inguinal, excisional biopsy:
    • Anaplastic large cell lymphoma, ALK positive
Differential diagnosis
Board review style question #1



A 42 year old man was diagnosed with anaplastic large cell lymphoma, ALK positive in an inguinal lymph node. The image shows an immunohistochemical stain with characteristic staining pattern. What stain is this?

  1. ALK1
  2. CD3
  3. CD4
  4. CD30
Board review style answer #1
D. CD30. Cytoplasmic stain with Golgi zone accentuation.

Comment Here

Reference: Anaplastic large cell lymphoma, ALK positive
Board review style question #2
What is the downstream genetic pathway involved in the pathogenesis of anaplastic large cell lymphoma?

  1. MAPK
  2. NFκB
  3. STAT3
  4. Wnt
Board review style answer #2

Anaplastic large cell lymphoma, ALK positive

Angioimmunoblastic T cell lymphoma

Angioimmunoblastic T cell lymphoma
Definition / general
Essential features
Terminology
  • Angioimmunoblastic lymphadenopathy with dysproteinemia (AILD)
  • Immunoblastic lymphadenopathy
  • Lymphogranulomatosis X
ICD coding
  • ICD-O: 9705/3 - angioimmunoblastic T cell lymphoma
  • ICD-10: C86.5 - angioimmunoblastic T cell lymphoma
Epidemiology
Sites
Pathophysiology
  • T follicular helper (TFH) cells are essential checkpoints for the B cell activation and differentiation from centroblast to centrocytes; consequently these cells become either plasma cells or memory B cells (Blood 2017;129:1095)
    • TFH tolerance is an important mechanism in the prevention of autoimmune diseases
    • Dysregulation of TFH checkpoints may cause a germinal center anarchy and consequent AITL
Etiology
  • Somatic mutations (Blood 2017;129:1095, Nat Genet 2014;46:371):
    • TET2, DNMT3A, IDH2-R172 and RHOA
  • Immunodysregulation (Blood 2017;129:1095, Leukemia 2022;36:165)
    • Upregulation of TFH cells promotes a high differentiation of B cells, plasmacytic differentiation and hypergammaglobulinemia
      • B cells of the tumor microenvironment decrease the expression of CD73 and CXCR5
    • Expansion of a distinct population of CD8+ T cells with exhausted phenotype and upregulation of the chemokines XCL2 and XCL1
  • Possible relationship to Epstein-Barr virus (EBV) with disease progression (Blood 2017;129:1095, Am J Surg Pathol 2016;40:335)
    • B cells in AITL usually show active EBV infection
Clinical features
Diagnosis
Laboratory
Radiology description
  • CT usually shows low bulky disease, which has variable uptake values on PET (Blood 2017;129:1095)
Prognostic factors
Case reports
Treatment
  • Single agent oral therapies or intensive chemotherapy combination followed by autologous stem cell transplantation (Blood 2017;129:1095, J Clin Oncol 2009;27:3951, Leukemia 2020;34:2592, Blood 2008;111:4463, J Oncol Pract 2019;15:137)
    • Good response but short remission duration
    • No gold standard chemotherapy, usually anthracycline based regimen
      • CHOP (cyclophosphamide, doxorubicin, vincristine, prednisone)
        • 53% of complete remission in up front setting
      • CHOEP (CHOP + etoposide)
        • 51% of complete remission in up front setting
      • Romidepsin + CHOP
        • 51% of complete remission in relapsed / refractory setting
      • Bevacizumab + CHOP
      • R-CHOP (rituximab + CHOP)
      • Alemtuzumab + CHOP
      • ACVBP (doxorubicin, cyclophosphamide, vindestine, bleomycin, prednisone)
      • ACVBP (doxorubicin, cyclophaphamide, vindesine, bleomycin, prednisone)
      • Bortezomibe + ACVBP
      • mBACOD (methotrexate, bleomycin, doxorubicin, cyclophosphamide, vincristine, dexamethasone)
      • PEGS (cisplatinum, etoposide, gemcitabine, methylprednisolone)
      • ICE (iphosphamide, carboplatin, etoposide), DHAP (dexamethasone, cytarabine, cisplatinum) and ESHAP (etoposide, methylprednisolone, cisplatinum, cytarabine)
        • Relapsed / refractory setting
    • Single agents
      • Used in a relapsed / refractory setting
      • Pralatrexate
      • Romidepsin
      • Brentuximab vedotin
      • Belinostat
      • Bendamustine
      • Lenalidomide
      • Hypomethylating agents
        • Azacitidine and decitabine
          • Counterbalance the DNA hypermethylation caused by TET2 and IDH2 mutation
    • Consolidation with stem cell transplantation
      • Able to achieve 70% of complete remission and progression free survival of 42% in 2 years
      • Complete remission patients have better survival compared with those with partial remission (PR)
      • Graft versus host disease (GVHD) may affect up to 60%
Microscopic (histologic) description
  • Lymph node (Leuk Lymphoma 2016;57:2804, Blood 2017;129:1095, Histopathology 2014;64:171, Hum Pathol 2010;41:79, Mod Pathol 2009;22:753, Am J Surg Pathol 2007;31:1077, Leukemia 2020;34:2592)
    • Partial or complete architecture effacement
      • Pattern I: architecture partially preserved and hyperplastic follicles with tingible body macrophages and indistinct mantle zones
      • Pattern II: architecture mostly preserved with residual follicles
      • Pattern III: architecture completely effaced without residual follicles
    • AITL cells are the minority population and are located adjacent to high endothelial venules (HEV) or around follicles
      • Small to intermediate size, clear cytoplasm and minimal atypia
        • Clear cytoplasm in ~30% of cases
    • Polymorphous inflammatory background composed of reactive lymphocytes, histiocytes, eosinophils, plasma cells, reactive B cells, Reed-Sternberg-like cells, immunoblasts and follicular dendritic cells
  • Peripheral blood (Hum Pathol 2010;41:79, Leukemia 2006;20:296, Medicine (Baltimore) 2007;86:282)
    • Limited evaluation
    • Lymphopenia
    • Cases with pronounced involvement usually show polymorphic lymphoid population composed of few atypical cells, lymphoplasmacytic, immunoblasts, granular lymphocytes and plasma cells; neoplastic cells are small to large sized lymphocytes with regular or slightly indented nuclei, moderated condensed chromatin, small or indistinct nucleoli and abundant basophilic cytoplasm
    • Cases with hypergammaglobulinemia may show rouleaux formation
  • Bone marrow (Hum Pathol 2010;41:79, Am J Clin Pathol 2007;128:854, Leukemia 2006;20:296, Medicine (Baltimore) 2007;86:282)
    • Bone marrow aspirates are not optimal for diagnosis: very few or absent atypical lymphocytes
    • Usually multifocal and poorly defined nodular pattern in core biopsies
      • Rare cases of paratrabecular pattern
      • Minority of cases present with HEV, stromal edema or patent sinusoids
    • Heterogeneous cellular background
    • Hemophagocytic syndrome is rare
    • Reticulin stain is usually disrupted in lymphoid aggregates
  • Skin (Medicine (Baltimore) 2007;86:282, Am J Surg Pathol 2007;31:1068)
    • Broad histological presentation
      • Histologic picture of vasculitis is more common
      • Band-like subepidermal, perivascular, periadnexial or diffuse infiltrate
      • Epidermotropism not present
Microscopic (histologic) images

Contributed by Roberto N. Miranda, M.D.
Pattern I: partial nodal involvement

Pattern I: partial nodal involvement

Pattern III: complete effacement of nodal architecture

Pattern III: complete effacement

Pattern III: capsular involvement

Pattern III: capsular involvement

Polymorphic infiltrate

Polymorphic infiltrate

Polymorphic infiltrate with eosinophilia

Polymorphic infiltrate with eosinophilia


Bone marrow involvement

Bone marrow involvement

Diffuse reactivity with CD3 Diffuse reactivity with CD3

Diffuse reactivity with CD3

CD20

CD20

CD21, pattern I

CD21, pattern I


CD21, pattern III

CD21, pattern III

PD-1 positivity

PD-1 positivity

CXCL13 positivity

CXCL13 positivity

CD10 in small and intermediate size lymphoma cells

CD10 in small and intermediate size lymphoma cells

CD30 positivity

CD30 positivity


Epstein-Barr virus (EBER) reactivity

Epstein-Barr virus (EBER) reactivity

Proliferation marker Ki67

Proliferation marker Ki67

CD3 reactivity in bone marrow

CD3 reactivity in bone marrow

CD10 reactivity

CD10 reactivity

Positive stains
Negative stains
Flow cytometry description
Molecular / cytogenetics description
Sample pathology report
  • Axillary lymph node, excisional biopsy:
    • Angioimmunoblastic T cell lymphoma (see comment)
    • Comment: According to clinical notes, patient is a 65 year old man with generalized lymphadenopathy, B symptoms, splenomegaly, anemia and thrombocytopenia. The peripheral blood shows anemia and thrombocytopenia. H&E sections show a complete effacement of the lymph node architecture by a diffuse and polymorphic infiltrate. Paracortical areas show increased vascularity with an arborizing pattern and the endothelial cells are prominent. The polymorphic infiltrate is composed of small to intermediate size atypical cells with slight nuclear irregular contours admixed with plasma cells, Reed-Sternberg-like cells, neutrophils and reactive small lymphocytes.
    • Immunohistochemical studies show a neoplastic population positive for CD2, CD3, CD4, CD5, CXCL13, CD10 and PD-1. The aberrant cells are negative for CD7, CD8, CD20, CD30, CD138 and ALK1. Kappa and lambda highlight polytypic plasma cells. The CD21 marker highlights follicular dendritic cell meshworks, which appear expanded around clusters of vessels. Epstein-Barr virus encoded small RNAs (EBER) in situ hybridization was positive in scattered large cells. Concurrent flow cytometry immunophenotype showed an aberrant T cell population positive for CD2, CD4, CD5 and CD10. The aberrant cells are negative for sCD3, CD7, CD8, CD19, CD20, CD56, TCRαβ (partial loss) and TCRγδ. A next generation sequencing panel demonstrated TET2, DNMT3, RHOA and IDH2 mutations. Polymerase chain reaction for Ig heavy chain (IGH) and T cell receptor gamma (TRG) were germline and T cell receptor beta (TRB) was monoclonal.
Differential diagnosis
Board review style question #1

What is the characteristic immunophenotype of angioimmunoblastic T cell lymphoma (AITL)?

  1. εCD3+ / CD4+ / CD7+ / CD8+ / CXCL13+ / PD-1-
  2. εCD3+ / CD4+ / CD7- / CD8- / CXCL13+ / PD-1+
  3. εCD3+ / CD4- / CD7- / CD8+ / CXCL13+ / PD-1+
  4. εCD3+ / CD4+ / CD7+ / CD8- / CXCL13- / PD-1+
Board review style answer #1
B. εCD3+ / CD4+ / CD7- / CD8- / CXCL13+ / PD-1+. Angioimmunoblastic T cell lymphoma (AITL) cells are usually positive for εCD3, CD4 and TFH markers (e.g. CXCL13, PD-1, CD10 and ICOS). Loss of CD7 is also observed.

Comment Here

Reference: Angioimmunoblastic T cell lymphoma
Board review style question #2
Which of the following is true about angioimmunoblastic T cell lymphoma (AITL)?

  1. Histologic pattern I is characterized by a complete effacement of the lymph node architecture
  2. Peripheral blood involvement is common
  3. The most common clinical presentation is localized disease and type A symptoms
  4. The most common mutations are TET2, DNMT3A, IDH2 and RHOA
Board review style answer #2
D. The most common mutations in angioimmunoblastic T cell lymphoma (AITL) are TET2, DNMT3A, IDH2 and RHOA. There are 3 histological patterns in the lymph node: pattern I (architecture partially preserved and hyperplastic), pattern II (architecture mostly effaced but residual follicles can be identified) and pattern III (architecture completely effaced). Peripheral blood is involved in a subset of cases. The most common clinical presentation is with advanced stage disease and type B symptoms. The most common mutations are TET2, DNMT3A, IDH2 and RHOA.

Comment Here

Reference: Angioimmunoblastic T cell lymphoma

B cell monoclonal lymphocytosis
Definition / general
  • Circulating clonal B cells (< 5 x 109/L) in patients who have no other signs of a lymphoproliferative disorder
  • Most common phenotype is chronic lymphocytic leukemia-like (CLL-like), though other phenotypes do exist (atypical CLL and non-CLL phenotypes) (Blood 2015;126:454)
Essential features
  • Monoclonal B cell lymphocytosis (MBL): monoclonal population of B cells in the blood of < 5 x 109/L
  • Asymptomatic and without extramedullary involvement
  • Subtypes include CLL-like MBL, MBL with atypical CLL phenotype, MBL with non-CLL phenotype
  • CLL-like subtype can be divided into low count monoclonal B cell lymphocytosis (< 0.5 x 109/L, progression to CLL is exceedingly rare) and high count monoclonal B cell lymphocytosis (> 0.5 x 109/L, progression to CLL 1 - 2% per year)
ICD coding
  • ICD-O:
    • 9591/1 - monoclonal B cell lymphocytosis, non-CLL type
    • 9823/1 - monoclonal B cell lymphocytosis, CLL type
Epidemiology
  • Prevalence: reported between < 1 - 18% (depending on assay sensitivity and populations studied)
  • M:F = 1.5:1 (Blood 2014;123:1319)
  • Incidence increases with age: < 1% at < 40 years to as high as 75% at > 90 years (Blood 2009;114:33)
  • Possible risk factors for onset: family history of CLL, certain single nucleotide polymorphisms, infections (including hepatitis C, pneumonia, influenza, cellulitis, upper respiratory infections and herpes zoster)
Sites
  • Blood
  • Bone marrow: involvement is common (median of 20% of marrow cellularity composed of clonal B cells); in the absence of other criteria, the percentage of marrow infiltration does not change the diagnosis from monoclonal B cell lymphocytosis to CLL (Blood 2015;126:454)
Pathophysiology
  • Proposed natural history: oligoclonal expansions of CD5+ B cells may progress to monoclonal B cell lymphocytosis, which may progress to CLL (J Exp Med 2012;209:2183)
  • High count CLL-like monoclonal B cell lymphocytosis appears to be biologically similar to CLL, while low count is genetically different (based on biased usage of IGHV genes) (Leukemia 2010;24:512)
  • Correlations with chronic infections suggest chronically stimulating the immune system may play a role in pathophysiology (Blood 2015;126:454)
  • CLL-like monoclonal B cell lymphocytosis is considered a precursor lesion to CLL: virtually all CLLs evolve from monoclonal B cell lymphocytosis, though not all monoclonal B cell lymphocytosis cases progress to CLL (Blood 2011;118:6618)
Clinical features
  • By definition, asymptomatic
  • May be an incidental finding or identified in work up for lymphocytosis
Diagnosis
  • Monoclonal population of B cells in the blood of < 5 x 109/L (Leukemia 2010;24:512, Blood 2015;126:454)
    • CLL-like monoclonal B cell lymphocytosis: CD5+, CD19+, dim CD20+, CD23+, dim surface immunoglobulin (or lack surface immunoglobulin)
      • Low count monoclonal B cell lymphocytosis: circulating population < 0.5 x 109/L
      • High count monoclonal B cell lymphocytosis: circulating population > 0.5 x 109/L
    • Monoclonal B cell lymphocytosis with atypical CLL phenotype: CD5+, CD19+, CD20+ (bright), CD23 variable, moderate to bright surface immunoglobulin
    • Monoclonal B cell lymphocytosis with non-CLL phenotype: dim or negative CD5, CD19+, CD20+, moderate to bright surface immunoglobulin
  • Asymptomatic and without lymph node, visceral or other extramedullary involvement
  • No features of another B cell lymphoproliferative disorder
  • While the diagnosis is based on absolute counts for clonal B cells, a standard method of determining such an absolute count does not currently exist
    • One method is to use dual platform flow cytometry (using CBC parameters from the hematology instrument in conjunction with the flow cytometry percentage); however, there are numerous variables that affect the method’s accuracy and precision
    • While single platform flow cytometric testing using beads or a volumetric approach can have improved accuracy and precision, the method is not widely available for leukemia / lymphoma type immunophenotyping assays (Cytometry B Clin Cytom 2021;100:142)
Prognostic factors
  • Absolute monoclonal B cell count
  • CLL-like monoclonal B cell lymphocytosis is more likely to persist than atypical or non-CLL phenotypes (Blood 2011;118:6618)
  • Mutated immunoglobulin heavy chain variable (IGHV) genes predict a longer time to first treatment and improved overall survival (Leukemia 2010;24:512)
Case reports
Treatment
  • Low count monoclonal B cell lymphocytosis: no specific clinical follow up
  • High count monoclonal B cell lymphocytosis: annual complete blood count and periodic lymph node examination; no treatment (Leukemia 2010;24:512)
Microscopic (histologic) description
Peripheral smear description
  • May or may not see an atypical lymphocyte population, depending on the number of monoclonal B cells
Peripheral smear images

Contributed by Elizabeth Courville, M.D.
Atypical monotonous lymphocyte population

Atypical monotonous lymphocyte population

Mature circulating atypical lymphocytes

Mature circulating atypical lymphocytes

Flow cytometry description
  • CLL-like monoclonal B cell lymphocytosis:
  • Monoclonal B cell lymphocytosis with atypical CLL phenotype:
  • Monoclonal B cell lymphocytosis with non-CLL phenotype:
  • Reference: Blood 2015;126:454
Flow cytometry images

Contributed by Meghan Hupp, M.D.
CLL-like

CLL-like

Non-CLL phenotype

Non-CLL phenotype

Molecular / cytogenetics description
  • Majority of CLL-like monoclonal B cell lymphocytosis cases have mutated IGHV genes (up to 90% in high count)
  • CLL-like monoclonal B cell lymphocytosis cases can have similar cytogenetic alterations to that of CLL, although at different frequencies (Br J Haematol 2012;157:86)
  • Genetic abnormalities frequently identified in chronic lymphocytic leukemia (deletion of the long arm of chromosome 13, deletion of the long arm of chromosome 11, deletion of the short arm of chromosome 17 and the presence of trisomy 12) are also seen in monoclonal B cell lymphocytosis
  • In low count MBL, deletions of chromosome 13 long arm (deletion 13q) were most frequently identified (Blood 2011;118:6618)
  • In contrast, by FISH, it appears that the distribution of genetic abnormalities in high count MBL is comparable to that of CLL (Blood 2015;126:454)
  • Elucidation of underlying genetic differences between MBL and CLL is an area of active investigation (Semin Oncol 2016;43:201)
Sample pathology report
  • Peripheral blood, flow cytometry:
    • CD5 positive, kappa monotypic B cell population (see comment)
    • Comment: This immunophenotypic assay identified a monotypic B cell population with a chronic lymphocytic leukemia / small lymphocytic lymphoma (CLL / SLL)-like immunophenotype. The main differential lies between CLL / SLL and monoclonal B cell lymphocytosis with a CLL-like immunophenotype. The monotypic B cell population comprises an estimated 10% of the total lymphocytes. Clinical correlation is recommended, including evaluation for lymphadenopathy / organomegaly.
Differential diagnosis
  • Chronic lymphocytic leukemia:
    • Monoclonal population absolute count will differentiate CLL from CLL-like monoclonal B cell lymphocytosis (≥ and < 5 x 109/L, respectively); the characteristic morphology and phenotype of CLL should be seen as well to make a CLL diagnosis
      • Of note, some patients have counts that oscillate between monoclonal B cell lymphocytosis and CLL for some time
  • Small lymphocytic lymphoma with minimal peripheral blood involvement:
    • Physical exam and imaging with or without biopsy to rule out extramedullary disease
  • Mantle cell lymphoma with peripheral blood involvement:
    • Especially important to rule out in monoclonal B cell lymphocytosis with atypical CLL phenotype; the presence of t(11;14) may help distinguish
  • Other small B cell lymphoma with low level peripheral blood involvement (particularly monoclonal B cell lymphocytosis with non-CLL phenotype or atypical CLL phenotype):
    • Some cases of monoclonal B cell lymphocytosis with non-CLL phenotype have 7q aberrations and develop splenomegaly, suggesting a connection with splenic marginal zone lymphoma (Blood 2014;123:1199)
  • Reactive (polyclonal) lymphocytosis:
    • May be in the differential prior to flow cytometry evaluation
Board review style question #1
A 65 year old woman has a flow cytometry study performed for lymphocytosis. She has a white blood cell count of 9.1 x 109/L with 58% lymphocytes. By flow cytometric immunophenotyping, a monoclonal B cell population is identified comprising 30% of leukocytes (absolute count of 2.73 x 109/L) with the following immunophenotype: CD5+, CD19+, CD20+ (moderate to bright), CD23 negative and moderate monotypic surface kappa light chain expression. Which of the following would be most helpful in establishing a diagnosis?

  1. Erythrocyte sedimentation rate and C reactive protein levels
  2. Fluorescence in situ hybridization to evaluate for t(11;14)
  3. Lumbar puncture with cell count
  4. Whole genome sequencing
Board review style answer #1
B. Fluorescence in situ hybridization. The patient has a CD5+ B cell clone with an absolute count of 2.73 x 109/L. The differential includes monoclonal B cell lymphocytosis with atypical CLL phenotype and mantle cell lymphoma with peripheral blood involvement. FISH for t(11;14) would assist in differentiating these 2 entities.

Comment Here

Reference: B cell monoclonal lymphocytosis
Board review style question #2

Which of the following is true regarding CLL-like monoclonal B cell lymphocytosis?

  1. It always progresses to chronic lymphocytic leukemia (CLL)
  2. It is associated with significant lymphadenopathy
  3. It is divided into low count and high count based on the absolute monoclonal B cell count
  4. The incidence decreases with age
Board review style answer #2
C. It is divided into low count and high count based on the absolute monoclonal B cell count

Comment Here

Reference: B cell monoclonal lymphocytosis

Breast implant associated ALCL

Burkitt lymphoma
Definition / general
  • Highly aggressive non-Hodgkin B cell lymphoma
  • Association with Epstein-Barr virus (EBV), especially endemic subtype
  • Characteristic t(8;14) between c-MYC (8q24) and IGH (14q32) genes
Essential features
  • Highly aggressive yet potentially curable, non-Hodgkin B cell lymphoma
  • 3 subtypes
    • Endemic subtype (Africa): affects children
    • Sporadic subtype (Western countries): affects children and young adults
      • Most common pediatric lymphoma in Western countries
    • Immunodeficiency related subtype: affects HIV patients essentially
  • Characteristic t(8;14) between c-MYC (8q24) and IGH (14q32) genes
  • Association with EBV (~100% in endemic subtype, ~25 - 40% in other subtypes)
  • Extranodal presentation often predominates
    • Jaw / orbital mass in endemic subtype
    • Abdominal mass in sporadic subtype
  • Central nervous system and bone marrow involvement confer a poor prognosis
Terminology
  • Burkitt tumor
  • Malignant lymphoma, undifferentiated, Burkitt type
  • Malignant lymphoma, small noncleaved, Burkitt type
ICD coding
  • ICD-O: 9687/3 - Burkitt lymphoma
  • ICD-10: C83.7 - Burkitt lymphoma
Epidemiology
  • 3 subtypes (endemic, sporadic, immunodeficiency related)
    • All with a male predominance
  • Endemic type (Lancet 2012;379:1234)
    • Young children
    • Distribution overlaps with that of holoendemic malaria (Burkitt lymphoma belt)
      • Equatorial Africa, Papua New Guinea
      • 50% of pediatric malignancies and 90% of pediatric lymphomas
    • Associated with EBV in ~100% of cases
  • Sporadic type (Curr Oncol 2020;27:83)
    • Bimodal distribution (young children and young adults)
    • Found in low risk regions for malaria
      • Eastern Europe, North America, East Asia
      • 30 - 50% of pediatric lymphomas
      • 1 - 2% of adult lymphomas
    • Associated with EBV infection in 25 - 40% of cases
  • Immunodeficiency related type (Lancet Haematol 2020;7:e594)
    • Essentially HIV infected adults (40% of HIV associated lymphomas)
    • Occurs early in the progression of the disease (high CD4 count)
    • Associated with EBV infection in 25 - 40% of cases
Sites
  • Extranodal involvement predominates
  • All 3 types at risk for central nervous system involvement (poor prognosis)
  • Endemic type
    • Jaw, orbits, retroperitoneal area
    • Bone marrow involvement is uncommon
  • Sporadic type
    • Gastrointestinal tract, especially the ileocecal junction
    • Head and neck (lymph nodes, pharynx, tonsils, sinuses, except the jaw)
    • Bone marrow involvement is common (20%)
  • Immunodeficiency associated type
    • Lymph nodes, gastrointestinal tract, bone marrow
  • References: Lancet 2012;379:1234, Lancet Haematol 2020;7:e594
Pathophysiology
  • c-MYC is a proto-oncogene on 8q24 involved the regulation of cell cycle progression (Curr Opin Hematol 2014;21:326)
  • Rearrangement of c-MYC gene with the promoter sequence of either
    • Ig heavy chain gene (IGH, 14q32)
    • Ig light chain kappa (IGK, 2p12) or lambda (IGL, 22q11) genes
  • Constitutive activation of c-MYC gene leading to unregulated germinal center derived B cell proliferation
  • c-MYC dysregulation alone not likely sufficient to cause oncogenesis
  • Other genetic factors have been identified
    • Recurrent mutations, such as
      • Cell cycle: ID3, TP53
      • Nucleosome remodeling: SMARCS4
      • Focal adhesion: GNA13
    • Copy number variation
      • Gain of 1q, 9q, 12q, 13q, 20q, 22q, Xq
      • Loss of 4q, 13q, 17p
  • Cofactors (Lancet 2012;379:1234)
    • EBV
      • Blocks apoptosis in B cells with c-MYC rearrangement
      • Promotes genetic instability
    • Plasmodium species
      • Unclear role
      • Promotes EBV reactivation
      • Decreases immune control of latently EBV infected B cells
      • Induces c-MYC rearrangement
    • HIV
      • Pathophysiology is unclear
      • No direct role of immunodeficiency (HIV patients have high CD4 count)
      • Possible dysregulation of activated B cells
      • HIV proteins might be directly involved in lymphomagenesis:
Etiology
  • Primary event is likely c-MYC rearrangement arising in germinal center derived B cells
  • However, c-MYC rearrangement is not sufficient and additional genetic events are required for lymphomagenesis (see Pathophysiology) (Nat Genet 2012;44:1321)
Clinical features
  • Mass growing very quickly
    • Fastest growing human tumor (doubling time = 24 - 48 hours)
    • Patients have symptoms for only a few weeks prior to diagnosis
  • Most common presenting signs and symptoms (Lancet 2012;379:1234):
    • Endemic type
      • Jaw mass, periorbital swelling (50 - 70%)
    • Sporadic type
      • Abdominal mass or pain, intussusception, gastrointestinal bleeding (60 - 80%)
    • Immunodeficiency associated type
      • Abdominal symptoms, cytopenia
Diagnosis
  • c-MYC translocation detected by karyotyping or FISH study usually confirms the diagnosis of Burkitt lymphoma; however, c-MYC translocation is not specific to Burkitt lymphoma and can be encountered in other lymphomas, including high grade B cell lymphoma (double / triple hit), diffuse large B cell lymphoma, plasmablastic lymphoma and others
  • Additionally, 5 - 10% of otherwise typical Burkitt lymphoma cases show no evidence of c-MYC translocation by cytogenetic study and fluorescence in situ hybridization (FISH), which may be explained by either test failure due to detection limitations or mechanisms other than translocation being responsible for the elevated MYC protein and Burkitt lymphoma pathogenesis
  • Reference: BMC Cancer 2015;15:668
Laboratory
  • Elevated serum lactate dehydrogenase (LDH) (poor prognosis)
Prognostic factors
  • Highly aggressive lymphoma but potentially curable
  • Event free survival in high income countries
  • Lower survival in low income countries (e.g. ~ 45% at 4 years in Uganda) (Pediatr Blood Cancer 2019;66:e27813)
  • Extent of disease is the main prognostic factor
  • Other factors of poor prognosis
    • Early relapse (within 6 months)
    • High serum LDH level
Case reports
Treatment
  • Paucity of randomized clinical trials (Int J Hematol 2019;110:265)
  • Several chemotherapy regimens have been described
  • Commonly recommended regimens:
    • R-CODOX-M / R-IVAC (rituximab, cyclophosphamide, vincristine, doxorubicin, methotrexate, ifosfamide, etoposide, cytarabine)
    • R-hyper-CVAD / R-MA (rituximab, cyclophosphamide, vincristine, doxorubicin, dexamethasone, methotrexate, cytarabine)
    • For poor candidates for high intensity chemotherapy, DA-R-EPOCH (rituximab, etoposide, prednisone, vincristine, cyclophosphamide, doxorubicin, methotrexate)
  • All the above regimens include central nervous system intrathecal prophylaxis (methotrexate with or without cytarabine)
  • Superiority of 1 over the others is unknown
  • If poor response or relapse, hematopoietic stem cell transplant might be considered
Clinical images

Images hosted on other servers
Skin involvement

Skin involvement

Classic jaw involvement

Classic jaw involvement

Gross description
  • Replacement of involved organs by fish flesh masses with hemorrhages and necrosis
Gross images

Contributed by Dr. Kaveh Naemi
Ileocecal valve tumor

Ileocecal valve mass

Microscopic (histologic) description
  • Sheets of monotonous intermediate size cells with starry sky appearance (Lancet 2012;379:1234)
  • Neoplastic cells
    • Molded against each other (squared off borders)
    • Gap between neoplastic cells secondary to retracted cytoplasm
    • Round nuclei with finely clumped chromatin and several paracentral nucleoli
    • High proliferation rate (numerous mitotic figures and apoptotic bodies)
    • Plasmacytoid differentiation and mild pleomorphism possible in immunodeficiency related Burkitt lymphoma
  • Tingible body macrophages phagocyting apoptotic debris (tingible bodies)
    • Are the "stars" scattered in the sheet of neoplastic cells (the "dark sky")
  • No histologic grading
Microscopic (histologic) images

Contributed by Saja Asakrah, M.D.
Axillary lymph node Axillary lymph node

Axillary lymph node

Axillary lymph node

CD20

Axillary lymph node

PAX5

Axillary lymph node

BCL6


Axillary lymph node

CD10

Axillary lymph node

Ki67 positivity

Axillary lymph node

BCL2

Axillary lymph node

EBER in situ hybridization

Virtual slides

Images hosted on other servers:
Lymph node in a 22 year old man

Lymph node in a 22 year old man

Oral cavity in a 12 year old boy

Oral cavity in a 12 year old boy

Cytology description
  • Hypercellular smear (Arch Pathol Lab Med 2004;128:1459)
  • Monotonous neoplastic cells with
    • Round or oval nuclei
    • Several small nucleoli
    • Thin rim of basophilic cytoplasm, often containing lipid vacuoles
  • Dirty background (numerous apoptotic bodies)
  • Scattered tingible body macrophages
Cytology images

Contributed by Saja Asakrah, M.D. and @85984 on Twitter
Axillary lymph node

Axillary lymph node

Burkitt lymphoma

Burkitt lymphoma

Peripheral smear description
  • Neoplastic cells with round or oval nuclei and a thin rim of basophilic cytoplasm, often containing lipid vacuoles
Peripheral smear images

Contributed by Saja Asakrah, M.D.
Blood smear Blood smear

Blood smear, Wright-Giemsa

Positive stains
Negative stains
Flow cytometry description
Flow cytometry images

Contributed by Saja Asakrah, M.D.
B tube

B tube

Molecular / cytogenetics description
  • Cytogenetics (Curr Opin Hematol 2014;21:326)
    • c-MYC reciprocal translocation is characteristic but nonspecific
    • Partner genes (DNA Repair (Amst) 2006;5:1213):
      • t(8;14)(q24;q32) c-MYC-IGH (~ 80%)
      • t(8;22)(q24;q11) c-MYC-IGL (~ 15%)
      • t(2;8)(p11;q24) c-MYC-IGK (~ 5%)
    • Additional chromosomal abnormalities possible (see Pathophysiology)
      • Karyotype overall simpler than that of other aggressive B cell lymphomas
  • In situ hybridization
    • EBER+ in EBV positive Burkitt lymphoma
    • FISH for c-MYC translocation
  • PCR
    • Positive for monoclonal immunoglobulin gene rearrangement
Molecular / cytogenetics images

Contributed by Saja Asakrah, M.D.
Tricolor / dual fusion FISH study

Tricolor / dual fusion FISH study

Break apart FISH study

Break apart FISH study

Karyotype

Karyotype

Sample pathology report
  • Colon, cecal mass, biopsy:
    • Burkitt lymphoma (see comment)
    • Comment: Sections show colonic mucosa with diffuse atypical lymphoid infiltrate composes of medium sized monotonous cells with frequent mitosis, apoptotic bodies and scattered tingible body macrophages, creating a starry sky appearance at low microscopic power.
    • Corresponding flow cytometry analysis identified a monotypic CD10 positive B cell population with lambda light chain restriction.
    • By immunohistochemistry, the atypical lymphocytes express B cell markers including CD20 and PAX5, co-express germinal center markers CD10 and BCL6 and are negative for BCL2, cyclin D1, SOX11, CD5, TdT and CD34. Ki67 proliferation fraction is markedly high and approaching 100%. Epstein-Barr virus (EBV) encoded RNA (EBER) in situ hybridization is positive within the vast majority of the atypical lymphocytes.
    • Fluorescence in situ hybridization, identified MYC rearrangement in 95% of cells. BCL2-IGH and BCL6 rearrangements are not detected. Karyotype analysis detected t(8;14) in 20 of the analyzed metaphase cells.
    • The constellation of the above findings are consistent with the diagnosis of Burkitt lymphoma.
Differential diagnosis
  • High grade B cell lymphoma with MYC and BCL2 or BCL6 rearrangement (i.e. double / triple hit):
    • Older patient
    • Morphologic features overlaps between Burkitt lymphoma and diffuse large B cell lymphoma
    • MYC and BCL2 rearrangement in double hit lymphoma; MYC, BCL2 and BCL6 rearrangement in triple hit lymphoma
    • Complex karyotype
  • High grade B cell lymphoma, NOS:
    • Morphologic features overlap between Burkitt lymphoma, diffuse large B cell lymphoma and lymphoblastic lymphoma (blastoid morphology)
    • Immunophenotype is not typical for Burkitt lymphoma
    • c-MYC rearrangement can be positive
    • Usually associated with a more complex karyotype
  • Mantle cell lymphoma, blastoid variant:
  • Diffuse large B cell lymphoma, NOS:
    • Older patient
    • Starry sky appearance uncommon
    • Larger neoplastic cells with vesicular nuclear chromatin
    • Ki67 might be high but usually < 90%
    • c-MYC rearrangement (when present) associated with complex karyotype
  • B lymphoblastic lymphoma:
    • Starry sky appearance uncommon
    • Blastic appearance of the neoplastic cells (fine nuclear chromatin and inconspicuous nucleoli)
    • Positive for blastic markers (CD34 and TdT)
    • Negative for surface Ig and BCL6 (in most cases)
    • c-MYC rearrangement is uncommon
  • Burkitt-like lymphoma with 11q alteration:
    • Resembles Burkitt lymphoma morphologically
    • Lacks c-MYC rearrangement
    • Associated with chromosome 11q alterations, particularly gain or amplification of 11q12/q13-q23/q24 and loss of the rest of the chromosome
  • Plasmablastic lymphoma:
    • Strongly associated with HIV
    • Starry sky appearance is common
    • Plasmacytoid differentiation
    • Positive for plasma cell markers (CD38, CD138, IRF4 / MUM1) and negative for pan B markers
    • EBV is positive in the majority of cases
    • c-MYC rearrangement is positive in approximately 50% of cases
Board review style question #1

Regarding this brain specimen, which immunostain expression would be the most helpful to support the suspected diagnosis?

  1. ALK positivity
  2. BCL2 negativity
  3. MYC negativity
  4. Diffuse and strong CD30 positivity
  5. GFAP negativity
Board review style answer #1
B. BCL2 negativity

The section shows a sheet of monotonous medium to large size CD20 positive cells with squared off borders and a starry sky appearance, raising concern for an aggressive B cell lymphoma. These cells are positive for CD10 and BCL6, suggesting a germinal center origin. Ki67 reactivity in almost 100% of the cells is suspicious for Burkitt lymphoma. Unlike diffuse large B cell lymphoma and high grade B cell lymphoma, BCL2 would be negative in Burkitt lymphoma.

Diffuse and strong CD30 positivity would be suggestive of anaplastic large T cell lymphoma. ALK positivity would be seen in ALK positive anaplastic large T cell lymphoma and ALK positive diffuse large B cell lymphoma. MYC is positive in Burkitt lymphoma, mirroring MYC overexpression. Finally, although GFAP would indeed be negative in Burkitt lymphoma, this immunostain would not be helpful as a glioma would not be in the differential.

Comment Here

Reference: Burkitt lymphoma
Board review style question #2

This specimen is a touch imprint obtained from an axillary lymph node of a 35 year old white man with HIV. Which of the following viruses is associated with this disease and with what frequency?

  1. Epstein-Barr virus in 30% of cases
  2. Epstein-Barr virus in > 95% of cases
  3. Human herpesvirus 8 in 50% of cases
  4. Human herpesvirus 8 in > 95% of cases
  5. Human T cell leukemia virus type 1 in > 95% of cases
Board review style answer #2
A. Epstein-Barr virus in 30% of cases

The touch imprint shows a sheet of monotonous cells with nuclei containing fine chromatin and prominent nucleoli, surrounded by a thin rim of deeply basophilic cytoplasm containing vacuoles; this raises concern for Burkitt lymphoma. A tingible body macrophage is present in the upper center.

Unlike the endemic variant of Burkitt lymphoma seen in Equatorial Africa and Papua New Guinea, where Epstein-Barr virus (EBV) is present in almost 100% of cases, the immunodeficiency associated variant is mainly seen in HIV patients and is associated with EBV in 25 - 40% of cases. Human herpesvirus 8 (HHV8) is associated with primary effusion lymphoma, multicentric Castleman disease, Kaposi sarcoma and HHV8 associated diffuse large B cell lymphoma. Human T cell leukemia virus type 1 is associated with adult T cell lymphoma.

Comment Here

Reference: Burkitt lymphoma

CHL lymphocyte depleted
Definition / general
  • Subtype of classic Hodgkin lymphoma (CHL) with numerous Reed-Sternberg cells and scant nonneoplastic background lymphocytes
Essential features
Terminology
  • Hodgkin lymphoma disease, lymphocyte depletion, NOS
  • Classic Hodgkin lymphoma, lymphocyte depletion, NOS
  • Hodgkin lymphoma, lymphocyte depletion, diffuse fibrosis
  • Hodgkin lymphoma, lymphocyte depletion, reticular
ICD coding
  • ICD-O:
    • 9653/3 - Hodgkin lymphoma, lymphocyte depletion, NOS
    • 9654/3 - Hodgkin lymphoma, lymphocyte depletion, diffuse fibrosis
    • 9655/3 - Hodgkin lymphoma, lymphocyte depletion, reticular
  • ICD-10: C81.30 - lymphocyte depleted Hodgkin lymphoma, unspecified site
  • ICD-11: 2B30.13 - lymphocyte depleted classical Hodgkin lymphoma
Epidemiology
Sites
Pathophysiology
Clinical features
Diagnosis
  • Excisional tissue biopsy or resection
  • Fine needle aspiration or core biopsy may be difficult to distinguish between CHL subtypes given the significance of tissue architecture and background inflammatory infiltrate for diagnosis
Laboratory
Radiology description
Radiology images

Images hosted on other servers:
Mesenteric and retroperitoneal lymphadenopathy

Mesenteric and retroperitoneal lymphadenopathy

Prognostic factors
  • Unfavorable prognostic factors (J Clin Oncol 2011;29:3914):
    • Advanced disease stage
    • Advanced age
    • Presence of B symptoms
    • ≥ 3 lymph node areas involved
    • Large mediastinal mass
    • High ESR (erythrocyte sedimentation rate)
    • HIV positive
Case reports
Treatment
Microscopic (histologic) description
  • Predominance of Reed-Sternberg cells and scarce background lymphocytes
    • Reed-Sternberg cells have multilobated nuclei with prominent eosinophilic inclusion-like nucleoli
  • May have coagulative necrosis or sinusoidal invasion
  • 2 patterns:
    • Diffuse fibrosis:
      • Prominent fibroblastic proliferation (nonbirefringent fibrillary stroma) without well formed fibrous bands
      • Numerous histiocytes
      • Scattered Reed-Sternberg cells
      • Scant lymphocytes
      • Lack of plasma cells or eosinophils
    • Reticular:
      • Rich in Reed-Sternberg cells (often sheets) with anaplastic, pleomorphic or sarcomatous features
      • Scant background small lymphocytes
      • Capsular and perinodal infiltration is common
  • References: Leuk Lymphoma 2009;50:937, Pathol Res Pract 2013;209:201
Microscopic (histologic) images

AFIP images
Diffuse fibrosis

Diffuse fibrosis

Numerous Hodgkin cells

Numerous Hodgkin cells

Cytology description
  • Predominantly large, atypical cells with irregular / convoluted nuclear contour, binucleation / multinucleation and frequent mitotic figures
  • Rare small lymphocytes
Positive stains
Negative stains
Flow cytometry description
  • Usually noncontributory as large Hodgkin and Reed-Sternberg cells are delicate and frequently destroyed during flow cytometric processing
Electron microscopy description
Electron microscopy images
Molecular / cytogenetics description
  • High rate of IGH gene rearrangements, likely related to the high density of Reed-Sternberg cells relative to nonneoplastic lymphoid cells in the specimen (Leuk Lymphoma 2009;50:937)
Sample pathology report
  • Retroperitoneal lymph node, excisional biopsy:
    • Classic Hodgkin lymphoma, lymphocyte depleted type (see comment)
    • Comment: H&E sections show an effaced lymph node by sheets of large, bizarre cells with irregular, multilobated nuclei, vesicular chromatin, prominent nucleoli and abundant cytoplasm. Rare background small lymphocytes are noted.
    • Immunohistochemical stains show that the atypical cells are positive for CD30, CD15 (subset), PAX5 (weak) and EBER (in situ hybridization for EBV encoded RNA). The atypical cells are negative for CD20, CD45, OCT2, BOB1 and ALK1. CD3 highlights rare small T cells.
    • Concurrent flow cytometric analysis was negative for monoclonal B cells or aberrant T cells.
Differential diagnosis
Board review style question #1
A middle aged HIV+ man presents with abdominal pain and a large retroperitoneal mass is detected on imaging. Core biopsy was performed, showing sheets of bizarre, atypical cells with large nucleoli and frequent binucleation. Scattered background small lymphocytes are present. The atypical cells are positive for CD30, EBER and PAX5 (weak) but negative for CD20. Molecular studies demonstrate immunoglobulin gene rearrangement. What is the most likely diagnosis?

  1. Anaplastic large cell lymphoma
  2. Classic Hodgkin lymphoma, lymphocyte depleted
  3. Classic Hodgkin lymphoma, lymphocyte rich
  4. Diffuse large B cell lymphoma, NOS
  5. EBV positive diffuse large cell lymphoma
Board review style answer #1
B. Classic Hodgkin lymphoma, lymphocyte depleted

Comment Here

Reference: CHL lymphocyte depleted
Board review style question #2

A 35 year old man has extensive subdiaphragmatic lymphadenopathy. Core biopsy of one of the lymph nodes shows an effaced architecture with diffuse, disordered fibrosis and abundant histocytes. Scattered large, binucleated, CD30 positive and EBER positive atypical cells are noted. Plasma cells, eosinophils and small lymphocytes are essentially absent. Which of the following is true about this disease entity?

  1. Atypical cells are strongly positive for PU.1, BOB1, CD79a and CD20
  2. Best detected by flow cytometry
  3. More common in developed nations
  4. Most likely occurs in female patients
  5. Often associated with HIV infection
Board review style answer #2
E. Often associated with HIV infection

Comment Here

Reference: CHL lymphocyte depleted

CHL lymphocyte rich
Definition / general
  • Subtype of classic Hodgkin lymphoma (CHL) with Hodgkin / Reed-Sternberg (HRS) cells in a nodular or less commonly diffuse background of small lymphocytes (Blood 2022;140:1229, Leukemia 2022;36:1720)
Essential features
  • B cell lymphoma derived from germinal center B cells
  • Scattered HRS cells in a nodular or diffuse background of small lymphocytes without eosinophils and neutrophils
  • Immunophenotype of HRS cells: PAX5+ (dim), CD30+, CD15 variable, CD20 variable
ICD coding
  • ICD-O: 9651/3 - classic Hodgkin lymphoma, lymphocytic rich
  • ICD-10: C81.4 - lymphocyte rich Hodgkin lymphoma
  • ICD-11: 2B30.11 - lymphocyte rich, classical Hodgkin lymphoma
Epidemiology
  • Rare subtype of CHL that accounts for 5.8% of classic Hodgkin lymphoma (Cancer Med 2018;7:953)
  • Most patients are adults (30 - 50 years)
  • Male predominance (70%)
  • EBV infection in 30 - 50% of cases; the virus is within the HRS cells and usually shows a type II latency pattern of infection
Sites
  • Peripheral lymph nodes (most often cervical) are typically affected (Leuk Lymphoma 2019;60:3426)
  • Mediastinal involvement (15%) and bulky disease are rare
Pathophysiology
  • Pathophysiology is incompletely understood
  • HRS cells derived from preapoptotic germinal center B cells with a disrupted B cell program (Leukemia 2021;35:968)
  • HRS cells harbor monoclonal IGH rearrangements with somatic mutations of their Ig variable region genes and lack the capacity of Ig expression (Leukemia 2021;35:968)
  • Genetic lesions that promote HRS cell proliferation, survival, immune evasion and interaction with their microenvironment, including alterations in NFκB and JAK / STAT signaling pathways (Leukemia 2021;35:968)
  • EBV could play a crucial role by infecting HRS cells with crippled germinal center B cell receptor (BCR), rescuing them from apoptosis and thus inducing lymphomagenesis (Blood 2005;106:4339)
  • EBV infection activates the NFκB pathway promoting survival and growth (Histopathology 2021;79:451)
Etiology
  • Remains unknown
Clinical features
  • Similar to those of nodular lymphocyte predominant Hodgkin lymphoma (WHO 5) / nodular lymphocyte predominant B cell lymphoma (ICC), except that relapses are less frequent in lymphocyte rich CHL (LRCHL) (J Clin Oncol 1999;17:776)
  • B symptoms are rare
  • Most patients are diagnosed at stage I / II (J Clin Oncol 2005;23:5739)
Diagnosis
  • Histopathological analysis of an excisional lymph node biopsy or tissue biopsy supplemented with ancillary techniques (immunohistochemistry, EBER ISH)
Laboratory
Prognostic factors
  • Approximately 90 - 95% of patients with early stage CHL and 80 - 85% with advanced stage CHL are cured (Am J Hematol 2020;95:978)
  • Prognosis is slightly better than other CHL subtypes and similar to nodular lymphocyte predominant Hodgkin lymphoma (NLPHL), unless for the relapses, that are more common in NLPHL
  • Treatment is individualized according to defined risk groups based on following prognostic factors
Case reports
Treatment
Gross description
Gross images

Contributed by Laurence de Leval, M.D., Ph.D. and Carmen Bárcena, M.D.
Macroscopy

Macroscopy

Microscopic (histologic) description
  • 2 patterns of growth: nodular or less often, diffuse proliferation with numerous reactive small lymphocytes and scattered HRS cells (Leukemia 2022;36:1720)
  • Neutrophils and eosinophils are absent from the nodules
  • In lymphocyte rich classic Hodgkin lymphoma (LRCHL) nodular cases
    • Nodules are composed of small lymphocytes and may contain eccentrically located germinal centers (small or regressed)
    • Nodules contain dense meshwork of follicular dendritic cells (CD21+)
    • HRS cells are within the nodules but outside of the germinal centers
    • Some HRS cells resemble lymphocyte predominant cells (LP cells) or lacunar cells
  • In LRCHL diffuse cases
    • Small lymphocytes of the background can be intermingled with histiocytes with or without epithelioid morphology
Microscopic (histologic) images

Contributed by Laurence de Leval, M.D., Ph.D. and Carmen Bárcena, M.D.
LRCHL, nodular pattern

Nodular pattern

CD20

CD20

HRS cells

HRS cells

CD20

CD20

PAX5

PAX5

OCT2

OCT2


CD30

CD30

CD15

CD15

MUM1

MUM1

CD3

CD3

PD-1

PD-1

EBER in situ hybridization

EBER in situ hybridization

Positive stains
Negative stains
Molecular / cytogenetics description
Sample pathology report
  • Lymph node, left neck, excision:
    • Lymphocyte rich classic Hodgkin lymphoma (see comment)
    • Comment: H&E stained sections display a lymph node with architectural effacement by a nodular proliferation of abundant small lymphocytes and scattered large atypical lymphoid cells. The neoplastic cells have monolobated / bilobated / multilobated nuclei, vesicular chromatin, prominent nucleoli and amphophilic cytoplasm, consistent with HRS cells.
    • Immunophenotypically, the large atypical cells are positive for PAX5 (weak), OCT2, CD30 (strong), CD15, MUM1 and GATA3 and negative for CD20 and CD45.
    • TFH markers reveal T cell rosettes around HRS cells.
Differential diagnosis
Board review style question #1

What is the characteristic immunophenotype of the large atypical cells shown in the image above?

  1. CD20+, CD30-, CD15-, BCL6+, IgD-
  2. CD20+, PAX5+, CD23+, CD30+
  3. CD45+, OCT2+, CD20+, CD30 -, CD15-, IgD+
  4. CD45-, PAX5+, CD30+, CD15+
  5. PAX5-, CD4+, CD30+, ALK-
Board review style answer #1
D. CD45-, PAX5+, CD30+, CD15+. This is the characteristic immunophenotype of Hodgkin / Reed-Sternberg cells. Answer A is incorrect because the large atypical cells of T cell / histiocytic rich large B cell lymphoma express this immunophenotype. Answer B is incorrect because this immunophenotype is characteristic of primary mediastinal B cell lymphoma. Answer C is incorrect because the LP cells in nodular lymphocyte Hodgkin lymphoma (WHO 5) / nodular lymphocyte predominant B cell lymphoma typically express this immunophenotype (ICC 2022). Answer E is incorrect because this immunophenotype is not specific. T cells could show this immunophenotype.

Comment Here

Reference: CHL lymphocyte rich
Board review style question #2
What is the typical composition of the background of lymphocyte rich classic Hodgkin lymphoma (LRCHL) with nodular pattern?

  1. Small atypical B cells: CD20+, BCL2+, BCL6+, CD10+
  2. Small B cells: CD20+, CD5+, CD23+, LEF1+
  3. Small B cells IgD+ admixed with CD4+ PD-1+ T cells
  4. Small T cells, eosinophils, plasma cells and neutrophils
Board review style answer #2
C. Small B cells IgD+ admixed with CD4+ PD-1+ T cells. The background is typically composed of B cells corresponding to expanded mantle zones (IgD+) admixed with CD4+, PD-1+ T cells, rosetting Hodgkin / Reed-Sternberg (HRS) cells. Answer B is incorrect because this is the characteristic composition of small lymphocytic lymphoma. Answer D is incorrect because this background is not specific but in the present of HRS cells it is diagnostic of mixed cellularity classic Hodgkin lymphoma. Answer A is incorrect because this is the characteristic composition of follicular lymphoma.

Comment Here

Reference: CHL lymphocyte rich

CHL mixed cellularity
Definition / general
Essential features
  • B cell lymphoma derived from germinal center B cells
  • HRS cells in a reactive mixed inflammatory microenvironment consisting of small lymphocytes, eosinophils, plasma cells, neutrophils and histiocytes
  • Immunophenotype of HRS cells: PAX5+ (dim), CD30+, CD15 variable, CD20 variable
  • EBER+ in 75% of cases
ICD coding
  • ICD-O: 9652/3 - Hodgkin lymphoma, mixed cellularity, NOS
  • ICD-10: C81.2 - mixed cellularity (classical) Hodgkin lymphoma
  • ICD-11: 2B30.12 - mixed cellularity classical Hodgkin lymphoma
Epidemiology
  • Second most common subtype of classic Hodgkin lymphoma
  • Accounts for 20 - 25% of classic Hodgkin lymphoma
  • Bimodal incidence: pediatric population and elderly patients
  • M > F
  • More common in patients with immunodeficiency and in developing countries (Expert Rev Hematol 2021;14:547)
  • EBV infection in 75% of cases; virus is located within the HRS cells and usually shows a type II latency pattern of infection (N Engl J Med 1989;320:502)
Sites
  • Lymph nodes (70%) > spleen (30%) > bone marrow (10%) > liver (3%) > other organs (1 - 3%) (Leuk Lymphoma 2019;60:3426)
  • Rarely limited to a single region and is more likely to involve extralymphatic tissues compared to other classic Hodgkin lymphoma subtypes
  • Mediastinal involvement is rare
Pathophysiology
  • Pathophysiology incompletely understood
  • Hodgkin / Reed-Sternberg cells derived from preapoptotic germinal center B cells with a disrupted B cell program (Leukemia 2021;35:968)
  • HRS harbor monoclonal IgH rearrangements with somatic mutations of their immunoglobulin (Ig) variable region genes, losing the capacity of Ig expression (Leukemia 2021;35:968)
  • Genetic lesions that promote HRS cell proliferation, survival, immune evasion and interaction with their microenvironment, including alterations in NFκB and JAK-STAT signaling pathways (Leukemia 2021;35:968)
  • EBV could play a crucial role by infecting and rescuing Hodgkin / Reed-Sternberg cells with a crippled B cell receptor in the germinal centers from apoptosis and thus inducing lymphomagenesis (Blood 2005;106:4339)
  • EBV infection activates the NFκB pathway, promoting survival and growth (Histopathology 2021;79:451)
Clinical features
  • B symptoms are common
  • May be diagnosed in all clinical stages (more commonly with stage II and III disease)
Diagnosis
  • Excisional lymph node biopsy or tissue biopsy with ancillary techniques (immunohistochemistry)
Laboratory
Prognostic factors
  • About 90 - 95% of patients with early stage CHL and 80 - 85% with advanced stage CHL are cured (Am J Hematol 2020;95:978)
  • Treatment is individualized according to defined risk groups, based on the following prognostic factors:
Treatment
Microscopic (histologic) description
  • Diffuse or interfollicular proliferation of HRS cells (< 10% of the cellularity) in a reactive microenvironment composed of lymphocytes, eosinophils, neutrophils, plasma cells, histiocytes, fibroblasts (Leukemia 2022;36:1720)
  • CD4+ T cells are the most abundant immune cells in the CHL microenvironment (Cytometry B Clin Cytom 2008;74:1, Blood 2004;103:1755, Lab Invest 2008;88:482)
  • In EBV+ cases, there may be numerous epithelioid histiocytes and even granulomas
  • Fine interstitial fibrosis may be seen but without collagen broad bands and with no capsular thickening
  • Bone marrow is involved in 10% of cases and shows scattered HRS cells in a background that is rich in histiocytes with ill defined granulomas merging with inflammatory cells and normal hematopoiesis
Microscopic (histologic) images

Contributed by Laurence de Leval, M.D., Ph.D. and Carmen Bárcena, M.D.
Interfollicular zones

Interfollicular zones

Interfollicular expansion

Interfollicular expansion

Reed-Sternberg / Hodgkin cells

Reed-Sternberg / Hodgkin cells

Reed-Sternberg / Hodgkin cells and microgranulomas

Reed-Sternberg / Hodgkin cells and microgranulomas

Polymorphous microenvironment

Polymorphous microenvironment

Focal necrosis

Focal necrosis


CD45

CD45

PAX5

PAX5

MUM1

MUM1

CD30

CD30


CD15

CD15

EBER in situ hybridization

EBER in situ hybridization

EBV LMP1

EBV LMP1

PDL1 SP263

PDL1 SP263

Positive stains
Molecular / cytogenetics description
Sample pathology report
  • Lymph node, right neck, excision:
    • Mixed cellularity classic Hodgkin lymphoma (see comment)
    • Comment: H&E stained sections show a lymph node with architectural effacement by an interfollicular mixed inflammatory infiltrate comprising small lymphocytes, eosinophils, neutrophils, plasma cells, with scattered admixed large atypical lymphoid cells. The neoplastic cells have monolobated / bilobated / multilobated nuclei, vesicular chromatin, prominent nucleoli and amphophilic cytoplasm, consistent with Hodgkin and Reed-Sternberg cells.
    • Immunophenotypically, the large atypical cells are positive for PAX5 (weak), CD30 (strong), CD15 and MUM1. These cells are negative for CD20 and CD45.
    • T cell markers reveal a rich background composed of small lymphocytes.
    • In situ hybridization for EBV encoded RNA (EBER) is positive on the large atypical cells.
Differential diagnosis
  • EBV+ diffuse large B cell lymphoma, NOS:
    • Large cells are usually CD45+, CD20+, CD79a+, PAX5+ (strong), CD30+, CD15- and may show monotypic Ig expression
    • EBV is located within both large and small lymphoid cells
    • Frequently extranodal
    • Geographic necrosis is common
    • Adults and elderly patients
  • EBV+ mucocutaneous ulcer:
    • Solitary ulcers in mucocutaneous sites with or without locoregional lymph node involvement
    • In immunosuppression settings or in the elderly
    • HRS-like cells are CD30+, CD15-
  • Lymphomatoid granulomatosis:
    • Angiocentric and angiodestructive lymphoproliferation, composed of EBV positive B cells, may include HRS-like cells (CD20+, CD30+, CD15-)
    • Pulmonary involvement is mandatory for the diagnosis (Blood 2020;135:1344)
  • T cell / histiocytic rich large B cell lymphoma (THRLBCL):
    • Composed of a background rich in small lymphocytes and histiocytes with scattered, large neoplastic B cells
    • CD20+, BCL6+, CD30-, CD15-, EBER-
    • Eosinophils, plasma cells and neutrophils are uncommon
    • These characteristics argue against a diagnosis of CHL (Leukemia 2022;36:1720)
  • Posttransplant lymphoproliferative disorder, polymorphic type:
    • A spectrum of maturation of lymphoid cells, including small lymphocytes, lymphoplasmocytoid cells, plasma cells, immunoblasts and large HRS-like cells
    • HRS-like cells are usually CD45+, CD20+, PAX5+ (strong), CD30+, CD15-
    • Most EBV positive cases reveal a type III EBV latency pattern
    • Classic Hodgkin lymphoma is uncommon in the posttransplant setting
  • Angioimmunoblastic T cell lymphoma (AITL) and other subtypes of follicular helper T cell lymphoma:
    • Proliferation of atypical neoplastic T follicular helper (TFH) cells
    • Proliferation of high endothelial venules
    • Expansion of the follicular dendritic cell meshwork
    • HRS-like cells are usually CD20+, CD30+, CD15- (or sometimes CD15+)
    • Rosettes of neoplastic TFH cells around HRS-like cells
    • Monoclonal T cell receptor gene rearrangement
  • Reactive conditions:
    • Infectious mononucleosis:
      • Lymph node architecture may be partially effaced and may reveal a mottled appearance
      • Hyperplasic follicles with prominent germinal centers
      • Paracortical expansion with polymorphic lymphoid cells, including immunoblasts, polytypic plasma cells and HRS-like cells (CD30+, CD15-), numerous mitoses and a high proliferative index (Ki67)
      • Monocytoid cells and sinus histiocytosis may be present
Board review style question #1

What is the characteristic immunophenotype of the large, atypical cells shown in the image above?

  1. CD2+, CD30+, CD15-, ALK+
  2. CD20+, PAX5+, CD23+, CD30+
  3. CD45+, OCT2+, CD20+, CD30-
  4. CD45-, PAX5+, CD30+, CD15+
  5. PAX5-, CD4+, CD30+, ALK-
Board review style answer #1
D. CD45-, PAX5+, CD30+, CD15+. This is the characteristic immunophenotype of Hodgkin / Reed-Sternberg cells. The immunophenotype in answer A is diagnostic of ALK positive anaplastic large cell lymphoma. The lymphocyte predominant (LP) cells in nodular lymphocyte Hodgkin lymphoma (WHO) / nodular lymphocyte predominant B cell lymphoma (ICC) typically express the immunophenotype in answer C. The immunophenotype in answer E is not specific. T cells could show this immunophenotype.

Comment Here

Reference: Mixed cellularity classic Hodgkin lymphoma
Board review style question #2
Which subtype of classic Hodgkin lymphoma is most frequently encountered in patients with HIV / AIDS?

  1. Lymphocyte depleted
  2. Lymphocyte rich
  3. Mixed cellularity
  4. Nodular sclerosis
Board review style answer #2
C. Mixed cellularity. Mixed cellularity classic Hodgkin lymphoma is the most common subtype of CHL associated with immunodeficiency states, including HIV infection / AIDS, immunodeficiency secondary to immunosenescence and immunodeficiency secondary to solid organ transplantation.

Comment Here

Reference: Mixed cellularity classic Hodgkin lymphoma

CHL nodular sclerosis
Definition / general
Epidemiology
  • 70% of classic Hodgkin lymphoma in Europe and USA
  • More common in rich countries, highest risk in those with high socioeconomic status
  • Similar incidence in males and females (the only classic Hodgkin lymphoma subtype without male predominance)
  • Peaks at 15 - 34 years of age
Sites
  • Cervical lymph nodes, mediastinum (80%), bulky disease (54%), spleen or lung (8 - 10), bone (5%), bone marrow (3%), liver (2%)
Clinical features
  • Most present with stage II disease (2/3 have stage I or II disease)
  • B symptoms: 40%
Case reports
Treatment
  • Better overall prognosis than other types of classic Hodgkin lymphoma
  • > 90% survival at 5 years for early stage disease
  • Adverse prognostic indicator: massive mediastinal disease
Microscopic (histologic) description
  • Nodular growth pattern with broad fibroblast poor birefringent collagen bands surrounding at least one nodule
  • Usually confined within thickened lymphonodular capsule
  • Highly variable numbers of HRS cells, small lymphocytes and other inflammatory cells; often numerous eosinophils, histiocytes and neutrophils; occasional foamy macrophages
  • Mitoses uncommon
  • HRS cells have more lobated nuclei, smaller lobes, less prominent nucleoli, more cytoplasm than other types of classic Hodgkin lymphoma
  • Lacunar cells: formalin fixation artifact of delicate folded or multilobate nuclei surrounded by abundant pale cytoplasm often disrupted or retracted during cutting of sections with formalin fixation (but not B5 or Zenkers), leaving a lacune (empty hole); associated with necrosis and histiocytes (necrotizing granuloma-like)
  • Syncytial variant: prominent aggregates of lacunar cells
  • Grading by number of HRS cells (British National Lymphoma Investigation: 1 - scattered, 2 - aggregates in > 25% of nodules) and number of eosinophils (German HL Study Group: > 5%) is for research protocols but not for routine clinical purposes
Microscopic (histologic) images

Images hosted on other servers:

Lacunar cells

Bands of collagen

Reed-Sternberg cells with bilobed mirror image nuclei, prominent nucleoli and abundant amphophilic cytoplasm

Positive stains
Negative stains
Differential diagnosis

CLL / SLL
Definition / general
  • Indolent mature B cell lymphoma
  • Clonal proliferation of B cells with characteristic phenotype
Essential features
  • Indolent mature B cell lymphoma
  • Most common adult leukemia in the Western world
  • Characteristic immunophenotype with CD19, CD5, CD20, CD23, CD200 positivity
  • 2 types with distinct biological behavior based on mutation status of rearranged IGH gene
Terminology
  • Chronic lymphocytic leukemia (CLL): neoplasm of mature B cells with characteristic immunophenotype showing peripheral lymphocytosis (≥ 5 x 109/L) with or without nodal or extranodal manifestation (Blood 2016;127:2375)
  • Monoclonal B cell lymphocytosis: clonal B cells with or without CLL-like immunophenotype in peripheral blood < 5 x 109/L and without nodal manifestation (see B cell monoclonal lymphocytosis)
  • Small lymphocytic lymphoma (SLL): < 5 x 109/L CLL-like cells in peripheral blood with nodal or extranodal manifestation, usually with bone marrow involvement
ICD coding
  • ICD-O: 9823/3 - Chronic lymphocytic leukemia / small lymphocytic lymphoma
  • ICD-10: C91.1 - Chronic lymphocytic leukemia
  • Related codes:
    • ICD-O: 9823/1 - Monoclonal B cell lymphocytosis, CLL type (or atypical CLL type)
    • ICD-O: 9591/1 - Monoclonal B cell lymphocytosis, non-CLL type
    • ICD-10: D72.8 - Lymphocytosis - for monoclonal B cell lymphocytosis
Epidemiology
  • Most common adult leukemia in the Western world (Future Oncol 2017;13:1873)
  • Median age is between sixth and seventh decade, practically nonexistent in children
  • Male preponderance (M:F = 1.5 - 2:1)
  • Incidence is low in Asian countries (Int J Oncol 2013;43:561)
Sites
  • Lymph nodes, bone marrow, spleen and peripheral blood
  • Usually widespread disease, clinical staging system is used
Pathophysiology
  • Clonal restriction of specific B cell populations with reduced competition of remaining B cell clones due to aging
  • B cell receptors show evidence of selection by antigens, B cell receptor signaling is crucial in survival of CLL cells (Front Oncol 2020;10:592205)
  • Evidence for autoreactivity of B cell receptors (antigens recognized by studies are related to normal tissue maintenance, apoptosis, atherosclerosis, common infections, etc.) (Front Oncol 2020;10:592205)
  • Some evidence indicating self stimulation by B cell receptors (Cell Res 2013;23:182)
  • 30% of cases show stereotyped B cell receptors with identical or nearly identical structure (Leukemia 2019;33:287)
  • 2 types of CLL that do not show conversion during disease course:
    • CLL-UM (unmutated): few mutations in the IGH gene (≥ 98% homology with germline sequence), associated with more proliferation, more aggressive disease course
    • CLL-MUT (mutated): many mutations in the IGH gene (< 98% homology), associated with less proliferation, better prognosis
  • Gradual accumulation of genetic alterations, however, no clear malignant transformation event recognizable
Etiology
  • Incidence is not increased following radioactive incidents
  • Familial predisposition in 5 - 10% cases
Clinical features
  • Frequently no symptoms
  • Lymphadenopathy, splenomegaly are common
  • Anemia, thrombocytopenia and neutropenia related symptoms frequently occur
  • Autoimmune hemolytic anemia is common (Best Pract Res Clin Haematol 2010;23:47)
  • Extramedullary involvement may occur; liver, skin, GI mucosa, kidneys are most commonly involved
Diagnosis
  • Laboratory diagnosis of lymphocytosis
  • Detection of generalized adenopathy or splenomegaly
  • CT scans are helpful to detect lymph node enlargement
  • Immunophenotyping is essential: flow cytometry of peripheral blood or bone marrow or immunohistochemistry of biopsy material
  • Lymph node biopsy is not generally required, unless to establish diagnosis of Richter transformation (Blood 2018;131:2745)
Laboratory
  • Lymphocytosis
  • Anemia, thrombocytopenia are common
  • Monoclonal gammopathy may occur
  • Hypogammaglobulinemia commonly occurs (Blood 2015;126:573)
Radiology images

Images hosted on other servers:
CT scan

CT scan

Prognostic factors
  • Clinical stage (based on physical manifestations and blood parameters): Rai and Binet systems
  • Beta-2 microglobulin: high levels associated with high tumor burden and adverse prognosis
  • TP53 mutation / 17p deletion is adverse, is associated with poor response to traditional chemotherapeutic regimens; patients may respond to newer agents (J Oncol Pract 2017;13:371)
  • Unmutated IGH gene is adverse
  • Complex karyotype (10 - 15%, 3 or more unrelated abnormalities) is adverse (Blood 2019;133:1205)
  • 13q deletion is favorable, 11q deletion is unfavorable, trisomy 12 is associated with intermediate prognosis
  • CD38 (> 30% positive in CLL cells by flow cytometry) is adverse, ZAP70 expression (> 20%) is adverse
  • CD49d (> 30% positive in CLL cells) has been associated with progression (Blood 2020;135:1244)
  • Large, confluent proliferation centers (adverse) (Blood 2016;127:2375)
  • Methylation profile, various gene mutations may correlate with epigenetic maturation status, response to therapy or prognosis but are not currently used in clinical practice (Nat Genet 2016;48:253)
  • Pattern of bone marrow infiltration is no longer considered important
Case reports
Treatment
  • Treatment is tailored based on clinical progression (progressive bone marrow failure, progressive or bulky lymphadenopathy, short lymphocyte doubling time, etc.), biological fitness of patient, presence of TP53 mutation / 17p deletion and IGH mutation status (Curr Oncol Rep 2020;22:36)
  • Cytostatic agents: chlorambucil, fludarabine, bendamustine, cyclophosphamide, etc.
  • Monoclonal antibodies: rituximab, obinutuzumab (anti-CD20), alemtuzumab (anti-CD52)
  • Kinase inhibitors: ibrutinib, acalabrutinib (Btk inhibitors), idelalisib (PI3K inhibitor)
  • BCL2 inhibitors: venetoclax
  • Lenalidomide
Gross description
  • Lymph node: enlarged lymph node with homogeneous, fleshy cut surface
  • Spleen: often miliary pattern of white pulp expansion, homogeneous infiltration with massive splenomegaly also occurs (Am J Clin Pathol 2003;120:335)
Gross images

Contributed by Béla Kajtár, M.D., Ph.D.
Paraaortic lymph nodes

Paraaortic lymph nodes



Images hosted on other servers:
Splenic involvement

Splenic involvement

Frozen section description
  • Diffuse infiltrate of small, mature lymphocytes with inconspicuous nuclei and scant cytoplasm
  • Immunophenotyping is necessary for diagnosis
Microscopic (histologic) description
  • Diffuse effacement of parenchyma by small, mature lymphocytes
  • Round nucleus, clumped chromatin with only small nucleoli and scant cytoplasm
  • Ill defined, pale proliferation centers (pseudofollicles) are common, composed of prolymphocytes and paraimmunoblasts
  • Bone marrow: nodular or diffuse infiltration; paratrabecular aggregates are not common

Notes:
  • Occasionally extensive plasmacytoid differentiation
  • Partial lymph node infiltration is possible with perifollicular or interfollicular infiltration (Haematologica 2011;96:1144)
  • Prolymphocyte: small to medium sized mature lymphocyte with clumped chromatin, somewhat larger central nucleolus
  • Paraimmunoblast: large cell with round nucleus, dispersed chromatin and central, enlarged nucleolus, often with basophilic cytoplasm
  • No grading system is used, prolymphocytic transformation is defined based on peripheral blood
  • Histologically aggressive CLL: confluent or very large proliferation center or > 40% Ki67 index; not the equivalent of Richter transformation (Blood 2018;131:2761)
  • Richter transformation: usually high grade B cell lymphoma (DLBCL) appearing in the context of CLL; confluent areas of large cells are required for diagnosis
  • Hodgkin transformation: rare form of Richter transformation, usually clonally unrelated and EBV positive, requires classical Hodgkin lymphoma pattern; scattered Reed-Sternberg cells may be present in CLL (Blood 2018;131:2761)
  • Prolymphocytic transformation: ratio of prolymphocytic cells in peripheral blood increases (< 55%), transformation into B cell prolymphocytic leukemia does not occur, by definition
Microscopic (histologic) images

Contributed by Béla Kajtár, M.D., Ph.D.
Proliferation centers

Proliferation centers

Diffuse lymphocytic infiltrate

Diffuse lymphocytic infiltrate

Proliferation center

Proliferation center

CD20

CD20

LEF1

LEF1

Bone marrow involvement

Bone marrow involvement

Virtual slides

Images hosted on other servers:
SLL / CLL, H&E SLL / CLL, H&E

SLL / CLL, H&E

Cytology description
  • Small, mature lymphocytes with round nucleus, clumped chromatin and only small nucleoli
Cytology images

Contributed by Béla Kajtár, M.D., Ph.D.
Touch prep SLL

Touch prep SLL

Peripheral smear description
  • Lymphocytosis composed of small, mature lymphocytes with round nucleus, clumped chromatin and only small nucleoli
  • Smudged cells (Gumprecht cells) commonly seen representing mechanically damaged cells (J Clin Oncol 2009;27:1844)
  • Prolymphocytes usually < 15%; 15 - 55% in case of atypical CLL; > 55% defines B cell prolymphocytic leukemia (see Prolymphocytic leukemia)
Peripheral smear images

Contributed by Béla Kajtár, M.D., Ph.D.
CLL in PB CLL in PB

CLL in peripheral blood

Positive stains
Negative stains
Flow cytometry description
Flow cytometry images

Contributed by Béla Kajtár, M.D., Ph.D.
CLL, scatter plot

CLL, scatter plot

Molecular / cytogenetics description
  • Unmutated IGH (sequencing) (40 - 60% of cases), testing is necessary before treatment
  • TP53 mutations (sequencing) 10% at diagnosis, 30% at relapse, testing is necessary before treatment (Hematology Am Soc Hematol Educ Program 2016;2016:149)
  • 17p deletion (5 - 30%) may be independent from TP53 mutation, testing is necessary before treatment
  • 13q deletion (50 - 55%), 11q23 deletion (5 - 20%), 12 trisomy (10 - 20%), 6q deletion by FISH is recommended (Biomed Res Int 2014;2014:435983, Blood 2018;131:2745)
  • Karyotyping to show complex karyotype is recommended in clinical studies, not generally indicated in routine clinical practice (Blood 2018;131:2745)
  • IGH translocations rarely occur, IGH-BCL2+ CLL does exist (2 - 5%) (Am J Hematol 2019;94:338)
Molecular / cytogenetics images

Contributed by Béla Kajtár, M.D., Ph.D.
FISH images

FISH images

Karyogram

Karyogram

Sample pathology report
  • Bone marrow, left posterior iliac crest, core biopsy, clot section, aspirate smears and peripheral blood smears:
    • Hypercellular bone marrow with maturing hematopoietic activity and 85% diffuse lymphocytic infiltrate with immunophenotype consistent with chronic lymphocytic leukemia (CLL) (see comment)
    • Comment:
      • Flow cytometric analysis of bone marrow demonstrates 62% lymphocytes with CD19+, CD5+, CD10-, CD43+, CD23+, CD200+, CD38-, CD20(dim)+, sIgL-kappa(dim)+ immunophenotype consistent with CLL. Peripheral blood smears indicate > 5 x 109/L CLL cells.
      • FISH showed biallelic 13q14 deletion in 58%, 11q23 monoallelic deletion in 32% of cells, no 12 trisomy or 17p deletion was noted (see details in separate cytogenetic report).
      • Additional molecular studies (TP53 and IGH sequencing) are in progress and will be reported separately.
    • Peripheral smear: Manual review of the peripheral blood shows unremarkable platelets, no morphological alteration of red blood cells and lymphocytosis with few smudged cells. 12% of lymphocytes show enlarged nuclei and nucleoli (prolymphocytes). A manual 500 cell differential count reveals 87% lymphocytes, 2% monocytes, 10% neutrophils and 1% eosinophils.
    • Bone marrow biopsy: Quality: adequate. Cellularity: 80%. Hematopoiesis: trilineage maturation is present but diminished without dysplastic features or increased blasts. Megakaryopoiesis: normal, cell number is reduced. Granulopoiesis: normal, no blast increase. Erythropoiesis: normal. Infiltrate: 85% diffuse lymphocytic infiltrate composed of mature lymphocytes, approximately 15% are prolymphocytes. Special stains: Reticulin: loose network of reticulin without significant intersections (minimal reticulin fibrosis). Trichrome: negative for collagen deposition.
    • Bone marrow clot section: Quality: adequate. Cellularity: 80% morphologic features are similar to those observed in the core biopsy.
    • Bone marrow aspirate: Quality: adequate. Granulocytes: decreased; normal maturation; no dysplastic features. Erythrocytes: decreased, normal maturation, no dysplastic features, no ringed sideroblasts. Megakaryocytes: significantly decreased; no dysplastic features. Blasts: less than 1% of nucleated cells. Infiltrate: diffuse lymphocytic infiltrate (78%) consisting of small, mature lymphocytes as well as 13% prolymphocytes. A manual 500 cell differential count reveals 23% erythroblasts, 1% myelocytes, 7% metamyelocytes, 2% neutrophils, 1% eosinophils and 66% lymphocytes (of which 13% are prolymphocytes).
Differential diagnosis
Board review style question #1

Which of the following is necessary for diagnosis of CLL?

  1. Immunophenotyping
  2. Sequencing of IGH genes
  3. Sequencing of TP53 gene
  4. Testing for BCL2 rearrangement
  5. Testing for CCND1 rearrangement
Board review style answer #1
A. Immunophenotyping

Comment Here

Reference: CLL / SLL
Board review style question #2
Which of the following constitutes Richter transformation?

  1. Appearance of complex karyotype as part of progression of CLL
  2. Appearance of TP53 mutation as part of progression of CLL
  3. Diffuse large B cell lymphoma appearing as progression of CLL
  4. Nodal CD5+ diffuse large B cell lymphoma
  5. Unmutated CLL turning into mutated CLL
Board review style answer #2
C. Diffuse large B cell lymphoma appearing as progression of CLL

Comment Here

Reference: CLL / SLL

Classic Hodgkin lymphoma
Definition / general
  • Classic Hodgkin lymphoma (CHL): B cell derived lymphoma characterized by distinctive immunophenotype and relatively few malignant cells in a nonneoplastic inflammatory background
  • 4 subgroups (nodular sclerosis, mixed cellularity, lymphocyte rich, lymphocyte depleted) with distinct clinical, morphologic and epidemiologic characteristics
Essential features
  • B cell derived lymphoma characterized by distinctive immunophenotype and relatively few malignant cells in a nonneoplastic inflammatory background
  • 4 subgroups (nodular sclerosis, mixed cellularity, lymphocyte rich, lymphocyte depleted) with distinct clinical, morphologic and epidemiologic characteristics
  • Involves lymph nodes (cervical > axillary, mediastinal, paraaortic)
  • Bimodal age distribution: peaks at 15 - 35 years and 50 - 70 years
  • Multiple mechanisms to evade immunosurveillance, including upregulation of programmed death ligand 1 / programmed death ligand 2 (PDL1 / PDL2)
Terminology
  • Classical Hodgkin lymphoma
  • Hodgkin disease
ICD coding
  • ICD-10: C.81* - classic Hodgkin lymphoma
  • ICD-10: C.81.90 - classic Hodgkin lymphoma, unspecified, unspecified site
  • ICD-11: 2B30.1Z - classical Hodgkin lymphoma, unspecified
Epidemiology
  • CHL: ~90 - 95% of Hodgkin lymphomas
  • Bimodal age distribution: peaks at 15 - 35 years and 50 - 70 years
  • Epidemiology varies by subgroup (Br J Haematol 2019;184:45):
    • Nodular sclerosis (~70% of CHL in Western countries): F > M, age 15 - 35 years, most common in urban areas, wealthier nations and individuals with high socioeconomic status, minority (10 - 25%) EBV+
    • Mixed cellularity (~20% of CHL): children and adults > 60 years, M > F, more common in developing countries and rural areas, majority (70 - 80%) EBV+
    • Lymphocyte rich (~5% of CHL): M > F, age 30 - 50 years
    • Lymphocyte depleted (<1% of CHL in Western countries): M > F, age 30 - 70 years
Sites
  • Lymph nodes (cervical > axillary, mediastinal, paraaortic)
  • Mediastinal involvement in 80% of nodular sclerosis CHL
  • Extranodal sites: spleen (20%), liver, lungs, bone marrow (5%)
  • Typical anatomic distribution varies by subgroup (Br J Haematol 2019;184:45):
    • Nodular sclerosis: mediastinal, cervical, supraclavicular lymph nodes
    • Mixed cellularity: peripheral lymph nodes, spleen, bone marrow
    • Lymphocyte rich: peripheral lymph nodes
    • Lymphocyte depleted: retroperitoneal lymph nodes, abdomen, bone marrow
Pathophysiology
  • Pathophysiology incompletely understood
  • Malignant Reed-Sternberg cells derived from preapoptotic germinal center B cells with a disrupted B cell program (Blood 2000;95:1443)
  • In a subset of cases, IgVH mutations lead to dysfunctional / absent B cell receptor, prohibiting antigenic selection; incompatible with survival of normal B cells (J Exp Med 1996;184:1495)
  • Postulated to circumvent apoptosis by differentiating toward distinct subset of CD30+ post germinal center memory B cells (J Clin Invest 2018;128:2996)
  • Cell proliferation and survival promoted by alterations in signaling pathways, including JAK / STAT, NFkB, PI3K / AKT and MAPK / ERK (Leukemia 2021;35:968, Cancers (Basel) 2021;13:682)
  • EBV infection thought to drive aberrant signaling in EBV+ cases (Histopathology 2021;79:451)
  • Reed-Sternberg cell cytokine secretion recruits nonneoplastic inflammatory infiltrate; reciprocal cytokine / chemokine secretion by immune cells supports Reed-Sternberg cell survival and proliferation
  • Reed-Sternberg cell evasion of immunosurveillance:
Etiology
  • Cause incompletely understood
  • Factors associated with increased risk of developing CHL:
Diagrams / tables

Images hosted on other servers:
Reed-Sternberg cell and microenvironment

Reed-Sternberg cell and microenvironment

Clinical features
  • Peripheral lymphadenopathy
  • 30 - 40% of patients present with B symptoms (fever, night sweats, weight loss), pruritis
  • Stage at presentation (Blood Rev 2002;16:111):
    • Nodular sclerosis, lymphocyte rich usually present with early stage disease
    • Mixed cellularity and lymphocyte depleted associated with HIV infection and EBV often present with advanced stage disease
    • Immunosuppression (including HIV infection) associated with advanced stage disease
Diagnosis
  • Excisional lymph node biopsy or tissue biopsy
  • FNA and core needle biopsy may be insufficient given importance of tissue architecture for diagnosis and potential rarity of Reed-Sternberg cells
Laboratory
  • Nonspecific
  • Elevated C reactive protein
  • Eosinophilia (20%)
Radiology description
  • Lower cervical, supraclavicular and mediastinal lymphadenopathy
  • Bulky mediastinal disease in 50% of nodular sclerosis subtype
  • PET / CT recommended for staging (J Clin Oncol 2014;32:3059)
  • PET / CT has high sensitivity for detecting bone marrow involvement
Radiology images

Images hosted on other servers:
PET scan before and after therapy

PET scan before and after therapy

Prognostic factors
  • Differences in prognosis between different subgroups largely gone with modern therapy
  • Immunosuppression, HIV infection associated with worse prognosis
  • Staging critical for accurate prognostication and treatment
    • Early stage disease includes favorable and unfavorable groups
      • Unfavorable prognosis associated with B symptoms, bulky mediastinal disease, elevated erythrocyte sedimentation rate (ESR), involvement of multiple lymph node groups, extranodal disease, older age (> 40 - 50 years)
    • Advanced stage disease: prognosis associated with International Prognostic Score, based on 7 variables that independently predict outcome (N Engl J Med 1998;339:1506)
  • 80 - 90% cure rates with stage adapted therapy
  • 10 - 25% of patients have primary refractory disease or relapse
  • Increased percentage of CD68 positive macrophages in a diagnostic lymph node biopsy associated with shorter progression free and disease specific survival and increased failure rate after secondary treatment, including autologous stem cell transplant (N Engl J Med 2010;362:875)
Case reports
Treatment
  • ABVD (doxorubicin, bleomycin, vinblastine, dacarbazine) with or without IFRT (involved field radiation therapy)
  • BEACOPP (bleomycin, etoposide, doxorubicin, cyclophosphamide, vincristine, procarbazine, prednisone) or AVD with brentuximab vedotin (anti-CD30 antibody drug conjugate) sometimes used in advanced disease
  • Options for relapsed / refractory disease include brentuximab vedotin, chemotherapy, checkpoint inhibitors, high dose therapy and autologous stem cell transplant, allogeneic stem cell transplant
  • Risks of treatment related morbidity, including therapy related malignancy (> 4x risk of cancer compared with general population), cardiovascular complications from mediastinal IFRT (Br J Haematol 2019;184:9)
  • Interim PET scan after 2 cycles of chemotherapy may be used to guide treatment (PET adapted therapy) (Br J Haematol 2019;184:9)
Gross description
  • Nodular sclerosis subtype: enlarged lymph node, thickened capsule, fibrotic bands with prominent nodularity
  • Nodular involvement of spleen
Gross images

Images hosted on other servers:

Fleshy lymph node

Liver with multifocal involvement

Frozen section description
  • Effacement of nodal architecture
  • Nodularity
  • Mixed inflammatory background (small lymphocytes, neutrophils, eosinophils, histiocytes)
  • May see scattered large Reed-Sternberg cells with multilobated nuclei and prominent nucleoli
Frozen section images

Contributed by Emily Mason, M.D., Ph.D.
Reed-Sternberg cells

Reed-Sternberg cells

Microscopic (histologic) description
  • Total or partial effacement of nodal architecture
  • Nodular sclerosis subtype: nodularity, dense collagen bands, thickened capsule
  • Diagnostic cell = Reed-Sternberg cell: large size (up to 100 microns), bilobated to multilobated nucleus, prominent eosinophilic nucleolus, ample amphophilic cytoplasm
  • Mononuclear variants = Hodgkin cells: single round to oval nucleus with prominent eosinophilic nucleolus
  • Mummified forms: degenerated Hodgkin and Reed-Sternberg cells, condensed hyperchromatic nuclei
  • Lacunar cells: characteristic of nodular sclerosis subtype, lobulated nuclei, pale retracted cytoplasm (artifact of formalin fixation), creating lacunae-like spaces
  • Mixed inflammatory background (Br J Haematol 2019;184:45):
    • Nodular sclerosis: small lymphocytes, neutrophils, eosinophils, histiocytes, plasma cells, necrosis and microabscesses common, with banding fibrosis
    • Mixed cellularity: small lymphocytes, neutrophils, eosinophils, histiocytes, plasma cells, epithelioid granulomas common
    • Lymphocyte depleted: reduced background infiltrate
    • Lymphocyte rich: nodules of small B cells, epithelioid histiocytes
  • Diagnostic Hodgkin and Reed-Sternberg cells may be rare to frequent depending on subtype
    • Syncytial variant: variant of nodular sclerosis subtype; Hodgkin and Reed-Sternberg cells present in aggregates and sheets, with typical inflammatory background and immunophenotype of nodular sclerosis subtype
Microscopic (histologic) images

Contributed by Emily Mason, M.D., Ph.D.
Hodgkin and Reed-Sternberg cells

Hodgkin and Reed-Sternberg cells

Nodular sclerosis CHL

Nodular sclerosis CHL

Scattered Reed-Sternberg cells

Scattered Reed-Sternberg cells

Lacunar cell

Lacunar cell

Syncytial variant

Syncytial variant


Weak PAX5 expression

Weak PAX5 expression

CD30 expression

CD30 expression

Variable CD20 expression

Variable CD20 expression

EBV positivity

EBV positivity

Syncytial variant, CD30 stain

Syncytial variant, CD30 stain

Virtual slides

Images hosted on other servers:
Nodular sclerosis CHL

Nodular sclerosis CHL

Cytology description
  • Mixed inflammatory background (small lymphocytes, neutrophils, eosinophils, histiocytes)
  • May see scattered large Reed-Sternberg cells with multilobated nuclei and prominent nucleoli
Cytology images

Contributed by Emily Mason, M.D., Ph.D.
Reed-Sternberg cell

Reed-Sternberg cell

Positive stains
Negative stains
Flow cytometry description
  • Hodgkin and Reed-Sternberg cells typically not identified by flow cytometry
  • Flow cytometry shows background infiltrate
  • Typically, mixed T cells with increased CD4:CD8 ratio
Electron microscopy description
  • Large rounded or oblong nuclei with folding, indentation and lobulation (Am J Pathol 1976;85:195)
  • Occasional multinucleation
  • Very large nucleoli
Electron microscopy images

Images hosted on other servers:
Reed-Sternberg cell nucleus Reed-Sternberg cell nucleus

Reed-Sternberg cell nucleus

Molecular / cytogenetics description
  • Hodgkin and Reed-Sternberg cells = B cells with clonal immunoglobulin heavy chain rearrangements (microdissection of Hodgkin and Reed-Sternberg cells required for detection) (Blood 2000;95:1443)
  • Immunoglobulin gene expression silenced in Hodgkin and Reed-Sternberg cells (Blood 2000;95:1443)
  • Hodgkin and Reed-Sternberg cells show numerical chromosome abnormalities in almost all cases (Blood 1995;86:1464)
  • 90% of CHL shows copy number gain or amplification at 9p24.1; causes increased expression of PDL1 and PDL2 (J Clin Oncol 2016;34:2690)
  • Gene mutations in components of the JAK / STAT and NFkB signaling pathways promote cell proliferation and survival (Leukemia 2021;35:968, Cancers (Basel) 2021;13:682)
Molecular / cytogenetics images

Images hosted on other servers:
PDL1 / PDL2 gain, amplification

PDL1 / PDL2 gain, amplification

Sample pathology report
  • Lymph node, left neck, excision:
    • Classic hodgkin lymphoma, nodular sclerosis subtype (see comment)
    • Comment: H&E stained sections show an enlarged lymph node with near total architectural effacement by a nodular mixed inflammatory infiltrate composed of small lymphocytes, plasma cells, neutrophils and numerous eosinophils, with scattered admixed large abnormal cells. The large abnormal cells have oval to occasionally multilobated nuclei, vesicular chromatin, prominent nucleoli and abundant cytoplasm, consistent with Reed-Sternberg cells and mononuclear variants. Lacunar cells are present. The lymph node capsule is thickened and banding fibrosis is present
    • Immunostains show that the large abnormal cells are positive for PAX5 (weak), CD30 and CD15 (subset) and are negative for CD20, CD45 and CD3. In situ hybridization for EBV encoded RNA (EBER) is negative. CD3 highlights background small T cells.
    • Flow cytometry showed polytypic B cells and mixed T cells subsets.
Differential diagnosis
Board review style question #1

Which of the following is most likely to be negative in the large abnormal cells in the image above?

  1. CD15
  2. CD30
  3. CD45
  4. MUM1
  5. PDL1
Board review style answer #1
Board review style question #2
Which of the following is most consistent with the immunophenotype of Reed-Sternberg cells?

  1. Positive: CD3, CD30, CD45, ALK1; negative: CD15, CD20, EBER, MUM1
  2. Positive: CD15, CD20, MUM1, ALK1; negative: CD3, CD15, CD45, EBER
  3. Positive: CD15, CD30, MUM1; negative: CD3, CD20, CD45
  4. Positive: CD15, CD45, EBER; negative: CD20, CD30, MUM1
  5. Positive: CD20, CD30, CD45, ALK1; negative: CD15, CD3, EBER
Board review style answer #2
C. Positive: CD15, CD30, MUM1; negative: CD3, CD20, CD45

Comment Here

Reference: Classic Hodgkin lymphoma
Board review style question #3
Which of the following subtypes of classic Hodgkin lymphoma is most commonly associated with EBV?

  1. Lymphocyte depleted
  2. Lymphocyte rich
  3. Mixed cellularity
  4. Nodular lymphocyte predominant
  5. Nodular sclerosis
Board review style answer #3
C. Mixed cellularity

Comment Here

Reference: Classic Hodgkin lymphoma

Composite lymphoma
Definition / general
  • Defined as two or more morphologically and immunophenotypically distinct lymphomas or lymphoid neoplasms that occur in the same organ or tissue site (Cancer 1977;40:959)
Essential features
  • Two or more distinct lymphomas or lymphoid neoplasms in the same location
  • 1 - 4.7% of all lymphomas
  • One lymphoma component often overshadows the other component
  • Ancillary studies very helpful
Terminology
Epidemiology
  • Accounts for 1% to 4.7% of all lymphomas (J Clin Exp Hematop 2016;56:55)
  • Combinations include (Am J Clin Pathol 2005;123:215):
    • Hodgkin lymphoma (HL) with a B cell or a T cell non-Hodgkin lymphoma (NHL)
    • B cell NHL with a T cell NHL
    • 2 distinct B cell or T cell NHLs at the same anatomic site
  • Most reported cases are B cell lymphoma composite with a Hodgkin lymphoma or two composite B cell lymphomas of different types (Hum Pathol 2014;45:768)
  • Few cases of combined classic and nodular lymphocyte predominant Hodgkin lymphoma (NLPHL) also described (Ann Oncol 1994;5 Suppl 1:7)
Sites
  • Lymph node is the most common involved anatomic site
  • Also spleen or other extranodal sites including GI, respiratory system, salivary glands, bone, skin, liver and bone marrow (J Egypt Natl Canc Inst 2007;19:171)
Etiology
  • For composite B cell lymphoma (J Egypt Natl Canc Inst 2007;19:171):
    1. Clonal selection
    2. Genomic instability and congenital predisposition
    3. Common precursor cell
  • For composite B cell NHL and HL: common precursor cell origin (J Egypt Natl Canc Inst 2007;19:171)
  • For composite T cell lymphoma with HL or B cell lymphoma:
    • EBV infection in B cells or RS cells causes reactive T cell proliferation
    • T cell neoplasm first provokes a B cell response with a pathogenic role for EBV; such B cells infected with EBV later transform to RS cells of Hodgkin lymphoma
    • A state of immunosuppression associated with some cases of T cell lymphoma lead to a prominent EBV associated B cell proliferation
    • Origin: (a) from a common EBV infected progenitor cell, (b) from the same clonal IgH gene rearrangement, (c) totally coincidental existence of two independent de novo neoplastic growth with two separate histogenic processes (J Egypt Natl Canc Inst 2007;19:171)
  • For composite AITL and B cell lymphoma:
    • The neoplastic cells in AITL produce B cell stimulatory factors leading to a continuous stimulation of B cells and promoting their malignant transformation
    • EBV is known to be associated with AITL and this specific microenvironment might be associated with the increasing risk of development of an EBV positive B cell lymphoma
    • TET2 mutations are commonly seen in AITL T cell tumor clones
      • These mutations have been identified in some monocytes and hemopoietic precursor cells and might also be shared by the B cell clones
      • As such, this genetic lesion might cause the simultaneous development of both T and B cell lymphomas (Lancet Oncol 2014;15:e435)
    • Chronic exposure to common antigens or carcinogens simultaneously stimulates or transforms B and T cells lineages (Hum Pathol 2014;45:768)
Clinical features
  • Seen in wide age range (26 - 88 years) but more common in elderly people with male predominance (2.5:1)
  • Presents either as concurrent or sequential (metachronous) disease in the same organ
  • In case of sequential disease, the second malignancy may be identified with recurrence of the primary type (J Egypt Natl Canc Inst 2007;19:171)
Diagnosis
  • Biopsy (usually lymph nodes) investigated with morphological, immunohistochemical and molecular techniques
  • The morphological criteria include cytological features of the tumor cells and the bystander cells and the growth pattern of the lymphoma (Lancet Oncol 2014;15:e435)
  • Diagnosis can be challenging because one lymphoma component often overshadows the other component
  • Ancillary studies, such as flow cytometry, immunohistochemistry, oncogenic virus tests, (such as EBV, cytogenetic study and molecular tests) can be very helpful in detection of masked neoplastic components (Hum Pathol 2014;45:768)
Prognostic factors
Case reports
Treatment
  • Overall therapeutic strategy needs to consider both disease components and the treatment goals vary dependent on the histological subtype for lymphomas (Lancet Oncol 2014;15:e435)
Microscopic (histologic) images

Images hosted on other servers:

Composite mantle cell lymphoma and CLL / SLL

Flow cytometry description
  • Usefulness of multiparametric flow cytometry in detecting composite lymphoma: study of 17 cases in a 12 year period (Am J Clin Pathol 2011;135:541)
  • Multiparameter flow cytometry is necessary for detection, characterization and diagnostics of composite mature B cell lymphoproliferative neoplasms (Int J Hematol 2013;98:589)
Differential diagnosis

DLBCL / high grade B cell lymphoma with MYC and BCL2 rearrangements
Definition / general
  • Aggressive mature B cell lymphoma
  • Category of high grade B cell lymphoma newly defined in the revised 2016 WHO (Blood 2016;127:2375, Swerdlow: WHO Classification of Tumours of Haematopoietic and Lymphoid Tissues, 2017)
  • All B cell lymphomas harboring a MYC rearrangement (chromosome 8q24.2) and a rearrangement in BCL2 (18q21.3) or in BCL6 (3q27.3) with some exceptions
    • Exceptions: Grade 3B follicular lymphomas with retained follicular architecture and B lymphoblastic leukemia / lymphoma with MYC and BCL2 translocations (may also arise out of follicular lymphoma) (Hum Pathol 2015;46:260)
  • Large B cell lymphoma with double hit arising out of a follicular lymphoma should still be designated as high grade B cell lymphoma
Essential features
  • Aggressive, mature B cell lymphoma
  • Presence of MYC rearrangement at chromosome 8q24.2 and a rearrangement in BCL2 (at chromosome 18q21.3) or in BCL6 (at chromosome 3q27.3)
  • Should be distinguished from diffuse large B cell lymphoma (DLBCL), NOS and Burkitt lymphoma for both biological and clinical reasons (Curr Opin Hematol 2012;19:299)
Terminology

Note:
  • Lymphomas with pre-existing or co-existing indolent lymphoma should be diagnosed as such (example: high grade B cell lymphoma with MYC and BCL2 rearrangements, transformed from follicular lymphoma)
  • Incorporates some cases of old terminology "B cell lymphoma, unclassifiable, with features intermediate between diffuse large B cell lymphoma and Burkitt lymphoma" (2008 WHO classification, no longer used)
  • Often called double hit lymphoma if two rearrangements are present (MYC and BCL2 or MYC and BCL6) or triple hit lymphoma if all three rearrangements are present (MYC, BCL2 and BCL6) (Hematology Am Soc Hematol Educ Program 2014;2014:9)
  • MYC translocation is required
  • Most double hit lymphomas with MYC and BCL2 translocations are double expressers (> 40% of cells Myc+; > 50% of cells BCL2+ by IHC) but these are not synonymous and this protein expression is not required
  • Most double expresser DLBCL, NOS are not double hit lymphomas and this cannot be used as a surrogate for cytogenetic testing
ICD coding
  • ICD-0: 9680/3 - B cell lymphoma, unclassifiable, with features intermediate between diffuse large B cell lymphoma and Burkitt lymphoma
Epidemiology
  • Mostly elderly patients
  • Median age sixth or seventh decade
  • Slight male predominance
Sites
  • Mainly lymph node
  • Frequently involve more than 1 extranodal site (bone marrow, CNS)
  • > 50% of patients present with widespread disease
Etiology
  • MYC, BCL2 double hit thought to arise from mature germinal center B cells; MYC, BCL6 more variable cell of origin
  • MYC translocation appears to be a secondary event, although this has been best established in cases arising out of follicular lymphoma (Blood Rev 2017;31:37)
  • Many additional cytogenetic abnormalities generally found
Diagrams / tables

Images hosted on other servers:

Diagnostic approach

Clinical features
  • More than half present with advanced disease (stage IV according to the Lugano Staging system, updated from the former Ann Arbor classification) (J Clin Oncol 2014;32:3059)
  • More than one extranodal localization, bone marrow and CNS may be involved (Curr Opin Hematol 2012;19:299, Blood 2014;124:2354)
  • High international prognostic index (IPI)
  • Elevated lactate dehydrogenase (LDH)
Diagnosis
  • Rearrangements of MYC, BCL2 and BCL6 should be detected by classic cytogenetics, FISH or other molecular / genetic tests (Blood 2016;127:2375)

Note:
  • Increased copy number, amplification or mutations are insufficient to qualify for this category in the absence of mentioned rearrangements; however, a complex karyotype is frequently seen
Prognostic factors
  • Relatively low response rate with R-CHOP (cyclophosphamide, doxorubicin, vincristine and prednisone, plus the monoclonal antibody rituximab) with short overall survival (4.5 - 18.5 months), prompting investigation of alternative therapeutic approaches (J Clin Oncol 2010;28:3360, Blood 2015;126:2466)
  • MYC partner: MYC / IG translocation may have a worse outcome compared to non-IG partner in some studies although this remains an area of investigation (Blood 2015;126:2466, Haematologica 2014;99:726)
  • Tumor morphology (may warrant further study, per WHO): majority of double hit lymphomas have DLBCL morphology and can be indistinguishable morphologically
  • Extent of the disease
  • International prognostic index (IPI) score (N Engl J Med 1993;329:987)
  • CD30 expression in > 20% of tumor cells has been associated with a better prognosis in DLBCL; however, this study found that 0/46 cases with MYC aberrations (rearrangement or amplification) were positive for CD30 at this level, excluding double / triple hit lymphomas from group (Blood 2013;121:2715)
Case reports
Treatment
  • R-CHOP (cyclophosphamide, doxorubicin, vincristine and prednisone, plus the monoclonal antibody rituximab)
  • Complete response is relatively low (Hematology Am Soc Hematol Educ Program 2014;2014:107)
  • Overall survival is short (median survival of 4.5 - 18.5 months with R-CHOP)
Microscopic (histologic) description
  • Variable morphology
  • Subset (~50%) with morphology of DLBCL, NOS (Histopathology 2016;68:1090)
    • Diffuse growth pattern
    • Relatively few small lymphocytes
    • Some fibrosis
    • Starry sky macrophages may be present
    • Variable degree of mitosis and apoptosis
    • Nuclei with variable size and contour, some 3 - 4 times the size of normal lymphocytes (larger than Burkitt lymphoma cells)
    • Cytoplasm more abundant and less basophilic than in Burkitt lymphoma
  • Subset (~50%) with morphology of Burkitt lymphoma or has features intermediate between DLBCL and Burkitt lymphoma (Blood 2011;117:2319):
    • Diffuse proliferation
    • Medium to large nuclei
    • Very few admixed small lymphocytes
    • No stromal reaction or fibrosis
    • Starry sky macrophages generally present
    • High mitotic rate and apoptosis
    • Some relatively monomorphic (similar to Burkitt lymphoma) and more variation in nuclear and nucleolar features
    • Cytoplasm less basophilic than in Burkitt lymphoma
  • Other cases may have blastoid cytomorphology (Histopathology 2012;61:945)
    • Tumor cells are mature B cells positive for CD10 and BCL6 and negative for TdT
    • Medium sized cells resembling small centroblasts
    • Nuclei with finely granular chromatin and inconspicuous nucleoli
    • Small rim of cytoplasm
Microscopic (histologic) images

Contributed by Genevieve M. Crane, M.D., Ph.D.

High grade B cell
lymphoma with
MYC and BCL2
translocations

BCL2

Myc



Images hosted on other servers:

H&E

Positive stains

Note:
Negative stains
Flow cytometry description
  • Some cases lack sIg (multiple IG loci involved by translocations)
Flow cytometry images

Contributed by Genevieve M. Crane, M.D., Ph.D.

Lack of sIg

Molecular / cytogenetics description
  • MYC (8q24.2) rearrangement:
    • IG partners (usually IGH, less frequently IGK or IGL): 65%
    • Non-IG partners
  • BCL2 (18q21.3) rearrangement
  • BCL6 (3q27.3) rearrangement

Note:
  • In current classification, a combination of a chromosomal rearrangement of one gene and copy number increase or amplification of other genes is not sufficient to classify a case as a double hit high grade B cell lymphoma
Molecular / cytogenetics images

Images hosted on other servers:

Karyotype

Differential diagnosis
Board review style question #1
Which of the following is confirmation of double hit lymphoma?

  1. High MYC and Ki67 expression on IHC
  2. MYC and BCL2 expression on IHC, each 40% expression
  3. MYC and BCL6 gene rearrangement on FISH
  4. MYC and CCND1 expression on IHC
  5. MYC and CCND1 gene rearrangement on FISH
Board review style answer #1
C. MYC and BCL6 gene rearrangement on FISH

Comment Here

Reference: With MYC and BCL2 or BCL6 rearrangements
Board review style question #2
Evaluation of CD30 expression in double hit lymphoma is recommended because it

  1. Can be suggestive of further FISH studies
  2. Can be used for targeted therapy
  3. Defies a new subclass of double hit lymphoma
  4. Is mainly associated with BCL6 gene rearrangement rather than BCL2 gene rearrangement
  5. Is a prognostic factor
Board review style answer #2
B. Can be used for targeted therapy

Comment Here

Reference: With MYC and BCL2 or BCL6 rearrangements

DLBCL, NOS
Definition / general
  • Large B cell lymphoma with a diffuse growth pattern which does not meet the criteria for other WHO defined large B cell lymphoma categories
Essential features
  • Medium to large atypical lymphoid cells
  • Nucleus ≥ size of histiocyte nucleus or 2 lymphocyte nuclei
  • Does not meet criteria for other WHO defined large B cell lymphomas
  • 2 main molecular subtypes: activated B cell (ABC) subtype and germinal center B cell (GCB) subtype
ICD coding
  • ICD-O: 9680/3 - malignant lymphoma, large B cell, diffuse, NOS
Epidemiology
Sites
Etiology
  • May arise de novo or by transformation from a low grade lymphoma
Clinical features
  • Rapidly enlarging mass (≥ 5 cm in 76% of cases) (J Clin Oncol 1998;16:2780)
  • B symptoms (33%) (J Clin Oncol 1998;16:2780)
  • Some asymptomatic
  • May represent transformation of existing low grade B cell lymphoma (e.g., follicular lymphoma, marginal zone lymphoma, chronic lymphocytic leukemia [CLL] / small lymphocytic lymphoma [SLL], nodular lymphocyte predominant Hodgkin lymphoma)
Diagnosis
  • Morphologic evaluation, immunophenotyping, cytogenetics (FISH) and clinical history
Laboratory
Prognostic factors
  • Unfavorable prognostic features:
    • Clinical: ↑ age, ↑ LDH, Eastern Cooperative Oncology Group (ECOG) ≥ 2, Ann Arbor stage III / IV, extranodal disease, bulky disease (J Clin Oncol 2010;28:2373, Blood 2014;123:837)
    • Cell of origin status (Blood 2004;103:275):
      • Activated B cell subtype (~40%) and germinal center B cell subtype (~60%) associated with 5 year progression free survival rates of 40 - 50% versus 70 - 80%, respectively
    • MYC translocation (Blood 2015;126:2466)
    • Double expresser status: immunohistochemical coexpression of MYC (≥ 40% tumor cells) and BCL2 (≥ 50% tumor cells) in the absence of co-occurring MYC and BCL2 rearrangements
    • Bone marrow involvement associated with increased risk of CNS relapse
Case reports
Treatment
  • R-CHOP (rituximab, cyclophosphamide, doxorubicin, vincristine and prednisone); other rituximab based regimens
Gross images

Contributed by Claudia Mendez, M.D.
DLBCL involving the kidney

DLBCL involving the kidney

Frozen section description
  • Large atypical lymphoid cells with a diffuse growth pattern consistent with a large cell lymphoma can be appreciated on frozen sections; however, ancillary immunophenotypic and cytogenetic studies are required to establish B cell lineage and further differentiate specific DLBCL subtypes
Microscopic (histologic) description
  • Partial or complete effacement of normal tissue architecture by a diffuse infiltrate of large (or occasionally medium size) atypical B lymphoid cells with vesicular chromatin and prominent nucleoli
    • Large is defined as nucleus size ≥ histiocyte nucleus or > 2 lymphocyte nuclei
  • Morphologic variants:
    • Immunoblastic variant: single central nucleolus
    • Centroblastic variant: 2 - 4 nucleoli
    • Anaplastic variant: anaplastic nuclei, can mimic anaplastic large cell lymphoma or Reed-Sternberg cells
Microscopic (histologic) images

Contributed by Matthew M. Klairmont, M.D.
Diffuse growth pattern

Diffuse growth pattern

Cytologic features

Cytologic features

Bone marrow involvement

Bone marrow involvement

Omental involvement Omental involvement

Omental involvement

CD20 (bone marrow)

CD20 (bone marrow)

Virtual slides

Images hosted on other servers:

Lymph node involved by DLBCL, NOS

DLBCL transformed from follicular lymphoma

Cytology description
  • Cytologic evaluation can identify large atypical lymphoid cells with vesicular chromatin and prominent nucleoli, consistent with a large cell lymphoma; however, classification as DLBCL, NOS requires immunophenotypic and cytogenetic studies
Positive stains
Negative stains
Immunohistochemical subclassification
  • Cell of origin subtype prediction by IHC using the Hans algorithm (positivity defined as staining in ≥ 30% of cells) (Blood 2004;103:275):
    • Germinal center B cell (GCB) subtype: CD10+ or CD10- / BCL6+ / MUM1-
    • Non-GCB subtype (classifier that encompasses ABC subtype and unclassifiable cases by GEP): CD10- / BCL6- or CD10- / BCL6+ / MUM1+
  • Double expresser status (30% of DLBCL, NOS):
    • Positive staining for BCL2 (> 50% tumor cells) and MYC (> 40% tumor cells) in the absence of underlying chromosomal rearrangements (double expresser) is associated with increased risk of CNS relapse and inferior survival (J Clin Oncol 2017;35:2515)
    • If MYC and BCL2 rearrangements confirmed by FISH, classified as high grade B cell lymphoma with MYC and BCL2 rearrangements (also known as double hit DLBCL)
Flow cytometry description
  • Flow cytometry shows a population of monoclonal B cells with high forward scatter (indicative of large size)
Flow cytometry images

Contributed by Matthew M. Klairmont, M.D.
Scatter plots of splenic mass

Scatter plots of splenic mass

Molecular / cytogenetics description
  • Gene expression profiling identifies activated B cell and germinal center B cell molecular subtypes of DLBCL, NOS
    • Since gene expression profiling is not widely available in the clinical setting, immunohistochemical algorithms that approximate these molecular subtypes (i.e., Hans algorithm) are used as an alternative (see Immunohistochemical subclassification)
  • FISH used to differentiate DLBCL, NOS from high grade B cell lymphoma with MYC and BCL2 and / or BCL6 rearrangements:
    • BCL2 rearrangement (20 - 30%; < 5% activated B cell, 50% germinal center B cell)
    • BCL6 rearrangement (30%; 30% activated B cell, 15% germinal center B cell)
    • MYC rearrangement (8 - 14% for activated B cell and germinal center B cell; ~50% have co-occurring BCL2 or BCL6 rearrangement)
  • PCR B cell gene rearrangement studies show clonal immunoglobulin gene rearrangement
  • Mutational landscape very heterogeneous with recurrent alterations identified in over 150 genes
    • Most frequently mutated genes include KMT2D (25%), MYD88 (17%), PIM1 (15%), CREBBP (12%), HIST1H1E (11%), BCL2 (10%), SPEN (10%), ARID1A (10%), TP53 (9%) (Cell 2017;171:481)
  • Multiple integrative genomic classification models recently proposed but none currently incorporated into WHO classification guidelines (Cell 2017;171:481, N Engl J Med 2018;378:1396, Nat Med 2018;24:679, Blood 2020;135:1759)
    • Common DLBCL genomic subtypes that have emerged from multiple independent studies include those characterized by:
      • MYD88 L265P and CD79B mutations: mainly activated B cell subtype, 18q gain, genetically similar to primary extranodal lymphomas, poor survival
      • BCL6 translocations and NOTCH2 mutations: mainly activated B cell subtype, constitutive B cell receptor signaling, genetically similar to marginal zone lymphoma, more favorable survival
      • BCL2 translocations and mutations in EZH2 / other chromatin modifiers: mostly germinal center B cell subtype with genetic features similar to other B cell lymphomas of germinal center origin
Videos

Overview of histologic features

Sample pathology report
  • Lymph node, left cervical, excisional biopsy:
    • Diffuse large B cell lymphoma, NOS, GCB subtype (see comment)
    • Comment: H&E sections show effacement of the normal nodal architecture by sheets of large atypical lymphoid cells with vesicular chromatin and prominent nucleoli. By immunohistochemistry, the tumor cells are positive for CD20, CD10 and BCL6, while negative for CD3, CD5, MUM1, BCL2, MYC and cyclin D1. EBER ISH is negative. The Ki67 proliferation index is 70%. Cytogenetic FISH studies are negative for rearrangements of BCL2, BCL6 and MYC. The morphologic, immunophenotypic and cytogenetic findings are diagnostic of diffuse large B cell lymphoma, NOS, GCB subtype (Hans algorithm).
Differential diagnosis
Board review style question #1

A 60 year old man presents with an inguinal mass and recent onset of B symptoms. H&E sections of a core needle biopsy of the mass show sheets of large, pleomorphic lymphoid cells with vesicular chromatin and prominent nucleoli. By immunohistochemistry, the tumor cells are positive for CD20, CD5, CD10, BCL6 and MYC, while negative for CD3, MUM1, BCL2, SOX11 and cyclin D1. EBER ISH is negative. The Ki67 proliferation index is 90%. FISH studies are negative for rearrangements of BCL2, BCL6 and MYC. This lymphoma is best classified as

  1. Mantle cell lymphoma, pleomorphic subtype
  2. Burkitt lymphoma
  3. Diffuse large B cell lymphoma, NOS, with germinal center B cell phenotype
  4. Diffuse large B cell lymphoma, NOS, with nongerminal center B cell phenotype, with double expression of MYC and BCL6
  5. Diffuse large B cell lymphoma, NOS, with nongerminal center B cell phenotype
Board review style answer #1
C. Diffuse large B cell lymphoma, NOS, germinal center B cell subtype. Negative cyclin D1 and SOX11 IHC virtually rules out mantle cell lymphoma (answer A). Burkitt lymphoma is composed of intermediate size cells with round nuclei which lack significant pleomorphism and have a Ki67 proliferation index of approximately 100% (answer B). Coexpression of MYC and BCL2 (not BCL6) has been associated with inferior clinical outcomes and should be noted in the report (answer D). Co-occurring rearrangements of BCL6 and MYC would support the diagnosis of high grade B cell lymphoma with MYC and BCL2 and / or BCL6 rearrangements. The morphologic, immunophenotypic and cytogenetic findings are diagnostic of DLBCL, NOS, with a germinal center B cell phenotype (answer C). Per the Hans algorithm, CD10 positivity in DLBCL is predictive of a germinal center B cell subtype gene expression profile. Nongerminal center B cell phenotype: CD10- / BCL6- or CD10- / BCL6+ / MUM1+ (answer E).

Comment Here

Reference: DLBCL, NOS
Board review style question #2
A 65 year old woman presents with a rapidly enlarging neck mass. H&E sections of a core needle biopsy of the mass show sheets of large atypical lymphoid cells with vesicular chromatin and prominent nucleoli which by immunohistochemistry are positive for CD20. Which of the following immunohistochemical findings would support the diagnosis of diffuse large B cell lymphoma, NOS?

  1. Cyclin D1+
  2. ALK+
  3. TdT+
  4. CD21-
  5. BCL6-
Board review style answer #2
D. CD21-. CD21 marks follicular dendritic cell meshworks, the absence of which is a characteristic feature of DLBCL. Positive cyclin D1 typically supports the diagnosis of mantle cell lymphoma (answer A). ALK expression does not support the diagnosis of DLBCL, NOS (answer B). It is, however, a defining feature of the ALK+ large B cell lymphoma, which is also characteristically CD20 negative. TdT is an immaturity marker characteristically expressed in acute lymphoblastic leukemia (answer C). Although high grade mature B cell lymphomas can on very rare occasion exhibit TdT expression, this finding does not support the diagnosis of DLBCL). BCL6 is a germinal center marker used in the Hans algorithm to predict cell of origin status (answer E).

Comment Here

Reference: DLBCL, NOS

DLBCL-primary testicular
Definition / general
  • Large B cell lymphoma that is confined to and presumably arises in testis
  • Recently classified under large B cell lymphomas (LBCL) of immune privileged sites in the most recent revision of the WHO, 5th edition; this is a new umbrella term that acknowledges common biological features of aggressive B cell lymphomas (Leukemia 2022 Jun 22 [Epub ahead of print])
    • This new entity now combines the previous entity of primary DLBCL of CNS with DLBCL of the vitreoretina and testis that were previously included among DLBCL, NOS
  • Recently considered a distinct entity by the International Consensus Classification of Mature Lymphoid Neoplasms, closely related to primary DLBCL of the central nervous system on the basis of common clinical and molecular features (Blood 2022 Jun 2 [Epub ahead of print])
Essential features
  • Comprises 80 - 90% of all primary testicular lymphomas with a mean age of presentation between 60 and 70 years of age
  • Composed of large atypical lymphoid cells with a predominant centroblastic morphology that replace the testicular parenchyma
  • Positive for pan B cell markers and typically positive for BCL6, MUM1 and BLC2
Terminology
  • Primary DLBCL of testis
  • Primary testicular lymphoma
ICD coding
  • ICD-O: 9680/3 - malignant lymphoma, large B cell, diffuse, NOS
  • ICD-10: C83.3 - diffuse large B cell lymphoma
Epidemiology
  • Typically presents in adult men with a median age between 60 and 70 at diagnosis
    • Most common testicular malignancy in men > 60 years of age
  • Most common testicular malignancy to present with bilateral involvement
  • Increased incidence in HIV positive men (Cancers (Basel) 2021;13:4049)
    • HIV positive patients with primary testicular diffuse large B cell lymphoma (DLBCL PT) are younger (median age of 36)
Sites
  • Most patients present with localized disease
  • Approximately 20% of patients have advanced stage with spread to contralateral testis, CNS or other extranodal sites (J Clin Oncol 2009;27:5227)
Pathophysiology
  • Evaluation of the cell of origin through IHC and gene expression profiling has demonstrated that DLBCL PT has a non-GCB phenotype in 60 - 96% of cases (Mod Pathol 2006;19:1521, J Pathol 2006;210:163)
  • Characterized by deregulation of intercellular immune signaling mechanisms, including NFkB pathway activation (Blood 2016;127:869)
    • Oncogenic toll-like receptor (TLR) signaling (via MYD88 activating mutations) and B cell receptor (BCR) signaling (via CD79b mutation) are central to the pathogenesis, leading to NFkB pathway activation
    • Prosurvival signals through mutations in PIM1/2, CARD11 and TNFAIP3 further contribute to the dysregulation in NFkB signaling
  • Mutations within genes mediating immune surveillance are also central to DLBCL PT pathogenesis
    • Losses of the HLA class I and II loci are associated with reduced expression of major histocompatibility complexes (MHC)
    • Gains and amplifications of the programmed death ligand (PDL) 1 (CD274) and 2 (CD273) loci, that increase protein expression and contribute to the evasion of an antitumor immune response
Etiology
  • Unknown in immunocompetent patients
Diagrams / tables

Images hosted on other servers:

Dysregulated NFkB signaling pathway

Clinical features
  • Firm, painless testicular mass without preference for either side (Blood 2014;123:486)
    • Median tumor size at presentation is 6 cm
  • Systemic symptoms are infrequent, seen in approximately 25 - 40% of cases
    • Fever, anorexia, night sweats, weight loss
  • Most patients present with localized disease and about 20% present with advanced stage disease, including spread to contralateral testis, CNS or other extranodal sites
Diagnosis
  • Imaging modalities that may assist in diagnosis include:
    • Ultrasonography
    • Magnetic resonance imaging (MRI)
  • Biopsy of the involved testis or orchiectomy
  • Secondary testicular involvement by systemic lymphoma should be excluded
  • Reference: Ann Oncol 2016;27:v91
Laboratory
  • Increased serum lactate dehydrogenase (LDH), increased serum B2 microglobulin and hypoalbuminemia have been associated with an adverse prognosis (J Clin Oncol 2003;21:20)
Radiology description
  • Ultrasonography will show focal or diffuse areas of hypoechogenicity with hypervascularity in affected testis
  • MRI allows simultaneous evaluation of both testes, paratesticular spaces and spermatic cord
Radiology images

Images hosted on other servers:

CT pelvis and FDG PET

Prognostic factors
  • High relapse rate; recurrence most often occurs in the CNS or the contralateral testis (Br J Haematol 2017;176:210)
  • Involvement / recurrence in the CNS is associated with a dismal prognosis; relapse in the CNS has been reported in up to 30% of the cases within 1 - 2 years from diagnosis
  • Adverse prognostic factors for progression free survival include age > 70 years, advanced stage, B symptoms, involvement of extranodal sites other than testis and tumor diameter > 10 cm (J Clin Oncol 2009;27:5227, J Clin Oncol 2003;21:20, Leuk Lymphoma 2010;51:1217)
Case reports
Treatment
Gross description
  • Enlarged testicle with a fleshy, diffuse or lobulated firm mass
  • Cut surface is relatively homogeneous and bulging, with a tan-yellow color
  • Often demonstrates a sharp demarcation from testicular parenchyma
  • Reference: Medicine (Baltimore) 2020;99:e19463
Gross images

Contributed by Miguel Gonzalez-Mancera, M.D.

Well demarcated testicular mass

Microscopic (histologic) description
  • Diffuse sheets of large lymphoma cells (Pathology 2020;52:53)
    • Initially infiltrate between seminiferous tubules before total replacement of the parenchyma
    • Lymphoma cells frequently invade and fill the seminiferous tubules
    • Frequent mitotic figures can be seen (Hematol Oncol 2014;32:72)
    • Necrosis is common
  • Lymphoma cells show centroblastic morphology in most cases (90%) and less commonly, immunoblastic morphology (10%)
  • Associated changes in the testis include spermatogenic arrest, interstitial fibrosis and tubular hyalinization
Microscopic (histologic) images

Contributed by Miguel Gonzalez-Mancera, M.D.

Diffuse infiltration

High mitotic activity

Centroblastic morphology

Immunoblastic morphology

Lymphoma infiltrating testicular parenchyma


CD20

CD10

BCL6

MUM1

Positive stains
Negative stains
Molecular / cytogenetics description
  • IGH rearrangements in virtually all cases
  • BCL2 and MYC rearrangements occur in 10% and 15% of cases, respectively; BCL6 rearrangement occurs in 40% (Leuk Lymphoma 2014;55:1410)
  • Genetic analysis (Onco Targets Ther 2019;12:10165, Leukemia 2014;28:719):
    • MYD88: amplifications, mutations and deletions (60 - 82%)
    • CD79B: mutations and deletions (19 - 34%)
    • NFKBIZ: copy number gain (42%)
    • CDKN2A: copy number alterations (71%)
    • CD274: rearrangements, copy number alterations, an increased protein expression (35%)
    • CD273: rearrangements, copy number alterations, an increased protein expression (47%)
Sample pathology report
  • Right testis, orchiectomy:
    • Diffuse large B cell lymphoma, nongerminal center type (see comment)
    • Comment: In the proper clinical circumstances, findings are consistent with primary diffuse large B cell lymphoma of the testis. Clinical correlation is required to exclude systemic or extratesticular primary disease.
Differential diagnosis
Board review style question #1

A man presents with right testicular mass and undergoes orchiectomy. Microscopic findings reveal a diffuse process of large lymphoid cells replacing the testicular parenchyma (see image). In which age group is this entity most commonly seen?

  1. < 15 years
  2. 30 - 40 years
  3. 60 - 70 years
  4. Typically seen at any age
Board review style answer #1
C. 60 - 70 years. Diffuse large B cell lymphoma of the testis has a mean age of presentation of 60 - 70 years.

Comment Here

Reference: DLBCL-primary testicular
Board review style question #2
Which of the following stains would be expected to be negative in diffuse large B cell lymphoma of the testis?

  1. BCL2
  2. CD20
  3. EBER ISH
  4. MUM1
Board review style answer #2
C. EBER ISH. EBV infection has not been associated with diffuse large B cell lymphoma of the testis.

Comment Here

Reference: DLBCL-primary testicular

EBV+ DLBCL
Definition / general
  • Epstein-Barr virus (EBV) positive diffuse large B cell lymphoma is a clonal B cell lymphoid proliferation in patients without a history of immunodeficiency or lymphoma
  • Other well defined EBV positive lymphoid disorders, such as plasmablastic lymphoma, diffuse large B cell lymphoma associated with chronic inflammation and EBV positive mucocutaneous ulcer, are excluded from this category
Essential features
  • EBV positive clonal large B cell lymphoma in patients without prior history of lymphoma or immunodeficiency
  • Associated with older age, predominant in males and often advanced stage at presentation
  • Large transformed cells with a spectrum of morphologies seen in sheets or admixed in a polymorphous infiltrate
Terminology
  • EBV positive diffuse large B cell lymphoma of the elderly
  • Senile EBV associated B cell lymphoproliferative disorder
  • Age related EBV positive lymphoproliferative disorder
ICD coding
  • ICD-O: 9680/3 - malignant lymphoma, large B cell, diffuse, NOS
  • ICD-10: C83.30 - diffuse large B cell lymphoma, unspecified site
Epidemiology
  • Mostly older patients (> 50 years), with peak in the eighth decade; second smaller peak in the third decade (Oncotarget 2015;6:13933)
  • M > F
  • Higher incidence in Asia and Latin America compared with Europe
Sites
  • Nodal and extranodal involvement is common
  • Frequent extranodal sites: skin, lung, tonsil, gastrointestinal tract
  • Younger patients (< 50 years) more likely to have predominantly nodal disease
  • Bone marrow involved in ~10% of patients
  • Reference: Clin Cancer Res 2007;13:5124
Pathophysiology
  • In elderly patients, pathogenesis is thought to be related to multifactorial causes of immunosenescence impacting both innate and adaptive immune responses
  • Alterations of the immune microenvironment may also contribute
  • EBV dependent activation of proliferative, cell cycle and antiapoptotic cellular mechanisms via NFkB pathway, PI3K / AKT, MEK-ERK, MAPK pathways, enhancing CDK2 and phosphorylation of Rb protein and activation of BCL2 (Blood 2013;122:328)
Clinical features
Diagnosis
  • Tissue biopsy (lymph node / extranodal sites)
    • Core beedle biopsy
    • Excisional biopsy
Laboratory
  • May have EBV DNA in blood or serum; however, this can also be seen in EBV negative diffuse large B cell lymphoma
  • Elevated LDH
Prognostic factors
  • Prognostic data derived mainly from retrospective studies
  • Worse prognosis in older patients (> 45 years)
  • In elderly patients > 70 years, B symptoms are associated with worse prognosis
  • Monomorphic EBV positive diffuse large B cell lymphoma associated with worse prognosis in younger patients as compared with cases with T histiocyte rich or polymorphic large B cell patterns
  • Prognosis varies with region (Asian data is controversial) (Blood 2013;122:328)
    • Median survival is 2 years in Asian patients
    • Worse prognosis than EBV negative diffuse large B cell lymphoma in Asian patients
    • CD30 expression associated with worse prognosis in European countries
  • Survival outcome has improved with the addition of rituximab to treatment regimens (Am J Hematol 2018;93:953)
Case reports
Treatment
  • No prospective comparative studies have been performed to evaluate treatment options
  • No standard approach, treatment strategies similar to de novo EBV negative diffuse large B cell lymphoma
Microscopic (histologic) description
  • Architectural effacement of site of involvement
  • Geographic necrosis and angioinvasion may be present
  • Neoplastic component comprised of large, atypical cells that can have an immunoblast, centroblast or Reed-Sternberg-like morphology
  • 2 architectural patterns described (no known prognostic relevance) (Mod Pathol 2012;25:968)
    • Monomorphic type: sheets of large cells resembling diffuse large B cell lymphoma
    • Polymorphic pattern: variable amount of admixed reactive small lymphocytes, histiocytes and plasma cells (3 different subtypes described below)
      • Canonical large B cell-like features: high density of large neoplastic cells and scattered Reed-Sternberg-like cells
      • Hodgkin lymphoma-like features: lower density of Reed-Sternberg-like cells
      • Posttransplant lymphoproliferative disease (PTLD)-like features: lower density neoplastic cells without Hodgkin-like features, may show plasmacytoid differentiation
  • Typically, activated B cell subtype by Hans criterion
  • No cutoff of EBV positivity in neoplastic cells; cutoffs ranging from 10 - 20% have been used in studies
  • Type II and less often, type III latency pattern based on EBNA2 and LMP1 staining patterns
Microscopic (histologic) images

Contributed by Sudhir Perincheri, M.B.B.S., Ph.D.
Polymorphic type

Polymorphic type

Reed-Sternberg-like cells

Reed-Sternberg-like cells

CD20

CD20

EBER

EBER

EBER

Monomorphic type

Necrosis

Necrosis

Positive stains
Negative stains
Molecular / cytogenetics description
  • Monoclonal IgH rearrangements
Sample pathology report
  • Lymph node, cervical lymph node, excision:
    • EBV positive, diffuse large B cell lymphoma
    • Microscopic description: The lymph node architecture is completely effaced by a variably sized infiltrate, including very large cells with irregular nuclear membranes, vesicular nuclei, distinct nucleoli and moderate amount of cytoplasm. There is a mixed infiltrate composed of small lymphocytes and histiocytes. The background is hyalinized, with areas of necrosis and foreign body giant cells. There are areas with residual small lymphocytes.
    • Immunostains: The large neoplastic cells are positive for CD20, PAX5, CD30 and negative for CD15 and CD10. BCL6 stains a subset of the large cells. EBER and LMP1 are positive in the large cells. CD3 and CD5 stain the background small T cells. Ki67 stains the majority of the large cells with an overall staining of approximately 50%.
Differential diagnosis
Board review style question #1

A 68 year old man with no prior relevant history presented with generalized lymphadenopathy. Excisional biopsy of a cervical lymph node shows large neoplastic cells with admixed small lymphocytes (see figure above). The large neoplastic cells are positive for CD20 and EBER. Which of the following immunophenotypes is most compatible with EBV positive diffuse large B cell lymphoma?

  1. Positive for PAX5, CD30, CD15 and CD10
  2. Positive for PAX5, BCL6, CD10 and CD138
  3. Positive for PAX5, CD30, BCL6 and MUM1
  4. Positive for CD79a, CD30, CD15 and negative for CD45
Board review style answer #1
C. Positive for PAX5, CD30, BCL6 and MUM1

Comment Here

Reference: EBV+
Board review style question #2
Which of the following is true of EBV positive diffuse large B cell lymphoma?

  1. Only seen in patients > 50 years of age
  2. CD30 expression is associated with poor prognosis
  3. Younger patients have predominantly extranodal disease
  4. Disease usually shows a type I EBV latency pattern
Board review style answer #2
B. CD30 expression is associated with poor prognosis

Comment Here

Reference: EBV+

EBV+ mucocutaneous ulcer
Definition / general
  • Indolent and rare mature B cell neoplasm associated with age related or iatrogenic immunosuppression
  • Almost all Epstein-Barr virus positive mucocutaneous ulcer (EBVMCU) patients achieve remission with the reduction or discontinuation of immunosuppressants
Essential features
  • Indolent and rare mature B cell neoplasm
  • Shallow, sharply circumscribed, unifocal mucosal or cutaneous ulcer
  • Associated with various forms of immunosuppression (including immune senescence) and often spontaneous regression
  • Proliferation of EBV+, variable sized, atypical CD30+ B cells
ICD coding
  • ICD-O: 9680/1 - EBV+ mucocutaneous ulcer
Epidemiology
Sites
Etiology
  • Iatrogenic or age associated immunosuppression
Diagrams / tables

Images hosted on other servers:

Differentiating characteristics of EBV+ DLBCL and EBVMCU

Clinical features
Diagnosis
  • Biopsy
Prognostic factors
Case reports
Treatment
Microscopic (histologic) description
  • Shallow, mucosal or cutaneous ulcers with adjacent acanthosis of the epidermis or mucosa and sometimes pseudoepitheliomatous changes (Semin Diagn Pathol 2017;34:60)
  • Variable angiotropism
  • Polymorphic infiltration with various inflammatory cells such as plasma cells, histiocytes and granulocytes
  • Atypical cells (EBV+) range in size from small to large and may resemble classic Hodgkin Reed-Sternberg (HRS) cells
  • A rim of small T lymphocytes at the base of the lesions is usually noted
Microscopic (histologic) images

Contributed by Sergio Pina-Oviedo, M.D.
Surface ulceration and necrosis

Surface ulceration and necrosis

Atypical cells with Hodgkin / RS cell-like morphology

Atypical cells with Hodgkin / RS cell-like morphology

EBER ISH positive in atypical cells

EBER ISH positive in atypical cells

CD20 positive B cells

CD20 positive B cells

CD3 positive T cells

CD3 positive T cells

Positive stains
Negative stains
Sample pathology report
  • Tongue, left base, biopsy:
    • Atypical B cell lymphoproliferative process, EBV+ (see comment)
    • Comment: Examination of hematoxylin and eosin stained slides reveals stratified nonkeratinized squamous epithelium accompanied by a submucosal infiltrate dominated by small lymphocytes and plasma cells. The epithelium displays focal ulceration, while the infiltrate showcases a focal area of abundant apoptotic debris that includes frequent scattered large atypical immunoblastic cells. These large cells exhibit nucleomegaly with prominent nucleoli and multinucleation.
    • Immunohistochemical stains are performed, revealing that CD20 and PAX5 highlight small B cells in nodules as well as the scattered large atypical cells. Similarly, CD79a demonstrates a comparable pattern in the small B cells. CD3 highlights an abundance of small T cells in the background, with CD4 and CD8 immunostains indicating that most of the T cells are CD4+. BCL6 is negative and BCL2 is positive in B and T cell areas. CD56 highlights rare scattered small cells, while CD30 is positive in the large atypical cells and CD15 highlights scattered granulocytes. The c-myc immunostain is negative but CD138 highlights the abundant plasma cells. In situ hybridization studies of kappa and lambda light chains demonstrate polytypic light chain expression. MUM1 highlights the plasma cells and in situ hybridization for EBV (EBER) is positive in the large atypical cells.
    • In summary, examination of the patient's tissue reveals ulcerated squamous mucosa with an atypical submucosal lymphoplasmacytic infiltrate consisting of aggregates of B cells and scattered large atypical lymphocytes that test positive for EBER, CD30 and CD20. The immunomorphologic observations of the infiltrate are indicative of EBV mucocutaneous ulcer and clinical correlation is necessary.
Differential diagnosis
  • EBV+ polymorphic lymphoproliferative disorder:
    • Often linked to immunosuppression, such as posttransplant
    • Systemic symptoms like fever, weight loss and lymphadenopathy
    • Involves multiple anatomical locations, mainly lymph nodes, spleen and liver
    • Heterogeneous lymphoid population, including B cells, T cells and plasma cells
    • Reed-Sternberg-like cells may be present but are not definitive
    • Immunophenotype similar to EBVMCU; B cells often express EBV latent membrane proteins (EBER, LMP1, EBNA2) in a range of cell sizes
    • Monoclonal or oligoclonal IG rearrangements
Board review style question #1

A 34 year old woman presents with a painful ulcer on her upper lip (as shown in the image above). The lesion has been present for ~4 weeks and has not responded to topical treatments. The lesion is about 1.5 cm in diameter, has a raised border and a yellowish base. She reports no history of trauma to the area but reports a recent onset of fatigue and malaise. The patient has a history of Hodgkin lymphoma, which was treated with chemotherapy and radiation therapy 2 years ago. A biopsy of the lesion shows diffuse infiltration of large atypical lymphoid cells, positive for CD20, CD30, PAX5 and Epstein-Barr virus (EBV) and negative for CD15 and CD56. There is a mixed inflammatory infiltrate, including lymphocytes and plasma cells. What is the most likely diagnosis?

  1. Classic Hodgkin lymphoma
  2. EBV+ mucocutaneous ulcer
  3. Extranodal NK / T cell lymphoma
  4. Lymphocyte predominant Hodgkin lymphoma
  5. Posttransplant associated lymphoproliferative disorder (PTLD)
Board review style answer #1
B. EBV+ mucocutaneous ulcer. EBV+ mucocutaneous ulcer (EBVMCU) is the most likely diagnosis for this patient due to the presence of a localized ulcer on her upper lip and the histological findings of a polymorphous infiltrate with large atypical cells, positive for CD20 and CD30 and negative for CD15, which align with the typical profile of EBVMCU. Additionally, the presence of Epstein-Barr virus (EBV) in the atypical cells, as well as the patient's history of immunosuppression following treatment for Hodgkin lymphoma, further support the diagnosis. EBVMCU is a self limited, often recurring disease that usually occurs in the setting of immune dysregulation or immunosuppression, fitting well with the patient's clinical context.

Answer A is incorrect because the clinical presentation and the immunophenotypic profile of the cells are not typical for classic Hodgkin lymphoma (CHL). CHL usually presents as lymphadenopathy rather than a mucocutaneous ulcer. Furthermore, CHL is characterized by the presence of Reed-Sternberg cells, which are typically CD30 positive, CD15 positive and CD20 negative, in contrast to the cells in this case, which are CD15 negative and CD20 positive.

Answer C is incorrect because extranodal NK / T cell lymphoma is typically associated with Epstein-Barr virus (EBV) infection but the neoplastic cells are typically T cells or NK cells, not B cells. The cells in this case are positive for CD20, a B cell marker, which makes NK / T cell lymphoma unlikely. Additionally, extranodal NK/T cell lymphoma frequently involves sites such as the nasal cavity, whereas this lesion is on the lip.

Answer D is incorrect because lymphocyte predominant Hodgkin lymphoma (LPHL) is characterized by the presence of LP cells, which are variants of Reed-Sternberg cells. LP cells typically express CD20 but not CD30. This case presents cells that are both CD20 and CD30 positive, which is not characteristic of LPHL. Furthermore, LPHL typically presents as lymphadenopathy, not as a mucocutaneous ulcer.

Answer E is incorrect because despite the patient's history of immunosuppression, the findings do not match those of a posttransplant lymphoproliferative disorder (PTLD). Polymorphic LPDs can appear in multiple settings of immune deficiency, not just posttransplant. They usually manifest as mass forming lesions composed of heterogeneous infiltrate of cells, including those similar to Hodgkin / Reed-Sternberg cells. While they share some immunophenotypic similarities with EBV+ mucocutaneous ulcers, polymorphic LPDs more often show monoclonal or oligoclonal IG rearrangements and have a broader range of potential sites of involvement, more commonly lymph nodes and various organs. The solitary painful lip ulcer in this case is more consistent with an EBV+ mucocutaneous ulcer.

Comment Here

Reference: EBV+ mucocutaneous ulcer

EBV+ nodal T and NK cell lymphoma
Definition / general
  • Primary nodal lymphomas composed of NK or mature T cells with EBV infection in immunocompetent individuals
Essential features
  • Primary nodal T cell lymphoma with EBV positive tumor cells in immunocompetent patients
  • Tumor cells have high grade morphology and frequently show large cells with vesicular nuclei
  • These cases largely represent neoplastic expansions of EBV positive cytotoxic T cell lymphocytes (usually CD8+ with coexpression of TIA1 and granzyme B)
  • The majority of cases have been reported in Asia, are usually diagnosed at an advanced stage and feature a poor median survival time of 3 - 8 months
Terminology
  • EBV positive nodal T or NK cell lymphoma
  • Epstein-Barr virus associated primary nodal T / NK cell lymphoma
ICD coding
  • ICD-10: C84.90 - mature T / NK cell lymphoma, unspecified, unspecified site
  • ICD-11: 2A90 - mature T cell lymphoma, specified types, nodal or systemic
Epidemiology
Sites
Pathophysiology
  • Most of the cases derive from cytotoxic T cell lymphocytes
Etiology
Clinical features
Diagnosis
  • Histological and immunohistochemical examination of lymph node biopsies
Laboratory
Case reports
Treatment
  • Limited data with no standardized treatment
  • No apparent benefit with use of etoposide or anthracycline based chemotherapy approaches
  • Poor response to chemotherapy (Hum Pathol 2015;46:981)
Microscopic (histologic) description
  • Diffuse proliferation of atypical lymphocytes with effacement of the nodal architecture
  • Generally, without conspicuous coagulative necrosis and angioinvasion
  • Neoplastic cells are generally pleomorphic, medium to large and a subset of the tumor cells feature anaplastic morphology
  • References: Am J Surg Pathol 2015;39:462, Hum Pathol 2015;46:981
Microscopic (histologic) images

Contributed by Carlos A. Murga-Zamalloa, M.D.
Diffuse atypical infiltrate

Diffuse atypical infiltrate

Morphological atypia

Morphological atypia

Pleomorphic infiltrate Pleomorphic infiltrate

Pleomorphic infiltrate

Pleomorphism and marked cellular atypia

Pleomorphism and marked cellular atypia

EBV positive lymphocytes (EBER in situ hybridization)

EBV positive lymphocytes (EBER in situ hybridization)

Positive stains
Negative stains
Molecular / cytogenetics description
Sample pathology report
  • Right axillary lymph node, excisional biopsy:
    • Involved by nodal EBV positive NK / T cell lymphoma (see comment)
    • Comment: In situ hybridization for EBV (EBER) shows reactivity in > 30% of the atypical cells.
    • The neoplastic cells feature a cytotoxic immunophenotype (TIA1 / CD8 positive)
    • The overall findings are consistent with EBV+ nodal NK / T cell lymphoma
Differential diagnosis
Additional references
Board review style question #1

The image depicts EBER in situ hybridization in a lymph node. Which of the following is true regarding EBV+ nodal T and NK cell lymphoma?

  1. EBV is positive in background nonneoplastic B cells
  2. Expression of the cytotoxic marker TIA1 and granzyme B is usually absent
  3. Most cases are CD3 / CD4 positive, while CD3 / CD8 positive cases are infrequent
  4. TCR alpha / beta is more common than TCR gamma / delta expression
  5. Usually features a good prognosis with response to anthracycline based chemotherapy
Board review style answer #1
D. TCR alpha / beta is more common than TCR gamma / delta expression. More cases feature TCR alpha / beta expression (~50%) rather than TCR gamma / delta (5%). Answer A is incorrect because in contrast to angioimmunoblastic T cell lymphoma, EBV infection drives the oncogenic process and is positive in the neoplastic T cells and not in the background B cell lymphocytes. Answer B is incorrect because this subtype of T cell lymphoma is characterized by the expansion of cytotoxic T cell lymphocytes with frequent expression of TIA1 and granzyme B. Answer C is incorrect because most cases reported feature neoplastic T cells that are CD3 / CD8 positive; < 5% reported feature CD3 / CD4 positive neoplastic T cells. Answer E is incorrect because patients with a diagnosis of EBV+ nodal T and NK cell lymphoma have poor response to standard chemotherapeutic agents and a median survival of 3 - 8 months.

Comment Here

Reference: EBV+ nodal T and NK cell lymphoma
Board review style question #2

The image depicts EBER in situ hybridization in a lymph node. Which of the following is true regarding EBV+ nodal T / NK cell lymphoma?

  1. By definition, > 80% EBER positivity of the tumor cells are required for the diagnosis of EBV+ nodal T / NK cell lymphoma
  2. Expression of CD30 is detected in most cases
  3. Mutations in the RHOA gene are frequently detected in this lymphoma
  4. There is an overlap in immunophenotypic features with systemic EBV positive T cell lymphoma (SETBL) of childhood; however, EBV+ nodal T / NK lymphoma is predominantly detected in adult populations with primarily nodal disease
  5. This group of T cell lymphomas is clinically characterized by primary orofacial involvement with secondary nodal disease
Board review style answer #2
D. There is an overlap in immunophenotypic features with systemic EBV positive T cell lymphoma (SETBL) of childhood; however, EBV+ nodal T / NK lymphoma is predominantly detected in adult populations with primarily nodal disease. SETBL and EBV+ nodal T / NK lymphoma constitute neoplastic proliferations of cytotoxic T cell lymphocytes; however, SETBL is diagnosed in pediatric populations and EBV+ nodal T / NK lymphoma is predominantly detected in adult populations. Answer A is incorrect because there are no established criteria regarding the percentage of neoplastic T cells for diagnosis; however, many reports included cases with 30% of neoplastic tumor cells positive for EBV by EBER in situ hybridization. Answer B is incorrect because expression of CD30 can be detected in ~20% of the cases and when positive, only a limited number of neoplastic T cells are positive. Answer C is incorrect because mutations in RHOA are predominantly detected in peripheral T cell lymphomas of the T follicular helper type (TFH type), not in EBV+ nodal T / NK lymphomas. Answer E is incorrect because in contrast to extranodal T / NK nasal type lymphoma, EBV+ nodal T / NK lymphoma is primarily nodal without involvement of extranodal sites in the nasal cavity, nasopharynx, oropharynx, Waldeyer ring and epiglottis.

Comment Here

Reference: EBV+ nodal T and NK cell lymphoma

Enteropathy associated T cell lymphoma
Definition / general
  • Intestinal lymphoma derived from intraepithelial T cells which occurs in patients with celiac disease (Blood 2011;118:148)
Essential features
Terminology
  • Enteropathy associated T cell lymphoma type I (obsolete)
  • Intestinal T cell lymphoma
  • Enteropathy type T cell lymphoma (obsolete)
ICD coding
  • ICD-O: 9717/3 - intestinal T cell lymphoma
  • ICD-10: C86.2 - enteropathy type (intestinal) T cell lymphoma
Epidemiology
Sites
Etiology
Pathophysiology
Clinical features
Diagnosis
Laboratory
Radiology description
  • 18F-Fluoro-deoxy-glucose positron emission tomography (18F-FDG-PET) (Gut 2003;52:347)
    • Patients with long standing celiac disease and a standard uptake value > 6 need further investigation for EATL
  • Computed tomography (CT) has a higher accuracy than small bowel enema (Blood 2012;119:2458)
    • Findings that increase the suspicion for EATL
      • Bowel wall thickening
      • Mesenteric lymph node cavitation
      • Intussusception
      • Small sized spleen (< 120 cm3)
    • Video capsule endoscopy (Curr Hematol Malig Rep 2016;11:504)
      • May determine the extent and the precise location of the lesions
      • Useful in cases of intestinal bleeding
Staging
  • Staging procedures as performed in other lymphomas (Blood 2012;119:2458)
    • Bone marrow examination
    • CT of thorax, head and neck
  • Patients usually present with advanced stage of disease (Blood 2011;118:148)
Prognostic factors
  • Poor prognosis (Blood 2011;118:148)
    • Median overall survival (OS): 10 months
    • 5 year overall survival: 20%
  • Prognostic index for peripheral T cell lymphoma (PIT score)
    • Predictive of survival
    • Parameters: lactate dehydrogenase (LDH), age, bone marrow involvement and performance status
  • Factors associated with better overall survival (Dig Liver Dis 2013;45:377):
    • Serum albumin level > 21.6 g/L
    • Chemotherapy treatment
    • Surgical resection
  • Factors associated with worse overall survival (Blood 2011;118:148):
    • Large tumor mass (> 5 cm)
    • Nonambulatory performance status
    • Elevated serum lactate dehydrogenase and C reactive protein (CRP)
Case reports
Treatment
  • Surgery (Blood 2012;119:2458)
    • Debulking and resection of tumor mass
    • Role in complications, mainly in cases of obstruction and perforation
  • Standard dose combination chemotherapy (Blood 2011;118:148, Blood 2012;119:2458)
    • Less than 50% achieve complete remission
    • Common regimes
      • Cyclophosphamide, doxorubicin, vincristine, prednisolone (CHOP) or CHOP-like regimen
        • Patients who do not respond to first line CHOP usually have poor prognosis
      • Bleomycin, doxorubicin, cyclophosphamide, vincristine and prednisone (BACOP)
      • Vincristine, doxorubicin, high dose methotrexate and prednisolone (VAMP)
      • Prednisolone, doxorubicin, cyclophosphamide, etoposide, mechlorethamine, vincristine and procarzabine (ProMACE-MOPP)
      • Cisplatin, cytarabine, etoposide and methylprednisolone (ESHAP)
      • Carmustine, etoposide, cytarabine and melphalan (BEAM)
  • Combination of high dose chemotherapy followed by consolidation with autologous stem cell transplantation consolidation (Blood 2011;118:148, Blood 2013;121:2529)
    • If patient is eligible, may lead to long term remission
  • Radiation therapy is controversial (Blood 2012;119:2458)
  • Novel treatments (Blood 2012;119:2458)
    • Alemtuzumab
    • Cladribine
    • Romidepsin
Gross description
Microscopic (histologic) description
Microscopic (histologic) images

Contributed by Roberto N. Miranda, M.D.
Ileocecal ulcer

Ileocecal ulcer

Ileocecal infiltration

Ileocecal infiltration

Angiocentric and angioinvasive lesion

Angiocentric and angioinvasive lesion

Mucosal and  submucosal infiltration

Mucosal and submucosal infiltration


Cytologic atypia

Cytologic atypia

Intraepithelial lymphocytes

Intraepithelial lymphocytes

Sinusoidal infiltration

Sinusoidal infiltration

Sinusoidal infiltration

Sinusoidal CD30 positivity


CD3 positivity

CD3 positivity

CD30 positivity

CD30 positivity

CD4 negativity

CD4 negativity

CD8 negativity

CD8 negativity

TIA1 positivity

TIA1 positivity

Positive stains
Molecular / cytogenetics description
Sample pathology report
  • Small intestine, jejunum biopsies:
    • Enteropathy associated T cell lymphoma (see comment)
    • Comment:
      • According to clinical notes, the patient has a previous history of celiac disease diagnosed 7 years ago and recently started with anemia, weight loss and diarrhea. Endoscopic examination showed multiple and active ulcers raising mucosal masses in jejunum and mosaic pattern on the adjacent mucosae. A computed tomography realized on the same day showed enlarged mesenteric lymph nodes.
      • Multiple biopsies from the jejunum are reviewed. Hematoxylin and eosin sections show an ulcerated lesion and dense infiltration in mucosae and submucosae by atypical lymphocytes. The abnormal lymphocytes are of medium to large size, with prominent nucleoli and moderate to abundant cytoplasm and surrounded by a background with occasional histiocytes, eosinophils and neutrophils. The mucosa adjacent to the atypical infiltrate showed numerous small intraepithelial lymphocytes. Immunohistochemical studies demonstrated that the neoplastic cells are positive for CD3, CD7, TIA1 and TCR-β. The neoplastic cells are negative for CD4, CD5, CD8, CD20, CD56, PAX5, TCRγδ and EBER. The adjacent intraepithelial lymphocytes show a similar reactivity to the lymphoma cells.
      • Concurrent polymerase chain reaction for T cell receptor gamma chain demonstrated monoclonal gene rearrangement.
      • In summary, the clinicopathologic findings support a diagnosis of enteropathy associated T cell lymphoma in a patient with celiac disease.
Differential diagnosis
Board review style question #1
Which of the following is true about enteropathy associated T cell lymphoma (EATL)?

  1. It does not associate with celiac disease
  2. Large and unifocal mass is the most common endoscopic finding
  3. There is an association with HLA-DQ2.2, HLA-DQ5 and HLA-DQ8
  4. Usually presents with peripheral lymphadenopathy
Board review style answer #1
C. There is an association with HLA-DQ2.2, HLA-DQ5 and HLA-DQ8. EATL is characteristically associated with celiac disease and in particular persons with major histocompatibility complex expressing HLA-DQ2.2, HLA-DQ5 and HLA-DQ8. Generalized lymphadenopathy is not a feature of intestinal lymphomas. Endoscopically, multiple small ulcerated lesions are noted.

Comment Here

Reference: Enteropathy associated T cell lymphoma (EATL)
Board review style question #2

What is the most consistent immunophenotype of enteropathy associated T cell lymphoma (EATL)?

  1. CD3+, CD4+, CD5+, CD7+, CD8-, EBER+, TCR-β-
  2. CD3+, CD4-, CD5-, CD7+, CD8-, EBER-, TCR-β-
  3. CD3+, CD4-, CD5+, CD7-, CD8-, EBER+, TCR-β+
  4. CD3+, CD4+, CD5-, CD7+, CD8-, EBER-, TCR-β-
Board review style answer #2
B. CD3+, CD4-, CD5-, CD7+, CD8-, EBER-, TCR-β-. EATL is a T cell lymphoma and thus expresses CD3, usually lacks CD4 and CD8 as well as TCR-β and is negative for Epstein-Barr virus.

Comment Here

Reference: Enteropathy associated T cell lymphoma (EATL)

Epstein-Barr virus related lymphoid proliferations and lymphomas
Definition / general
  • EBV is the human herpesvirus 4 (HHV4) and belongs to the Herpesviridae family
  • There are 4 patterns of EBV latency, defined based on the pattern of EBV proteins and small RNAs expressed in in vitro infections and in EBV induced tumors
Essential features
  • Epstein-Barr virus (EBV) is the most common viral infection in humans, with 95% of the worldwide population showing an asymptomatic lifelong carrier stage state (Am J Clin Pathol 2023;159:14)
  • Most EBV infections are acquired in childhood and are asymptomatic
  • Patients who acquire the infection in the second decade usually develop the clinical features of infectious mononucleosis (IM), a self limited disease
  • EBV plays an important role in the oncogenesis of a wide spectrum of neoplasms including solid tumors, B and T cell lymphomas, as well as classic Hodgkin lymphoma (CHL)
Microbiology
  • EBV is the human herpesvirus 4 (HHV4) and belongs to the Herpesviridae family
  • There are 2 subtypes: EBV1 and EBV2
    • Distinguished by their sequences of the EBV nuclear antigens (EBNA) and the EBV encoded RNAs (EBER) (Viruses 2023;15:714)
    • Subtype 1 is the more prevalent worldwide and has greater transforming potential (Viruses 2023;15:714)
  • EBV is restricted to primate hosts and has B cell lymphotropism
    • Can also infect epithelial cells where it causes growth cell arrest
  • EBV can transform in vitro B lymphocytes to lymphoblastoid cell lines (LCLs) (Viruses 2023;15:714)
  • EBV has a double stranded linear DNA genome containing ~100 genes surrounded by a protein capsid
    • 172 kb in length
    • Protein capsid contains glycoproteins important for cell tropism and receptor recognition
    • Membrane antigen (MA) consists of 3 glycoproteins on the viral envelope (gp350, gp250, gp85) that mediate cell binding of virus
    • Major viral envelope glycoprotein gp350 / 220 binds to complement receptor 2 (CD21) cell receptor on B lymphocytes
    • Glycoprotein gp42 is essential for penetration of B cells by forming a complex with the histocompatibility complex (MHC) II
EBV latent nuclear and membrane antigens functions
  • 6 EBV nuclear antigens (EBNA 1 - 6) are expressed in infected cells
    • EBNA1
      • Only known EBV protein to be consistently expressed in all EBV related cancers
      • Main function is to ensure EBV double stranded DNA replication as cells multiply
      • Avoids cytotoxic T lymphocytes surveillance
      • Enhances cell survival, inhibits apoptosis
    • EBNA2
      • Transcriptional activation of viral EBNA and latent membrane proteins (LMP) as well as of cellular genes such as MYC
      • Rewires host gene regulatory programs in multiple autoimmune diseases
      • Functionally replaces the intracellular region of Notch receptor
    • EBNA LP
      • Assists EBNA2 mediated transcriptional activation
      • Suppresses the innate cell response to viral DNA
    • EBNA3A and EBNA3C
      • Required for efficient transformation of B cells in vitro
      • Mediate inhibition of various cyclin dependent kinase inhibitors (Bim, BLIMP1 and p16 / p15 / p18 expressions)
      • Epigenetically control gene transcriptions
    • EBNA3B
      • Functions as a tumor suppressor
  • EBV has 3 LMP that are expressed in infected cells and are involved in immortalization
    • LMP1
      • Main transforming antigen of EBV
      • Mimics CD40 signaling in B cells
      • Drives growth and differentiation
      • Increases cytokine production (IL6, IL8)
      • Prevents p53 mediated apoptosis and upregulates BCL2
      • Escape from immune surveillance via PI3K / AKT / mTOR pathway
    • LMP2A
      • Not essential for EBV induced transformation in vitro
      • Drives proliferation and survival of B cells mimicking BCR (B cell receptor) signaling
    • LMP2B
      • Modulates the effects of LMP2A on BCR function
  • EBV has 2 small noncoding RNAs (EBER1 and EBER2) and BamH1A region transcripts (BARTs, also known as miRNAs)
    • EBERs can increase tumorigenicity, promote cell survival and induce interleukin 10 (IL10) expression in Burkitt lymphoma cell lines (Adv Cancer Res 2010;107:119)
    • EBV is clonal in EBV associated neoplasms
      • Viral clonality can be demonstrated by analysis of restriction enzyme digested EBV DNA terminal repeat fragments
      • Detection of clonal EBV episomes in tumor suggests that malignant cells are also clonal
  • See Table 1
  • References: Viruses 2023;15:714, Pathology 2020;52:154
Latency programs
  • There are 4 patterns of EBV latency, defined based on the pattern of EBV proteins and small RNAs expressed in in vitro infections and in EBV induced tumors (Viruses 2023;15:714)
  • Expression of EBER is a constant feature of all EBV infected cells and therefore, is the most consistent marker to demonstrate EBV infection (see Table 1)
  • EBV latency within B cells usually starts with type III in primary infection, to latency II while selecting pressure locally and type I that allows virus survival without expression of other viral genes (Pathology 2020;52:40)
    • Type III: during initial infection of a resting naive B cell or in the context of immunodeficiency (Pathology 2020;52:40)
    • Type II: under the selective pressure of an effective cell mediated immune response, the virus downregulates expression of several viral proteins (mostly EBNA2) entering into an intermediate stage; infected B cells differentiate into memory B cells (Pathology 2020;52:40, Semin Diagn Pathol 2020;37:32)
    • Type I: restricted to the expression of EBER and EBNA 1 and is responsible for the maintenance and replication of the episomal EBV genome (Pathology 2020;52:40)
    • Type 0: viral antigen expression is maximally suppressed and memory B cells are its main reservoir (Pathology 2020;52:40, Semin Diagn Pathol 2020;37:32)
  • Latency III expresses all latent genes and is seen in (see Table 2)
    • EBV genome within LCLs
    • Transiently seen in the normal course of EBV infection in healthy individuals
    • Infectious mononucleosis
    • Lymphoid proliferations and lymphomas associated with immune deficiency and dysregulation
    • Primary central nervous system (CNS) lymphoma (human immunodeficiency virus [HIV] associated)
    • Diffuse large B cell lymphoma (DLBCL) associated with chronic inflammation
    • EBV positive breast implant associated large B cell lymphoma (Mod Pathol 2021;34:2154)
  • Latency II (also referred to as the default program), in which LMPs are expressed
    • Hodgkin lymphoma
    • EBV positive DLBCL
    • Lymphomatoid granulomatosis
    • Angioimmunoblastic T cell lymphoma
    • Extranodal NK / T cell lymphoma, nasal type
    • Aggressive NK cell leukemia
    • EBV associated T and NK cell lymphomas
  • Latency I is described as a program where EBNA1 is the only viral protein expressed
    • Immunocompetent / immunocompromised Burkitt lymphoma
    • Primary effusion lymphoma (PEL)
    • Plasmablastic lymphoma (PBL), oral type, in immunodeficient HIV patients
  • Latency 0, in which none of the EBV antigens are expressed and immune recognition is avoided; typically observed in circulating memory B cells in healthy individuals
  • See Table 3
Pathophysiology
  • Target cells: oropharyngeal epithelium and B lymphocytes (Viruses 2023;15:714)
    • B cells are the major reservoir of the virus before epithelial amplification in the oral cavity
    • Oropharyngeal epithelial cells are permissive for viral replication
    • EBV can shuttle between different cell types under certain circumstances: T cells, NK cells, monocytes, smooth muscle cells, follicular dendritic cells (FDC) and neurons
  • First cell type to get infected is debated (Viruses 2023;15:714)
    • Epithelial cells theory: the virus replicates and releases virions which then infect the colocalized B cells in the lymphoepithelial structures of the Waldeyer ring
    • B cells theory: the virus enters tonsillar crypts, crosses the thin layer of epithelium and infects naive B cells residing in the follicular mantle
  • Entry of EBV is through endocytosis (Viruses 2022;14:2372)
    • B cells
      • Viral surface glycoprotein gp350 / 220 attaches to CD21 / CD35 on the surface of B cells (Viruses 2022;14:2372)
      • Second glycoprotein, gp42, binds to HLA class II molecules, triggers conformational changes and causes fusion of both membranes (Viruses 2022;14:2372)
    • Epithelial cells
      • BMRF2 glycoprotein facilitates EBV attachment through binding with β1, α5 and α3 integrins (Viruses 2022;14:2372)
      • BMRF2 may also function in the cell to cell transmission of EBV (Viruses 2022;14:2372)
  • Initial replication occurs within oropharyngeal mucosal epithelium and spreads through infection of B cells in the lymphoid tissues in Waldeyer ring
  • Infection of tonsillar naive B cells leads to a latency III program, with a full spectrum of latent proteins expressed
  • Most of the proliferating cells are eliminated by NK cells and the emerging latent antigen specific primary T cell response (lytic phase)
    • Some studies suggested that the infection of T and NK cells could occur through the immunological synapse, in an attempt by these cells to kill the EBV infected B cells (Front Oncol 2023;13:1240359)
    • Molecule of HLA class II present on NK cells may interact with glycoproteins gp42 and gp85 from EBV, fundamental in the internalization (Front Oncol 2023;13:1240359)
  • Some infected cells escape from immune surveillance by downregulating antigen expression and undergoing germinal center (GC) reaction, where a more limited set of viral genes are expressed in what constitutes the latency II program
  • Stable reservoir of resting viral genome positive memory B cells is established when these EBV infected GC B cells migrate to peripheral blood, where viral antigen expression is silenced (latency 0)
  • When EBNA1 is expressed intermittently during the division of these memory B cells, the viral genome is distributed to the daughter memory B cells (latency 1)
  • During latent phase, EBV can persist in the host cells by replicating extrachromosomal nucleic acids (episomes)
  • EBV maintains latency in memory B cell pool via expression of 2 EBV encoded small RNAs (EBER1 and EBER2), EBV nuclear antigens (EBNAs) and 3 LMPs
    • EBNA1 maintains episomal state of EBV DNA in infected cells
    • Protein EBNA2 blocks B lymphocyte differentiation and allows cell proliferation
    • LMPs are membrane spanning proteins that function as constitutive active receptors to sustain B cell activation and differentiation independently of ligand binding
      • LMP1 is a major oncogene that functions as a constitutively active member of tumor necrosis factor receptor family
  • CD8 positive cytotoxic T cells and NK cells kill infected B cells
  • Virus is periodically reactivated and shed in saliva
Oncogenic mechanisms
  • Oncogenes lead inhibition of cell apoptosis and host cell cycle disturbances that promote G1 / S phase transition
  • EBV genes activate oncogenes such as MYC and BCL2 and signaling pathways such as JNK, NFκB, PI3K / Akt and JAK / STAT
  • Virus also inhibits tumor suppressor genes such as TP53, DOK1, PRDM1, PKR, PTEN, DICE1 and others
  • p21cip1 and p27kip1 is downregulated by EBERs, while BCL2 is upregulated and thereby inhibits the release of CDK4 and CDK2 and promotes the cell cycle from G1 phase to S phase
  • Histone modification, DNA methylation, noncoding RNA expression and chromatin remodeling
  • Reference: Microb Pathog 2023;183:106292
Laboratory diagnosis
  • Serology (Viruses 2023;15:656)
    • Method of choice for unequivocal diagnosis
    • Mainly includes the exploration of 3 or 4 markers using automated immunoassays
      • Antiviral capsid antigen (VCA) or total anti-EBV immunoglobulin M (IgM)
      • Anti-VCA
      • Anti-early antigen (EA) or total anti-EBV immunoglobulin G (IgG)
      • Anti-Epstein-Barr nuclear antigen 1 (EBNA1) IgG
    • Combination of these markers can differentiate a primary from a past infection
      • Antibodies to EA IgG generally occur in acute phase of illness
        • VCA IgM appears with onset of symptoms and usually persists for 2 - 3 months
          • With or without anti-VCA IgG but always without anti-EBNA1 IgG
        • VCA IgG appears with onset of symptoms and is positive for life
          • May be absent at the very start of symptoms
      • Antibodies to EBNA1 IgG and EBNA2 IgG
        • EBNA1 IgG appears after 2 - 3 months
        • 3 - 5% of individuals and 10 - 20% of immunocompromised patients, anti-EBNA1 IgG is not detected after primary infection
          • Not related to specific symptoms or risks
        • EBNA2 IgG can appear early in 30% of patients in course of disease
      • Past infection is defined by the presence of anti-VCA IgG and anti-EBNA1 IgG without anti-VCA IgM
      • Antibodies to VCA IgG, VCA IgM and EBNA1 IgG can be used to distinguish acute and past infections in immunocompetent patients
        • VCA IgG and VCA IgM in absence of EBNA1 IgG indicates acute infection
        • VCA IgM antibodies decrease and VCA IgG antibodies increase during convalescence
        • VCA IgG and EBNA1 IgG in absence of VCA IgM indicate past infection
    • Rapid antibody responses to EBV are correlated with a reduced severity of primary infection
    • Serology is less useful in immunocompromised patients who may have atypical antibody profiles
  • Peripheral blood smear
    • Patients with IM can present with atypical lymphocytes (Downey cells)
      • 3 types (Lancet 2020;395:225)
        • Type I: slightly larger than normal lymphocytes, with indented or so called bean shaped nuclei, reduced cytoplasm with few azurophilic granules and vacuoles
        • Type II: larger with abundant pale cytoplasm and often abutting or partly surrounding adjacent red blood cells
        • Type III: also large but they have one or more prominent nucleoli and abundant, deeply basophilic cytoplasm
      • They represent activated CD8 T lymphocytes responding to EBV infected cells (Curr Top Microbiol Immunol 2015;390:211)
  • Heterophile antibodies (Curr Top Microbiol Immunol 2015;390:211)
    • Not specific
    • Heterophile test detects IgM class antibodies that agglutinate mammalian erythrocytes
    • Peak 2 - 5 weeks after onset of clinical symptoms and are useful in diagnosis of primary EBV infection
    • Heterophile antibodies have sensitivity of 63 - 84% and specificity of 84 - 100%
      • 25% false negative rate in first week of illness and in young children
      • False positive tests may occur with malignancies, autoimmune disease, other infections
  • Polymerase chain reaction (Methods Mol Biol 2017;1532:33)
    • Mainly used to monitor EBV viral loads in specific clinical situations
      • Diagnosis and monitoring antiviral therapy in patients with posttransplant lymphoproliferative disorder (PTLD)
      • Immunocompromised patients who may have defective antibody production
        • Because serological reactivation does not correlate with viral load in this patient group, EBV serological testing is not useful at all
      • Young children with heterophile negative IM
    • Most used EBV gene targets in these assays are BALF5 gene (coding for thymidine kinase), BamHIW region, EBNA1 gene, EBER1 gene
    • Whole blood is the specimen of choice
      • EBV DNA is located within memory B cells
      • Cerebrospinal fluid can be tested by PCR to diagnose EBV infection of central nervous system
  • See Table 2
EBV associated reactive B cell proliferations
Infectious mononucleosis
  • Acute infection that usually follows primary infection
  • Fatigue, fever, exudative pharyngitis, anterior and posterior cervical lymphadenopathy
  • Infection is usually self limited
  • Microscopic features
    • Histologic changes are variable and depend on disease duration
      • Early stage: reactive follicles, with mild paracortical expansion
      • Progression: follicles sparse due to interfollicular expansion
      • Advanced stage: follicles may become effaced, with interfollicular expansion
        • Interfollicular areas with numerous immunoblasts or mixed infiltrate
        • Immunoblasts can be binucleated, mimicking Hodgkin / Reed-Sternberg (HRS) cells
      • Focal necrosis may be present
    • Immunohistochemistry
      • HRS-like cells are CD45 positive, CD15 negative, CD30 (weak positive) admixed with B and T immunoblasts
      • EBV latency type III
      • EBV encoded RNA (EBER) present in numerous infected cells, ranging from small and intermediate lymphocytes to HRS-like immunoblasts
      • EBNA1 positive, EBNA2 variable and EBNA3 positive
      • LMP1 positive and LMP2 positive
      • Most T lymphocytes in the background are CD8+
  • Reference: Semin Diagn Pathol 2018;35:54
EBV associated B cell lymphoproliferative disorders
EBV positive mucocutaneous ulcer
  • Occurs in the context of attenuated immunosurveillance, such as the use of iatrogenic immunosuppression, acquired immunodeficiency, primary immunodeficiency, posttransplant setting or age associated immunodeficiency
  • Isolated, self limited, shallow, well circumscribed lesion in the oropharyngeal mucosa, skin or gastrointestinal tract
    • Presence of ≥ 2 lesions is best classified as EBV+ polymorphic B cell lymphoproliferative disorder (LPD) (nodal and extranodal, recently proposed entity) or EBV+ DLBCL
  • No lymphadenopathy, organomegaly or bone marrow infiltration, no lactate dehydrogenase (LDH) elevation
  • EBV DNA in blood is not increased
    • Useful criterion in the differential diagnosis with EBV positive DLBCL, NOS
  • Most cases regress spontaneously or with reduction of immunosuppression
  • Microscopic features
    • Sharply circumscribed ulcer; skin or mucosal sites
    • Atypical intermediate size or large lymphoid cells
      • HRS-like cells can be present
      • Large cells can be numerous (large cell lymphoma-like)
    • Background shows variable number and proportion of small lymphocytes, histiocytes, plasma cells and eosinophils
    • Base of lesion is demarcated by rim of small T lymphocytes
    • Immunophenotype
      • Large lymphoid cells are B cells
        • Non-germinal center phenotype: CD20 positive, PAX5 positive, OCT2 positive, MUM1 / IRF4 positive, CD10 negative
        • CD30 positive, CD15 (~50%)
      • Rim of CD8+ lymphocytes at base of lesion
    • EBV evaluation
      • EBER might be positive mainly in the large HRS-like cells but it is frequently observed in a wide range of background cells, including small lymphocytes
      • LMP1 positive and EBNA2 positive (50% of cases)
      • Small lymphocytes within the lesion are T cells; increased CD8 / CD4
  • References: Pathology 2020;52:40, Hum Pathol 2018;79:18

Lymphoproliferative disorders associated with immune deficiency and dysregulation
  • Hyperplasias arising in immune deficiency / dysregulation (WHO terminology): mass lesion with preserved tissue architecture (nondestructive)
    • Typically lack monoclonal Ig gene rearrangements
  • Polymorphic lymphoproliferative disorders arising in immune deficiency / dysregulation (WHO terminology): mass lesion with effaced / destroyed tissue architecture arising after transplant
    • Monoclonal or oligoclonal Ig gene rearrangements in ~60% of cases
    • Cytogenetic abnormalities in ~33% of cases
  • Both types are due to impaired host immunosurveillance
    • Posttransplant setting
      • Most common scenario for polymorphic LPDs
    • Cancer therapy associated
    • Autoimmune diseases
    • HIV setting
  • Almost always associated with EBV infection
    • Latency type III pattern: EBERs positive, LMPs positive, all EBV nuclear antigens positive
    • Follicular hyperplasia type: EBER positive in 40 - 50%
      • EBER is often localized to reactive GCs
  • Microscopic features
    • 3 types of nondestructive lesions are recognized
      • Plasmacytic hyperplasia: medullary and interfollicular plasma cells, lymphocytes
      • IM-like hyperplasia: paracortical expansion by CD30 positive immunoblasts
      • Follicular hyperplasia: widely spaced, reactive follicles, usually reflect seroconversion from EBV negative to EBV positive
        • Germinal centers of hyperplastic lymphoid follicles may contain numerous cells positive for EBER
    • Polymorphic LPDs
      • Effacement by variable size lymphocytes, immunoblasts, histiocytes and plasma cells
        • Reed-Sternberg and Hodgkin-like cells are common
      • These lesions do not meet criteria for lymphoma
        • There are no sheets of large cells
  • See Table 6
  • References: Pathology 2020;52:40, Hum Pathol 2018;79:18, Am J Surg Pathol 2018;42:116, Laryngoscope 2011;121:1718
EBV positive T / NK lymphoproliferative disorders of childhood
Hydroa vacciniforme lymphoproliferative disorder
  • Chronic EBV+ LPD of childhood with a broad spectrum of clinical aggressiveness and usually a long clinical course
  • Pathogenesis is unknown; however, genetic predisposition might play a role (Blood 2019;133:2753)
    • Classic
      • Indolent, self limited, more common in White people
      • Localized papulovesicular eruptions on sun exposed skin
      • No systemic symptoms; rarely progresses to a more aggressive form (systemic)
    • Systemic
      • More common in Asia and Latin America
      • Skin lesions in sun exposed and nonexposed areas
      • Mild to severe disease
      • Systemic symptoms: fever, lymphadenopathy, liver involvement
      • Patients might respond initially to immunomodulating therapies but eventually will require treatment similar to chronic active EBV (CAEBV) disease
  • EBV DNA is elevated in the blood and the levels are not discriminatory between the 2 forms
  • Most cases have clonal TCR gene rearrangements
  • Microscopic features
    • Intraepidermal spongiotic vesiculation
    • Lymphoid infiltrate predominantly in the dermis but may extend into the subcutaneous tissue
    • Mainly perivascular and periadnexal, often with angiodestructive features
    • Epidermotropism (exocytosis) and ulcers are common
    • Small neoplastic cells without atypia
    • Immunophenotype
      • Mostly CD8 positive, some cases are CD4 positive; rarely CD4+ / CD8+
      • Occasional cases have an NK cell phenotype; tend to be more panniculitis-like
        • Might mimic subcutaneous panniculitis-like T cell lymphoma (SPTCL)
      • TIA1 positive, granzyme B positive, perforin positive; CD30 is often expressed
      • EBER is positive in a variable number of cells; LMP1 is negative
  • Reference: Virchows Arch 2023;482:227


Severe mosquito bite allergy
  • Rare disease; exaggerated reaction to mosquito bites in children
  • Localized, clear or hemorrhagic bullae in mosquito bitten areas
    • Cases with systemic manifestations with organ infiltration should be classified as CAEBV disease
    • Patients have an increased risk of developing hemophagocytic syndrome and progression to an overt NK / T cell lymphoma
  • Symptomatology comes and goes every time a mosquito bites
  • Microscopic features
    • Tends to be panniculitis-like
    • Features of angiodestruction are usually seen
    • Most of the infiltrating cells have an NK cell phenotype
    • EBV evaluation
      • Association with decreased CD4 positive cells, high serum IgE titers and high EBV DNA copy number
      • EBV positive NK cell lymphocytosis in peripheral blood
      • LMP1 is rarely positive by IHC in tissue but can be detected by PCR in peripheral blood (latency II)
      • EBER is positive in a fraction of the NK cells
        • Much higher density of EBV positive cells should raise suspicion of NK cell lymphoma / leukemia
  • Reference: Hum Pathol 2018;79:18


Chronic active EBV disease (T and NK cell phenotype)
  • CAEBV disease refers only to T and NK cell lineage disease
    • Excludes cases of B cell lineage that usually arise in patients with primary immunodeficiency
  • Absence of underlying immunosuppression
  • More often in children but also in young adults
  • IM-like illness persistent for > 3 months
    • Fever, lymphadenopathy, hepatosplenomegaly, anemia, skin rash, diarrhea and uveitis
    • Rare manifestations: myocarditis, coronary aneurism, interstitial pneumonia, digestive tract ulcer / perforation, CNS involvement
    • Symptoms range from mild to severe
  • Protracted clinical course
    • Patients with EBV infected T cells have a shorter survival time than patients with NK cell disease, prominent systemic symptoms and high titers of EBV DNA in blood
    • Increased risk of progression to EBV+ T or NK cell lymphoma
  • Microscopic features
    • Small lymphocytic inflammatory changes without evidence of malignant lymphoproliferation
    • Diagnosis is usually made in liver or a lymph node biopsy
      • Liver: sinusoidal and portal infiltration suggestive of viral hepatitis
      • Lymph nodes: either follicular or paracortical hyperplasia, focal necrosis or small granulomas
    • Ancillary testing
      • Lineage: T cells (60%) or NK cells (40%)
      • CD4 positive > CD8 positive > γ δ T cells
      • EBER is positive
      • CD56 is expressed in 40% of cases
      • Monoclonal TCR (40 - 63%); monoclonal EBV (84%)
      • Somatic mutations in DDX3D and KMT2D have been demonstrated
  • References: Am J Clin Pathol 2023;159:14, Virchows Arch 2023;482:227
Lymphomas where EBV+ is essential for diagnosis
EBV positive diffuse large B cell lymphoma, NOS
  • No history of immunodeficiency or previous lymphoma
    • Preferential distribution among elderly patients might be related to physiological immunosenescence
  • Extranodal mass with or without lymphadenopathy in ~70% of cases
    • Only lymphadenopathy in ~30% of cases
  • Microscopic features
    • 2 types, no clinical or prognostic relevance
      • Polymorphous
        • Most frequent
        • HRS-like and LP-like cells often present, scattered in a reactive background with small lymphocytes, histiocytes and plasma cells
      • Monomorphous (large cell lymphoma)
        • Large neoplastic cells with immunoblastic or centroblastic morphology, without a polymorphous inflammatory background
      • Areas of necrosis; often geographic pattern
      • Angiocentric / angiodestructive lesions can be present
    • Immunophenotype
      • CD20 positive, CD22 positive, CD79a positive, PAX5 positive
      • 90% of cases are CD10 negative and BCL6 negative
      • CD30 expression associated with a poorer prognosis in elderly patients but not in young patients; only a few are CD15 positive
      • IRF4 / MUM1 positive
    • EBV evaluation
      • EBV DNA load may have prognostic significance in DLBCL patients regardless of tumor EBV status (Viruses 2023;15:656)
        • Supports the hypothesis that EBV could disappear from the tumor after contributing to lymphomagenesis in an earlier phase of cancer development
        • It has been suggested that high levels of pretreatment EBV DNA in whole blood (> 30,800 copies/mL) are associated with a worse prognosis (Viruses 2023;15:656)
      • EBER positive: no defined cutoff; WHO classification suggests a cutoff of 80%
        • EBER highlights more positive cells than LMP1
      • Lymphoma cells usually have EBV type II latency pattern
        • EBNA2 is expressed only in some cases (14%)
  • References: Pathology 2020;52:40, Viruses 2023;15:656


Diffuse large B cell lymphoma with chronic inflammation
  • Rare; associated with hypoxic microenvironment
  • Mass producing DLBCL that occurs in a setting of longstanding chronic inflammation
  • Mostly involves body cavities or narrow spaces
  • Presents with large tumor mass
  • Reference: Pathology 2020;52:40


Fibrin associated diffuse large B cell lymphoma
  • Arises from fibrinous material in wall of pseudocysts
    • It has been suggested that EBV positive DLBCL associated with breast implants is related with this subset of neoplasms
    • This is a controversial concept since fibrin is not a known pathogenic mechanism
    • Minute tumor burden; associated with hypoxic microenvironment
  • Consistently associated with EBV
  • Does not produce mass
  • Does not directly produce symptoms
  • Discovered incidentally on histologic examination of surgical pathology specimen
  • Highly favorable clinical outcome with complete surgical excision
    • Chemotherapy for DLBCL is not required
  • Reference: Pathology 2020;52:40


Lymphomatoid granulomatosis
  • Extranodal, predominantly pulmonary angiocentric or angiodestructive lymphoproliferative disorder
  • Lung is the most frequent site of involvement
    • Multiple bilateral pulmonary nodules (most frequent)
    • Other sites of involvement: skin and central nervous system
  • Microscopic features
    • Neoplastic B cells are EBV positive
      • Latency III: LMP1 positive, EBNA2 positive
    • Many admixed T cells, histiocytes, plasma cells and variable number of immunoblasts
    • Histologic grade based on number of EBV positive large B cells and extent of necrosis
      • < 5 EBV positive B cells per high power field (HPF) in grade 1
      • 5 - 50 EBV positive cells per HPF in grade 2
      • > 50 EBV positive cells per HPF in grade 3; this latter criterion may not be fulfilled when tumor is extensively necrotic and reactivity for EBER is decreased
      • Cases with sheets of EBV positive large B cells should be classified as EBV+ DLBCL
    • Ancillary tests
      • Large cells are B cells
      • CD45RB / LCA positive, EBER positive, CD30 variable, CD15 negative
      • Smaller reactive cells are T cells: CD4 > CD8
      • Monoclonal IGH rearrangements in grade 2 (50%) and grade 3 (70%) of lymphomatoid granulomatosis cases
  • References: Hum Pathol 2018;79:18, Pathology 2020;52:40
Lymphomas where EBV+ is not essential for diagnosis
Plasmablastic lymphoma
  • Diffuse proliferation of large neoplastic cells with plasmablastic cytologic and immunophenotypic features
  • Generally associated with immunodeficiency, a subset occurs in immunocompetent patients
  • Predominantly extranodal disease
  • Aggressive clinical course, poor prognosis
  • Microscopic features
    • Large neoplastic cells with a variable degree of immunoblastic or plasmablastic features
    • Starry sky pattern can be seen
    • High mitotic and apoptotic rates
    • Ancillary tests
      • Negative for B cell markers (CD20, CD79a, PAX5) and CD45
      • CD138 positive, MUM1 / IRF4 positive, CD38 positive
      • EBER positive in ~70% of cases
      • High proliferation rate
      • MYC rearrangements in 40 - 50% of cases
  • References: Pathology 2020;52:40, Hum Pathol 2018;79:18


Burkitt lymphoma
  • Highly aggressive B cell neoplasm characterized by monomorphic, intermediate size cells with basophilic cytoplasm
    • Germinal center B cell immunophenotype with MYC overexpression
      • CD20 positive, CD10 positive, BCL6 positive, BCL2 negative, LMO2 negative and Ki67 proliferation rate ~100%
    • MYC translocation partnered with immunoglobulin (Ig) gene loci
      • IGH: 80 - 85%, IGL: 10%, IGK: 5%
  • Infection by Epstein-Barr virus (EBV) in a subset of cases
    • EBER positive, EBV LMP1 negative, EBNA1 positive, consistent with latency type 1
    • Association with epidemiologic variants
      • > 95% of cases of endemic Burkitt lymphoma
      • ~20 - 30% of cases of sporadic Burkitt lymphoma
      • ~30 - 40% of cases of immunodeficiency associated Burkitt lymphoma
  • Recent studies recommend dividing Burkitt lymphoma into 2 groups: EBV positive and EBV negative, regardless of epidemiologic context and geographic location
    • EBV positive Burkitt lymphoma shows significantly higher levels of somatic hypermutation and harbors fewer driver mutations
  • References: Pathology 2020;52:40, Leukemia 2022;36:1720


Acquired immunodeficiency syndrome (AIDS) related diffuse large B cell lymphoma
  • Most common lymphoma arising in the context of HIV infection
    • No correlation between viral load and occurrence of lymphoma
  • Mostly nodal disease, gastrointestinal tract is the most common extranodal site of involvement (19%)
  • EBER is positive in ~40% of the cases
  • Lymphomagenesis is induced by multiple mechanisms
    • Genetic alterations
    • Chronic B cell activation by immune dysfunction
    • Loss of immunoregulatory control of oncogenic herpesviruses (EBV, HHV8)
  • Similar morphological, immunophenotypic and molecular features to HIV negative cases
    • Usually has a non-germinal center immunophenotype, may express CD30
    • Monoclonal IGH rearrangements, frequent somatic hypermutation of IGH variable regions
  • May present as primary central nervous system lymphoma, usually with immunoblastic morphology
    • Long history and advanced stage of AIDS
    • Very low CD4 positive count
    • Immunoblastic morphology common
    • EBV positive in 80 - 100% of cases
  • Reference: Pathology 2020;52:40


Classic Hodgkin lymphoma
  • Overall EBV positivity rate is 30 - 50% in the U.S. and Europe but ~100% in Vietnam, Kenya and Honduras
    • More frequent in childhood (< 10 years) and in older adults (> 60 years)
      • Possibly represents 2 distinct phenomena, one related to earlier age of EBV infection and the other to the decline in immune function
    • Also correlates with lower socioeconomic status
  • EBV is detectable only in HRS cells and absent in background cells
    • EBV infected HRS cells show latency pattern II
      • Positive for EBERs, LMP1, LMP2A and EBNA1
  • Epidemiologically, there are 3 different incidence patterns
    • Pattern 1: seen in developing nations and in patients of low socioeconomic status, there is an early childhood peak in incidence with tumors that are predominantly mixed cellularity (MC) subtype and EBV positive
    • Pattern 2: seen in countries with transitional economies, has peaks of incidence in childhood and a second decade peak and shows a relatively more equal frequency of the MC and nodular sclerosis (NS) subtypes
    • Pattern 3: seen in developed countries and in patients of high socioeconomic status, shows a peak incidence in the third decade with tumors that are mainly NS subtypes and EBV negative
  • Association with EBV is also variable among pathological subtypes (Pathology 2020;52:154)
    • Nodular sclerosis: 10 - 25%
    • Mixed cellularity: 72 - 85%
    • Lymphocyte rich: 57%
    • Lymphocyte depleted: 75%
  • It has been proposed that EBV rescues crippled germinal center B cells from apoptosis
    • EBV provides the necessary signals required for cellular growth, survival and cellular genetic alterations, thus generating a pool of cells that may become progenitors of HRS cells
    • Full contribution of EBV to CHL molecular pathogenesis remains to be fully elucidated
  • Association of EBV and CHL in the setting of iatrogenic immunodeficiency has been identified in up to 80% of cases
  • References: Pathology 2020;52:154, Viruses 2023;15:656


Primary effusion lymphoma
  • Arises from HHV8 infected B cells with plasmablastic differentiation
    • HHV8 / KSHV (Kaposi sarcoma herpes virus) positive is essential for diagnosis
    • Pan B cell markers are negative
    • EBER positive in ~70% of cases
  • Diagnosis is usually established on cytologic preparations of effusion fluid
    • Biopsy specimens may show neoplastic cells adherent to mesothelial surfaces
  • EBV is constant in setting of HIV infection
    • Latency pattern I
    • Potential role in promoting establishment of latency in HHV8 by inhibiting its lytic replication
  • See Table 4
  • References: Pathology 2020;52:40, Hum Pathol 2018;79:18
EBV positive T and NK cell aggressive lymphomas
Extranodal NK / T cell lymphoma
  • Predominantly extranodal lymphoma of either NK or T cell lineage
    • Nasal cavity, nasopharynx, paranasal sinuses, palate (upper aerodigestive tract)
    • Extranasal presentation: skin and intestines are the most common sites
  • Geographical predilection for Asia and Latin America
    • HLA::DPB1 is postulated as the susceptibility gene
  • Microscopic features
    • Characterized by necrosis, angiocentricity and angiodestruction, cytotoxic immunophenotype and EBV infection
    • Ancillary testing
      • CD2 positive, cytoplasmic CD3ε positive, CD94 positive, CD4 negative, CD5 negative, CD8 negative, TCRβ (BF1) negative
        • Both T and NK cells express CD3ε
      • TIA1 positive, granzyme B positive, perforin positive, CD56 variable
      • CD30 positive in 30 - 50% of cases, PDL1 positive in 40 - 100% of cases
      • EBER positive, presence of EBV in the majority of viable lymphoma cells
        • Strong association, regardless of ethnicity
        • Mechanism of infection of T cells by EBV is not entirely clear
        • Latency pattern II has been reported
  • Reference: Hum Pathol 2018;79:18


Aggressive NK cell leukemia
  • Overlaps with extranodal NK / T cell lymphoma when there is multiorgan involvement
    • Considered leukemic counterpart of extranodal NK / T cell lymphoma
  • Characteristically in adults, may present in young adults
  • High fever, general malaise, hepatosplenomegaly, liver failure and pancytopenia
  • Neoplastic NK cells in peripheral blood (PB) and bone marrow (BM)
  • Fulminant clinical course
  • Common association with hemophagocytic lymphohistiocytosis (HLH)
  • Microscopic features
    • Varying degrees of leukemic infiltrate in BM, liver, lymph nodes and spleen
    • Lymph nodes with diffuse infiltration by neoplastic cells and striking hemophagocytosis
    • Broad cytologic spectrum from subtle to large pleomorphic cells
    • Immunophenotype
      • CD3ε positive, CD56 positive, CD2 positive, FASL positive, CD16 positive (75%)
      • Surface CD3 negative, CD5 negative, CD57 negative
      • EBER positive (90%)
  • Reference: Am J Clin Pathol 2023;159:14


Primary nodal EBV positive T / NK cell lymphoma (EBV TNKL)
  • Rare, aggressive lymphoma
  • Mainly elderly patients in East Asia
  • Associated with underlying immunodeficiency (autoimmune [AI], HIV, PTLD)
  • Microscopic features
    • Primarily in lymph nodes but can involve extranodal sites
      • If nasal involvement, consider a diagnosis of extranodal NK / T cell lymphoma
      • Monomorphic infiltrate, lack angiodestruction or coagulative necrosis, mainly T cell lineage
      • CD3 positive, CD2 positive, CD8 positive, CD56 negative or positive, CD5 negative, CD4 negative
      • TIA1 positive, granzyme B positive, perforin positive
      • EBV latency II
    • Poor outcome: median overall survival of 4 - 6 months
    • Low genomic complexity compared with extranodal NK / T cell lymphoma (ENKTL) or panniculitis-like T cell lymphoma (PTCL), NOS
      • Mutation in epigenetic modifier genes: TET2, DNMT3A
        • Favor nodal EBV TNKL over extranasal ENKTL
  • Reference: Virchows Arch 2023;482:227


Systemic EBV positive T cell lymphoma of childhood
  • More often in children and young adults
  • No underlying immunodeficiency
  • Occurs shortly after acute primary EBV infection and rare in the setting of CAEBV disease
  • Abnormal serology against EBV with lack of anti-VCA IgM
  • High fever, hepatosplenomegaly, pancytopenia, coagulopathy, abnormal liver function
  • Lymphadenopathy might occur and be the dominant feature
  • Fulminant clinical course that results in death within days to weeks
  • Virtually always associated with HLH
    • Overlapping clinical and morphologic features with HLH
  • Microscopic features
    • Cytologic atypia ranges from minimal to severe
    • Histiocytic hyperplasia and striking hemophagocytosis in BM, spleen and liver
    • Lymph nodes look depleted with paracortical EBV+ CD8+ T cells
    • Ancillary tests
      • CD8+ cytotoxic αβ T cells, CD3, CD2, TIA1 and granzyme B but lack CD56
      • Rarely, cases associated with CAEBV show a CD4+ immunophenotype
      • EBER+ cells are less numerous than the infiltrating CD8+ cells
    • See Table 5
  • Reference: Am J Clin Pathol 2023;159:14
Diagrams / tables

Table 1: EBV viral gene expression patterns during different types of latency
Genes Latency III Latency II Latency I Latency 0
Epstein-Barr nuclear antigen 1 (EBNA1) + + + -
Epstein-Barr nuclear antigen 2 (EBNA2) + - - -
Epstein-Barr nuclear antigen 3 (EBNA3) + - - -
Epstein-Barr nuclear antigen (EBNA) LP + - - -
Latent membrane protein 1 (LMP1) + + - -
Latent membrane protein 2 (LMP2) + + - -
Epstein-Barr encoded RNAs (EBERs) + + + +
BHRF1 micro RNAs (miRNAs) + - - -
BamHI A rightward transcript (BART) micro RNAs (miRNAs) + + + +


Table 2: Main Epstein-Barr virus serological profiles
Anti-viral capsid antigen (VCA) IgG Anti-viral capsid antigen (VCA) IgM Anti-Epstein-Barr nuclear antigen (EBNA) IgG Interpretation
- - - Seronegative individual
Variable + - Primary infection
+ - + Past infection
+ - - Past infection (adults) or primary infection (children)
+ + + Past infection or end of primary infection
- - + Indeterminate


Table 3: Viral latency type in EBV associated lymphoproliferative disorders and lymphomas
Disease Percentage of EBV related cases Latency pattern Viral proteins expressed EBER expression pattern
EBNA1 EBNA2 LMP1 LMP2 EBER
B cell lymphoproliferative disorders
Infectious mononucleosis 100 III + + + + + Many small and large cells are EBV+ (mainly B cells and rare positive T and NK cells); most of the EBV+ cells are present in the paracortical area
EBV positive mucocutaneous ulcer 100 I or II + - + + + EBV expression is variable with most cases showing scattered EBV+ cells
Lymphoproliferative disorders associated with immune deficiency and dysregulation > 90 III or II + + + + +
  • Nondestructive follicular hyperplasia: EBV is within reactive GCs
  • Nondestructive plasmacytic hyperplasia: few EBV+ polyclonal B cells
  • Polymorphic LPD: numerous EBV+ cells with variable size are noted
T / NK lymphoproliferative disorders of childhood
Hydroa vacciniforme lymphoproliferative disorder 100 II + - + + + EBV expression in around 50% of lesional cells
Severe mosquito bite allergy 100 II + - - - + EBV is positive in few of the lesional NK cells; a much higher density of EBV+ cells should raise suspicion of NK cell lymphoma
Chronic active EBV disease (CAEBVD) 100 II + - + + + EBV is uniformly expressed in many cytotoxic T cells in most cases
B cell lymphomas
EBV positive diffuse large B cell lymphoma, NOS 100 II or III + + + + + Most of the large atypical lymphoma cells are EBV+, a cutoff of 80% has been proposed
Diffuse large B cell lymphoma with chronic inflammation 100 II or III + + + + + Most of the lymphoma cells are diffusely positive for EBER
Fibrin associated large B cell lymphoma 100 II or III + + + + + Most of the lymphoma cells are diffusely positive for EBER
Primary effusion lymphoma Most of the lymphoma cells are positive for EBER in the EBV positive cases
 HIV associated 100 I + - - - +
 HIV unrelated 70 - 90 I + - - - +
Lymphomatoid granulomatosis 100 III + + + + + The large neoplastic B cells are EBV positive; the number of EBV+ cells determines the grade
Plasmablastic lymphoma 60 - 75 I + - - - + Most of the lymphoma cells are diffusely EBV positive
Burkitt lymphoma Most lymphoma cells are positive for EBER in EBV positive cases
 Endemic > 95 I + - - - +
 Sporadic 20 - 80 I + - - - +
AIDS related DLBCL The pattern of EBV expression coincides with the histological subtype of lymphoma occurring in the immunocompetent state
 Immunoblastic 70 - 100 III + + + + +
 Nonimmunoblastic 10 - 30 III + + + + +
 CNS lymphomas 80 - 100 III + + + + +
Hodgkin lymphoma
  • EBV is positive in all Hodgkin / Reed-Sternberg cells
  • Some cases show partial expression, likely due to RNA degradation
  • Background cells are negative
 EBV unrelated 20 - 90 II + - + + +
 EBV associated 100 II + - + + +
EBV positive T and NK cell lymphomas
Extranodal, NK / T cell lymphoma 100 I or II + - Variable Variable + Virtually all lymphoma cells are positive for EBER
Aggressive NK cell leukemia 90 II + - + + + Most lymphoma cells are EBV positive
Primary nodal EBV positive T / NK cell lymphoma 100 II + - + + + Most lymphoma cells are EBV positive
Systemic EBV positive T cell lymphoma of childhood 100 II + - + + + Most lymphoma cells are EBV positive


Table 4: Differential diagnosis of EBV positive B cell lymphoproliferative disorders and lymphomas
Features Infectious mononucleosis EBV+ mucocutaneous ulcer EBV+ classic Hodgkin lymphoma EBV+ diffuse large B cell lymphoma
Clinical
Age Young, elderly Elderly Young and elderly Elderly
Lymphadenopathy Present Absent Present, nodal or mediastinal Present, high stage
LDH elevation Present, mild to moderate Absent Present Present
Extranodal disease Absent Present Extremely rare as primary disease Can be present, late stages
Clinical course Self limited in majority of cases Waxing and wanning Progressive Aggressive, poor outcome
Morphology
Architecture Paracortical Ulcer Effacement Effacement
Circumscription Absent Present, lymphocytic rim at base Absent Absent, diffuse involvement
Large cells Reed-Sternberg-like cells Reed-Sternberg-like cells Reed-Sternberg cells Sheets of large neoplastic cells, some RS-like cells
EBV latency type III II / III II III / II
Immunohistochemistry
CD45 Positive in most cells Positive in most cells Negative in HRS cells Positive in neoplastic cells
CD20 Positive in large cells Positive in large cells Mostly negative in HRS cells, faint reactivity in HRS cells in ~20% of cases Positive in large cells
PAX5 Positive, strong Positive, strong Positive, weak Positive, strong
BOB.1 Positive, strong Positive, strong Negative, can be weak Positive, strong
MUM1 Positive Positive Positive Positive
BCL6 Negative Negative Negative Can be positive
CD10 Negative Negative Negative Negative
CD30 Positive in HRS-like cells, usually dim Positive in HRS-like cells, usually dim Positive in HRS cells, strong Positive
CD15 Positive in up to 50% of cases Positive in up to 50% of cases Positive, variable Positive in up to 50% of cases
PDL1 Negative Negative Positive in > 80% Can be positive (40 - 60%, extranodal)
Diagnostic molecular testing
B cell Polyclonal Clonal in 50% of the cases Clonal Monoclonal IGH gene rearrangements
T cell Polyclonal Oligoclonal and restricted TCR rearrangement patterns Polyclonal, restricted pattern in elderly patients Oligoclonal and restricted TCR rearrangement patterns
Genetic features No immune evasion features No immune evasion features Immune evasion (host evasion) Immune evasion (host evasion)


Table 5: Differential diagnosis of EBV positive T and NK lymphomas
Feature Extranodal NK / T cell lymphoma Aggressive NK cell leukemia EBV positive nodal T and NK cell lymphoma Systemic EBV+ T cell lymphoma of childhood
Clinical presentation
Age Adults Young to middle aged adults Older adults Children, young adults
Site at presentation Nasopharynx (70 - 80%), others (20 - 30%): skin, gastrointestinal (GI) Bone marrow, spleen, peripheral blood, rarely lymph nodes (20%) Lymph nodes, no nasal involvement by definition Systemic proliferation: bone marrow, liver or spleen, CNS
Behavior Localized disease, frequent dissemination Fatal Aggressive Fulminant
Median survival 26 - 76 months Weeks 4 months Days to weeks
Hemophagocytic syndrome Generally absent Present Uncommon Always present
Morphology
Cytology of neoplastic cells Variable atypia, spectrum from small to large cells Large granular atypical lymphocytes, distinct nucleoli and clear cytoplasm (smears) Pleomorphic medium sized cells, with centroblastic, anaplastic or plasmacytoid features Small to intermediate sized with subtle to absent atypia (most common) or large atypical cells
Necrosis Common Frequently present Variable Absent
Angiocentricity and angiodestruction Present Frequently present Uncommon Absent
Apoptosis Present Frequently present Variable Absent
Ancillary testing
CD2 Positive Positive Positive Positive
CD3 Often negative; subset is positive Negative Positive Positive
CD3ε Positive Positive Positive Positive
CD4 Negative Negative Negative Usually negative
CD8 Positive Usually negative Positive (> 80%) Usually positive
CD56 Positive Positive Mostly negative (positive < 20%) Negative
Cytotoxic granules Positive Positive Positive Positive
EBER All neoplastic cells All neoplastic cells, a subset is negative (< 15%) All neoplastic cells Majority of neoplastic cells


Table 6: Differential diagnosis of EBV+ B cell lymphoproliferations with Hodgkin-like features
Disease Clinical features Morphology Immunophenotype Lineage, clonality and molecular features
EBV+ mucocutaneous ulcer 
  • Mostly elderly with well circumscribed ulcerating lesions of skin and mucosas
  • No breach of compartmental boundaries
  • No lymphadenopathy or hepatosplenomegaly
  • No LDH elevation
  • Full resolution upon reduction / withdrawal of immunosuppressive therapy (IS)
  • Waxing / wanning course in elderly with immunosenescence
  • No progression to lymphoma 
  • Good circumscription with a band-like infiltrate following contour of the overlying ulcerated surface
  • Hodgkin-like features with HRS-like cells in a mixed inflammatory background
  • Wide range of lesional cell sizes (highlighted by EBER)
  • Angioinvasion
  • Rim of lymphocytes at base 
  • Retained B cell phenotype (postgerminal center)
  • CD45 variable
  • CD20 variable
  • CD79a+
  • PAX5+ (strong)
  • OCT2+ (strong)
  • BOB1+ (strong)
  • MUM1+
  • BCL6 variable
  • CD10-
  • CD30+
  • CD15+ (up to 50%)
  • EBV latency III (50%)
  • PD-L1-
  • B cell clonal 50%
  • T cell (background lymphocytes) oligoclonal and restricted TCR rearrangement patterns
  • Genetic features not studied fully
  • No immune evasion features 
EBV+ diffuse large B cell lymphoma, NOS 
  • Elderly with high stage nodal and extranodal disease
  • B symptoms and elevated LDH
  • Isolated lesions and bulky tumor masses breaching compartmental boundaries
  • Aggressive lymphoma with poor outcomes 
  • Diffuse, destructive and infiltrative proliferations
  • Range of morphologies with common Hodgkin-like features and polymorphism; wide range of lesional cell sizes (highlighted by EBER)
  • Angioinvasion
  • Copious necrosis
  • Mucosal / cutaneous lesions show no distinctive architecture 
  • Retained B cell phenotype (postgerminal center)
  • CD45 variable
  • CD20 variable
  • Cd79a+
  • PAX5+ (strong)
  • OCT2+ (strong)
  • BOB1+ (strong)
  • MUM1+
  • BCL6 variable
  • CD10-
  • CD30+
  • CD15+ (up to 50%) EBV latency III (9 - 12%)
  • PD-L1 variable (40 - 60%) (extranodal) 
  • B cell clonal
  • T cell (background lymphocytes) oligoclonal and restricted TCR rearrangement patterns
  • Host response lymphoma partly characterized by immune evasion
  • A proportion with CIITA and PD-L1 / L2 gene loci abnormalities 
EBV+ classic Hodgkin lymphoma 
  • Young and elderly with mediastinal and nodal presentation
  • B symptoms and elevated LDH
  • Extremely rare primary extranodal presentation in skin and mucosas
  • Extranodal involvement usually as part of widespread disease
  • Excellent outcomes with appropriate combined therapies 
  • Diffuse / nodular destructive and infiltrative proliferations
  • HRS cells in a mixed inflammatory background
  • Only large HRS cells with no intermediate or small cell component (highlighted by EBER)
  • No angioinvasion
  • Exceptionally rare mucosal / cutaneous lesions show no distinctive architecture 
  • Shut off B cell expression program
  • CD45-
  • CD20-
  • CD79a variable
  • PAX5+ (weak)
  • OCT2 variable (weak)
  • BOB1 variable (weak)
  • MUM1+
  • BCL6 variable
  • CD10-
  • CD30+
  • CD15+ (variable)
  • EBV latency II
  • PD-L1 variable (> 80%) 
  • B cell clonal
  • T cell (background lymphocytes) polyclonal; oligoclonal / restricted patterns in elderly
  • Host response lymphoma characterized by immune evasion
  • Higher prevalence of CIITA and PD-L1 / L2 gene loci 
Clinical images

Contributed by Francisco Bravo, M.D.
Hydroa vacciniforme lymphoproliferative disorder

Hydroa
vacciniforme
lymphoproliferative
disorder

Microscopic (histologic) images

Contributed by Roberto N. Miranda, M.D. and Roman Segura-Rivera, M.D.
Infectious mononucleosis in an excised lymph node Infectious mononucleosis in an excised lymph node

Infectious mononucleosis in an excised lymph node

Interfollicular expansion in infectious mononucleosis

Interfollicular expansion in infectious mononucleosis

Infectious mononucleosis Infectious mononucleosis Infectious mononucleosis

Infectious mononucleosis


Infectious mononucleosis

Infectious mononucleosis

Follicular hyperplasia arising in immune deficiency / dysregulation Follicular hyperplasia arising in immune deficiency / dysregulation Follicular hyperplasia arising in immune deficiency / dysregulation

Follicular hyperplasia arising in immune deficiency / dysregulation

Germinal center reaction

Germinal center reaction


Hydroa vacciniforme

Hydroa vacciniforme

Hemophagocytic lymphohistiocytosis

Hemophagocytic
lymphohistiocytosis

EBV positive plasmablastic lymphoma EBV positive plasmablastic lymphoma

EBV positive plasmablastic lymphoma


Extranodal NK / T cell lymphoma showing angiocentricity

Extranodal NK / T cell lymphoma showing angiocentricity

Extranodal NK / T cell lymphoma

Extranodal NK / T cell lymphoma

Extranodal NK / T cell lymphoma in the intestine Extranodal NK / T cell lymphoma in the intestine

Extranodal NK / T cell lymphoma in the intestine

Peripheral smear images

Contributed by Roberto N. Miranda, M.D. and Roman Segura-Rivera, M.D.
Downey cells

Downey cells

Board review style question #1

Which of the following is true about Epstein-Barr virus (EBV) infection?

  1. EBV is the human herpesvirus 5 (HHV5) and belongs to the Herpesviridae family
  2. EBV subtype 2 is the more prevalent worldwide and has greater transforming potential
  3. Expression of EBER is a constant feature of all EBV infected cells, independent of the latency pattern acquired
  4. Most EBV infections are acquired in adulthood and are asymptomatic
Board review style answer #1
C. The expression of EBER is a constant feature of all EBV infected cells, independent of the latency pattern acquired. Answer A is incorrect because EBV is the human herpesvirus 4 (HHV4) and part of the Herpesviridae family. Answer D is incorrect because most EBV infections are acquired in childhood and are asymptomatic. Answer B is incorrect because subtype 1 is the most prevalent worldwide and has a greater transforming potential.

Comment Here

Reference: Epstein-Barr virus
Board review style question #2

What lymphomas and lymphoproliferative disorders (LPDs) commonly present with Epstein-Barr virus (EBV) type I latency pattern?

  1. Primary effusion lymphoma, hydroa vacciniforme LPD and Hodgkin lymphoma
  2. Primary effusion lymphoma, lymphomatoid granulomatosis and acquired immunodeficiency syndrome (AIDS) related diffuse large B cell lymphoma
  3. Primary effusion lymphoma, plasmablastic lymphoma and Burkitt lymphoma
  4. Primary effusion lymphoma, severe mosquito bite allergy and Hodgkin lymphoma
Board review style answer #2
C. Primary effusion lymphoma, plasmablastic lymphoma and Burkitt lymphoma. Primary effusion lymphoma, plasmablastic lymphoma and Burkitt lymphoma are usually latency type I. Answers A and D are incorrect because hydroa vacciniforme, LPD, Hodgkin lymphoma and severe mosquito bite allergy are usually latency type II. Answer B is incorrect because lymphomatoid granulomatosis and AIDS related diffuse large B cell lymphoma are usually latency pattern III.

Comment Here

Reference: Epstein-Barr virus

Extranodal NK / T cell lymphoma
Definition / general
Essential features
Terminology
  • Synonyms
    • Polymorphic reticulosis
    • Malignant midline reticulosis
    • Lethal midline granuloma
    • Angiocentric T cell lymphoma (Blood 1994;84:1361)
ICD coding
  • ICD-O: 9719 / 3 - extranodal NK / T cell lymphoma, nasal type
  • ICD-10: C86.0 - extranodal NK / T cell lymphoma, nasal type
Sites
Pathophysiology
Etiology
Clinical features
Diagnosis
Radiology description
  • Both CT and MRI are useful for evaluation of extension in nasal lesions (Expert Rev Hematol 2016;9:861)
    • No pathognomonic findings
  • PET / CT is used to identify extranasal lesions (Expert Rev Hematol 2016;9:861)
    • F-18 fluorodeoxyglucose avid
    • Standard uptake value is typically less than aggressive B cell lymphomas
Prognostic factors
Case reports
Treatment
Clinical images

Contributed by Auris Huen, M.D.
Skin lesion

Skin lesion

Microscopic (histologic) description
Microscopic (histologic) images

Contributed by Roberto N. Miranda, M.D. and Carlos A. Torres-Cabala, M.D.
Nasal lesion of ENKTCL

Nasal lesion of ENKTCL

Diffuse infiltrate

Diffuse infiltrate

Cytological atypia

Cytological atypia

EBER positivity

EBER positivity

Angioinvasion in ENKTCL

Angioinvasion in ENKTCL

Atypical infiltrate in skin

Atypical infiltrate in skin


CD3 positivity

CD3 positivity

CD7 positivity

CD7 positivity

CD56 positivity

CD56 positivity

Perforin positivity

Perforin positivity

Hemophagocytosis

Hemophagocytosis

ENKTCL NT in bone marrow

ENKTCL NT in bone marrow


Bone marrow CD3

Bone marrow CD3

Bone marrow EBER

Bone marrow EBER

Skin involvement by PTCL, NOS

Skin involvement by PTCL, NOS

 cell LGL leukemia

Cytological atypia in PTCL, NOS

 cell LGL leukemia

T cell LGL leukemia

CD3 in T cell LGL

CD3 in T cell LGL


CD57 in T cell LGL

CD57 in T cell LGL

TIA1 in T cell LGL

TIA1 in T cell LGL

Skin in EBV+ DLBCL

Skin in EBV+ DLBCL

CD20 in DLBCL

CD20 in DLBCL

EBER in DLBCL

EBER in DLBCL

Necrosis in tuberculosis

Necrosis in tuberculosis


Necrosis in histoplasmosis

Necrosis in histoplasmosis

Histoplasmosis GMS stain

Histoplasmosis GMS stain

Cytology description
Positive stains
Negative stains
Molecular / cytogenetics description
Sample pathology report
  • Left maxillary sinus, biopsy:
    • Extranodal NK / T cell lymphoma, nasal type (see comment)
    • Comment: According to clinical records, patient is a 56 year old Hispanic male with recent clinical history of nasal obstruction, purulent discharge, epistaxis and B type symptoms. In the last 5 months, the patient has received several antibiotic regimens with no improvement of the clinical course. A recent CT of the head and neck showed a 6 cm mass in the left maxillary sinus with invasion of the adjacent bone.
    • The biopsy is adequate and shows respiratory type mucosa with underlying dense and monotonous lymphoid infiltrate with focal vascular invasion. The lymphocytes are of medium size, folded nuclei, indistinct nucleoli and moderate amount of cytoplasm. Frequent mitoses are observed. There is focal ulceration and extensive coagulative necrosis.
    • Immunohistochemical studies are performed and the atypical cells are positive for CD30, CD56, granzyme B and TIA1. Ki67 proliferation index is 60%. In situ hybridization for EBV encoded RNA (EBER) shows reactivity in most tumor cells. The abnormal cells are negative for sCD3, CD5, CD8, CD20, PAX5 as well as for TCRδ and TCRαβ. Acid fast bacilli stain and Gomori methenamine silver stain for fungal organisms are negative.
    • Next generation sequencing demonstrated mutation of STAT3.
    • Polymerase chain reaction for Ig heavy chain (IGH) and T cell receptors gamma (TRG) and beta (TRB) chains revealed a germline pattern.
Differential diagnosis
Board review style question #1
Which of the following is true about extranodal NK / T cell lymphoma, nasal type (ENKTCL)?

  1. Frequently presents as an indolent disease with favorable prognosis
  2. JAK / STAT pathway is usually activated
  3. More common in young (second and third decades) patients
  4. Necrosis and angiotropism are usually absent
Board review style answer #1
B. JAK / STAT pathway is usually activated

Comment Here

Reference: Extranodal NK / T cell lymphoma, nasal type
Board review style question #2
What is the most common immunophenotype of extranodal NK / T cell lymphoma, nasal type?

  1. εCD3+ / sCD3- / CD5- / CD16+ / CD56+ / EBER+
  2. εCD3+ / sCD3+ / CD5- / CD16+ / CD56+ / EBER+
  3. εCD3- / sCD3+ / CD5- / CD16- / CD56- / EBER+
  4. εCD3- / sCD3- / CD5- / CD16- / CD56+ / EBER+
Board review style answer #2
A. εCD3+ / sCD3- / CD5- / CD16+ / CD56+ / EBER+

Comment Here

Reference: Extranodal NK / T cell lymphoma, nasal type

Extranodal NK / T cell lymphoma

Fibrin associated large B cell lymphoma
Definition / general
  • Very rare, indolent, non mass forming, Epstein-Barr virus (EBV)+ large B cell lymphoproliferative disorder
  • Noted incidentally on histological examination of tissues excised for some other reason (Am J Surg Pathol 2017;41:299)
  • Neoplastic cells are typically found within fibrinous or amorphous material
Essential features
  • Very rare, indolent, microscopic, EBV+ large B cell lymphoproliferative disorder
  • Localized to a restricted anatomic site
  • Non mass forming, noninvasive with no direct clinical signs or symptoms
  • Noted incidentally on histological examination of tissues excised for other reasons
  • Location of lymphoma cells limited to fibrinous or amorphous eosinophilic material
  • No infiltration or effacement of adjacent normal tissues
  • Excellent prognosis, usually with surgical excision alone
Terminology
  • Fibrin associated (EBV+) large B cell lymphoma (FA DLBCL)
  • Subtype of: diffuse large B cell lymphoma associated with chronic inflammation
ICD coding
  • ICD-10: C83.39 - diffuse large B cell lymphoma, extranodal and solid organ sites
  • ICD-11: 2A81.7 - diffuse large B cell lymphoma associated with chronic inflammation
  • ICD-O: 9680/3 - malignant lymphoma, large B cell, diffuse, NOS
Epidemiology
  • EBV associated
  • Median age 55.5 years
  • M:F = 3:1
  • No ethnic or racial predilection
Sites
Pathophysiology
  • Unknown at this time
Etiology
Clinical features
Diagnosis
  • Requires a high index of suspicion
  • Lymphoma cells incidentally noted only on microscopic examination of tissues
  • Exclude systemic or invasive disease and primary or iatrogenic immunosuppression
  • Localized to a restricted anatomic site
Radiology description
  • No evidence of lymphoma on imaging (if performed)
Prognostic factors
Case reports
Treatment
  • Surgical excision alone is usually adequate
Gross description
  • No gross tumor mass or evidence of lymphoma
Microscopic (histologic) description
  • Small foci of lymphoma cells in fibrinous or amorphous eosinophilic material
    • Aggregates / clusters
    • Ribbons
    • Single cells
  • Neoplastic cells are large and atypical
    • Irregular nuclear contours
    • Coarse chromatin
    • Distinct nucleoli
    • Amphophilic cytoplasm
    • May be plasmacytoid or bizarre appearing
    • Mitotic figures and apoptotic bodies present
  • Lack of lymphomatous infiltration or effacement of adjacent normal tissues
  • Usually lacks prominent inflammatory background in vicinity of lymphoma cells
Microscopic (histologic) images

Contributed by Jordan M. Hall, M.D., Hyunkyu Shin, M.D., Dr. Christian Schürch, M.D., Ph.D., Falko Fend, M.D. and Claudia Wickenhauser, M.D., Ph.D. (Case #529)

Knee synovium

Knee joint space fibrin

PAX5

CD30

Ki67


EBER ISH

H&E staining H&E staining H&E staining H&E staining H&E staining

H&E staining


H&E staining H&E staining H&E staining

H&E staining

CD138

CD138

MUM1

MUM1

CD3

CD3


CD20

CD20

CD79a

CD79a

ALK

ALK

MYC

MYC

HHV8

HHV8

Kappa

Kappa


Lambda

Lambda

Cyclin D1

Cyclin D1

CD56

CD56

EBER

EBER

BCMA

BCMA

Positive stains
Negative stains
Molecular / cytogenetics description
Sample pathology report
  • Soft tissue, right subpatellar space, excision:
    • Fibrin associated diffuse large B cell lymphoma (see comment)
    • Comment: Histopathologic evaluation reveals fragments of synovium with patchy chronic inflammation and abundant eosinophilic fibrinous material. Within and limited to the fibrin, there are scattered small collections of large atypical lymphoid cells noted. Immunohistochemistry shows these cells are positive for CD20, PAX5, CD30 and CD43, express an activated B cell immunophenotype (CD10 negative, BCL6 negative, MUM1 positive) and show a high proliferative rate (> 90% Ki67 positivity). These cells are also positive for Epstein-Barr virus encoded small RNA (EBER) by chromogenic in situ hybridization. Negative stains include CD3, CD15, CD68, CD163, CD138, ALK1 and SOX11. Impression: The histopathologic findings and available clinical history of arthroplasty without clinical suspicion for neoplasia support a diagnosis of FA DLBCL. This is a clinically indolent lymphoproliferative disorder that is confined to a restricted anatomic space, typically an incidental finding on histopathologic examination only and usually adequately treated with surgical excision alone. That being said, correlation with the complete clinical and radiologic workup is recommended to exclude the possibility of more aggressive systemic disease.
Differential diagnosis
Board review style question #1

    Which of the following is detected in the tumor cells of fibrin associated diffuse large B cell lymphoma?

  1. EBV
  2. HHV8
  3. HIV
  4. HSV
  5. KSHV
Board review style answer #1
Board review style question #2

    Fibrin associated diffuse large B cell lymphoma is currently included in the 2017 revised WHO classification as a subtype of which entity?

  1. Anaplastic large cell lymphoma
  2. Diffuse large B cell lymphoma associated with chronic inflammation
  3. EBV positive diffuse large B cell lymphoma, NOS
  4. High grade B cell lymphoma
  5. Primary effusion lymphoma
Board review style answer #2
B. Diffuse large B cell lymphoma associated with chronic inflammation

Comment Here

Reference: Fibrin associated large B cell lymphoma
Board review style question #3
    Which of the following is a true feature of fibrin associated large B cell lymphoma (FA-LBCL)?

  1. Distinguishing this tumor from chronic inflammation associated (CI-)LBCL has no clinical importance because both are inflammation associated indolent neoplasms
  2. The neoplastic cells are derived from germinal center B cells
  3. This neoplasm is strongly associated with EBV infection with type III latency
  4. This tumor is a highly aggressive lymphoid neoplasm due to its association with acute fibrous inflammation
  5. With discohesive neoplastic cells, this tumor readily infiltrates pre-existing anatomic structure
Board review style answer #3
C. This neoplasm is strongly associated with EBV infection with type III latency. Except for rare cases as presented here, FA-LBCL is consistently associated with EBV infection showing type III latency with LMP1 and EBNA2 positivity. Answer A is incorrect because CI-LBCL exhibits a more aggressive clinical course than FA-LBCL with similar morphology, so these two entities should be clearly distinguished. Answer B is incorrect because the neoplastic cells of FA-LBCL show an activated (nongerminal center) B cell phenotype (CD10-, BCL6+/-, MUM1+). Answer D is incorrect because FA-LBCL is an indolent lymphoid neoplasm, so it is often discovered incidentally during careful examination. Answer E is incorrect because this tumor is mostly confined to cystic or pseudocystic spaces without infiltration of pre-existing anatomic structures or mass formation.

Comment Here

Reference: Fibrin associated large B cell lymphoma

Flow cytometry
Definition / general
  • Flow cytometry is a technique that measures various parameters (optical characteristics and emitted fluorescence) of cells in a flowing fluid suspension
Essential features
  • Sensitive method for simultaneously obtaining information on various parameters, including optical characteristics and fluorescence of cells in suspension
  • Components include fluidics, optical systems and electronic systems
  • Applications are limited to cells in suspension and provide no information about cell to cell interactions
  • Provides valuable clinical application in hematologic diseases and body fluids diagnostics
Terminology
  • Cell sorting: separating cells, identified by specified characteristics
  • Compensation: mathematical algorithm for removing bleeding / spillover of 1 fluorophore into multiple detectors
  • Forward scatter: light scattered in the forward direction after interacting with a particle
  • Gating: specifying populations of cells with common characteristics to investigate further
  • Side scatter: light scattered at 90 degrees after interacting with a particle
CPT coding
  • Fresh tissue or body fluids
    • 88108: cytospin technical
    • 88184: first marker flow cytometry, technical
    • 88185: each additional marker, flow cytometry technical
    • 88187: flow cytometry interpretation, 2 - 8 markers
    • 88188: flow cytometry interpretation, 9 - 15 markers
    • 88189: flow cytometry interpretation, 16 or more markers
Sites
  • Peripheral blood
  • Bone marrow
  • Body cavity fluids (pleural, peritoneal, pericardial, joints)
  • Cerebrospinal fluids
  • Tissue (e.g., lymph nodes)
Diagrams / tables

Contributed by Abdallah Flaifel, M.D.
cell block TdT

Cell block TdT



Images hosted on other servers:

Schematic of a flow cytometer

Overview
  • Basic principles of flow cytometry (Crit Rev Biotechnol 2017;37:163)
    • Single cells are dissociated in a liquid medium
    • Cells are then stained with 1 or more fluorochrome tagged markers
    • Laser beams with specific wavelengths strike the cells
    • Cells tagged with fluorochrome dye absorb photons and emit fluorescence
    • Light scatter and fluorescence from individual cells are detected by photomultiplier tubes
    • Electronic impulse is converted to digital data (analog to digital) and is recorded by the computer
    • Data acquisition software stores digital data as flow cytometry standard file
    • Software packages display data as uni / bivariate histograms, scatter / dot plots, contour plots or density plots
  • Components (Curr Protoc Immunol 2018;120:5.1.1, Crit Rev Biotechnol 2017;37:163)
    • Fluidics
      • Composed of a coaxial system: inner (sample) and outer (sheath)
        • The aim of the system is to maintain a stable flow of cells
        • Sheath fluid protects the sample from turbulence caused by resistance to the flow by the tube walls
      • Air pressure of the sample controls the rate of flow of cells
        • Increasing sample fluid pressure increases the rate of flow
        • Lowering the sample fluid pressure decreases rate of flow
    • Optical system
      • Light source
        • Laser (light amplification by stimulated emission of radiation)
          • Emits light with a specific wavelength
          • Primary laser (488 nm wavelength) or secondary red diode laser (635 nm wavelength) may be utilized
          • Excitation of the fluorochrome from laser beam results in fluorescence; the selection of fluorochrome should be appropriately matched with the wavelength of the laser used
      • Light scattering
        • Forward scatter (FSC): directly proportional to the size and surface area of the cell; it is collected by the FSC detector located on the same axis as the laser beam
        • Side scatter (SSC): directly proportional to the granularity and internal complexity of the cell
      • Fluorescence emission
        • Fluorochrome dye can be used to characterize various properties in cells
        • Fluorochrome is commonly tagged with an antibody
        • Fluorochrome will absorb and emit light at a specific wavelength
      • Light collection: via a set of special filters and optical mirrors
    • Electronic system (Methods Mol Biol 2011;699:1)
      • Photons of light, generated by light scatter and fluorophore emission, hit the photodetectors
      • Photon signal is converted into an electrical current called a photocurrent
      • Photocurrent is then converted into a voltage pulse
        • Voltage pulse is directly proportional to forward / side scatter and the number / brightness of fluorophores
      • Voltage pulse may then be amplified (linear or logarithmic) or digitized by an analog digital converter (ADC)
        • Newer flow cytometers (termed digital systems) digitize photocurrent early without prior amplification or processing
      • Digitization involves organization of the continuous analog data into digital channels through process known as binning
      • Analog digital converter outputs a data file in a standard FSC format
Advantages of flow cytometry
  • Sensitive method for simultaneously obtaining information on various parameters and processes, including expression of surface markers or the presence of intracellular cytokines and proteins
  • High throughput system that can excel in characterizing heterogeneous cell populations
  • Capable of sorting cells based on specific features
  • Indispensable tool for the classification and immunophenotyping hematologic neoplasms
  • Reference: J Invest Dermatol 2012;132:1
Disadvantages of flow cytometry
  • Limited to cells in suspension, so information on tissue architecture and cell - cell interactions is not available
  • Analyses requiring more fluorophores are subject to signal spillover
  • Analysis is complicated by the amount of data generated
  • Results are limited to the specific immunofluorescence panels that are being tested
  • Less accurate in diagnosing certain lymphomas (i.e., Hodgkin lymphoma)
  • Limited utility in evaluating infectious etiologies (e.g., EBV, HHV8)
  • Reference: J Invest Dermatol 2012;132:1
Effectiveness
  • Efficient flow cytometry satisfies the following
    • High sensitivity
      • Threshold: the capability to distinguish dim cells from the particle free background
      • Resolution: the ability to separate dim cells from unstained ones
    • Relative measured values of fluorescence (which depends on the instrument’s linearity accuracy)
    • Reproducibility of the results
  • Quality control
    • Periodically: laser time delay, laser alignment
    • Every day: photomultiplier voltage settings, compensation, gating control for multicolor flow
    • External quality assessment
  • Reference: Clin Lab Med 2007;27:671
Clinical applications
  • Hematologic diseases
    • Diagnosis and sub classifying non-Hodgkin lymphoma (NHL)
      • WHO approaches to classify the lymphomas based on the lineage of the cells: B cell and T / NK cell; precursor versus mature
      • Useful in immunophenotyping different lymphomas
        • B cell markers: CD19, CD20, CD79a, PAX5, CD10, CD23
        • T cell markers: CD2, CD3, CD4, CD5, CD7, CD8
        • NK cell markers: CD2, CD3, CD56
        • Plasma cell markers: CD38, CD138, CD56
      • Fine needle aspiration of lymph node with flow cytometry
        • Rapid onsite evaluation
        • Material processed for
          • Routine smears
          • Cell block
          • Flow cytometry
          • Cytogenetics
        • False negative in flow cytometry after fine needle aspiration (FNA) can be due to
          • Predominantly necrotic tissue or fibrosed lymph node
          • Material diluted with blood
          • Few and scattered atypical cells as in Hodgkin lymphoma
      • Assessment of minimal residual disease (Blood Cancer J 2020;10:108)
        • Ultrasensitive method of determining disease progression, relapse and response to therapy
        • May be used with or without next generation sequencing to detect specific populations with a sensitivity between 10-5 and 10-6
        • Usually performed on bone marrow aspirate fluid (multiple myeloma and acute leukemias) but may also use peripheral blood in certain instances (e.g., CLL or ALL)
        • Limitations include hemodilution and sampling error
      • One major application of flow cytometry is demonstrating the clonality of lymphoma
        • Light chain restriction: monoclonal B cell proliferation has altered kappa to lambda chain ratio
  • Flow cytometry of cytology samples
    • Effusion
      • Detection of carcinoma in fluid
        • Higher synthetic phase (S phase) cells may be used as indirect evidence of malignancy
        • Epithelial cell adhesion molecule (EpCAM [CD326]) or its antibody (BerEP4) can be used to identify epithelial cells (Appl Immunohistochem Mol Morphol 2009;17:202)
        • Possible pitfalls
          • Poorly differentiated carcinoma may not express any markers
          • Benign cells may express epithelial markers, especially in peritoneal washings and instrumentalizations
      • Effusion due to involvement by a lymphoreticular neoplasm
        • Cytology along with flow cytometry have 100% sensitivity and 94% specificity in diagnosing lymphoma (Diagn Cytopathol 2006;34:335)
        • Some panels suggested include
          • Acute leukemia: MPO, Tdt, CD45 / CD34 / CD7 / CD13 and CD33 / CD56 / CD19
          • Lymphoma: CD45 / CD3 / CD4 / CD8 and CD19 / CD20 / CD10
          • For suspected T cell lineage, must include: CD3 / CD2 / CD5 / CD7 / CD8, CD26
          • For suspected B cell lineage, must include CD5 / CD23 / CD10 / CD38 / CD138 / FMC7 / surface kappa and lambda light chain
          • For suspected myeloma: CD20 / CD19 / CD38 / CD138 / CD27 / CD45 / cytoplasmic kappa / lambda
        • Primary effusion lymphoma
          • Cytomorphology shows large cells with nuclear pleomorphism and nucleoli
          • Flow cytometry
            • Positive: CD45, CD30, CD38, CD138, CD43
            • Negative for B cell markers: CD19, CD20 and no light chain restriction
            • Absent or aberrant expression of T cell markers: CD2, CD3, CD5, CD7
    • Bronchoalveolar lavage (BAL) (Am J Respir Crit Care Med 2012;185:1004)
      • Cellular analysis may provide useful adjunct information for patients who lack radiographic evidence of usual interstitial pnuemonia
      • Predominant inflammatory cellular pattern could help clinicians determine etiology of interstitial lung disease
  • Identification of therapeutic targets
    • Selection of antibody based therapies
      • Rituximab: anti-CD20
      • Epratuzumab: anti-CD22
      • Gemtuzumab: anti-CD33
      • Blinatumomab: directed against CD19 and CD3
  • Nonhematologic disease
    • Assessment of nonhematologic neoplasms (Am J Clin Pathol 2020;153:99)
      • May utilize antibodies directed against epithelial markers such as MOC31, Ber-EP4, CK5 / 8 and MUC1
      • Targeted panel of CD45, CD33 and EpCAM (CD326) has been used to distinguish monocytes / macrophages, mesothelial cells and epithelial cells
  • DNA content analysis
    • Use of propidium iodide that interposes into the DNA helical structure
      • Fluorescence is directly proportional to the amount of DNA
      • DNA content aneuploidy can be determined in a tumor cell
Cytology images

Contributed by Abdallah Flaifel, M.D.
Pericardial fluid cytospin

Pericardial fluid cytospin

Cell block TdT

Cell block TdT

Flow cytometry images

Contributed by Nicholas Ward, M.D.
B-ALL flow cytometry B-ALL flow cytometry

B ALL flow cytometry

B-ALL flow cytometry B-ALL flow cytometry B-ALL flow cytometry

B ALL flow cytometry

Board review style question #1
In a typical flow cytometry, the side scatter (SSC) and forward scatter (FSC) provide information about which of the following?

  1. Architecture and granularity
  2. Cytoplasmic complexity and size
  3. Intracellular signaling and viability
  4. Size and surface markers
Board review style answer #1
B. Cytoplasmic complexity and size

Comment Here

Reference: Flow cytometry
Board review style question #2
Which of the following is a clinical application of flow cytometry?

  1. Demonstrating clonality of lymphoma
  2. Easily identifying Hodgkin lymphoma
  3. Identifying benign cells in instrumented effusion fluid
  4. Identifying relevant information about architecture of carcinoma
Board review style answer #2
A. Demonstrating clonality of lymphoma. Answer B is incorrect because Reed-Sternberg cells are admixed in a rich inflammatory background which consists mainly of T cells, B cells, eosinophils, histiocytes and plasma cells. Answer C is incorrect because benign cells are scant in instrumented fluid. Answer D is incorrect because flow cytometers analyze suspensions of single cells.

Comment Here

Reference: Flow cytometry

Fluid overload associated LBCL
Definition / general
  • Body cavity large B cell lymphoma with no associated solid lymphoma involvement in immunocompetent individuals
  • Negative for HHV8 infection and does not occur in the setting of chronic inflammatory / infectious related effusions
Essential features
  • B cell lymphoma composed of large malignant cells present in body cavity effusions without any solid tissue lymphoma involvement
  • Predominantly diagnosed in elderly and immunocompetent individuals with comorbidities that put them at risk for developing body cavity effusions (e.g., congestive heart failure, cirrhosis)
  • Not associated with HHV8 infection
  • EBV infection can be detected only in a minority of cases
  • Tumor cells are frequently positive for B cell markers (CD20, PAX5) and most are of nongerminal center type per the Hans algorithm
  • References: Mod Pathol 2022;35:1411, J Blood Med 2021;12:833, J Am Soc Cytopathol 2015;4:37, Blood Adv 2020;4:4442, Histopathology 2018;72:930
Terminology
  • Primary effusion lymphoma (PEL)-like lymphoma
  • HHV8 negative effusion based large B cell lymphoma
  • HHV8 negative malignant effusion lymphoma
  • PEL-like lymphoma
  • HHV8 unrelated PEL-like lymphoma
  • Type II PEL
ICD coding
  • ICD-10: C83.8 - other nonfollicular lymphoma
  • ICD-11: 2A81.9 - primary effusion lymphoma
Epidemiology
  • Represents < 1% of all diagnosed lymphomas
  • Predominantly elderly patients (median age: 79)
  • M:F = 1.26:1
  • HIV positive (2% of individuals)
  • Risk of fluid overload (e.g., congestive heart failure, cirrhosis) in 50 - 79% of cases
  • References: Leuk Lymphoma 2017;58:80, Mod Pathol 2022;35:1411
Sites
  • Body cavities, predominantly the pleural cavity
  • May also be present in pericardial and peritoneal fluids
  • Solid lymphoma involvement is absent
  • Reference: Mod Pathol 2022;35:1411
Pathophysiology
  • Most of the reported cases derive from postgerminal center B cell lymphocytes (Blood 2019;133:377)
Etiology
Clinical features
Diagnosis
  • Nongynecological cytology analysis of effusion fluid
  • Flow cytometry analysis of effusion fluid
Laboratory
  • Findings depend on the site of fluid effusion and are otherwise nonspecific
  • Lactate dehydrogenase (LDH) is typically elevated
Prognostic factors
  • Overall survival was 11 months for non-Japanese residents and the median overall survival was 63.6 months for Japanese residents
  • Presence of MYC rearrangements is associated with the worst survival
  • Age of ≥ 79 is an independent unfavorable predictor
  • References: Mod Pathol 2022;35:1411, Haematologica 2002;87:339
Case reports
Treatment
  • No standardized therapeutic approach is available due to the limited number of cases reported
  • First line management usually includes rituximab combined with systemic chemotherapy
  • Therapeutic drainage of effusions as the sole therapeutic intervention is performed in patients unsuitable for systemic chemotherapy
  • Reference: Mod Pathol 2022;35:1411
Cytology description
Cytology images

Contributed by Carlos A. Murga-Zamalloa, M.D.
Cell block preparation

Cell block preparation

Cell block preparation (CD20)

Cell block preparation (CD20)

Cell block preparation (MUM1)

Cell block preparation (MUM1)

Cell block preparation (HHV8)

Cell block preparation (HHV8)

Cytospin preparation (Wright-Giemsa) Cytospin preparation (Wright-Giemsa)

Cytospin preparation (Wright-Giemsa)

Positive stains
Negative stains
Molecular / cytogenetics description
  • BCL2 rearrangement (8 - 29% of cases)
  • BCL6 rearrangement (11 - 21% of cases)
  • MYC rearrangement (11 - 22% of cases)
  • Recurrent mutations in MYD88 (including L265P) are identified in some cases (Blood 2019;133:377)
Sample pathology report
  • Pleural fluid, thoracocentesis:
    • Involved by HHV8 negative large B cell lymphoma (see comment)
    • Comment: In the right clinical context and with the absence of involvement by lymphoma in solid tissues, the diagnosis is consistent with fluid overload associated LBCL.
Differential diagnosis
  • Primary effusion lymphoma:
    • HHV8 positive
    • Detected in immunodeficient patients
    • Usually negative for pan-B cell markers (CD20, PAX5)
    • Tumor cells are mostly positive for CD30 and CD138
  • Pyothorax associated lymphoma:
    • Occurs in the setting of chronic inflammation and is EBV driven
    • Patients are usually young with a longstanding history of pyothorax or pleuritis secondary to infectious etiologies (e.g., tuberculosis)
    • Associated with solid lesions in the pleura or lung
  • B cell lymphomas with associated pleural effusions:
Board review style question #1

Which of the following is true regarding fluid overload associated LBCL?

  1. Association with tuberculosis related chronic pleural effusions is commonly observed
  2. Clinical course is usually indolent
  3. Diagnosis is usually established from lymph node biopsies
  4. Patients are usually young individuals
  5. Tumor cells are frequently positive for pan-B cell markers
Board review style answer #1
E. Tumor cells are frequently positive for pan-B cell markers, including CD20, PAX5 and CD79a. Answer A is incorrect because in contrast to pyothorax associated lymphoma, fluid overload associated LBCL is not associated with infectious / chronic pleural effusions, such as those associated with tuberculosis. Answer C is incorrect because per definition, no solid lymphoma involvement is detected. Answer B is incorrect because although initial reports suggested indolent clinical behaviors, larger series demonstrate poor clinical outcomes. Answer D is incorrect because this entity is predominantly diagnosed in elderly individuals and the median age of diagnosis is 79 years.

Comment Here

Reference: Fluid overload associated LBCL
Board review style question #2
Which of the following is true regarding fluid overload associated LBCL?

  1. Cases are only delimited to the pleural cavity
  2. Cell of origin is predominantly germinal center type (by the Hans algorithm)
  3. Expression of CD30 is observed in virtually all the cases
  4. Patients are usually elderly immunocompetent individuals
  5. Presence of MYC rearrangements excludes the diagnosis
Board review style answer #2
D. Patients are usually elderly immunocompetent individuals. Most cases are detected in immunocompetent and elderly individuals. Answer A is incorrect because the tumor cells can involve other body cavities different from the pleural cavity, including the pericardial and peritoneal cavities. Answer C is incorrect because only 10 - 27% of the cases feature variable CD30 positive staining. Answer B is incorrect because the majority of the cases are nongerminal center types per the Hans algorithm. Answer E is incorrect because the presence of MYC rearrangements is associated with decreased survival and poor prognosis.

Comment Here

Reference: Fluid overload associated LBCL

Follicular lymphoma-duodenal type
Definition / general
  • Duodenal type follicular lymphoma is a variant follicular lymphoma (FL) characterized by:
    • Morphologic features and immunophenotype similar to nodal follicular lymphoma
    • Distinct entity in 2016 WHO
    • Unlike nodal follicular lymphoma, is almost always diagnosed at a low stage and stays localized to the small intestine
    • Treatment is local, not systemic
Essential features
  • Low grade cytology
  • Indolent clinical course
  • Usually carries the t(14;18) (q32;q21) translocation
Terminology
  • Primary intestinal follicular lymphoma
ICD coding
  • ICD-10: C82.90 - Follicular lymphoma, unspecified, unspecified site
  • ICD-O: 9695/3 - Follicular lymphoma, grade 1
Epidemiology
Sites
Pathophysiology / etiology
  • Origin from memory B cell with germinal center marker expression
  • t(14;18)(q32;q21) translocation: duodenal type follicular lymphoma is part of follicular lymphoma family
  • Immune microenvironment of duodenal type follicular lymphoma is distinct from nodal follicular lymphoma (Blood 2018;132:1695)
  • Some shared features with MALT lymphoma
    • Chronic inflammation signature
    • Restricted immunoglobulin heavy chain variable region genes (IGHV): suggests development related to chronic antigen stimulation (Cancer Sci 2014;105:608)
Diagrams / tables


Nodal Follicular Lymphoma Duodenal Follicular Lymphoma
 Grade 1 - 2 or 3   Grade 1 - 2 
 Stage III or IV   Stage I or II 
 BCL2, CD10, BCL6: Positive    BCL2, CD10, BCL6: Positive 
 AID: Positive   AID: Negative 
 CD21 stain: Dense stain in the center of
 germinal center of follicles 
 CD21 stain: Accentuated staining at the 
 periphery of germinal center of follicles 
 BCL2 and BCL6 rearrangements: +   BCL2 and BCL6 rearrangements: + 
CREBBP mutations present  CREBBP mutations present 
KMT2D mutations present   Lower KMT2D mutations present 
AID: Activation-induced cytidine deaminase
Clinical features
  • Second portion of the duodenum is the most common site
  • Asymptomatic or abdominal symptoms
  • Low grade histology (grade 1 - 2)
  • Low stage presentation
  • Incidental polyp in patients undergoing upper endoscopy
  • Multifocal involvement of the gut by polypoid and nodular lesions
  • Multiple lesions more in the jejunum and ileum rather than duodenum (J Clin Oncol 2011;29:1445)
  • Rarely ulcerated mucosal lesion
Diagnosis
  • Solitary or multiple nodules in endoscopy (1 - 5 mm)
  • Biopsy
Prognostic factors
  • A small subset of patients (< 10%) progress to nodal disease
  • 5 year progression free survival 98% in duodenal type follicular lymphoma with restricted duodenal involvement (Cancer Sci 2011;102:1532)
Case reports
  • 34 year old man with primary multifocal small bowel follicular lymphoma discovered incidentally on diagnostic endoscopy (Clin Gastroenterol Hepatol 2017;15:A27)
  • 52 year old man with duodenal type follicular lymphoma in which histological transformation into diffuse large B cell lymphoma developed 7 years after diagnosis (Oncotarget 2019;10:3424)
  • 52 year old woman who was eventually diagnosed with primary follicular lymphoma of the duodenum showed atypical endoscopic features, namely, erosions with peripheral whitish edematous mucosa (Case Rep Med 2012;2012:582607)
  • 58 year old man who was diagnosed with duodenal type follicular lymphoma and was treated with clarithromycin monotherapy (Case Rep Oncol 2018;11:239)
Treatment
  • Two approaches:
    • Watch and wait approach
    • Localized radiation with or without chemotherapy (+/-Rituximab)
Gross description
  • Almost all diagnoses are made on endoscopic biopsy
Microscopic (histologic) description
  • Several well circumscribed lymphoid aggregates
  • Composed of centrocytes (with infrequent centroblasts)
  • Lymphoma usually involves the mucosa and submucosa
  • Spares the muscularis propria
  • Neoplastic cells commonly involve duodenal villi
  • Hollowed out follicular dendritic cell meshworks (Mod Pathol 2009;22:940)
Microscopic (histologic) images

Contributed by Mahsa Khanlari, M.D.
Missing Image Missing Image

Small intestinal polyp

Missing Image Missing Image

CD10 and CD20 Immunohistochemistries

Missing Image

BCL2 IHC

Missing Image

CD21 IHC



Contributed by @ahmsab_MD on Twitter
Duodenal type follicular lymphoma

Duodenal type follicular lymphoma

Positive stains
Negative stains
Molecular / cytogenetics description
Sample pathology report
  • Biopsy, endoscopic, duodenum:
    • Follicular lymphoma involving duodenum (see comment)
    • Comment: The patient is a 45 year old female with history of vague abdominal pain. Histologic sections consist of endoscopic biopsies of duodenum showing involvement by an atypical lymphoid infiltrate. Lymphoid cells are arranged in a follicular distribution and consist of small lymphoid cells with mature nuclear chromatin and frequent centrocytic features. Centroblasts are rare and scattered.
    • By immunohistochemistry the atypical lymphoid cells are positive for CD20 and CD79a with coexpression of CD10, BCL6, LMO2 and BCL2. CD3 and CD5 highlight reactive T cells surrounding the neoplastic B cell aggregates. Ki67 proliferative rate is low within neoplastic follicles, ~5%. These findings support the above interpretation.
    • Correlation with clinical features and staging procedures is required to determine whether the patient’s follicular lymphoma is a primary duodenal type follicular lymphoma, or a systemic follicular lymphoma secondarily involving duodenum. In the sampled material the lymphoma is of low histologic grade (grade 1).
Differential diagnosis
Board review style question #1
    Which of the following follicular lymphoma variants usually has the t(14; 18) (q32;q21) IGH/BCL2 translocation?

  1. Diffuse follicular lymphoma variant
  2. Duodenal type follicular lymphoma
  3. Pediatric type follicular lymphoma (PTFL)
  4. Primary testicular follicular lymphoma
Board review style answer #1
B. Duodenal type follicular lymphoma

Comment Here

Reference: Duodenal type follicular lymphoma
Board review style question #2
Missing Image Missing Image Missing Image


    54 year old man presents with a dyspepsia. In his upper endoscopy a small polypoid lesion is detected in duodenum. Part of the immunohistochemical stains are provided in this case (left to right: BCL2, BCL6 and Ki67 immunostains). What is the most probable diagnosis?”

  1. Chronic lymphocytic leukemia / small lymphocytic lymphoma
  2. Follicular lymphoma
  3. Mantle cell lymphoma
  4. Reactive lymphoid follicle in a small intestine polyp
Board review style answer #2
B. Follicular lymphoma

Comment Here

Reference: Duodenal type follicular lymphoma

Follicular lymphoma-duodenal type

Follicular neoplasia in situ
Definition / general
  • In situ follicular neoplasia is a monoclonal proliferation of BCL2 positive B cells confined to follicle centers
  • B cells are positive for t(14;18)(q32;q21) (Blood 2002;99:3376)
Essential features
  • Histopathologic and immunohistochemical evaluation are essential
  • Clinical and pathological evidence of usual follicular lymphoma is lacking in other sites
  • BCL2 intensity in germinal centers is strong, stronger than adjacent mantle cuff
  • CD10 is strongly positive
  • Variable number of germinal centers may be involved
  • Lymph node architecture is preserved
Terminology
  • Formerly follicular lymphoma in situ
  • "In situ follicular lymphoma" terminology is discouraged, because risk for clinically significant lymphoma is small
ICD coding
  • ICD-10: D36.0 - benign neoplasm of lymph nodes
  • ICD-0: 9695/1 - in situ follicular neoplasia
Epidemiology
Sites
  • Lymph node otherwise showing follicular hyperplasia
  • May be seen in secondary lymphoid follicles of tonsil, spleen and lymphoid tissue in extranodal sites
Pathophysiology
  • Precursor lesion of conventional follicular lymphoma but progresses to follicular lymphoma in fewer than 5% of cases
  • B cells with BCL2 rearrangement within germinal centers
  • Other than BCL2 rearrangement, low level of genetic aberrations is found
  • Very early step in lymphomagenesis (Blood 2018 Nov 6 [Epub ahead of print])
Etiology
  • High environmental exposure to pesticides and herbicides associated with increase in cells carrying t(14;18)(q32;q21) (IGH / BCL2) translocation in the peripheral blood
Diagrams / tables

Images hosted on other servers:

Development of FL-like B cells

Clinical features
  • Always an incidental finding
  • Risk for clinically significant follicular lymphoma less than 5%
  • Low incidence of other lymphoma subtypes in affected patients, mainly mantle cell lymphoma, chronic lymphocytic leukemia, marginal zone lymphoma and diffuse large B cell lymphoma
Diagnostic criteria
  • BCL2 and CD10 strongly positive in lesional cells
  • Neoplastic cells confined to follicle centers of secondary follicles
  • Lymph node architecture is preserved
  • Lymph nodes may show other pathology or involvement by unrelated lymphoma, usually of B cell lineage
Laboratory
  • Low levels of B cells with the BCL2 rearrangement in the peripheral blood may be found by PCR
  • In blood, referred to as follicular lymphoma-like B cells (Haematologica 2013;98:1571)
Case reports
Treatment
  • Clinical evaluation for follicular lymphoma at other sites recommended at diagnosis
  • No treatment indicated if only in situ follicular neoplasia is found
Gross description
  • No gross finding is evident
  • Involved lymph node may be enlarged secondary to lymphoid hyperplasia or other pathology
Microscopic (histologic) description
  • Lesion generally not evident on routine H&E stained sections
  • Lymph node architecture is intact
  • Involved follicles are generally normal in size
  • The neoplastic cells (BCL2 strong+, CD10 strong+) are exclusively centrocytes, confined to the follicle centers
  • Individual germinal centers show varying degrees of involvement
  • By definition, no interfollicular infiltration is seen
  • Involved follicles show low proliferation fraction with Ki67
Microscopic (histologic) images

Contributed by Elaine S. Jaffe, M.D.

H&E overview

BCL2 overview

CD10 overview

BCL2 neoplastic follicle

H&E neoplastic follicle

CD10 neoplastic follicle


H&E reactive follicle

BCL2 reactive follicle

CD10 reactive follicle

Positive stains
  • Strong expression of BCL2 and CD10
  • BCL6 is variably positive
  • B cell phenotype intact: CD20, CD19, CD79a, PAX5
  • Low proliferation index (Ki67 ~10%) in contrast to reactive germinal center
Negative stains
Flow cytometry description
Molecular / cytogenetics description
Differential diagnosis
Board review style question #1
Which of the following is characteristic of in situ follicular neoplasia?

  1. Bone marrow involvement is common at diagnosis
  2. Common in childhood
  3. Lymphocytosis is a common finding
  4. Tend to involve extranodal sites
  5. Typically an incidental finding
Board review style answer #1
E. It is typically an incidental finding.

Comment here

Reference: Follicular neoplasia in situ
Board review style question #2
Which of the following is true about in situ follicular neoplasia?

  1. Composed exclusively of centrocytes
  2. High mitotic rate
  3. Interfollicular infiltration of neoplastic B cells is common
  4. Positive for BCL6 rearrangement
  5. Positive for CCND1 rearrangement
Board review style answer #2
A. In situ follicular neoplasia has a monomorphic appearance and is composed exclusively of centrocytes.

Comment here

Reference: Follicular neoplasia in situ

Follicular peripheral T cell lymphoma (F-PTCL)
Definition / general
  • A rare subtype of nodal lymphomas of T follicular helper (Tfh) cell origin with a follicular or perifollicular growth pattern exhibiting strong and consistent expression of T follicular helper markers
  • These cases distinctly lack histologic features commonly associated with angioimmunoblastic T cell lymphoma (AITL), such as interfollicular polymorphous infiltrate, proliferation of high endothelial venules or expanded follicular dendritic cell meshworks
Essential features
  • Clinical and laboratory findings similar to AITL
  • Histologically characterized by a follicular or perifollicular growth pattern with consistent expression of Tfh markers (2, preferably 3 markers)
  • Lacks histologic features commonly associated with angioimmunoblastic T cell lymphoma (AITL), including proliferation of interfollicular high endothelial venules or expanded follicular dendritic cell meshworks
  • t(5;9)(q33;q22) ITK-SYK rearrangement seen in about 20% of cases
Terminology
  • Follicular peripheral T cell lymphoma (F-PTCL)
  • Follicular T cell lymphoma (FTCL)
  • Peripheral T cell lymphoma with a nodular, follicular or perifollicular pattern
  • Peripheral T cell lymphoma with follicular involvement
ICD coding
  • ICD-O: 9702/3 - mature T cell lymphoma, NOS
Epidemiology
Sites
  • Most commonly presents as disseminated / advanced stage nodal disease
  • Less commonly at extranodal sites: skin, bone marrow, liver and spleen
Pathophysiology
  • F-PTCL shares with AITL not only morphologic features but also similar clinical features, genetic landscape and molecular signatures, which may suggest a common pathophysiology (Haematologica 2017;102:e148)
Etiology
Clinical features
Diagnosis
  • Diagnosis by a constellation of morphologic and immunophenotypic profiles on tissue biopsy (i.e. lymph node)
    • Demonstrates a mature T follicular helper phenotype and does not meet criteria for AITL in the WHO classification (Blood 2016;127:2375)
  • Staging: bone marrow biopsy
Laboratory
  • Positive Coombs test or hypergammaglobulinemia, findings typically associated with AITL, can occasionally be seen in FTCL
  • Increased LDH
  • Reference: Am J Surg Pathol 2009;33:682
Prognostic factors
  • Typically aggressive course with reported 50% mortality within 24 months of diagnosis (Am J Surg Pathol 2009;33:682)
  • Occasional indolent / protracted with localized presentation
Case reports
Treatment
  • Distinction between F-PTCL and PTCL, NOS has currently no impact on clinical management
    • New therapeutic approaches and targeted interventions, such as hypomethylating agents, are promising (Br J Haematol 2015;168:913)
Microscopic (histologic) description
  • 2 main architectural patterns in lymph nodes:
    • Progressive transformation of germinal center (PTGC)-like pattern
      • Well defined aggregates of T cell lymphoma cells immersed by numerous IgD positive B lymphocytes appearing as PTGC with H&E stain
    • Follicular lymphoma-like pattern: most common
      • Tumor cells form intrafollicular aggregates or nodules confined within lymphoid follicles often with attenuated / absent mantle zones
    • Follicular lymphoma-like and PTGC-like patterns are frequently together
  • Neoplastic cells are intermediate in size, monotonous with vesicular to coarsely granular nuclei, prominent nucleoli, moderate or abundant, clear or pale eosinophilic cytoplasm and only mild atypia
  • Interfollicular areas lack the polymorphic infiltrate and vascular proliferation characteristic of AITL
    • However, the presence of enlarged B cells, Hodgkin / Reed-Sternberg (HRS)-like cells or epithelioid histiocytes is common in interfollicular areas (Am J Surg Pathol 2012;36:1636)
  • Diagnosis of FTCL at extranodal sites can be highly challenging unless the diagnosis already has been established
  • Skin lesions typically manifest with an extensive pandermal nodular lymphocytic infiltrate with admixed histiocytes; variable extension into the subcutaneous fat is noted (Am J Dermatopathol 2017;39(5):374)
  • Bone marrow pattern of involvement by FTCL has been reported to be paratrabecular or interstitial (Hum Pathol 2012;43:1789)
  • FTCL at extranodal mucosal sites show association with lymphoepithelial lesions, similar to those of extranodal marginal zone B cell lymphoma of mucosa associated lymphoid tissue (MALT lymphoma) (Mod Pathol 2002;15:420)
Microscopic (histologic) images

Contributed by Nicholas Ward, M.D. and Roberto N. Miranda, M.D.
Mixed histology pattern

Mixed histology pattern

Atypical lymphoid aggregates

Atypical lymphoid aggregates

CD3

CD3

CD20

CD20


PD-1

PD-1

ICOS

ICOS

CD10

CD10

IgD

IgD


Neoplastic lymphocytes

Neoplastic lymphocytes

CD3

CD3

CD20

CD20

CD21

CD21

PD-1

PD-1

Cytology description
  • Broad spectrum of cytologic features (cannot diagnose based on cytology alone)
  • Neoplastic infiltrate typically features a monotonous lymphoid infiltrate exhibiting vesicular nuclei, prominent nucleoli, abundant pale / clear cytoplasm and only mild atypia; Hodgkin / Reed-Sternberg (HRS)-like cells or epithelioid histiocytes are both seen in most cases (Hum Pathol 2012;43:1789)
Positive stains
  • Pan T cell antigens CD2, CD3 and CD5 (with frequent loss of CD7)
  • Helper T cell phenotype CD4+ and CD8- (rare CD4 / CD8 double negative cases)
  • At least 2 (preferably 3) TFH cell markers (e.g. PD-1, CXCL13, BCL6, CD10 and ICOS)
    • Almost all cases show positivity for PD-1 or ICOS and most cases are positive for CD10, BCL6 or CXCL13
  • Largely intact follicular dendritic cell meshwork structure: CD21, CD23 and CD35 staining within remnant follicles, without evidence for expansion or arborization
  • Hodgkin / Reed-Sternberg-like large cells: CD30, CD15, PAX5 (weakly) and frequently EBV (Am J Surg Pathol 2013;37:816)
  • Ki67 immunoreactivity variable, ranging from 5 - 70%; median and mean values of approximately 45% (Hum Pathol 2012;43:1789)
Negative stains
Flow cytometry description
  • Flow cytometry may be helpful in identifying aberrant T cell population
  • CD3- / dim CD4+ T cell population is commonly identified in cases of peripheral T cell lymphomas of follicular type (Mod Pathol 2016;29:1173)
Flow cytometry images

Images hosted on other servers:

T cells expressing CD4 without surface CD3 (n, o, p, q)

Significant population of CD3- / dim CD4+ T cells (b, c)

Molecular / cytogenetics description
Molecular / cytogenetics images

Images hosted on other servers:

FISH for the detection of t(5;9)(q33;q22)

5' and 3' RACE identifies ITK-SYK fusion

Sample pathology report
  • Lymph node, neck, right, excision:
    • Follicular T cell lymphoma (see comment)
    • Comment: Sections show an enlarged lymph node featuring numerous follicles associated with a perifollicular expansion of neoplastic T cells that, by immunohistochemical staining, are consistent with that of T follicular helper cell derivation. No significant polymorphous infiltrate, proliferation of high endothelial venules or expanded follicular dendritic cell meshworks are noted. Overall, findings are consistent with that of a follicular T cell lymphoma.
    • Microscopic description: H&E stained sections show lymph node tissue featuring a PTGC-like pattern with well defined aggregates associated with dispersed interfollicular lymphoid aggregates of neoplastic cells. The neoplastic cells are medium sized with clear cytoplasm and irregular nuclei. A CD3 stain highlights interfollicular aggregates of neoplastic T cells with coexpression of Tfh markers: PD-1, ICOS and CXCL13. CD20 immunostain reveals numerous follicles as well as interfollicular IgD positive B cells, which surround neoplastic T cells. CD21 demonstrates intact follicular dendritic cell meshworks as well as staining interfollicular B cells.
Differential diagnosis
  • Angioimmunoblastic T cell lymphoma:
    • Associated with polymorphous infiltrate, florid proliferation of high endothelial venules and enlarged follicular dendritic cell meshworks
    • Subset of cases may recur as FTCL and vice versa, cases of FTCL can recur as AITL, supporting the notion that AITL and FTCL are part of the same spectrum
  • Peripheral T cell lymphoma with a T follicular helper phenotype:
  • Classic Hodgkin lymphoma (J Clin Exp Hematop 2021;61:97):
    • HRS-like cells have been reported in interfollicular areas of FTCL cases
    • Immunophenotype of the HRS-like cells in F-PTCL may overlap with the phenotype of classic Hodgkin lymphoma
    • Assessment of the neoplastic T cell population for expression of CD10 and CD30 or the presence of a T cell clone favor a diagnosis of F-PTCL rather than classic Hodgkin lymphoma
  • Follicular lymphoma:
    • Monotonous lymphoid infiltrate with abundant pale / clear cytoplasm and only mild atypia with notable absence of centrocytic / centroblastic morphology suggest the diagnosis of F-PTCL rather than follicular lymphoma
    • IHC, flow cytometry and FISH for IGH-BCL2 translocation may limit confusion with FTCL
  • Methotrexate associated LPD with features of FTCL (Pathol Int 2021 Sep 2 [Epub ahead of print]):
    • Histopathology and immunophenotype similar to FTCL
    • No apparent loss of T cell antigens; no monoclonality demonstrated
    • Lesion may regress upon removal of methotrexate
  • Reactive changes / paracortical hyperplasia:
    • Staining for CD4 and CD8 populations would reveal an admixture as opposed to a typically CD4 predominating infiltrate in F-PTCL
    • Lacks the occasional cytologic T cell atypia seen in FTCL
    • May express Tfh markers (Am J Clin Pathol 2021;156:409)
Board review style question #1
A 54 year old woman presented with cervical lymphadenopathy. Lymph node biopsy showed tumor cells predominantly confined within lymphoid follicles. The cells are medium sized with clear cytoplasm and irregular nuclei. Immunohistochemical stains showed that the tumor cells are positive for CD3, CD5, CD4, CD10, ICOS and PD-1 while negative for CD7, CD8 and CD20. Which of the following features would favor a diagnosis of follicular T cell (FTCL) lymphoma over angioimmunoblastic T cell lymphoma (AITL)?

  1. Absence of interfollicular high endothelial venules or expanded follicular dendritic cell meshworks
  2. Interfollicular polymorphous infiltrate
  3. Ki67 proliferation index of 40%
  4. Neoplastic cells with a centrocytic / centroblastic morphology
  5. Large Hodgkin-like cells positive for CD15 and CD30
Board review style answer #1
A. Absence of interfollicular high endothelial venules or expanded follicular dendritic cell meshworks. The main differential of follicular T cell lymphoma is AITL. FTCL is usually confined to lymphoid follicles, while most cells in AITL are extrafollicular and associated with a polymorphous infiltrate, proliferation of high endothelial venules and expanded and distorted follicular dendritic cell meshworks. Centrocytes and centroblasts are features of follicular (B cell) lymphoma.

Comment Here

Reference: Follicular peripheral T cell lymphoma (F-PTCL)
Board review style question #2

Which of the following translocations is seen in up to 20% of cases of follicular T cell lymphoma?

  1. t(1;19)(q23;p13): TCF3-PBX1
  2. t(5;9)(q33;q22): ITK-SYK
  3. t(8;9)(p22;p24.1): PCM1-JAK2
  4. t(9;22)(q34.1;q11.2): BCR-ABL1
  5. t(11;14)(q13;q32): IGH-CCND1
Board review style answer #2
B. t(5;9)(q33;q22): ITK-SYK

Comment Here

Reference: Follicular peripheral T cell lymphoma (F-PTCL)

Follicular-usual
Definition / general
  • Mature B cell neoplasm
  • Malignancy of germinal center B cells (centrocytes and centroblasts)
  • Biologically and histopathologically heterogeneous
  • Indolent lymphoma
Essential features
  • Characterized by at least partial follicular (nodular) distribution
  • Entirely diffuse pattern in rare cases
  • Cytologic composition (centrocyte / centroblast) used for lymphoma grading
    • More important than pattern (follicular / diffuse) in diagnosis / prognosis
  • Germinal center marker expression
Terminology
ICD coding
  • ICD-O: 9690/3 - follicular lymphoma
  • ICD-O: 9695/3 - grade 1
  • ICD-O: 9691/3 - grade 2
  • ICD-O: 9698/3 - grade 3A
  • ICD-O: 9698/3 - grade 3B
  • ICD-10: C82.90 - follicular lymphoma, unspecified, unspecified site
Epidemiology
Sites
  • Cervical and inguinal lymph nodes are more frequently affected
  • Extranodal sites that are commonly involved include bone marrow, spleen, liver and peripheral blood
Pathophysiology
  • Bone marrow event (Nat Rev Dis Primers 2019;5:83)
    • t(14;18) (q32;q21) translocation: repair failure during V(D) J recombination
    • t(14;18) (q32;q21) IGH / BCL2: 85% of follicular lymphomas
    • BCL2 expression in precursor follicular lymphoma cells: increase survival in germinal center
  • Germinal center events
    • Retain germinal center functionality (e.g. BCL6)
    • Class switch recombination with retention of IgM / IgD surface expression
    • This phenomenon is called allelic paradox
  • Postgerminal center events
    • Chromosomal alterations
    • Molecular changes
      • Chromatin modification (KMT2D, EZH2, CREBBP, ARID1A, MEF2B, EP300)
      • B cell receptor signaling (CARD11, IgHV, IgLV)
      • Cell cycle regulation (RB1, CDK4)
      • Transcription factor (FOXO1, MEF2B)
      • Immune evasion (EPHA7, TNFRSF14, CREBBP)
Etiology
  • Unknown etiology
  • Family history is important (especially first degree relatives) (Br J Haematol 2009;146:91)
  • Environmental factors: exposure to pesticides and herbicides
  • t(14;18) alone is not sufficient to cause lymphoma (Haematologica 2014;99:802)
  • Natural history of the disease starts in bone marrow and not germinal center (GC)
Diagrams / tables

World Health Organization grading of follicular lymphoma
Grade  Definition Pattern Immunohistochemistry and cytogenetics 
1 0 - 5 centroblasts/high power field  Follicular or diffuse IHC:
CD10: + (95 - 100%)
BCL2: + (85 - 90%)
FISH:
BCL2 translocation: + (80 - 90%)
BCL6 rearrangement: + (3%)

Ki67: < 20%*
2 6 - 15 centroblasts/high power field Follicular or diffuse
3A > 15 centroblasts/high power field
Centrocytes present
Follicular
If diffuse component:
Reported as diffuse large B cell
lymphoma and follicular lymphoma
(% of each component is reported); 
correlate with clinical features and
overall grade in cases with small
areas of diffuse pattern
IHC:
CD10: + (80 - 95%)
BCL2: + (50 - 75%)
FISH:
BCL2 translocation: + (60 - 70%)
BCL6 rearrangement: + (30 - 40%)

Ki67: > 20%
3B > 15 centroblasts/high power field
Lack centrocytes
Follicular
If diffuse component:
Reported as diffuse large B cell
lymphoma and follicular lymphoma
(% of each component is reported)
IHC:
CD10: + (40 - 85%)
BCL2: + (45 - 75%)
FISH:
BCL2 translocation: + (15 - 30%)
BCL6 rearrangement: + (40 - 50%)
CD10-IRF4/MUM1+: common

Ki67: > 50%

Notes:
Clinical features
Diagnosis
  • Symptoms: fatigue, fever or night sweats, weight loss or recurrent infections
  • Most patients with follicular lymphoma have no obvious symptoms of the disease
  • Tissue biopsy (lymph node / extranodal sites)
    • Needle biopsy (both core needle biopsy and fine needle aspiration biopsy)
  • Peripheral blood exam / bone marrow biopsy (especially in staging)
  • Reference: Nat Rev Dis Primers 2019;5:83
Laboratory
Radiology description
  • Widespread lymphadenopathy
Prognostic factors
  • Median overall survival of > 15 years
  • 5 year progression free survival of ~60%
  • Follicular lymphoma frequently transforms into an aggressive subtype
  • Short remission after treatment: poor prognosis (J Clin Oncol 2015;33:2516)
  • The Follicular Lymphoma International Prognostic Index (FLIPI):
    • Age (≥ 60 versus < 60 years)
    • Ann Arbor stage (III - IV versus I - II)
    • Hemoglobin level (< 120 versus ≥ 120 g/L)
    • Number of nodal areas involved (> 4 versus ≤ 4)
    • Serum lactate dehydrogenase (LDH) level (above normal versus normal or below)
  • FLIPI2 system (patients treated with rituximab)
    • High serum beta 2 microglobulin
    • Largest involved lymph node > 6 cm
    • Bone marrow involvement
    • Hemoglobin < 12 g/dL
    • Age > 60 years
  • M7 FLIPI (FLIPI combined with 7 genes) better identify the risk
  • Pathologic adverse prognostic factors
    • High histologic grade and diffuse areas > 25% with predominance of large cells (diffuse large B cell lymphoma)
    • High proliferation index
    • Low grade follicular lymphoma with high proliferation index behave similar to high grade follicular lymphoma
    • Partial involvement by follicular lymphoma has better prognosis
  • Cytogenetic adverse prognostic factors
    • Complex karyotype
    • Both BCL2 and a MYC rearrangement
    • Del6q25-27; del17p and mutations of TP53
Case reports
  • 55 year old woman presented with a 6 month history of a slow growing mass in her right neck diagnosed as floral variant of follicular lymphoma (Am J Hematol 2011;86:503)
  • 64 year old woman diagnosed with follicular lymphoma grade 1 and endometrioid carcinoma transformed to double hit lymphoma during chemotherapy (Int J Hematol 2019;110:375)
  • Low grade follicular lymphoma with evolution to a blastic phase (Am J Clin Pathol 1985;83:681)
  • Transformation of primary extranodal marginal zone B cell lymphoma of the uterus into a nodal follicular lymphoma grade IIIB (Ann Hematol 2019;98:797)
Treatment
  • Stage I or contiguous stage II (nonbulky)
    • Localized radiotherapy
  • Stage I or stage II (bulky) or noncontiguous stage II
    • Anti-CD20 monoclonal antibody with or without chemotherapy or radiotherapy
    • Observation, if clinically indicated
  • Stage III or stage IV
    • Anti-CD20 monoclonal antibody with or without chemotherapy (with or without maintenance anti-CD20)
    • Observation, if clinically indicated
  • Reference: Nat Rev Dis Primers 2019;5:83
Gross description
Frozen section description
  • Same criteria applied in microscopic evaluation
  • Cytogenetics, flow cytometry, preparation of imprints should be considered
Microscopic (histologic) description
  • Lymph node
    • Partial or complete effacement of lymph node
    • Neoplastic follicles:
      • Similarly sized
      • Nonpolarized
      • Attenuated or absent mantle zones
      • Lack tingible body macrophages
      • Composed of centrocytes or centroblasts
      • Infrequent mitoses (except grade 3)
    • With or without sclerosis
    • Grading: proportion of centroblasts defines the grade
    • Follicular dendritic cell meshworks are absent in diffuse areas
    • Partial involvement by follicular lymphoma
      • Both neoplastic and reactive follicles are present
      • Lymph node architecture is partially preserved
  • Floral variant
    • Irregular neoplastic follicles
    • Darker mantle zone lymphocytes penetrate into neoplastic follicles
    • Grade 3 in most cases
    • Resembles progressively transformed germinal center
  • Signet ring cell variant (differential diagnosis: carcinoma)
  • Other uncommon variants (refer to unusual variants of follicular lymphomas)
  • Bone marrow
    • Focal or extensive involvement
    • Paratrabecular aggregates of lymphoma cells
    • With or without interstitial or diffuse patterns
    • Pure follicular pattern in ~5% (Am J Clin Pathol 2002;118:780)
  • Liver
    • Portal tracts are preferentially involved
    • Large nodule in parenchyma in extensive disease
  • Spleen
    • Preferential involvement of white pulp
    • Two patterns
      • Expansion of the white pulp
      • Relatively preserved architecture
Microscopic (histologic) images

Contributed by Jennifer Chapman, M.D.

Follicular lymphoma

Back to back follicles

Mantle zone

Low grade morphology

CD20

CD3


Germinal center markers (CD10 and BCL6)

BCL2

Follicular dendritic cell meshworks

High grade morphology


Paratrabecular pattern of involvement in bone marrow

Cytology description
  • Variable mixture of centrocytes and centroblasts
    • Centrocytes
      • Small to large
      • Angulated nuclei
      • Dense chromatin
      • Scant cytoplasm
    • Centroblasts
      • Large cells with oval nuclei
      • Vesicular chromatin
      • 1 - 3 nucleoli
      • Moderate cytoplasm
      • > 3 times the size of lymphocytes
      • Differentiate from follicular dendritic cells
        • Large round nuclei
        • Dispersed, nearly clear chromatin
        • Single eosinophilic nucleolus
        • Indistinct cytoplasm
    • Absence / rare tingible body macrophages
  • Reference: Jaffe: Hematopathology, 2nd Edition, 2016
Cytology images

Contributed by Mahsa Khanlari, M.D.

Fine needle aspirate smear

Peripheral smear description
  • Low level involvement is common (~90%)
  • Absolute lymphocytosis with high count, 5 - 10% of cases (Br J Haematol 2014;164:659)
  • Neoplastic cells are small to intermediate in size
  • Highly indented nuclei, known as buttock cells
Peripheral smear images

Contributed by Jennifer Chapman, M.D.

Peripheral blood smear involvement

Positive stains
  • Monotypic surface Ig
    • lgM with or without lgD is the most common surface Ig
  • Pan-B cell markers: CD19, CD20, CD22 and CD79a
  • Germinal center markers: CD10, BCL6, HGAL, LMO2
    • Grade 3B can be more commonly negative
    • CD10 and BCL6 are weaker in interfollicular tumoral cells
  • BCL2 (variable)
    • Positive in 85 - 90% of follicular lymphoma grades 1 and 2
    • It may be negative in grade 3
    • False negative: mutations of the BCL2 epitope (anti-BCL2 antibody used)
  • Follicular dendritic cell meshworks in follicles
  • Proliferation rate of follicular lymphomas assessed by Ki67
    • Percentage correlates with grade
    • ~20% of low grade follicular lymphomas have high proliferation rate (> 40%)
    • These follicular lymphomas appear to behave more aggressively, similar to grade 3A follicular lymphoma
    • Report as follicular lymphoma grade 1 to 2 with a high proliferation fraction
    • Assess Ki67 in follicles
      • If interfollicular area > follicles, estimate Ki67 based on average for entire neoplasm
  • Reference: Swerdlow: WHO Classification of Tumours of Haematopoietic and Lymphoid Tissues, 4th Edition, 2017
Negative stains
Flow cytometry description
  • CD20 bright+, CD19 dim+, CD10+, CD5-, cyclin D1-
Flow cytometry images

Contributed by Mahsa Khanlari, M.D.

Flow cytometric immunophenotyping

Flow cytometric immunophenotyping, monotypic population

Molecular / cytogenetics description
  • Monoclonal immunoglobulin (Ig) heavy and light chain gene rearrangements
  • FISH can detect t(14;18)(q32;q21) IGH-BCL2 in up to 90% of follicular lymphoma cases
  • FISH is more sensitive than PCR based approaches
    • Variation in breakpoint regions
  • Juxtaposition of the Ig light chain promotors to BCL2: t(2;18) and t(18;22)
  • 3q27 BCL6 rearrangement or amplification mostly in follicular lymphoma 3B
  • Loss of 1p36 which contains TNFRSF14 is common
  • MYC rearrangement / activation of MYC is rare in follicular lymphoma (< 5%)
    • Associated with transformation to diffuse large B cell lymphoma
    • Categorized as double hit lymphoma if transformed and also has BCL2 or BCL6 translocation
  • Other genetic alterations associated with transformation to diffuse large B cell lymphoma
    • Inactivation of TP53 and CDKN2A
  • Frequent genetic alterations
    • BCL2 > KMT2D / MLL2 > EPHA7 > EZH2 > BCL6 > CREBBP > TNFRSF14
  • Less common genetic alterations
    • ARID1A, CARD11, TNFAIP3, MYC, FOXO1, TP53
  • Comparative genomic hybridization:
    • Gains: 1, 2p15, 6p, 7p, 7q, 8q, 12q, 18p, 18q, X
    • Losses: 1p36, 3q, 6q, 9p, 10q, 11q, 13q, 17p
  • References: Medeiros: Diagnostic Pathology - Lymph Nodes and Extranodal Lymphomas, 2nd Edition, 2017, Swerdlow: WHO Classification of Tumours of Haematopoietic and Lymphoid Tissues, 4th Edition, 2017
Molecular / cytogenetics images

Contributed by Mahsa Khanlari, M.D.

IGH-BCL2 dual color FISH

Sample pathology report
  • Inguinal lymph node, core needle biopsy:
    • Follicular lymphoma, low grade, grade 1 - 2 / 3, follicular pattern (see comment)
    • Comment: This patient presented recently with inguinal lymphadenopathy. Histologic sections consist of a core needle biopsy of lymph node demonstrating complete nodal architectural effacement by malignant lymphoma. Lymphoma cells are arranged in an entirely follicular distribution. Neoplastic follicles are composed predominantly of small cleaved cells (centrocytes) with fewer large noncleaved cells (centroblasts), the latter of which comprise < 15 cells per high power field (grade 1 - 2 / 3).
    • Immunoperoxidase studies confirm that the tumor cells are positive for CD20, CD10, BCL6, LMO2 and BCL2 (dim) and negative for CD3 and CD5. The proliferation rate of lymphoma cells, measured by Ki67 (MIB1), is ~10%. These morphologic and immunophenotypic findings support the above interpretation.
Differential diagnosis
  • Reactive follicular hyperplasia:
    • Follicles mostly in cortex
    • Widely spaced follicles
    • Variation in size and shape of follicles
    • Polarization of germinal centers into light and dark zones (higher proliferation)
    • Frequent mitoses
    • Tingible body macrophages in germinal centers
    • Sharply demarcated mantle zones
    • No evidence of monoclonality by IG rearrangement
    • Negative for t(14;18)(q32;q21)
  • Progressive transformation of germinal centers:
    • Nodules are 3 - 5x larger than background reactive follicles
    • It may be hard to separate from floral variant of follicular lymphoma on morphology
    • Germinal center B cells are BCL2-
    • No evidence of monoclonality
    • Negative for t(14;18)(q32;q21)
  • Castleman disease, hyaline vascular variant:
    • Follicles are widely spaced
    • Concentric mantle zones
    • Depleted germinal centers
    • Prominent follicular dendritic cells
    • BCL6- in most of the lymphoid cells of atretic follicles
    • Refer to unusual variants of follicular lymphomas (follicular lymphoma with Castleman-like features)
  • Nodular lymphocyte predominant Hodgkin lymphoma:
    • Larger nodules than follicular lymphoma
    • Vaguely circumscribed nodules
    • Most cells in nodules are small round lymphocytes
    • Absent centrocytes and centroblasts
    • Presence of lymphocyte predominant (LP) cells
    • LP cells and small B lymphocytes are CD10- and BCL2-
    • Negative for t(14;18)(q32;q21)
  • Lymphocyte rich classic Hodgkin lymphoma:
    • Large nodules with eccentrically located germinal centers
    • Admixed Reed-Sternberg and Hodgkin (RS+H) cells
    • RS+H cells are CD15+, CD30+, CD45-
    • BCL6+ germinal center B cells do not coexpress BCL2
  • Mantle cell lymphoma:
    • Can resemble low grade follicular lymphoma if in nodular or mantle zone pattern
    • Lymphoma cells are usually CD5+, CD43+, cyclin D1+, CD10-
    • t(11;14)(q13;q32) / IGH-CCND1
  • Nodal marginal zone lymphoma:
    • Lymphocytes with monocytoid features
    • Frequent plasmacytic differentiation
    • Colonize germinal centers
    • Monotypic B cell population; BCL2+
    • CD5-, CD10-, cyclin D1-, BCL6-, LMO2-
    • IRTA1+, MNDA+
Board review style question #1
All of the following genes are frequently associated with large cell transformation in follicular lymphoma, except

  1. BCL6
  2. CDKN2A
  3. MYC
  4. TP53
Board review style answer #1
Board review style question #2
Which of the following is true of follicular lymphoma?

  1. Abnormal expression of BCL2 protein by germinal center B cells is required for diagnosis of follicular lymphoma
  2. Follicular lymphoma is generally systemic in nature and stage IV at initial diagnosis
  3. Grading in follicular lymphoma is based on proliferative rate, as measured by Ki67 immunostaining
  4. High grade follicular lymphomas more frequently show upregulation of BCL2 protein expression compared to low grade follicular lymphomas
Board review style answer #2
B. Follicular lymphoma is generally systemic in nature and stage IV at initial diagnosis

Comment Here

Reference: Follicular lymphoma, nodal, usual variant

Graft versus host disease
Definition / general
  • Common complication of allogeneic bone marrow transplantation seen in 20 - 50% of HLA identical and 70% of nonidentical / unrelated recipients
  • Causes 1/3 of bone marrow transplant deaths

  • Acute GVHD: within 100 days of transplant; affects skin, GI tract, liver
  • Skin - maculopapular rash of palms, soles, trunk, later entire body; may progress to bullous patches and desquamation
  • GI - diarrhea (profuse, bloody), nausea, abdominal pain
  • Liver - elevated liver function tests, nausea, vomiting
  • Also oral GVHD with xerostomia

  • Chronic GVHD: after 200 days of transplant; autoimmune-like symptoms, wasting, recurrent infections, prolonged immunodeficiency
Pathophysiology
  • Due to donor T cytotoxic (CD8+) T cells introduced with bone marrow cells as bystanders that attack recipient tissue
  • Donor T cells recognize host HLA antigens, proliferate and secrete Interleukin-2 and then other cytokines including tumor necrosis factor, IL-1 and interferon, causing tissue damage
Treatment
  • Increased immunosuppression, irradiation (treatment opposite that of posttransplant lymphoproliferative disease)
Microscopic (histologic) description
    Skin: Transplant Proc 1974;6:367
  • Grade 1: vacuolar degeneration of basal epithelial cells or acanthocytes
  • Grade 2: also dyskeratosis, apoptosis of keratinocytes (eosinophilic bodies) surrounded by lymphocytes, spongiosis, edema of overlying epithelium
  • Grade 3: also splitting and degeneration of acanthocytes and basal cells, causing cleft formation and separation of dermoepidermal junction
  • Grade 4: sloughing of overlying epithelium

    Rectum:
  • Early: flatting / atrophy of mucosa, degeneration and loss of crypts; occasional apoptotic bodies surrounded by lymphocytes
  • Late: mucosal sloughing

    Salivary glands:
  • Grade 1: abnormal mononuclear infiltrates with or without ductal epithelial necrosis
  • Grade 2: also obliteration of ducts
Differential diagnosis
  • Chemotherapy effect, drug reaction, infection

HHV8 related DLBCL, NOS
Definition / general
  • HHV8 related diffuse large B cell lymphoma (DLBCL) is a neoplasm characterized by a monoclonal proliferation of HHV8 infected lymphoid cells that resemble plasmablasts and express IgM lambda
  • Neoplastic cell corresponds to a naïve IgM producing B cell without immunoglobulin somatic hypermutations
Essential features
  • Usually arises in association with multicentric Castleman disease (MCD) and HIV infection
  • Morphologically similar to plasmablasts but correspond to naïve IgM producing B cell without somatic hypermutations
  • Sheets of HHV8 latency associated nuclear antigen (LANA1) positive plasmablasts efface lymph node architecture
  • Atypical cells are strongly positive for cytoplasmic IgM with lambda light chain restriction and negative for EBV encoded small RNA (EBER)
Terminology
  • HHV8+ large B cell lymphoma arising in multicentric Castleman disease (obsolete)
  • HHV8 positive plasmablastic lymphoma (obsolete)
  • HHV8+ diffuse large B cell lymphoma, not otherwise specified
ICD coding
  • ICD-O: 9738/3 - large B cell lymphoma arising in HHV8 associated multicentric Castleman disease
  • ICD-10:
    • C83.3 - diffuse large B cell lymphoma
    • C83.30 - diffuse large B cell lymphoma, unspecified site
    • C83.31 - diffuse large B cell lymphoma, lymph nodes of head, face and neck
    • C83.32 - diffuse large B cell lymphoma, intrathoracic lymph nodes
    • C83.33 - diffuse large B cell lymphoma, intra-abdominal lymph nodes
    • C83.34 - diffuse large B cell lymphoma, lymph nodes of axilla and upper limb
    • C83.35 - diffuse large B cell lymphoma, lymph nodes of inguinal region and lower limb
    • C83.36 - diffuse large B cell lymphoma, intrapelvic lymph nodes
    • C83.37 - diffuse large B cell lymphoma, spleen
    • C83.38 - diffuse large B cell lymphoma, lymph nodes of multiple sites
    • C83.39 - diffuse large B cell lymphoma, extranodal and solid organ sites
  • ICD-11: 2A81.Y - other specified diffuse large B cell lymphomas (large B cell lymphoma arising in HHV8 associated multicentric Castleman disease)
Epidemiology
Sites
  • Characteristically involves lymph nodes or spleen
  • Can disseminate to other viscera including liver, lungs and gastrointestinal tract
  • Can also manifest as a leukemia with involvement of peripheral blood (Blood 2002;99:2331)
Pathophysiology
  • HHV8 infects singly scattered cells that are morphologically similar to plasmablasts with abundant cytoplasmic immunoglobulin but are, in fact, naïve IgM producing B cells that lack IG somatic hypermutations and are polyclonal at the molecular level
  • HHV8 positive plasmablasts form aggregates (microlymphomas), which can be polyclonal or monoclonal (Am J Clin Pathol 2017;147:171)
  • HHV8 positive aggregates rarely can expand to HHV8 positive large B cell lymphoma, which is monoclonal
Etiology
  • By definition, the large plasmablastic cells in all cases are positive for HHV8
  • Molecular mechanism is similar to other HHV8 positive entities (Blood 2004;104:3349)
Clinical features
  • Usually manifests with profound immunodeficiency, enlarging lymph nodes and massive splenomegaly
  • There may be manifestations of Kaposi sarcoma (Blood 2002;99:2331)
  • Rarely, may arise in absence of MCD (Mod Pathol 2017;30:745)
Diagnosis
  • Biopsy of involved lymph node with immunohistochemistry
  • Effacement of lymph node architecture by sheets of HHV8 positive, LANA1 positive and EBER negative plasmablasts (Am J Clin Pathol 2017;147:171)
Laboratory
  • Rarely, there is involvement of bone marrow and peripheral blood by HHV8 positive IgM lambda plasmablasts (Eur J Haematol 2013;91:497)
Prognostic factors
  • Extremely aggressive clinical course and poor prognosis
Case reports
  • 37 and 42 year old men with multifocal hepatic Kaposi sarcoma and intravascular HHV8 positive (EBV negative) systemic DLBCL, NOS (Autops Case Rep 2020;10:e2020206)
  • 56 year old man with an unusual EBV associated HHV8 positive large B cell lymphoma (Clin Case Rep 2019;7:587)
  • 59 year old woman with hemoglobin sickle cell disease and sequential development of HHV8 positive DLBCL and chronic myelomonocytic leukemia (Pathol Res Pract 2019;215:152704)
Treatment
  • No definite treatment guidelines
  • Can be treated with the same regimens described for patients with HIV associated DLBCL
Gross description
  • Involved lymph node is enlarged
  • Firm, fleshy mass
Microscopic (histologic) description
  • Effacement of lymph node / splenic architecture by confluent sheets of large plasmablastic cells
  • Large plasmablastic cells have amphophilic cytoplasm and vesicular, often eccentrically placed nuclei containing 1 or 2 prominent nucleoli (Am J Clin Pathol 2017;147:171)
  • High mitotic activity
Microscopic (histologic) images

Contributed by Annapurna Saksena, M.B.B.S, M.D. and Elaine S. Jaffe, M.D.
Diffuse growth pattern

Diffuse growth pattern

Atypical large lymphoid cells

Atypical large lymphoid cells

Multicentric Castleman disease

Multicentric Castleman disease


Multicentric Castleman disease, HHV8

Multicentric Castleman disease, HHV8

HHV8 / LANA positive atypical cells

HHV8 LANA positive atypical cells

HHV8 LANA nuclear staining

HHV8 LANA nuclear staining

EBER ISH negative atypical cells

EBER ISH negative atypical cells

Positive stains
Molecular / cytogenetics description
  • PCR reveals a monoclonal B cell population
  • Immunoglobulin (IG) genes are unmutated, lacking somatic hypermutation
Sample pathology report
  • Lymph node, right cervical, excision:
    • Diffuse large B cell lymphoma, HHV8 positive, NOS
Differential diagnosis
Board review style question #1

What pattern on HHV8 staining (shown above) is considered positive in HHV8 related diffuse large B cell lymphoma (DLBCL), NOS?

  1. Cytoplasmic
  2. Golgi region
  3. Homogenous nuclear
  4. Membranous
  5. Stippled nuclear
Board review style answer #1
E. Stippled nuclear

Comment Here

Reference: HHV8 related DLBCL, NOS
Board review style question #2
Which of the following is true about HHV8 related diffuse large B cell lymphoma (DLBCL), NOS?

  1. Clonal rearrangement pattern with somatic hypermutation of IG genes
  2. Clonal rearrangement pattern without somatic hypermutation of IG genes
  3. Polyclonal rearrangement pattern with somatic hypermutation of IG genes
  4. Polyclonal rearrangement pattern without somatic hypermutation of IG genes
Board review style answer #2
B. Clonal rearrangement pattern without somatic hypermutation of IG genes

Comment Here

Reference: HHV8 related DLBCL, NOS

HHV8 related germinotropic lymphoproliferative disorder
Definition / general
  • Monotypic HHV8 positive lymphoproliferative lesion occurring characteristically in HIV negative patients
Essential features
  • Coinfection of HHV8 and EBV
  • Immunocompetent patients
  • Localized lymphadenopathy
  • Aggregates or single plasmablasts / immunoblasts in the germinal centers
  • Good prognosis
Epidemiology
  • No known epidemiological association
  • Usually seen in HIV negative, immunocompetent patients
  • Occasional cases seen in HIV positive patients
Sites
  • Involves lymph nodes
Pathophysiology
  • Uncertain contribution of EBV in pathogenesis
  • Occasional EBV negative cases
  • Elusive role of vIL6
    • Activation of IL6 receptor signaling through vIL6 and human IL6 may play a role in differentiation of Kaposi sarcoma associated herpesvirus infected B cells into plasmablasts and development of lymphoproliferative lesions (Am J Surg Pathol 2017;41:795)
Etiology
  • Unclear origin of the plasmablasts; possible theories include:
Clinical features
  • Localized, sometimes multifocal lymphadenopathy in otherwise healthy individuals
Diagnosis
  • Lymph node biopsy
  • Morphology and immunohistochemistry
  • Immunophenotype and molecular studies
Radiology description
  • Very few descriptions of radiological features reported so far
Prognostic factors
Case reports
Treatment
  • Treatment approach is variable; some cases have not been treated, while others have been treated with surgery alone or with chemotherapy and radiation
  • Potential therapeutic role of PD1 / PDL1 immunotherapy has been suggested due to the PDL1 expression and it requires additional research
Microscopic (histologic) description
Microscopic (histologic) images

Contributed by Julie Teruya-Feldstein, M.D.
Follicles of varying sizes

Follicles of varying sizes

Hyalinization in some follicles

Hyalinization in some follicles

Plasmablasts Plasmablasts

Plasmablasts


EBER1+

EBER1

Plasmablasts MUM1+

Plasmablasts positive for MUM1

Plasmablasts CD30-

Plasmablasts negative for CD30

Plasmablasts BCL6-

Plasmablasts negative for BCL6

Plasmablasts CD10-

Plasmablasts negative for CD10

Plasmablasts CD79a-

Plasmablasts negative for CD79a

Positive stains
Negative stains
Molecular / cytogenetics description
  • Plasmablasts are polyclonal or oligoclonal
Sample pathology report
  • Lymph node, excision / core needle biopsy:
    • HHV8 related germinotropic lymphoproliferative disorder
Differential diagnosis
Board review style question #1

Which of the following is true about HHV8 related germinotropic lymphoproliferative disorder?

  1. There is coinfection of EBV and HHV8
  2. Usually seen in HIV positive patients
  3. Positive for LMP1, EBNA2 and BZLF1 expression and negative for cytoplasmic monotypic immunoglobulin heavy chain
  4. Unfavorable response to chemotherapy and radiation
Board review style answer #1
A. There is coinfection of EBV and HHV8

Comment Here

Reference: HHV8 related germinotropic lymphoproliferative disorder

Hairy cell leukemia
Definition / general
  • Hairy cell leukemia (HCL) is a rare, indolent lymphoproliferative neoplasm of mature B cells with a distinct clinical presentation that includes peripheral blood cytopenias, splenomegaly and a small number of circulating neoplastic cells with hair-like cytoplasmic projections
Essential features
  • Pancytopenia and monocytopenia
  • Splenomegaly
  • Immunophenotype positive for CD11c, CD19, CD20 bright, CD22 bright, CD25, CD103, CD123 and CD305 (LAIR)
  • BRAF V600E mutation is a hallmark genetic mutation in > 95% of cases
Terminology
  • Hairy cell leukemia: no changes were made to this entity in the recent 5th edition of the WHO classification of hematolymphoid tumors
  • Classic hairy cell leukemia (cHCL): used in some of the medical literature to best distinguish hairy cell leukemia from splenic B cell lymphoma / leukemia with prominent nucleoli (WHO 5th edition; formerly known as HCL variant [HCL-v])
  • Leukemic reticuloendotheliosis: obsolete by the WHO classification of tumors of hematopoietic and lymphoid tissues
ICD coding
  • ICD-O: 9940/3 - hairy cell leukemia
Epidemiology
  • Incidence: < 2% of all leukemias, < 1 per 100,000 persons/year in U.S.
  • M:F = 4:1
  • Median age of diagnosis is 55 years
  • More common in whites and less in African, Asian and Arab populations
  • Increased risk: farming and exposure to pesticides (Scand J Work Environ Health 1996;22:285)
  • Smoking has been reported as a significant negative association and appears to be protective (Br J Haematol 1995;91:154)
Sites
  • Common sites of involvement: bone marrow, peripheral blood and spleen
  • Uncommon sites of involvement: liver, lymph node and skin
Pathophysiology
  • BRAF V600E mutation is the causal genetic event in the vast majority of HCL
    • Mutation constitutively activates BRAF by autophosphorylation of the protein and downstream MEK-ERK signaling pathway, leading to increased expression of genes involved in survival and proliferation and promoting leukemic transformation (Blood 2016;128:1918)
Etiology
  • Epidemiologic data shows a multifactorial etiology that is influenced by ethnicity and geographical factors
  • Farming and exposure to pesticides, petroleum products, diesel and ionizing radiation are associated with an increased risk (Best Pract Res Clin Haematol 2015;28:175)
Clinical features
  • Pancytopenia and monocytopenia are common findings
  • Often reported symptoms are fatigue and infection
  • Splenomegaly: massive splenomegaly that can cause left sided abdominal pain is less commonly seen today due to early detection
  • Lymphadenopathy is not usually seen in HCL (Blood 2017;129:553)
Diagnosis
  • Diagnosis is based on peripheral blood and bone marrow morphology, flow cytometry immunophenotyping, immunohistochemistry and molecular studies
Laboratory
  • CBC and differential count: pancytopenia and monocytopenia (Blood 2017;129:553)
  • Peripheral blood smear morphology: circulating neoplastic hairy cells are present but at a low frequency and they can be difficult to identify
Prognostic factors
  • Chronic, relatively indolent leukemia that predominantly responds well to current chemotherapy; 5 year event free survival rate after treatment is ~90%
  • Poor prognostic factors include:
Case reports
  • 16 year old boy presented with hairy cell leukemia (Blood 2018;132:1216)
  • 41 year old man presented with unilateral sudden hearing loss as an initial manifestation of hairy cell leukemia (Int J Surg Case Rep 2019;60:200)
  • Man in mid-60s presented with severe Legionnaires disease pneumonia associated with undiagnosed hairy cell leukemia (BMJ Case Rep 2019;12:e230130)
  • 70 year old African American man with hairy cell leukemia presented with pancytopenia but without splenomegaly (Case Rep Hematol 2014;2014:354810)
  • 72 year old man presented with coexisting chronic lymphocytic leukemia and hairy cell leukemia (Blood 2019;133:1264)
  • 74 year old woman presented with osteolytic lesions in a case of hairy cell leukemia mimicking multiple myeloma (Lancet Oncol 2019;20:e187)
  • 79 year old man with relapsed hairy cell leukemia diagnosed by endoscopic ultrasound guided FNA (Cytojournal 2006;3:1)
Treatment
  • Not curative; asymptomatic patients usually do not require treatment
  • Standard therapy is purine analogues (i.e., cladribine, pentostatin), which are effective in the majority of HCL
  • Late relapses are not uncommon; choices for relapsed and refractory HCL include moxetumomab (anti-CD22), pasudotox (anti-CD22), vemurafenib (BRAF inhibitor) and bendamustine plus rituximab
  • Splenectomy as salvage therapy (Blood 2017;129:553)
Gross description
  • Spleen: splenomegaly, > 150 g; spleen shows a diffuse and marked enlargement without nodules
Microscopic (histologic) description
  • Spleen:
    • Red pulp involvement by sheets of bland tumor cells that are larger than lymphocytes with modest, pale blue agranular cytoplasm, round or folded nuclei and no distinct nucleoli
    • There is an increase in volume, surface and length of red pulp vessels that can classically form pools or lakes of blood lined by hairy cells
  • Bone marrow:
    • Bone marrow aspirate is often a dry tap due to extensive marrow fibrosis as demonstrable by reticulin stain; however, neoplastic hairy cells, as described in the peripheral blood description, can be seen (Blood 2003;102:1051)
    • Bone marrow biopsy: always involved
      • Neoplastic infiltrates can vary and have a patchy and subtle, interstitial or diffuse solid pattern
      • Presence of fibrosis is common; ~10% of cases can be markedly hypocellular and mimic aplastic anemia
Microscopic (histologic) images

Contributed by Buthaina Al-Maashari, M.D. and Dietrich Werner, M.D.
Bone marrow core biopsy

Bone marrow core biopsy

BRAF IHC

Cytology images

Images hosted on other servers:

Lymph node FNA

Peripheral smear description
  • Smear shows pancytopenia with rare hairy cells that are medium in size with moderately abundant pale blue cytoplasm, reniform nuclei, open chromatin, absent nucleoli and a characteristic serrated cytoplasmic border (Blood 2017;129:553)
Peripheral smear images

Contributed by Buthaina Al-Maashari, M.D. and Dietrich Werner, M.D.
Peripheral blood image

Circulating hairy cell



Images hosted on other servers:

Typical leukemic cell in hairy cell leukemia

Positive stains
  • Bone marrow IHC stain is important to appreciate tumor infiltration
  • Typical pattern for B cell lymphocytes includes CD19, CD20, CD79a and PAX5
  • Annexin A1 expression is specific to distinguishing HCL from other B cell lymphomas including HCL variant; however, correlation with B cell markers is needed, particularly in hypocellular and low tumor burden cases since annexin is also expressed by T cells and myeloid cells (Lancet 2004;363:1869)
  • TRAP (tartrate resistant acid phophotase): done by immunohistochemistry; in the aspirate smears, TRAP test by enzyme stain is not commonly done anymore since it is a more difficult technical and interpretative procedure
    • TRAP IHC in the core biopsy is currently more commonly used
  • DBA-44: combined TRAP and DBA-44 is sensitive and fairly specific (Am J Surg Pathol 2005;29:474)
  • Cyclin D1: positive staining; dim staining compared to what is seen in mantle cell lymphoma (Mod Pathol 2000;13:1308)
  • BRAF V600E: positive overexpression by immunohistochemistry studies; point mutations are also detected by molecular studies
  • Reticulin is increased due to the presence of fibrosis
  • TIA1 is seen in 56% of cases (Mod Pathol 2004;17:840)
Negative stains
Flow cytometry description
Flow cytometry images

Contributed by Buthaina Al-Maashari, M.D. and Dietrich Werner, M.D.
Flow cytometry analysis Flow cytometry analysis Flow cytometry analysis Flow cytometry analysis

Peripheral blood analysis of a 67 year old man with cytopenias


Flow cytometry analysis Flow cytometry analysis Flow cytometry analysis Flow cytometry analysis

Peripheral blood analysis of a 67 year old man with cytopenias

Electron microscopy description
Molecular / cytogenetics description
  • BRAF V600E mutation is a hallmark for classical HCL; this mutation is not seen in HCL variant
  • Rare cases that are BRAF V600E mutation negative will be positive for IGHV4-34
  • Histone methyltransferase KMT2C (MLL3) and CDKN1B are commonly mutated genes in cHCL
  • BRD4, CEBPA, CREBBP, RUNX1, EP300 and MED12, Notch signaling (NOTCH1 and NOTCH2) and DNA repair
  • Deletions of chromosomes 7q and 13q and gains of chromosome 5 (Blood 2017;130:1644)
Sample pathology report
  • Bone marrow, (R)PSIS, aspirate, biopsy and peripheral blood:
    • Positive for classical hairy cell leukemia involving ~40% of the sample (see comment)
    • Comment: Peripheral blood with pancytopenia and rare circulating neoplastic hairy cells, < 1%. Hypocellular bone marrow, 15 - 20% with decreased trilineage hematopoiesis. Positive for hairy cell leukemia involving ~40% of the sample. Tumor burden is based on the core biopsy with the help of immunohistochemistry that is felt to be more accurate for this case. Flow cytometry shows a small clonal population in ~2% of the sample that shows positive coexpression for the specific hairy cell leukemia markers CD123, CD103 and CD25. Next generation sequencing is positive for BRAF V600E mutation. Flow cytometry and molecular studies support the diagnosis.
Differential diagnosis
  • Splenic B cell lymphoma / leukemia with prominent nucleoli, formerly known as HCL variant (HCL-v):
    • Usually present with absolute lymphocytosis and less severe cytopenias
    • Can have a similar flow cytometry immunophenotype with bright CD11c and positive coexpression for CD103 but are usually negative for CD25, CD123 and CD200
    • Negative for BRAF V600E mutations; these cases can be IGHV4-34 positive
  • Splenic marginal zone lymphoma:
    • Can have circulating neoplastic cells with cytoplasmic projections; these cells are called villous lymphocytes
    • Usually show involvement of the white pulp in the spleen and are negative for classical hairy cell markers CD123, CD103, CD25 and CD305 (LAIR) by flow cytometry
    • Negative by IHC for annexin 1 and cyclin D1 and negative for BRAF V600E mutations
  • Splenic diffuse red pulp lymphoma (SDRPL):
    • Can present with splenomegaly and is positive for CD11c
    • Negative for classical hairy cell markers CD123, CD103, CD25 and CD305 (LAIR) by flow cytometry
    • Negative by IHC for annexin 1 and cyclin D1 and negative for BRAF V600E mutations
Board review style question #1

A peripheral blood smear on a middle aged man with cytopenias shows these rare circulating white blood cells that express CD103 by flow cytometry. What is the most likely diagnosis?

  1. CMV infection
  2. EBV infection
  3. Hairy cell leukemia
  4. HIV infection
  5. Splenic marginal zone lymphoma
Board review style answer #1
C. Hairy cell leukemia

Comment Here

Reference: Hairy cell leukemia
Board review style question #2
Molecular testing detected a BRAF V600E mutation on a bone marrow sample that contained a B cell clonal population that fell on the monocyte gate by flow cytometry and is estimated to constitute ~4% of the total sample. What is your diagnostic interpretation?

  1. Chronic lymphocytic leukemia
  2. Diffuse large B cell lymphoma
  3. Hairy cell leukemia
  4. Mantle cell lymphoma
  5. Monoclonal B cell lymphocytosis
Board review style answer #2
C. Hairy cell leukemia

Comment Here

Reference: Hairy cell leukemia

Hairy cell leukemia variant
Definition / general
  • Rare B cell chronic lymphoproliferative disorder
  • Not biologically related to classic hairy cell leukemia (cHCL)
  • Exhibits a heterogeneous spectrum of clinical, morphologic, immunophenotypic and genetic features
  • Overlap features with classic hairy cell leukemia and other hairy cell-like B cell neoplasms
  • Provisional entity within the category of splenic B cell lymphoma / leukemia, unclassifiable, along with splenic diffuse red pulp small B cell lymphoma
Essential features
  • First recognized by Cawley et al. in 1980 (Leuk Res 1980;4:547)
  • Clinical laboratory findings
    • Leukocytosis and presence of monocytes
  • Bone marrow biopsy / aspirate: aspirable (reticulin fibrosis is normal / not increased)
  • Morphological findings
    • Intermediate size cells with blastic or convoluted nuclei, prominent nucleoli and circumferential shaggy contours
    • Other morphologic subtypes include convoluted and blastoid variant (Best Pract Res Clin Haematol 2015;28:253)
  • Immunophenotypic findings
    • Absence of CD25, CD123, annexin A1 and tartrate resistant acid phosphatase (TRAP)
  • Wild type BRAF
  • Resistant to conventional HCL therapy (e.g. cladribine)
  • Poorer prognosis (median survival ~ 9 years) (Leukemia 2001;15:184)
Terminology
  • Prolymphocytic variant of hairy cell leukemia
  • Atypical hairy cell leukemia
  • Japanese variant (HCL JV): predominant in females and may respond better to treatment (Leukemia 1993;7:181)
  • Synonyms
    • HCL variant
ICD coding
  • ICD-O: 9591/3 - non-Hodgkin lymphoma, NOS
  • ICD-10: C85.17 - unspecified B cell lymphoma, spleen
Epidemiology
  • Incidence
    • Constitutes 0.4% of chronic lymphoid malignancies; ~ 10% of all HCL cases
  • Age (Cancer Treat Rev 2011;37:3)
    • Middle aged to elderly patients
    • Reported median age 71 years
  • Sex
Sites
  • Mainly spleen, bone marrow and peripheral blood
  • Liver: less than 33% of patients
  • Lymph node, other solid organs: uncommon
Pathophysiology
  • Activated B cell at a late stage of maturation
Diagrams / tables

Contributed by Pallavi Khattar, M.D.
Differential diagnosis

Differential diagnosis



Images hosted on other servers:
Therapeutic algorithm for HCL variant

Therapeutic algorithm for HCL variant

Clinical features
Diagnosis
  • Requires constellation of clinical features, peripheral blood smear, bone marrow, immunophenotyping and molecular studies
Laboratory
  • Clinical laboratory values
    • Leukocytosis (average WBC ~ 30 x 109/L) and lymphocytosis (versus splenic diffuse red pulp small B cell lymphoma (SDRPL) with low lymphocytosis and cHCL with pancytopenia and monocytopenia)
    • Anemia (~ half of the patients)
    • Thrombocytopenia (~ half of the patients)
    • Absolute monocyte count is typically within the normal range (versus classic hairy cell leukemia (cHCL) with monocytopenia) (Mod Pathol 2018;31:1717)
Radiology description
  • Abdominal CT scan
    • Splenomegaly
    • Hepatomegaly
Prognostic factors
  • Chronic clinical course with a tendency for more aggressive behavior than cHCL
  • Median survival 9 years with only 15% survival over 15 years (Leukemia 2001;15:184, Cancer Treat Rev 2011;37:3)
  • Reported complete remission (combined purine analog and rituximab treatment or immunotherapy alone)
  • Unfavorable prognostic factors: (Br J Haematol 2012;158:347)
    • Significant anemia
    • Older age
    • Mutations in TP53
Case reports
Treatment
  • Therapeutic algorithm for hairy cell leukemia variant (HCL variant; see Diagram 1) (Ann Oncol 2015;26:v100)
    • Observation and close monitoring: asymptomatic patients with moderate splenomegaly and no cytopenia
    • Symptomatic disease (progressive splenomegaly or elevated leukocyte counts with cytopenia)
    • Relapsed / refractory cases (Am J Hematol 2017;92:1382)
    • Interferon alpha and purine analogs alone are unsatisfactory and fail to achieve a high complete remission rate
Microscopic (histologic) description
  • Peripheral smear
    • Leukemic cells
      • Variable morphology
      • Exhibit hybrid features of prolymphocytic leukemia and cHCL
      • Morphological subtypes e.g. blastic and convoluted
      • Cells are intermediate size
      • Nuclear features range from condensed chromatin with the prominent central nucleoli of a prolymphocytic cell to dispersed chromatin with highly irregular nuclear contours (Blood 2016;128:1018)
      • Cytoplasm abundant basophilic cytoplasm, circumferential stellate / hairy projections
    • Bone marrow
      • Aspirable: no significant reticulin fibrosis (MF = 0) (versus hairy cell leukemia showing marked fibrosis and dry tap)
      • Infiltration may be subtle; usually interstitial and lesser intrasinusoidal distribution (Mod Pathol 2018;31:1717)
      • Immunohistochemical staining is helpful to highlight the pattern and extent of infiltration
    • Spleen
      • Red pulp: diffusely involved and expanded
      • White pulp: follicles atretic / absent
      • Leukemic cells are present in the dilated sinusoids
      • Blood lakes may be present
    • Liver
      • Leukemic cell infiltration involves both the portal tract and sinusoids
Microscopic (histologic) images

Contributed by Pallavi Khattar, M.D. and Case #437
Bone marrow biopsy Bone marrow biopsy

Bone marrow biopsy

CD20 immunostain CD20 immunostain

CD20 immunostain

Bone marrow aspirate Bone marrow aspirate

Bone marrow aspirate


Bone marrow aspirate Bone marrow aspirate

Bone marrow aspirate

Bone marrow biopsy Bone marrow biopsy

Bone marrow biopsy

Bone marrow aspirate Bone marrow aspirate

Bone marrow aspirate


Bone marrow aspirate

Bone marrow aspirate

CD20 immunostain

CD20 immunostain

CD3 immunostain

CD3 immunostain

DBA-44 immunostain

DBA-44 immunostain

Peripheral smear images

Contributed by Pallavi Khattar, M.D. and Case #437
Peripheral smear Peripheral smear Peripheral smear Peripheral smear

Peripheral smear

Positive stains
Negative stains
Flow cytometry description
Flow cytometry images

Contributed by Pallavi Khattar, M.D. and Case #437
Flow cytometry analysis Flow cytometry analysis

Flow cytometry analysis

Molecular / cytogenetics description
  • Negative for BRAF V600E mutation
  • No somatic mutations of IGHV: 33% of cases
    • Unmutated cases have a high frequency of TP53 dysfunction
  • VH gene family, IGHV4-34 (immunoglobulin heavy variable) overexpression (Blood 2012;119:3330)
  • Recurrent mutations in the gene MAP2K1 (15q22.31) (Nat Genet 2014;46:8)
  • Mutations in KMT2C, CCND3 and U2AF1 seen in a subset (Blood 2017;130:1644)
  • Large number of DNA copy number alterations
  • Most frequent gains on chromosome 5 and losses on 7q and 17p (Leukemia 2011;25:1189)
Sample pathology report
  • A - D: bone marrow, left posterior iliac crest, trephine biopsy, aspirate smears, peripheral blood and flow cytometry analysis:
    • CD103 positive / CD5 negative / CD10 negative B cell lymphoma (see comment)
    • Comment: Bone marrow biopsy demonstrates increased CD20+ atypical lymphoid cells in the interstitial distribution. Flow cytometry analysis exhibits monotypic lambda restricted CD5 negative / CD10 negative B cell population with bright CD20, CD11c, variable expression of CD103 and lack of CD25. Overall findings, in conjunction with morphologic, immunophenotypic features (lack of CD25 (on flow cytometry) and annexin A1 (by IHC)) are suggestive of splenic B cell lymphoma / leukemia, unclassifiable, favor hairy cell leukemia variant (HCL variant). However, the differential diagnosis includes splenic diffuse red pulp small B cell lymphoma (SDRPL). Additional cytogenetic and molecular studies are in progress and will be reported separately.
    • Peripheral smear: Manual review of the peripheral blood shows normochromic, normocytic anemia, thrombocytopenia and increased atypical mononuclear cells.
    • RBCs: Mild normochromic, normocytic anemia with anisopoikilocytosis.
    • WBCs: Increased atypical lymphocytes (~ 15%) on smear that are intermediate in size with cytoplasmic projections and occasional prominent nucleoli.
    • Platelets: Thrombocytopenia with unremarkable morphology.
    • Bone marrow biopsy: Quality: adequate. Cellularity: 30 - 40% (normocellular marrow for age). Hematopoiesis: trilineage maturation with increased interstitial lymphocytes (better highlighted on CD20 immunostain ~ 10 - 20% of biopsy cellularity).
    • Immunohistochemical stains: Show interstitial small lymphocytes that are positive for CD20 and PAX5, while negative for CD3, CD5, CD10, BCL6, BCL1, BRAF / VE1, CD25 and annexin A1.
    • Bone marrow clot section: Quality: adequate. Morphologic features are similar to those observed in the core biopsy.
    • Special stains: Reticulin stain: no increase in reticulin fibrosis (MF = 0).
    • Bone marrow aspirate: Quality: adequate. Increased atypical lymphocytes that are intermediate in size with cytoplasmic projections and occasional prominent nucleoli. Myeloid and erythroid lineage shows progressive maturation with no significant dysplastic features. Megakaryocytes are adequate in number with unremarkable morphology.
    • Iron stores (PERL stain): Yes positive stores, no ring sideroblasts.
    • Flow cytometry studies: Flow cytometry immunophenotypic studies: abnormal B cell population detected. Monotypic lambda restricted B cell population (40% of total, 72% of cells in lymphocyte gate) exhibiting the following immunophenotype: CD5-, CD10-, CD19+, CD20+ (bright), CD22+, CD25- and CD103+. Remaining antigens are negative or noncontributory.
Differential diagnosis
  • See Diagram 1
  • Hairy cell leukemia:
    • Pancytopenia and monocytopenia with characteristically few circulating cells
    • Reniform or oval nuclei, circumferential long villi, inconspicuous nucleoli
    • Spleen: diffuse infiltrate in red pulp cords, effaced white pulp, formation of blood lakes
    • Marked reticulin fibrosis (dry tap)
    • Immunophenotype: CD25+, CD103+, CD123+
    • Genetics: BRAF V600E (~ 100%)
    • Treatment: purine analogs, rituximab
  • Splenic diffuse red pulp small B cell lymphoma:
    • No monocytopenia
    • Broad based polar cytoplasmic extensions
    • Spleen: diffuse involvement of red pulp with cord and sinusoid infiltration, absence of white pulp involvement
    • Immunophenotype: CD25-, CD103-, CD123-
    • Genetic mutations: CCND3, NOTCH1, MAP2K1, TP53
    • Treatment: splenectomy
  • Splenic marginal zone lymphoma:
    • No monocytopenia
    • Smaller cells may have short cytoplasmic projections
    • Cells have clumpy chromatin, indistinct nucleoli, different staining pattern
    • Spleen: white pulp is infiltrated by the neoplastic cells, marginal zone differentiation
    • Immunophenotype: CD25+, CD103 variable, CD123-
    • Cytogenetic: deletion of 7q
    • Genetic: NOTCH2, VH1 / 2
    • Treatment: splenectomy, rituximab
Board review style question #1
Which of the following are bad prognostic markers in hairy cell leukemia variant?

  1. Asymptomatic patient with normal counts
  2. IGHV mutation status
  3. Mutation in TP53
  4. Young age
Board review style answer #1
C. Mutation in TP53. Mutations in TP53 are associated with worse prognosis in HCL variant. Other adverse prognostic factors are significant anemia and older age. IGHV mutation status does not have prognostic impact. (Br J Haematol 2012;158:347)

Comment Here

Reference: Hairy cell leukemia variant
Board review style question #2

Which of the following stains are most useful for differentiating classic hairy cell leukemia and hairy cell leukemia variant?

  1. CD21, CD23, CD35
  2. CD103, CD11c, CD25, annexin A1
  3. CD34, CD117, myeloperoxidase
  4. CD3, CD10, CD20, BCL2
Board review style answer #2
B. CD103, CD11c, CD25, annexin A1. Hairy cell leukemia variant is positive for CD11c (bright) and CD103 (variable) while negative for annexin A1, CD25 and CD123.

Comment Here

Reference: Hairy cell leukemia variant
Board review style question #3
Which one of the following is the most characteristic feature to differentiate hairy cell leukemia from hairy cell leukemia variant?

  1. Affects adult patients
  2. BRAF V600E mutation
  3. CD103 positivity
  4. Splenomegaly
Board review style answer #3
B. BRAF V600E mutation. Classical hairy cell leukemia is almost always positive for BRAF V600E mutation (100%) but HCL variant is not.

Comment Here

Reference: Hairy cell leukemia variant
Board review style question #4
Which among the following splenic B cell neoplasms is associated with the worst prognosis?

  1. Classic hairy cell leukemia
  2. Hairy cell leukemia variant
  3. Splenic diffuse red pulp small B cell lymphoma
  4. Splenic marginal zone lymphoma
Board review style answer #4
B. Hairy cell leukemia variant. It is associated with a worse prognosis than classic hairy cell leukemia and has been reported to show a median survival of ~ 9 years.

Comment Here

Reference: Hairy cell leukemia variant

Hepatosplenic T cell lymphoma
Definition / general
Essential features
  • Rare, aggressive extranodal neoplasm that originates from cytotoxic T cells; it usually expresses the T cell receptor (TCR) γδ (gamma delta) (Am J Surg Pathol 2017;41:82)
  • Hepatomegaly and splenomegaly are the most common clinical manifestations (Ann Diagn Pathol 2017;26:16)
  • Liver and bone marrow: sinusoidal infiltration (Histopathology 2014;64:171)
  • Spleen
    • Diffuse involvement; expansion of the red pulp cords and sinusoids, with atrophy or absence of white pulp (Hum Pathol 2018;74:5)
  • Cytology: variable
    • Usually monotonous, small to intermediate size cells with pale agranular cytoplasm, round / oval nuclei, moderately condensed chromatin and inconspicuous nucleoli (Blood 2003;102:4261)
Terminology
  • Obsolete term: erythrophagocytic Tγ lymphoma
ICD coding
  • ICD-O: 9716/3 - hepatosplenic T cell lymphoma
Epidemiology
Pathophysiology
Etiology
Clinical features
Radiology description
  • CT or MRI (Blood 2020;136:2018):
    • Homogeneous hepatosplenomegaly
  • Fluorodeoxyglucose (FDG) PET / CT (Blood 2020;136:2018):
    • Diffuse FDG uptake in affected areas
      • Usually liver, spleen and bone marrow
Prognostic factors
Case reports
Treatment
Gross description
  • Diffuse and homogeneous, enlarged liver and spleen; without distinct macroscopic lesions; lymph nodes are usually not enlarged (Blood 2003;102:4261)
Microscopic (histologic) description
Microscopic (histologic) images

Contributed by Roberto N. Miranda, M.D.

Bone marrow infiltration

Bone marrow with dyspoietic changes

CD3 positive

CD4 negative


CD8 negative

TCR βF1 negative

TCRγδ positive

Liver infiltration


HSTCL: CD3 positive

HSTCL: CD4 negative

CD8 positive

TIA1 positive


Small size morphology

Intermediate size morphology

Blastic morphology

Cytology description
Negative stains
Flow cytometry description
Flow cytometry images

Contributed by Roberto N. Miranda, M.D.

Positivity for CD2, TCRγδ and CD56

Molecular / cytogenetics description
Sample pathology report
  • Bone marrow (right iliac crest) core and clot specimen; aspirate smears and peripheral blood:
    • Hepatosplenic T cell lymphoma
    • Flow cytometry immunophenotype demonstrated aberrant T cells with loss of CD4 and CD8, positive for T cell receptor gamma delta chain (see comment)
    • Comment: According to clinical records, patient is a 28 year old man who has a history of Crohn's disease, for which he was receiving azathioprine and tumor necrosis alpha receptor antagonist. Patient reports malaise and fever for 3 months. On physical exam, patient had massive splenomegaly (8 cm below costal margin) and hepatomegaly. Laboratory findings showed leukopenia, severe anemia and thrombocytopenia. Additional testing proved negative for antiglobulin testing, negative for HIV, hepatitis B, hepatitis C and HTLV1.
    • Bone marrow core biopsy and clot sections show similar features. There is 70% cellularity with trilineage hematopoiesis. Megakaryocytes are adequate in number and range from small, hypolobated forms to megakaryocytes of normal size and shape. There are scattered small clusters of small to intermediate size lymphocytes with hyperchromatic nuclei with irregular nuclear contours.
    • Bone marrow aspirate smears show trilineage hematopoiesis with mild dysmegakaryopoiesis. There are approximately 15% lymphocytes, of small to intermediate size, with clear and agranular cytoplasm and central hyperchromatic nuclei with irregular nuclear contours.
    • Immunohistochemical studies were performed in bone marrow core biopsy and the T cell marker CD3 highlights lymphocytes in an interstitial and sinusoidal pattern, representing approximately 10% of marrow cells. Atypical lymphocytes are also highlighted with CD2, CD7, CD56, TIA1 and granzyme M. The T cell receptor gamma delta is positive in lymphocytes, while the T cell receptor alpha beta (βF1 clone) is negative. CD4 and CD8 are negative in atypical lymphocytes. The following markers are also negative in lymphocytes: CD1a, CD5, CD10, CD57, EBER (EBV in situ hybridization), TCL1 and TdT.
    • Concurrent flow cytometry immunophenotype demonstrates that approximately 15% of marrow cells are T lymphocytes, positive for CD2, CD7, CD52, CD56 and T cell receptor gamma delta. The atypical lymphocytes are negative for CD4, CD5, CD8, CD57 and T cell receptor alpha beta. The B lymphocytes are polytypic.
    • Molecular testing revealed monoclonal T cell receptor gene rearrangement of gamma (V gamma 4 family) and beta chains. Next generation sequencing showed mutation of STAT5B. The specimen was negative for IGH gene rearrangement.
    • Cytogenetic analysis revealed isochromosome 7 and trisomy 8
    • Clinical correlation and followup is recommended.
Differential diagnosis
Board review style question #1

Which of the following immunophenotypic patterns corresponds to hepatosplenic T cell lymphoma?

  1. CD2+, CD3+, CD4+, CD5-, TIA1+, granzyme B+, TCRγδ+
  2. CD2+, CD3+, CD4-, CD5-, TIA1+, granzyme M+, TCRγδ+
  3. CD2+, CD3-, CD4+, CD5+, CD7+, CD8+, TCRγδ+
  4. CD2+, CD3-, CD4+, CD5+, CD7-, CD8-, TCRγδ+
  5. CD2-, CD3-, CD4-, CD5+, CD7-, CD8-, TCRγδ+
Board review style answer #1
B. CD2+, CD3+, CD4-, CD5-, TIA1+, granzyme M+, TCRγδ+

Comment here

Reference: Hepatosplenic T cell lymphoma
Board review style question #2
Which of the following cytogenetic and molecular abnormalities are found in hepatosplenic T cell lymphoma?

  1. Del(6q) and RHOA1 mutation
  2. Inversion 14 and TCL1 rearrangement
  3. Isochromosome 7 and STAT5B mutation
  4. t(2;5) and JAK / STAT activation
  5. 6p25 and DUSP22 / IRF4
Board review style answer #2
C. Isochromosome 7 and STAT5B mutation

Comment here

Reference: Hepatosplenic T cell lymphoma

High grade B cell lymphoma, NOS
Definition / general
  • Aggressive mature B cell lymphoma previously included in the 2008 WHO classification as "B cell lymphoma, unclassifiable, with features intermediate between diffuse large B cell lymphoma and Burkitt lymphoma"
  • Newly defined high grade B cell lymphoma in the revised 2016 WHO (Blood 2016;127:2375)
  • These lymphomas do not harbor combined MYC rearrangement (chromosome 8q24.2) and BCL2 (chromosome 18q21.3) and / or BCL6 (chromosome 3q27.3) rearrangement
    • These neoplasms may carry a MYC rearrangement, or BCL2 and / or BCL6 rearrangement without MYC rearrangement, or extra copies of MYC, BCL2 and / or BCL6
Essential features
Terminology
  • High grade B cell lymphoma, NOS
    • May be applicable to de novo or transformed cases
  • Note:
    • This diagnosis is rare and should be rendered only when diffuse large B cell lymphoma (DLBCL) and Burkitt lymphoma (BL) have been ruled out
    • Cases with large cell morphology that have MYC rearrangement with or without amplification / gain of BCL2 should be considered DLBCL
    • Pediatric cases with simple karyotypes and features of both DLBCL and BL have better prognosis than high grade B cell lymphoma, NOS and should instead be classified as either DLBCL or BL
      • Half of these cases harbor MYC rearrangements and can demonstrate a gene expression profile resembling that observed in BL (Blood 2008;112:1374)
    • Care should be taken before diagnosing a transformed follicular lymphoma as high grade B cell lymphoma, especially if any retained follicular architecture exists
ICD coding
  • ICD-O: 9680/3 - B cell lymphoma, unclassifiable, with features intermediate between diffuse large B cell lymphoma and Burkitt lymphoma
Epidemiology
  • Rare; true frequency is unknown but increases with age
  • Typically reported in elderly patients in the sixth decade of life
  • Occurs equally in men and women
Sites
Etiology
  • Few studies have been conducted but this lymphoma may arise from germinal center B cells in 20 - 35% of cases with MYC rearrangement and amplification / gain of BCL2 (Mod Pathol 2015;28:208)
  • Cell of origin is unknown in cases without MYC or BCL2 rearrangements (Mod Pathol 2009;22:1507)
Diagrams / tables

Images hosted on other servers:
Diagnostic approach

Diagnostic approach

Clinical features
Diagnosis
  • Morphological evaluation reveals cells that more likely resemble BL than DLBCL
    • Nuclear size variations or a blastoid appearance uncharacteristic of BL are possible
  • Simultaneous rearrangement of MYC and BCL2 or BCL6 should be excluded by classic cytogenetics, FISH or other molecular / genetic tests
Laboratory
Prognostic factors
  • Despite some variability, generally associated with poor prognosis
    • Survival is typically measured in months
  • This category may have a slightly better outcome than high grade B cell lymphoma with MYC and BCL2 and / or BCL6 rearrangements (Am J Surg Pathol 2014;38:49, Br J Haematol 2013;162:40)
  • The worst prognosis has been reported in instances of amplification / gain of MYC with or without rearrangement of BCL2 or in instances of amplification / gain of BCL2 with or without rearrangement of MYC (Mod Pathol 2015;28:208, Cancer 2012;118:1566)
  • Poor outcome has been associated with age > 60 years, stage IV disease or a high International Prognostic Index (IPI) score (Br J Haematol 2013;162:40)
Case reports
Treatment
  • No standardized treatment exists
  • This category is thought to be resistant to typical DLBCL, NOS and BL regimens, including rituximab, cyclophosphamide, doxorubicin, vincristine and prednisone (R-CHOP)
  • Use of dose adjusted etoposide, prednisone, vincristine, cyclophosphamide, doxorubicin with rituximab (DA-EPOCH-R) has been favored in some settings (Hsi: Hematopathology - Foundations in Diagnostic Pathology, 3rd Edition, 2017)
Microscopic (histologic) description
  • 2 morphological variants have been defined (Pathology 2020;52:68)
  • Variant previously described as B cell lymphoma, unclassifiable, with features intermediate between DLBCL and BL (BCL-U):
    • Diffuse proliferation of intermediate sized, sometimes squared off cells similar to BL
    • Other cases demonstrate a spectrum of intermediate and large sized cells
    • Starry sky pattern is common
    • Increased mitotic figures and apoptotic bodies are appreciated
    • Sclerosis and admixed small lymphocytes are usually absent
  • Blastoid variant:
    • Intermediate sized cells with fine nuclear chromatin resembling lymphoblasts
    • Starry sky pattern and frequent mitotic figures are common
Microscopic (histologic) images

Contributed by Jon C. Aster, M.D., Ph.D. and Genevieve M. Crane, M.D., Ph.D.
H&E, 400x of an oropharyngeal mass

Oropharyngeal mass

CD10

CD10

MYC

MYC

Ki67

Ki67

HGBL, NOS, H&E

HGBL, NOS

High Ki67

High Ki67

Positive stains
  • CD19, CD20, CD79a and PAX5
  • CD10 (variable, especially present in cases with BL-like morphology)
  • BCL6 (most cases)
  • IRF4 / MUM1 (variable)
  • BCL2 (present in cases with BL-like morphology)
  • Ki67 (variable, but > 90% in many cases)
  • MYC (variable)
Negative stains
Flow cytometry description
Molecular / cytogenetics description
  • Clonal IGH rearrangements are present
  • 20 - 35% of cases demonstrate MYC rearrangements and amplification / gain of BCL2 (Mod Pathol 2015;28:208)
  • Significant proportion of cases with blastoid morphology lack MYC or BCL2 rearrangements (Histopathology 2012;61:945)
  • Complex karyotypes with ≥ 3 abnormalities are common
  • Genomic gains and losses include but are not limited to:
  • Genetic profile intermediate between DLBCL and BL has been observed including mutations in:
Sample pathology report
  • Stomach, ulcer, biopsy:
    • B cell lymphoma with aggressive features; pending FISH for further classification (see comment)
    • Microscopic description: The sections show gastric mucosa with a diffuse infiltrate comprised of intermediate to large sized lymphoid cells with round to irregular nuclei, vesicular chromatin, distinct nucleoli and moderate amounts of cytoplasm. Interspersed are small lymphocytes. Mitotic figures and apoptotic bodies are easily appreciated. Areas of necrosis are seen. Immunoperoxidase studies show that the larger cells are positive for CD20, CD10, BCL6 and BCL2 (weak, small subset) and are negative for CD5 and cyclin D1. CD3 study highlights interspersed T cells. MYC expression is noted in about 50% of the tumor cells. Ki67 shows a proliferative index of more than 95%. In situ hybridization study for EBV encoded RNA (EBER) is negative.
    • Comment: The morphologic and immunophenotypic findings support the diagnosis of involvement by diffuse large B cell lymphoma with high grade features based on the proliferation rate. CD10 and BCL6 coexpression suggests a follicular center cell derivation. The differential diagnosis includes diffuse large B cell lymphoma, NOS and so called double hit lymphoma. Additional materials have been sent for FISH analysis, the results of which will be reported separately and in an addendum.
    • Addendum: FISH analysis showed no c-MYC rearrangement in 50/50 interphase nuclei. A normal MYC FISH finding can result from absence of MYC rearrangement, from an atypical MYC rearrangement or from an insufficient number of neoplastic cells in the specimen. Features of a double hit lymphoma are not seen. The findings are most consistent with involvement by a high grade B cell lymphoma, NOS.
Differential diagnosis
Board review style question #1

A 57 year old woman presents with generalized lymphadenopathy and bilateral pulmonary nodules. A representative photograph of her lung biopsy is provided. FISH detected an MYC rearrangement without associated abnormalities in BCL2 or BCL6. Immunohistochemistry was positive for CD79a and CD20 but negative for TdT and cyclin D1. The most appropriate diagnosis is

  1. Burkitt lymphoma
  2. Diffuse large B cell lymphoma, NOS with MYC rearrangement
  3. High grade B cell lymphoma, NOS
  4. High grade B cell lymphoma with MYC and BCL2 or BCL6 rearrangements
Board review style answer #1
B. Diffuse large B cell lymphoma, NOS with MYC rearrangement

Comment Here

Reference: High grade B cell lymphoma, NOS
Board review style question #2
Cases of high grade B cell lymphoma, NOS usually demonstrate which of the following?

  1. BCL6 expression
  2. Concurrent MYC and BCL2 rearrangements
  3. Low to moderate Ki67 proliferation index
  4. Prominent inflammatory background of small lymphocytes
Board review style answer #2
A. BCL6 expression

Comment Here

Reference: High grade B cell lymphoma, NOS

Hydroa vacciniforme-like lymphoproliferative disease
Definition / general
  • Chronic EBV+ cutaneous lymphoproliferative disorder with a broad clinical spectrum, a usually protracted clinical course and a long term risk to progress to a systemic lymphoma
  • Occurs in children and associated with sun sensitivity (WHO 2008)
  • Increased frequency in Asians and Native Americans from Central and South America, and Mexico
Epidemiology
  • Mainly children and adolescents from Asia or Native Americans from Central America, South America and Mexico
  • Rare in adults
Sites
  • Sun exposed skin, particularly the face (cheeks, nose, lower lip) but also ears and dorsum of hands
Etiology
  • EBV transformed neoplastic cells (usually T but sometimes NK cells)
  • Hypersensitivity to sunlight
  • Related condition: mosquito bite hypersensitivity (the EBV+ cells are NK cells)
Postulated normal counterpart
  • Skin homing cytotoxic T cell or NK cell
Clinical features
  • Papulovesicular eruption usually followed by ulceration and scarring (mimics herpes)
  • Edema of face, eyelids and lips
  • Systemic symptoms (fever, wasting, lymphadenopathy, hepatosplenomegaly, myocarditis) may occur, particularly late in course of disease
Treatment
  • Variable clinical course with recurrent skin infections for up to 10 - 15 years before progression to systemic involvement
  • Much more aggressive once systemic spread has occurred
  • Mosquito bite allergy: clinically more aggressive and often associated with a hemophagocytic syndrome
Microscopic (histologic) description
  • Small to medium neoplastic cells without significant atypia
  • Infiltrates extend from epidermis to subcutis with necrosis, angiocentricity, angioinvasion, epidermal ulceration
Microscopic (histologic) images

Contributed by Hillary Rose Elwood, M.D.

Shave biopsy shows epidermal spongiosis, vesiculation and necrosis


Lymphocytic infiltrate is CD3+, TIA+, subset weak CD4+, subset CD30+ and negative for CD8 and CD56

Positive stains
  • Cytotoxic T cell phenotype; less often NK cell phenotype with CD56 expression
  • EBER-ISH
Negative stains
Molecular / cytogenetics description
  • Clonal TCR gene rearrangements in most cases (except some NK cell cases)
  • EBV is monoclonal
Molecular / cytogenetics images

Contributed by Hillary Rose Elwood, M.D.

EBER in situ hybridization


In situ mantle cell neoplasm
Definition / general
  • Cyclin D1 positive B cells with CCND1 rearrangements restricted to the mantle zones of reactive appearing lymphoid tissue (Blood 2016;127:2375)
Essential features
Terminology
  • In situ mantle cell neoplasia is accepted terminology
  • Previous terms are not recommended: in situ mantle cell lymphoma, in situ involvement by mantle cell lymphoma-like cells or mantle cell lymphoma-like B cells of uncertain / undetermined significance (Semin Diagn Pathol 2018;35:76)
Epidemiology
Sites
Pathophysiology
  • t(11;14) IGH::CCND1 translocation results in overexpression of cyclin D1 and dysregulation of the cell cycle (J Clin Invest 2012;122:3416)
  • Low levels of B cells with t(11;14) translocation have been identified in peripheral blood of healthy individuals (Mod Pathol 2012;25:1629)
  • Cyclin D1 positive B cells with CCND1 rearrangements may colonize the mantle zones of lymphoid follicles (Blood 2016;127:2375)
  • Risk of progression is very low (Blood 2016;127:2375)
Etiology
  • Unclear
Clinical features
Diagnosis
  • Usually an incidental finding discovered when cyclin D1 immunohistochemistry is performed on a lymph node
Radiology description
  • No specific radiologic findings
Case reports
Treatment
Microscopic (histologic) description
Microscopic (histologic) images

Contributed by Kyle Bradley, M.D.
Mantle zone and interfollicular involvement Mantle zone and interfollicular involvement

Mantle zone and interfollicular involvement

Early lymph node involvement by mantle cell lymphoma Early lymph node involvement by mantle cell lymphoma Early lymph node involvement by mantle cell lymphoma

Early lymph node involvement by mantle cell lymphoma


Follicular lymphoma with concurrent in situ mantle cell neoplasia Follicular lymphoma with concurrent in situ mantle cell neoplasia Follicular lymphoma with concurrent in situ mantle cell neoplasia Follicular lymphoma with concurrent in situ mantle cell neoplasia Follicular lymphoma with concurrent in situ mantle cell neoplasia

Follicular lymphoma with concurrent in situ mantle cell neoplasia

Positive stains
Negative stains
Flow cytometry description
Molecular / cytogenetics description
Sample pathology report
  • Lymph node, right axilla, excision:
    • In situ mantle cell neoplasm (see comment)
    • Comment: The H&E stained sections show morphologic features consistent with a benign lymph node with follicular hyperplasia. A cyclin D1 immunohistochemical stain demonstrates cyclin D1 positive cells confined to the inner mantle zones of lymphoid follicles without mantle zone expansion. These findings are consistent with a diagnosis of in situ mantle cell neoplasm. In situ mantle cell neoplasm typically follows an indolent course; however, rare cases may progress to overt mantle cell lymphoma.
Differential diagnosis
Board review style question #1
In situ mantle cell neoplasm is characterized by which of the following translocations?

  1. t(8;14) MYC::IGH
  2. t(9;22) BCR::ABL1
  3. t(11;14) IGH::CCND1
  4. t(14;18) BCL2::IGH
Board review style answer #1
C. t(11;14) IGH::CCND1. The characteristic translocation of in situ mantle cell neoplasia is t(11;14) IGH::CCND1, resulting in overexpression of cyclin D1. Answer B is incorrect because t(9;22) BCR::ABL1 is the Philadelphia chromosome associated with AML, CML and B ALL. Answer D is incorrect because t(14;18) BCL2::IGH is associated with follicular lymphoma. Answer A is incorrect because t(8;14) MYC::IGH is associated with Burkitt lymphoma.

Comment Here

Reference: In situ mantle cell neoplasm
Board review style question #2

In a lymph node biopsy, which of the following features supports a diagnosis of in situ mantle cell neoplasm?

  1. Cyclin D1 positive B cells extending into the interfollicular region
  2. Mantle zones prominently expanded by cyclin D1 positive B cells
  3. Monoclonal proliferation of BCL2 positive B cells confined to germinal centers
  4. Preserved lymphoid architecture
Board review style answer #2
D. Preserved lymphoid architecture. In situ mantle cell neoplasm is characterized by cyclin D1 positive B cells restricted to the mantle zone of reactive appearing lymphoid tissue. Lymphoid architecture is preserved. Answers A and B are incorrect because prominent expansion of the mantle zone, complete replacement of the mantle zone or significant extension into the interfollicular region or germinal center by cyclin D1 positive B cells warrants a diagnosis of mantle cell lymphoma rather than in situ mantle cell neoplasm. Answer C is incorrect because a monoclonal proliferation of BCL2 positive B cells confined to germinal centers in a hyperplastic appearing lymphoid follicle is consistent with in situ follicular B cell neoplasm.

Comment Here

Reference: In situ mantle cell neoplasm
Board review style question #3
Which of the following is true of in situ mantle cell neoplasm?

  1. Characterized by aggressive behavior
  2. May be seen in extranodal locations
  3. Peripheral blood involvement is not permitted
  4. Usually progresses to overt mantle cell lymphoma
Board review style answer #3
B. May be seen in extranodal locations. In situ mantle cell neoplasm is a rare finding that is most commonly seen in lymph nodes but may be seen in extranodal locations. Answer C is incorrect because peripheral blood involvement is permitted. Answers A and D are incorrect because it is typically a stable disease with indolent behavior and infrequent (< 10%) progression to overt mantle cell lymphoma.

Comment Here

Reference: In situ mantle cell neoplasm

Inborn error of immunity-associated lymphoid proliferations and lymphomas
Definition / general
  • Primary immunodeficiency disorders (PIDs) describe a group of diseases characterized by a decrease, absence or malfunction of at least 1 part of the immune system, caused by monogenic germline mutations, resulting in loss or gain of function of the encoded protein (Hematology Am Soc Hematol Educ Program 2019;2019:443)
  • Can be dominant or recessive, autosomal or X linked and with complete or incomplete penetrance
  • Generally manifest as increased susceptibility to a spectrum of infectious diseases, variable autoimmune or autoinflammatory conditions or malignancies
  • The 2019 International Union of Immunological Societies (IUIS) classification reports 406 distinct disorders, with 430 different gene defects and a diversity of phenotypes (J Clin Immunol 2020;40:66)
  • Due to compromised immunosurveillance and chronic infections, PID patients have a high risk of developing lymphoproliferative disorders, ranging from nonmalignant lymphoid hyperplasia to malignant lymphoma
Essential features
  • Nonneoplastic or neoplastic lymphoid proliferation are common in PIDs
  • Nonneoplastic lymphoid hyperplasia (such as follicular hyperplasia, marginal zone hyperplasia and paracortical hyperplasia) is associated with recurrent infections or autoimmune reactions
  • Overall, B cell lymphoma is more frequent than T cell neoplasm; the lymphoma cells can be EBER positive or negative
  • Diagnostic criteria of lymphoid neoplasm is nearly the same as immunocompetent counterparts
  • Genetic mechanisms underlying transformation are largely unknown
Epidemiology
  • Each individual primary immune disorder is generally rare but collectively primary immunodeficiency disorders are common
  • Overall risk for malignancy in patients with PID is 4% (range: 0.7 - 15%), 10,000 times greater than the general population
  • Risk for lymphoproliferative and related disorders varies among individual PID
Primary immune disorders associated with predominantly antibody deficiency or low immunoglobulins
    Immunoglobulin A deficiency
  • Prevalence 1:100 - 1,000
  • Results from inherited genetic differences in chromosomes 18, 14 and 6
  • Usually IgA < 7 mg/dL or absent; other immunoglobulin levels are normal or elevated
  • Most patients are asymptomatic; some have infection, autoimmune and inflammatory disorders of gut; 10 - 20 times increased risk for celiac disease (Int J Surg Case Rep 2017;30:69)
  • Microscopic (histologic) description: nodular lymphoid hyperplasia in the lamina propria of GI tract (see Microscopic (histologic) images)
  • Overall the immunostaining pattern on tissue biopsies are nonspecific; occasionally, expansion of CD3+, CD8+, CD57+ LGLs can be identified in peripheral blood, lymph node or bone marrow by either flow cytometry or immunostains
  • Molecular / cytogenetics description: usually IG and TCR gene rearrangements are poly or oligoclonal
  • Rarely, B cell lymphoma and gastric carcinoma are reported (Dis Colon Rectum 1984;27:822)

    Common variable immunodeficiency (CVID)
  • CVID is the most prevalent symptomatic PID in adults (1 in 25,000 - 50,000)
  • Results from variants in multiple genes of B cell development
  • CVID represents a heterogeneous group with antibody deficiency of at least 2 serum Ig isotypes with normal or low B lymphocytes; T cell defects include increased memory T cells, loss of T cell proliferate and T cell associated cytokine defects; other PIDs excluded
  • Good syndrome (GS) presents with thymoma and hypogammaglobulinemia in the fifth to sixth decade of life; patients can have low or absent B cells, variable defects in cell mediated immunity with a CD4 T lymphopenia, inverted CD4/CD8+ T cell ratio and reduced T cell mitogen proliferative responses (J Clin Immunol 2018;38:96)
  • CVID patients present recurrent infections, GI villous atrophy and inflammatory bowel disease (IBD), liver and lung focal nodular hyperplasia and other symptoms (J Allergy Clin Immunol 2014;133:535)
  • Microscopic (histologic) description: follicular hyperplasia with ill defined germinal centers lacking mantle cuffs or marginal zone hyperplasia; the differential diagnosis with B cell lymphoma, especially marginal zone B cell lymphoma is challenging (see Microscopic (histologic) images)
  • Immunohistochemistry: nonspecific; similar to immunocompetent counterparts
  • Aberrant T cell population, such as large granular lymphocytes (LGLs), can be identified in peripheral blood, lymph node or bone marrow by either flow cytometry or immunostains; TCR gene rearrangement can be monoclonal
  • Molecular / cytogenetics description: IG and TCR gene rearrangements are usually poly or oligoclonal; the clonal IG does not warrant a lymphoma diagnosis in CVID patients
  • 4.1% of CVID have malignant lymphoma, especially marginal zone lymphoma or diffuse large B cell lymphoma (DLBCL); usually EBV- (J Clin Immunol 2020;40:524)
    • BCL6 gene rearrangement is reported in these B cell lymphomas

    X linked agammaglobulinemia (XLA)
  • Prevalence 1:250,000 - 379,000
  • Results from mutations of Bruton tyrosine kinase or BTK on the X chromosome (Xq21.3-q22); recently, other genetic defects reported (Front Immunol 2020;11:2001)
  • B cells often < 2% with maturation arrest, all immunoglobulins level decreased and absence of humoral responses
  • Patients present recurrent infections since 6 - 9 months of age, with positive family history (33%); few patients have bronchiectasis and chronic lung disease, IBD or enteritis, neutropenia (Medicine (Baltimore) 2006;85:193, J Allergy Clin Immunol 2020;146:429)
  • Microscopic (histologic) description: nonspecific microscopic features
  • Overall the immunostaining pattern on tissue biopsies are nonspecific; occasionally, expansion of CD3+, CD8+, CD57+ LGLs can be identified in peripheral blood, lymph node or bone marrow by either flow cytometry or immunostains
  • Molecular / cytogenetics description: TCR gene rearrangements are oligoclonal and rarely monoclonal
  • Rare lymphoma is reported; 3.7% of XLA patients develop nonhematopoietic tumor in 1 study (Medicine (Baltimore) 2006;85:193, J Allergy Clin Immunol 2020;146:429)

    Warts, hypogammaglobulinemia, infections and myelokathexis (WHIM)
  • Approximately 0.23 cases per 1,000,000 live births
  • Results from autosomal dominant mutation of CXCR4, encoding GPCR in many leukocytes and involving T / APC synapses; the mutations lead to prolonged receptor activation; the same mutation has been reported in Waldenström macroglobulinemia (Br J Haematol 2016;172:735)
  • Severe congenital neutropenia and myelokathexis are characteristic; B or T lymphopenia and hypogammaglobulinemia were detected in 88% and 58% of patients, respectively (J Allergy Clin Immunol Pract 2019;7:1568)
  • Patients present with severe bacterial infection (78% of patients), recurrent pneumonia (61%), skin warts (61%, at a mean age of 11 years), HPV related malignancies (16%) (N Engl J Med 2019;380:163, J Allergy Clin Immunol Pract 2019;7:1568)
  • Microscopic (histologic) description: myelokathexis and right shifted granulocytosis in bone marrow biopsy
  • EBV+ B cell lymphoma is reported (J Clin Immunol 2019;39:532)
Primary immune disorders associated with combined immunodeficiency
    Severe combined immunodeficiency (SCID)
  • 1 in 50,000 worldwide (J Allergy Clin Immunol 2014;133:335)
  • Results from heterogeneous genetic defects involving VDJ (variable, diversity and joining regions) recombination process (RAG1, RAG2, DCLRE1C, PRKDC, NHEJ, LIG4), T cell maturation (such as ADA, AK2, PNP, CORO1A), T cell signaling (IL2RG, IL7R, JAK3) or T cell receptor (CD3D, CD3E, CD3Z) (Front Immunol 2021;11:623199, J Clin Immunol 2021;41:631, J Clin Immunol 2014;34:304, J Clin Immunol 2011;31:281)
  • Absence or very low level of CD3+ T cells, proliferate response to mitogens < 10% of control, very low serum Ig and impaired specific antibody responses; B cell or natural killer (NK) cell deficiency is variable; T cell receptor excision circle (TREC) levels are absent or very low at birth
  • Patients present with cytopenia of multiple lineages, autoimmunity (including vasculitis, enteropathy and endocrinopathy, at a median age of around 2 years) and granulomatous disease of multiple tissues (J Allergy Clin Immunol Pract 2019;7:1970)
  • Microscopic (histologic) description: nonspecific microscopic features
  • Overall the immunostaining pattern on tissue biopsies are nonspecific; occasionally, expansion of CD3+, CD8+, CD57+ LGLs can be identified in peripheral blood, lymph node or bone marrow by either flow cytometry or immunostains
  • Risk for lymphoma is probably increased


Primary immune disorders associated with immune dysregulation
    Primary hemophagocytic lymphohistiocytosis (HLH)
  • 1:50,000 live born children
  • Results from autosomal recessive mutations of PRF1 gene and other genes, including UNC13D, STXBP2, STX11, LYST (overlaps with Chédiak-Higashi syndrome); RAB27A (Griscelli syndrome type 2); AP3B1 (Heřmanský-Pudlák syndrome type 2)
  • Functionally impairs delivery of perforin and cytotoxic granules and prevents killing of antigen presenting or damaged cells and leads to NK / CTL overt activation (Blood Adv 2020;4:2578, Blood 2020;135:1332)
  • Diagnostic criteria commonly used:
  • Fever ≥ 38.5 °C
  • Splenomegaly
  • Cytopenias (affecting at least 2 of the below): hemoglobin < 9 g/dL (in infants < 4 week: hemoglobin < 10 g/dL) platelets < 100 x 109/L neutrophils < 1 x 109/L
  • Hypertriglyceridemia or hypofibrinogenemia (< 150 mg/dL)
  • Hemophagocytosis in bone marrow, spleen, lymph nodes, liver or other tissue
  • Low or absent NK cell activity
  • Ferritin > 500 ng/mL
  • Elevated sIL2R: > 2,400 U/mL or elevated based on the laboratory defined normal range (Pediatr Blood Cancer 2007;48:124)
  • Clinically overlaps with macrophage activation syndrome (MAS) and HLH episodes during the clinical course of other PIDs (Open Access Rheumatol 2018;10:117)
  • Viral infections, especially EBV, trigger primary as well as secondary forms of HLH
  • See Hemophagocytic lymphohistiocytosis
      X linked lymphoproliferative disease (XLP or Duncan disease)
    • Approximately 1 - 3 in 1,000,000 males
    • Firstly described in 1970s; XLP results from pathogenic mutation or deletion of SH2D1A gene, which causes its coding protein SLAM associated protein (SAP) unable to bind to signaling lymphocytic activation molecule (SLAM) and leads to aberrant IFNγ modulation, causing uncontrolled cell proliferation (Lancet 1978;1:554)
    • Reduced memory B cells, defects of CD8+ / CD4- T and NK cells effector functions; hypogammaglobulinemia due to CD4- T cell defects (Immunol Rev 2005;203:180)
    • Clinical presentation resembles that of infectious mononucleosis and includes hemophagocytic lymphohistiocytosis (HLH), lymphoma and dysgammaglobulinemia, often triggered by Epstein-Barr virus (Blood 2011;118:5060)
    • Clinically overlaps with X linked inhibitor of apoptosis (XIAP) deficiency, caused by BIRC4 gene mutation, with increased risk of EBV- related hemophagocytic lymphohistiocytosis; recurrent splenomegaly, IBD with Crohn's disease-like features, especially in early infancy
    • Microscopic (histologic) description: polyclonal infiltration by EBV infected B cells, as well as by reactive CD4+ and CD8+ T cells; macrophages with hemophagocytosis may be present
    • Immunohistochemistry: similar to immunocompetent host counterparts
    • Molecular / cytogenetics description: no specific genetic abnormalities reported in XLP associated LPD
    • Hematopoietic malignancies include largely B cell lymphomas (Burkitt, DLBCL) and rarely Hodgkin disease or T cell lymphomas; mostly extranodal; EBV positive or negative (Immunol Rev 2005;203:180)

      Autoimmune lymphoproliferative syndrome (ALPS or Canale-Smith syndrome)
    • Prevalence unknown
    • Results from pathogenic mutations in the FAS, FASL or CASP10 genes, which cause impairment of lymphocyte apoptosis
    • Required diagnostic criteria: chronic (> 6 months), nonmalignant, noninfectious lymphadenopathy or splenomegaly; elevated CD3+ TCR alpha / beta positive CD4-, CD8- double negative T cells (≥ 1.5% of the total lymphocytes or 2.5% of the CD3 positive lymphocytes in the setting of normal or elevated lymphocyte counts) (Blood 2020;136:1933)
    • Secondary diagnostic criteria: elevated biomarkers (s-FASL, IL10, vitamin B12, IL18), typical immunohistological findings, autoimmune cytopenia and elevated IgG and family history
    • Patients present lymphadenopathy, hepatosplenomegaly and cytopenia
    • Microscopic (histologic) description: prominent follicular hyperplasia, progressive transformation of germinal centers, expansion of CD4 / CD8 double negative T cells in peripheral blood, lymph nodes and spleen; or frank lymphoma similar to that seen in immunocompetent hosts (Haematologica 2017;102:364)
    • Immunohistochemistry: CD3+, CD4-, CD8-, CD45RA+, CD45RO-, CD25-, variable CD57, may have an increase in CD5+ B cells
    • Molecular / cytogenetics description: no specific genetic abnormalities reported in ALPS associated LPD
    • Hematopoietic malignancies include Hodgkin and non-Hodgkin lymphomas; a subset are EBV positive; rare histiocytic sarcoma reported (Am J Surg Pathol 2010;34:589)
    • Clinically overlaps with RAS associated lymphoproliferative disease (RALD), caused by somatic gain of function mutations in members of the RAS subfamily and lacked the defining increase in the CD3+ / TCRαβ+ CD4- / CD8- double negative T cell compartment, elevated biomarkers (i.e. vitamin B12) and the germline or somatic variants in FAS, FASL or CASP10 (Blood 2011;117:2883)
    • Differential diagnosis of RALD from Rosai-Dorfman syndrome and occasionally systemic lupus erythematosus can be challenging; both of them have reported the same variant of somatic origin, KRAS p.G13C (Clin Immunol 2017;175:143)
    • RALD and juvenile myelomonocytic leukemia (JMML) likely exist on a shared clinical continuum, where JMML undergoes additional genetic changes responsible for malignant transformation (Blood 2015;125:2753)
    • Patients with Gaucher disease (GD) may have ALPS-like features and FAS mediated apoptosis defects (J Allergy Clin Immunol Pract 2020;8:3535)


      PI3K related immunodeficiency
    • Prevalence unknown
    • Results from heterozygous gain of function mutations in PIK3CD gene or others genes in PI3K signaling pathway
    • Usually normal B cells but memory B cells decrease while transitional / immature B cells increase; with variable immunoglobulin levels; progressive CD4+ T lymphopenia, reversed CD4:CD8 ratio; overlap with CVID and hyper-IgM syndrome (Front Immunol 2018;9:2172, Science 2013;342:866, J Clin Med 2020;9:3335, Clin Rev Allergy Immunol 2020;59:323)
    • Patients present early in life with recurrent sinopulmonary infections, chronic virus infection (EBV, CMV), lymphadenopathy and extranodal lymphoid hyperplasia (J Allergy Clin Immunol 2016;138:210, J Allergy Clin Immunol 2017;139:597)
    • Microscopic (histologic) description: atypical follicular hyperplasia with prominent naked germinal centers with ill defined outlines, monocytoid B cell proliferate; resembling CMV lymphadenitis
    • Immunohistochemistry: IgM+ cells surrounding the naked germinal center and rare IgG+ cells
    • Molecular / cytogenetics description: usually IG and TCR gene rearrangements are poly or oligoclonal
    • Rare EBV associated lymphoma reported (Front Immunol 2018;9:1758)

  • Primary immune disorders associated with congenital syndromes or DNA repair defects
      Ataxia telangiectasia (AT)
    • 1 in 40,000 to 100,000 worldwide
    • Results from autosomal recessive mutations in ataxia telangiectasia mutated (ATM) gene encoding a serine / threonine protein kinase and playing a key role in DNA double strand break repair (Am J Hum Genet 1995;57:112)
    • PID in about 70% of AT patients; usually, B or T cell lymphopenia, decreased CD8 or CD4+ T cells, antibody production deficiency, Ig subclasses deficiency (mostly IgA and IgG2), hypogammoblobulinemia, occasionally hyper-IgM due to Ig class switch recombination defect (CSRD) and associated with high frequency of granulomas (J Pediatr 2004;144:505)
    • Patients present with progressive cerebellar ataxia, mucocutaneous telangiectasia, sinopulmonary infections, variable immunodeficiency, radiosensitivity and metabolic disorders; patients often die from chronic pulmonary disease or malignancies (Pediatr Allergy Immunol 2019;30:277)
    • High risk for leukemia and lymphoma (mostly T cell acute lymphoblastic leukemia and T cell prolymphocytic leukemia) in young age; EBV+ Hodgkin lymphoma and B cell non-Hodgkin lymphoma and other tumors are also common (J Clin Oncol 2015;33:202)
    • TCR genes alterations on chromosomes 7 and 14 are seen in AT associated leukemia, including TCL1

      Nijmegen breakage syndrome (NBS)
    • 1 in 100,000 worldwide
    • Results from biallelic mutations in NBN located at 8q21.3, encoding nibrin, 1 of 3 proteins that make up the MRN complex to activate and recruit ATM to double strand breaks (Acta Paediatr Scand 1981;70:557, Science 2005;308:551)
    • Severe hypogammaglobulinemia in 20%, IgA deficiency in 50% and reduced B and T cells in > 80% of NBS patients, resulting in a spectrum from silent phenotype to recurrent, chronic respiratory tract infections (J Clin Immunol 2015;35:538)
    • Patients present with microcephaly at birth with distinct, bird-like craniofacial features, growth retardation and intellectual disability
    • High risk for diffuse large B cell lymphoma, T cell acute lymphoblastic leukemia and less frequently for Hodgkin lymphoma and acute myeloid leukemia (Front Pediatr 2020;8:570084)
    • NBS overlaps with other PIDs; microcephaly and immunodeficiency are common in DNA ligase IV deficiency (LIG4 syndrome) and severe combined immunodeficiency with microcephaly, growth retardation and sensitivity to ionizing radiation due to NHEJ1 deficiency (NHEJ1 syndrome) (Orphanet J Rare Dis 2012;7:13)
    Primary immune disorders associated with other deficiencies
      Chronic granulomatous disease
    • 1 in 200,000 worldwide
    • Results from mutations in genes encoding any of the NADPH oxidase subunit proteins (gp91phox ~70% of cases; p47phox ~25%; p67phox ~2.5%; p22phox ~2.5%; and p40phox < 1%), leading to deficiency in production of antimicrobial reactive oxidative species (ROS) by granulocytes and monocytes (Blood Adv 2020;4:5976)
    • Patients present with recurrent infections by catalase positive organisms; skin, gut, lungs, liver are commonly involved (Hematol Oncol Clin North Am 2013;27:89)
    • Biopsy shows active chronic inflammation, pigmented macrophages, with or without necrotizing or non-necrotizing granuloma (Histopathology 2005;47:508)
    • Granuloma prevalence in all PID patients is approximately 1 - 4% (J Clin Immunol 2018;38:717)
    • Uncertain risk for lymphoma

      Type I interferonopathies
    • Unknown prevalence
    • Genetically and phenotypically heterogeneous group of autoinflammatory and autoimmune disorders, characterized by constitutive activation of the antiviral type I interferon (IFN) axis
    • Mutations in TREX1 and RNase H2 are reported in Aicardi-Goutieres syndrome (AGS) patients, retinal vasculopathy with cerebral leukodystrophy (RVCL), familial chilblain lupus (CHBL), systemic lupus erythematosus (SLE) (EMBO J 2018;37:e98506)
    • Mutations in STING gene cause STING associated vasculopathy with onset in infancy (SAVI) (Front Pediatr 2020;8:577918, J Allergy Clin Immunol Pract 2021;9:803)
    • Mutations in proteasome subunit genes cause chronic atypical neutrophilic dermatosis with lipodystrophy and elevated temperature (CANDLE) (Front Immunol 2017;8:927)
    • Mutations or deficiency in adenosine deaminase 2 (ADA2) cause early onset vasculopathy resembling polyarteritis nodosa (PAN), early onset of recurrent strokes or cytopenia (Arthritis Rheumatol 2019;71:1747)
    • Mutations in DNA polymerase POLA cause X linked reticulate pigmentary disorder (XLPRD), characterized by skin hyperpigmentation, sterile multiorgan inflammation, recurrent infections and distinct facial features; or O'Driscoll syndrome, characterized by growth retardation, microcephaly, hypogonadism and in some cases, overlapping immunological features to those seen in XLPDR (J Clin Immunol 2021;41:285)
    • Biallelic mutations in ACP5, encoding tartrate resistant acid phosphatase (TRACP), cause the inherited immuno-osseous disorder, spondyloenchondrodysplasia (SPENCD)
    • At least 1 autoimmune diagnosis is seen in most patients and some showing recurrent bacterial and viral infections and potential immunodeficiency (PLoS One 2020;15:e0230052, J Clin Immunol 2016;36:529)

      Hyper-IgE syndrome
    • Unknown prevalence
    • Genetically and phenotypically heterogeneous group of PID, characterized by a triad of eczema, recurrent skin and pulmonary infections and elevated IgE levels
    • Food and environmental allergies are common (J Allergy Clin Immunol Pract 2018;6:996)
    • Most cases are sporadic but autosomal recessive (AR), autosomal dominant (AD) and X linked variants were described
    • Heterozygous mutations in STAT3 gene with dominant negative effect cause autosomal dominant hyper-IgE syndrome (AD-HIES), showing eosinophilia, marked elevation in serum IgE (inversely correlated with patient age) and associated with abnormal B cell maturation (J Allergy Clin Immunol Pract 2018;6:996)
    • Biallelic mutations in DOCK8 gene, regulating cytoskeletal rearrangement and immune synapse formation, cause autosomal recessive hyper-IgE syndrome (AR-HIES), showing eosinophilia, elevated IgE levels, T cell lymphopenia, variable B cell number and Ig levels (J Allergy Clin Immunol 2010;125:743)
    • Other mutations (TYK2, PGM3, CARD11) have been reported in hyper-IgE patients (J Allergy Clin Immunol 2016;138:1222, J Pediatr 2004;144:93)
    • Omenn syndrome, IPEX, Wiskott-Aldrich syndrome (WAS) and other PIDs with similar phenotypes are excluded
    Treatment
    • Treatment of PID is dependent on the underlying PID and clinical presentation
    • Bone marrow transplant has been performed in some PIDs (J Allergy Clin Immunol Pract 2021;9:628)
    • Treatment of lymphoproliferative disease or lymphomas is similar to immunocompetent counterparts
    Microscopic (histologic) images

    Contributed by Jinjun Cheng, M.D., Ph.D.
    Lymphoid aggregate

    Lymphoid aggregate

    Lymphoid follicles

    Lymphoid follicles

    Marginal zone expansion

    Marginal zone expansion

    DLBCL

    DLBCL

    Flow cytometry images

    Contributed by Jinjun Cheng, M.D., Ph.D.
    Large granular lymphocytosis

    Large granular lymphocytosis

    Board review style question #1
    A 10 year old child was recently diagnosed with autoimmune lymphoproliferative syndrome (ALPS). The molecular studies identified a pathogenic mutation in the FAS gene. The patient's peripheral blood was submitted for flow cytometry study. Which of the following aberrant T cell populations is expected to be elevated in this peripheral blood?

    1. CD4- CD8- T cells
    2. CD4+ CD8+ T cells
    3. CD4+ TFH cells
    4. CD4+ Treg cells
    5. Natural killer (NK) cells
    Board review style answer #1
    A. Elevated CD3+ TCR alpha / beta positive CD4- CD8- double negative T cells (≥ 1.5% of the total lymphocytes or 2.5% of the CD3+ lymphocytes in the setting of normal or elevated lymphocyte counts) is commonly seen in peripheral blood or lymph node of ALPS patients.

    Comment Here

    Reference: Associated with primary immune disorders

    Indolent NK cell lymphoproliferative disease of the GI tract (pending)
    [Pending]

    Indolent T cell lymphoma of the GI tract
    Definition / general
    • Clonal T cell lymphoproliferative disorder (T-LPD) that involves the mucosa of gastrointestinal (GI) tract
    Essential features
    • Rare, clinically indolent clonal T cell lymphoproliferative disease confined to the GI tract
    • Can involve all GI sites, with small intestine and colon being most commonly involved
    • Most patients are middle aged men, frequently presenting with diarrhea
    • Nondestructive infiltrate of small, monotonous lymphoid cells that can be CD4+ / CD8- or CD4- / CD8+
    • Poor response to chemotherapy; chronic course with frequent relapses and prolonged survival
    ICD coding
    • ICD-O: 9702/1 - indolent T cell lymphoproliferative disorder of the gastrointestinal tract
    Epidemiology
    Sites
    • Most commonly involves small intestine and colon but any GI site can be involved (Blood 2013;122:3599)
    • Rarely associated with regional (abdominal) lymphadenopathy
    • Occasional reports of bone marrow involvement
    Etiology
    • Unknown
    Clinical features
    • Most common presenting symptoms are abdominal pain and diarrhea (Blood 2013;122:3599)
    • Other symptoms / signs include vomiting, weight loss, signs of malabsorption
    • Disease can clinically mimic inflammatory bowel disease
    • Some patients had a history of Crohn's disease
    • Chronic course with frequent relapses
    • Prolonged survival with persistent disease
    • 4 reported cases in the literature of progression to aggressive T cell lymphoma, with poor outcome (Int J Surg Pathol 2019;27:102)
    Diagnosis
    • Biopsy of affected GI site
    Prognostic factors
    • No specific prognostic factors identified
    Case reports
    Treatment
    • Poor response to conventional chemotherapy and immunotherapy; so far, there is no successful treatment
    Microscopic (histologic) description
    • Dense lymphoid infiltrate in the lamina propria / mucosa, occasionally extending into the submucosa (Blood 2013;122:3599)
    • Infiltrate is nondestructive; adjacent glands are displaced and distorted
    • Infiltrate is composed of monomorphic small lymphoid cells with scant pale cytoplasm, slightly irregular nuclear contours, mature chromatin, inconspicuous nucleoli
    • Cases with oral involvement manifest as ulcers with an underlying lymphocytic infiltrate
    • Typically no necrosis, ulceration or angiocentricity / angiodestruction (Blood 2013;122:3599)
    Microscopic (histologic) images

    Contributed by Anamarija M. Perry, M.D.
    Colon with indolent T-LPD Colon with indolent T-LPD

    Colon with indolent T-LPD

    Infiltration of lamina propria

    Infiltration of lamina propria

    Atypical infiltrate of indolent T-LPD

    Atypical infiltrate of indolent
    T-LPD


    CD8 immunohistochemical stain

    CD8 IHC stain

    CD4 immunohistochemical stain

    CD4 IHC stain

    TIA1 immunohistochemical stain

    TIA1 IHC stain

    Virtual slides

    Images hosted on other servers:
    Colonic mucosa with indolent T-LPD

    Colonic mucosa with indolent
    T-LPD

    Positive stains
    Negative stains
    Flow cytometry description
    • Immunophenotype similar to mature T lymphocytes
    • Aberrant loss of CD7 is rarely seen
    Molecular / cytogenetics description
    • Clonal TCR gene rearrangements present by PCR testing
    • t(9;17)(p24.1;q21.2) STAT3-JAK2 fusion found as recurrent abnormality, mainly in CD4+ cases (Blood 2018;131:2262)
    • Genetic alterations are common, including recurrent mutations and novel rearrangements, often involving JAK-STAT pathway genes (Haematologica 2020;105:1895)
    Differential diagnosis
    • Intestinal T cell lymphoma:
    • Celiac sprue:
      • In cases of indolent T-LPD that involve duodenum, differential diagnosis includes celiac sprue
      • Distribution of lymphocytes in indolent T-LPD is not characteristic for celiac sprue (no clustering at the tip)
      • There are no other histologic features of sprue (i.e. villous atrophy, hyperplastic and elongated crypts, increase in plasma cells in lamina propria)
    • Inflammatory bowel disease (IBD):
      • Crypt distortion is present in indolent T-LPD
      • However, other morphological features that are usually present in IBD are absent, including cryptitis, crypt abscesses, basal plasmacytosis and Paneth cell hyperplasia
    Board review style question #1

    A 52 year old man presents with a 10 year history of intermittent abdominal pain and diarrhea, which usually resolves in several weeks without treatment. Colonoscopy revealed erythematous irregular mucosa throughout the entire length of colon. Biopsy was taken and infiltrate is demonstrated in the image. The infiltrate involves mainly lamina propria, it is nondestructive and is positive for CD8 and TIA1 immunohistochemical stains. Molecular study was performed and showed clonal T cell gene rearrangement. Which of the following is true about this disease?

    1. Infiltrate is destructive and frequently involves the entire thickness of bowel wall
    2. It is successfully treated with CHOP chemotherapy
    3. Most cases are CD4- / CD8-
    4. Oral cavity is the most common site of involvement
    5. Subset of cases show STAT3-JAK2 fusion
    Board review style answer #1
    E. Subset of cases show STAT3-JAK2 fusion

    Comment Here

    Reference: Indolent T lymphoproliferative disease of the GI tract

    Intestinal T cell lymphoma, NOS
    Definition / general
    • Primary intestinal T cell lymphomas (ITCL) are rare malignancies that account for < 10% of intestinal lymphomas
    • Diagnosis of the not otherwise specified category (ITCL, NOS) is established when there is a failure of classification with the current schemes
    Essential features
    Terminology
    • ITCL, NOS
    • ITCL, unclassified
    ICD coding
    • ICD-10: C86.2 - enteropathy type (intestinal) T cell lymphoma
    • ICD-11: 2A90.7 - enteropathy associated T cell lymphoma
    Epidemiology
    Sites
    • The few cases reported demonstrate involvement in the small intestine
    Etiology
    Diagrams / tables

    Images hosted on other servers:
    Differential diagnosis

    Differential diagnosis of intestinal T lymphomas

    Clinical features
    Diagnosis
    • Computed tomography (CT) scans can be performed to show intestinal wall thickening; however, a biopsy is required for diagnosis
    Radiology description
    • Intestinal wall thickening in CT scans
    Prognostic factors
    Case reports
    Treatment
    • Similar to the systemic peripheral T cell lymphoma, NOS counterpart and includes induction with CHOP chemotherapy (cyclophosphamide, doxorubicin hydrochloride [hydroxydaunorubicin], vincristine sulfate [Oncovin] and prednisone)
    Gross description
    • Involved areas are characterized by thickened rugal folds
    Microscopic (histologic) description
    • Dense and extensive infiltrates that are composed of atypical lymphocytes that are medium to large in size, with irregular nuclear contours
    • Necrosis may be present
    • Epitheliotropism can be present
    • References: Diagn Pathol 2022;17:53, Clin Case Rep 2022;10:e05546
    Microscopic (histologic) images

    Contributed by Carlos A. Murga-Zamalloa, M.D.
    Epitheliotropism of atypical infiltrates

    Epitheliotropism of atypical infiltrates

    Infiltrates adjacent to ulcer site

    Infiltrates adjacent to ulcer site

    Submucosal extension of infiltrates

    Submucosal extension of infiltrates

    CD3 positive infiltrates

    CD3 positive infiltrates

    Aberrant CD5 loss

    Aberrant CD5 loss

    Positive stains
    Negative stains
    Sample pathology report
    • Small intestine, biopsies:
      • Atypical lymphocytic infiltrates, consistent with involvement by T cell lymphoma (see comment)
      • Comment: The overall morphological and immunohistochemical findings are most consistent with T cell lymphoma unclassified (not otherwise specified, NOS). Correlation with staging and imaging studies is required. In the right clinical context, the findings can be consistent with primary intestinal T cell lymphoma, not otherwise specified (ITCL, NOS).
    Differential diagnosis
    Additional references
    Board review style question #1

    Which of the following is the correct statement regarding intestinal T cell lymphoma, not otherwise specified?

    1. All patients characteristically feature persistent diarrhea
    2. Clinical course is indolent
    3. Diagnosis requires the exclusion of celiac disease
    4. Loss of the expression of pan T cell markers is rarely observed
    5. Lymphocytic infiltrates are benign appearing and indistinguishable from inflammatory bowel disease
    Board review style answer #1
    C. Diagnosis requires the exclusion of celiac disease

    Comment Here

    Reference: Intestinal T cell lymphoma, NOS

    Intravascular
    Definition / general
    • Extranodal large B cell lymphoma characterized by lymphoma cells, predominantly within lumina of blood vessels, especially capillaries, with the exclusion of larger arteries and veins
    • Few to no circulating lymphoma cells in peripheral blood
    Essential features
    • Rare, mature B cell lymphoma limited to intravascular spaces
    • Most common in skin and central nervous system
    • Classical variant most common clinical presentation (neurological and cutaneous manifestations)
    • Neoplastic cells are located in lumina of small to intermediate sized vessels
    • Lymphoma cells are large with prominent nucleoli and vesicular chromatin
    • Pan B cell markers positive, as well as BCL2 and MUM1
    • Aggressive behavior with short overall survival
    Terminology
    • Intravascular large B cell lymphoma (ILBCL)
    • Malignant angioendotheliomatosis, angioendotheliomatosis proliferans syndrome, intravascular lymphomatosis, angioendothelilotropic lymphoma (all obsolete)
    ICD coding
    • ICD-O: 9712/3 - intravascular large B cell lymphoma
    Epidemiology
    Sites
    • Selective growth within lumina of small blood vessels, particularly capillaries
    • Extranodal sites, including bone marrow, with sinusoidal or perivascular involvement
    • Most common sites of involvement: skin, nervous system, kidneys, lungs, endocrine glands
    • Lymph node involvement rare (Blood 2018;132:1561)
    Pathophysiology
    • Localization to blood vessel lumens partially explained by lack of CD29 (β1 integrin) and CD54 (ICAM1), both of which are important for transvascular lymphocyte migration (Blood 2018;132:1561)
    Etiology
    • Unknown
    Clinical features
    • Classic / western form:
      • Clinical presentation ranges from a few mild symptoms (fever of unknown origin, pain, organ specific local symptoms) to severe symptoms (B symptoms and signs of multiorgan failure) (Cancer Sci 2021;112:3953)
      • CNS involvement (present in 35%) with heterogeneous symptoms, including sensory and motor deficits or neuropathies, meningoradiculitis, paresthesias, hemiparesis, seizures and altered mental state
      • Skin lesions (present in 40%) are heterogeneous, including painful indurated erythematous eruptions, cellulitis, peau d’orange, small red palpable spots, nodules with or without ulceration, tumors and erythematous and desquamated plaques (Blood 2018;132:1561)
      • Lymphadenopathy typically absent
      • May present with disseminated intravascular coagulation leading to critical bleeding after biopsy (Cancer Sci 2021;112:3953)
    • Cutaneous variant:
      • 25% of patients
      • More frequent in western countries
      • Patients usually younger, with a median age of 59
      • Single or multiple skin lesions; no other sites involved
      • Disease progression is less aggressive and associated with better overall survival (Blood 2018;132:1561)
    • Hemophagocytic syndrome associated form:
      • Presents in Asian populations with multiorgan failure, hepatosplenomegaly and pancytopenia
      • Bone marrow infiltration common
      • Skin and CNS involvement are rare
      • Rapid aggressive onset and progression with median survival of 2 - 8 months (Blood 2018;132:1561)
    Diagnosis
    Laboratory
    • Anemia, leukopenia, thrombocytopenia and unexplained hypoxemia are most frequently observed
    • Low levels of serum albumin
    • High LDH and ferritin (Br J Haematol 2019;187:328)
    Radiology description
    • Lung involvement presents with ground glass appearance and nodules (Blood 2018;132:1561)
    • Organ enlargement (liver, spleen, kidney and adrenal gland) is common (Cancer Sci 2021;112:3953)
    • Brain MRI shows hyperintense lesions in pons, nonspecific white matter lesions, infarct-like lesions or meningeal enhancement
    • PET CT can detect bone marrow involvement (Br J Haematol 2019;187:328)
    Prognostic factors
    • Median overall survival of 105 months, with chemotherapy 135 months
    • 5 year survival of approximately 50 - 60% (Br J Haematol 2019;187:328)
    Case reports
    Treatment
    Microscopic (histologic) description
    • Lymphoma cells can be found in any organ vessel lumina (Br J Haematol 2019;187:328)
    • Lymphoma cells are large with high nuclear to cytoplasmic ratio with single or multiple prominent nucleoli and scant cytoplasm
    • Morphologic spectrum from centroblasts to immunoblasts / plasmablasts, including rare forms with anaplastic morphology
    • Different growth patterns:
      • Discohesive - lymphoma cells are preferentially within the central portion of the blood vessels with a free floating appearance
      • Cohesive pattern - lymphoma cells almost completely fill the lumen to the point that assessment of vascular structure tends to be difficult
      • Marginating pattern - less frequent, lymphoma cells preferentially adhere to endothelium, leaving the central portion of the lumen free (Blood 2018;132:1561)
    • Infiltration patterns in bone marrow:
      • Pure intrasinusoidal infiltration - neoplastic cells are confined within the intrasinusoidal spaces
      • Intrasinusoidal infiltration with extravasation - neoplastic cells proliferate within intrasinusoidal space but with extravasation
      • Diffuse interstitial infiltration - neoplastic cells proliferate diffusely within bone marrow (Br J Haematol 2019;187:328)
    Microscopic (histologic) images

    Contributed by Kathryn Gibbons, M.D.
    ILBCL involving skin and subcutaneous tissue

    ILBCL involving skin and subcutaneous tissue

    Large lymphoma cells in small vessels

    Large lymphoma cells in small vessels

    ILBCL involving the brain parenchyma

    ILBCL involving the brain parenchyma

    Vessel with involvement by ILBCL

    Vessel with involvement by ILBCL


    Large lymphoma cells in the vessels

    Large lymphoma cells in the vessels

    CD20

    CD20

    BCL6

    BCL6

    MUM1

    MUM1

    Virtual slides

    Images hosted on other servers:

    Lymphoma cells in blood vessels of subcutaneous adipose tissue

    Lymphoma cells limited to lumina of blood vessels

    Positive stains
    Negative stains
    Molecular / cytogenetics description
    Sample pathology report
    • Skin, left thigh, punch biopsy:
      • Intravascular large B cell lymphoma (see comment)
      • Comment: Biopsy shows large atypical CD20 positive B cells present within vascular spaces in the subcutaneous fat. These cells coexpress CD5, BCL6 and MUM1 and are negative for CD10 and CD3. Overall, these findings are consistent with intravascular large B cell lymphoma.
    Differential diagnosis
    Board review style question #1

    The image shown above is from a brain biopsy in a patient with fever and seizures. Large cells are strongly positive for CD20. What is the correct diagnosis?

    1. CNS vasculitis
    2. Intravascular large B cell lymphoma
    3. Marginal zone lymphoma
    4. Plasmablastic lymphoma
    Board review style answer #1
    B. Intravascular large B cell lymphoma

    Comment Here

    Reference: Intravascular large B cell lymphoma
    Board review style question #2
    Which of the following combinations of immunohistochemical stains is most consistent with immunophenotypic profile of intravascular large B cell lymphoma?

    1. CD20+, CD3-, CD5-, CD10+, BCL6+, BCL2+, c-MYC+
    2. CD20+, CD5+, Cyclin D1+
    3. CD20+, PAX5+, MUM1+, BCL2+
    4. CD20-, CD2+, CD3+, CD5-, CD7+, CD56+
    Board review style answer #2
    C. CD20+, PAX5+, MUM1+, BCL2+

    Comment Here

    Reference: Intravascular large B cell lymphoma

    LBCL with IRF4
    Definition / general
    • Uncommon subtype of large B cell lymphoma characterized by strong expression of IRF4 / MUM1
    • Commonly associated with IRF4 gene rearrangement
    Essential features
    • Children and young adults
    • Lymph nodes of head and neck or Waldeyer ring are affected
    • Diffuse, diffuse and follicular or follicular pattern
    • Medium to large cells
    • Germinal center B cell phenotype with diffuse IRF4 / MUM1 expression
    • Most cases show IRF4 rearrangement
    Terminology
    • Lymphoma of Waldeyer ring
    ICD coding
    • ICD-O: 9698/3 - follicular lymphoma, grade 3
    • ICD-10: C83.3 - diffuse large B cell lymphoma
    Epidemiology
    • Rare subtype accounting for 0.05% of diffuse large B cell lymphomas
    • Primarily affects children and young adults
    • Equal gender incidence (Blood 2011;118:139)
    Pathophysiology
    Clinical features
    • Isolated lymph node enlargement
    • Tonsillar enlargement
    Diagnosis
    • Biopsy of the involved lymph node / tissue with immunophenotyping and confirmation using FISH analysis for IRF4 rearrangement
    • Strong, diffuse positivity for IRF4 / MUM1 in neoplastic cells
    • Young age of the patient, head and lymph node / Waldeyer ring location
    Prognostic factors
    • Favorable response to treatment with immunochemotherapy
    Case reports
    Treatment
    • Responds well to chemoimmunotherapy with or without radiation
    Gross description
    • Involved lymph node / tonsil is enlarged
    • Firm, fleshy homogenous mass
    Microscopic (histologic) description
    Microscopic (histologic) images

    Contributed by Jayalakshmi Balakrishna, M.D.

    Diffuse pattern

    Expansile follicles

    Diffuse effacement
    with medium
    sized cells


    Medium sized atypical lymphoid cells

    Round to irregular nuclear contours and conspicuous nucleoli

    Follicular pattern


    CD20

    BCL2

    BCL6

    MUM1

    CD21

    Positive stains
    Negative stains
    Flow cytometry description
    • Low side light scatter and moderate forward light scatter
    • Pan B cell markers and CD10 positive
    • Surface immunoglobulin light chain positive with monoclonal pattern
    Molecular / cytogenetics description
    Molecular / cytogenetics images

    Contributed by Itziar Salaverria, Ph.D.

    FISH break apart probe for IRF4

    Sample pathology report
    • Lymph node, left cervical, excision:
      • Large B cell lymphoma with IRF4 rearrangement; see synoptic report
    Differential diagnosis
    Board review style question #1

    An 8 year old boy has tonsillar enlargement. Which stain is most likely positive in the neoplastic cells?

    1. CD1a
    2. CD3
    3. MUM1
    4. TdT
    Board review style answer #1
    Board review style question #2
    Which is the common partner gene of IRF4 in large B cell lymphoma with IRF4 rearrangement?

    1. BCL2
    2. c-Myc
    3. IgH
    4. IgK
    Board review style answer #2

    Large B cell lymphoma with 11q aberration
    Definition / general
    • Rare B cell lymphoma that resembles Burkitt lymphoma morphologically, phenotypically and by gene expression profiling
    • Negative for MYC rearrangement
    Essential features
    • Rare, germinal center-derived mature B-cell lymphoma characterized by proximal gains and telomeric losses of chromosome 11q (including 11q-gain/loss, telomeric loss or telomeric LOH pattern)
    • Features overlapping with Burkitt lymphoma, but lacks MYC rearrangement
    • Most patients are children and young adults presenting with lymphadenopathy
    • Diffuse infiltrate of medium sized to large cells, frequently with interspersed tingible body macrophages and apoptotic debris
    • Lymphoma cells positive for CD20, CD10, and BCL6, and negative for BCL2, with Ki67 > 95%
    • Lymphoma cells show LMO2 and sometimes CD56 expression (Leukemia 2022;36:1720)
    • Favorable prognosis with intensive chemoimmunotherapy
    Terminology
    • Large B cell lymphoma with 11q aberration (Blood 2022;140:1200)
    • High grade B cell lymphoma with 11q aberrations (Leukemia 2022;36:1720)
    • Not recommended: Burkitt-like lymphoma with 11q aberration
    • Burkitt-like lymphoma
    ICD coding
    • ICD-O: 9687/3 - Burkitt lymphoma, NOS
    Epidemiology
    Sites
    Diagnosis
    • Tissue biopsy of lymph node or extranodal site
    Prognostic factors
    Case reports
    Treatment
    Microscopic (histologic) description
    • Diffuse involvement by sheets of lymphoma cells
    • Occasional cases show a vaguely nodular pattern (Haematologica 2019;104:1822)
    • Some cases are similar to Burkitt lymphoma, with tingible body macrophages, exhibiting starry sky pattern
    • Other cases have fewer macrophages but frequently show apoptotic bodies
    • Lymphoma cells are medium-sized to large and usually have more cytological pleomorphism than typical Burkitt lymphoma with more irregular nuclei
    • Some cases morphologically resemble diffuse large B cell lymphoma (Haematologica 2019;104:1822, Am J Clin Pathol 2017;149:17)
    Microscopic (histologic) images

    Contributed by Anna Shestakova, M.D., Ph.D.
    Diffuse lymph node infiltration by lymphoma cells

    Diffuse lymph node infiltration by lymphoma cells

    Lymphoma cells with interspersed tingible body macrophages

    Interspersed tingible body macrophages

    Atypical lymphoma cells

    Atypical lymphoma cells


    CD20 stain

    CD20 stain

    CD10 stain

    CD10 stain

    BCL6 stain

    BCL6 stain

    Ki67 stain

    Ki67 stain

    Virtual slides

    Images hosted on other servers:

    Abdominal mass in a 20 year old man

    Positive stains
    Flow cytometry description
    • Mature B cell lymphoma with germinal center immunophenotype, positive for CD19, CD20, CD10 and CD38 with surface light chain restriction (Mod Pathol 2018;31:732)
    Molecular / cytogenetics description
    Sample pathology report
    • Lymph node, cervical, core needle biopsy:
      • Burkitt-like lymphoma with 11q aberration
    Differential diagnosis
    Board review style question #1

    A 20 year old man presents with abdominal pain and recent 20 pound weight loss. He was found to have a 10 cm peripancreatic mass, which was biopsied and shown in the figure above. Cytogenetic studies are negative for MYC gene rearrangement. Chromosomal genetic microarray shows 11q aberration with proximal gain and telomeric loss. Which of the following is the typical immunophenotype seen in this entity?

    1. CD20+, CD10+, BCL6+, MUM1-, BCL2-, Ki67 > 95%
    2. CD20+, CD10+, BCL6+, MUM1+, BCL2+, Ki67 > 95%
    3. CD20+, CD10-, BCL6-, MUM1+, BCL2+, Ki67 > 95%
    4. CD20+, CD10-, BCL6-, MUM1+, BCL2-, Ki67 > 95%
    Board review style answer #1
    A. Burkitt-like lymphoma with 11q aberration usually shows the following immunophenotype: CD20+, CD10+, BCL6+, MUM1-, BCL2-, Ki67 > 95%

    Comment Here

    Reference: Large B cell lymphoma with 11q aberration

    Lymphomas arising in immune deficiency / dysregulation
    Definition / general
    • Lymphomas that develop in HIV+ patients (LHIV) are predominantly aggressive B cell lymphomas
    • Some are considered acquired immunodeficiency syndrome (AIDS); defining conditions and are the initial manifestation of AIDS
    • These disorders are heterogeneous and include lymphomas usually diagnosed in immunocompetent patients, as well as those seen much more often in the setting of HIV infection (WHO Classification of Tumours of Haematopoietic and Lymphoid Tissue, IARC 2008)
    Epidemiology
    • Occur in 4 - 10% of AIDS patients, 100× expected rates of non-Hodgkin lymphoma (Hum Pathol 2002;33:392)
    • Overall 60 - 200× increase of all types of NHL in HIV+ patients
    • Conversely, 10% of non-Hodgkin lymphomas in US / Europe are AIDS related
    • Most common LHIV: Burkitt lymphoma, diffuse large B cell lymphoma (DLBCL), primary effusion lymphoma (PEL), plasmablastic lymphoma (ScientificWorldJournal 2011;11:687)
    • Burkitt Lymphoma in HIV patients is 1000× more common than in non HIV+ individuals
    • Three categories:
      1. More specific to HIV+ patients: primary effusion lymphoma, plasmablastic lymphoma, primary CNS lymphoma (Arch Neurol 2010;67:291)
      2. Common also in HIV- patients: Burkitt lymphoma, DLBCL, extranodal MZL, MALT, rare peripheral T cell lymphoma, classic Hodgkin lymphoma (Adv Hematol 2011;2011 Epub 2010 Sep 23)
      3. Occurring in other immunodeficient states: polymorphic B cell posttransplant lymphoproliferative disease
    • Before highly active antiretroviral therapy (HAART), primary CNS lymphoma & Burkitt lymphoma had a 1000× increase in HIV+ patients; since HAART, risk of NHL declined dramatically from 53 to 23 standardized incidence ratio
    • Considerable risk decrease in 1996, stable thereafter
    • Decreased incidence of AIDS associated NHL after HAART also consistent with improved CD4 counts
    • HAART associated with enhanced survival (75% decrease in mortality)
    • Unexpected increase in the incidence of classic Hodgkin lymphoma (CHL) since HAART may be related to improved CD4 counts (CHL incidence lower in patients with severe immunosuppression)
    Sites
    • Usually extranodal: GI, CNS, liver, bone marrow
    • Rare peripheral blood involvement (occasional Burkitt lymphoma presenting as acute leukemia)
    • Unusual sites frequently involved: oral cavity, jaw, body cavities
    • Other possible extranodal sites: lung, skin, testis, heart, breast
    • Lymph node involvement: 1/3 of patients, since HAART – 1/2
    Etiology
    • Heterogeneous, with several pathogenetic mechanisms: chronic antigen stimulation, genetic abnormalities, cytokine dysregulation, EBV and HHV8
    • EBV+ lymphomas have decreased in the HAART era
    • Generally monoclonal but some are polyclonal / oligoclonal suggesting multistep lymphomagenesis
    • Chronic antigen stimulation reflected by: B cell stimulation, hypergammaglobulinemia, persistent generalized lymphadenopathy
    • Cytokine dysregulation: high levels of IL6 and IL10 (LHIV associated with EBV or HHV8)
    • EBV positivity: overall ~40% of LHIV, 80 - 100% of primary CNS lymphomas & PEL, 80% of DLBCL with immunoblastic features, 30 - 50% of Burkitt lymphoma, virtually 100% of CHL
    • HHV8 specifically associated with PEL, usually in the context of profound immunosuppression (late stage disease)
    • HIV is NOT directly involved in the malignant transformation of B cells (no HIV sequences detected in lymphoma cells)
    Clinical features
    • Frequent presentation with advanced clinical stage, bulky disease and high tumor burden
    • LDH markedly elevated
    • Significant relationship between lymphoma type and HIV disease status:
      • DLBCL: long standing AIDS, lower CD4 counts (mean < 100×106/L), higher rate of opportunistic infections
      • Burkitt lymphoma: less immunodeficient patients, higher CD4 counts ( > 200×106/L), shorter mean interval between the diagnosis of HIV seropositivity and lymphoma
    Treatment
    • Aggressive, poor outcome
    • Before HAART:
      • ~50% rate of complete remission for most histological subtypes
      • 2 year survival significantly lower for DLBCL than Burkitt lymphoma
      • International Prognostic Index (IPI): reliable indicator, also correlated with the degree of immunosuppression
      • Other adverse prognostic indicators: age > 35 years, IV drug use, Stage III / IV, CD4 count < 100×106/L
      • Outcome closely related to the severity of immunodeficiency (despite dose adjustment)
    • Since HAART:
      • Improved overall survival of patients with DLBCL approaching that of patients with de novo lymphoma
      • Most important prognostic factor for time of survival – achievement of complete remission
    • Long-term survival in ~1/3 of patients with favorable prognostic features
    • PEL – usually low complete remission rate and very poor prognosis
    Microscopic (histologic) description
    • Diffuse, pleomorphic, high grade B cells; frequent mitotic figures, increased cellular debris, necrosis common
    • Some aggressive B cell lymphomas are common with HIV negative patients, some are more specific for AIDS patients
    Immunohistochemistry & special stains
    • Usually B cell, occasionally CHL, rarely T cell
    Molecular / cytogenetics description
    • Consistently monoclonal
    • Abnormalities involving MYC, BCL6 and tumor suppressor genes

    Lymphomatoid granulomatosis
    Definition / general
    • Lymphomatoid granulomatosis is an angiocentric and angiodestructive lymphoproliferative disease involving extranodal sites composed of Epstein–Barr virus (EBV) positive B cell admixed with reactive T cells, which usually predominate (Blood 2016;127:2375)
    Essential features
    • Angiocentric and angiodestructive lymphoproliferative disease involving extranodal sites composed of EBV positive B cell admixed with reactive T cells
    Terminology
    • Angiocentric immunoproliferative lesion
    ICD coding
    • ICD-O: 9766/1 - lymphomatoid granulomatosis
    • ICD-O: 9766/3 - lymphomatoid granulomatosis, grade 3
    Epidemiology
    Sites
    Pathophysiology
    • Lymphomatoid granulomatosis due to the predominance of T cells on pathologic examination, was initially thought to be a T cell disorder (Am J Med 1982;72:467)
      • Subsequently, lymphomatoid granulomatosis was determined to be a B cell lymphoproliferative disorder secondary to EBV, with a prominent angiocentric T cell infiltrate (Mod Pathol 1990;3:435)
    • Lymphomatoid granulomatosis has a complex relationship with the host’s underlying immune function and defective immune surveillance of EBV infected B cells, particularly a functional defect in CD8+ cytotoxic T cells, is hypothesized to lead to disease development (Blood 1996;87:4531)
    • Particular study suggests that immunologic deficits are likely preexistent and that a quantitative or qualitative defect in mainly CD8+ cytotoxic T cells may be prerequisite for disease (Cancer 1982;49:2070)
    Etiology
    • Resembles an EBV driven lymphoproliferative disorder (Blood 1996;87:4531)
    • Also common in many immunodeficiency states, such as AIDS, allogenic organ transplant, Wiskott-Aldrich syndrome and X linked lymphoproliferative syndrome (Cancer Surv 1997;30:233)
    • Normal counterpart: mature B cell, transformed by EBV (Blood 2016;127:2375)
    Clinical features
    Diagnosis
    • Combination of light microscopy and immunohistochemical markers or in situ hybridization for EBV coded small RNA (EBER)
    Radiology description
    • Chest Xray may show single or multiple nodules with poorly defined margins
    • CT scans show well defined and poorly defined nodules throughout both lungs (AJR Am J Roentgenol 2000;175:1335)
    Radiology images

    Contributed by Nicholas Joseph Dcunha, M.B.B.S., M.D. and Elanthenral Sigamani, M.B.B.S., M.D.

    Mass in lung (CT scan)

    Prognostic factors
    • Prognostic features are difficult to determine due the rarity of the disease
    Case reports
    Treatment
    • In a prospective NCI study, patients with low grade lymphomatoid granulomatosis received primary therapy with IFN-α while patients with high grade lymphomatoid granulomatosis received primary therapy with DA-EPOCH-R (dose adjusted etoposide, prednisone, vincristine, cyclophosphamide, doxorubicin and rituximab) every 3 weeks for up to 6 cycles of therapy (Blood 2018;132:785)
    Gross description
    Microscopic (histologic) description
    • Angiocentric and angiodestructive polymorphous lymphoid infiltrate (Cancer 1979;43:360)
    • Lymphocytic vasculitis with infiltration of the vessel wall; may also show infarct like tissue necrosis or fibrinoid necrosis of vessel wall (Blood 1997;90:4099)
    • Infiltrate is predominantly composed of lymphocytes with admixed plasma cells, immunoblasts and histiocytes; background lymphocytes; may show atypia but are not overtly neoplastic
    • Lymphomatoid granulomatosis is composed of variable number of EBV positive B cells admixed with inflammatory background (Am J Surg Pathol 2010;34:e35)
    • EBV positive cells resemble immunoblasts or are multinucleated; may show atypia
    • Classic Reed-Sternberg cells are absent
    • Well formed granulomas are typically not seen in lungs and most other extranodal sites except skin where granulomatous reaction might be seen in subcutaneous tissue (Am J Surg Pathol 2001;25:1111)
    • Grading is based on the relative number of EBV positive B cells to the reactive lymphocyte background (Am J Surg Pathol 2015;39:141)
      • Grade I: EBV positive cells are < 5 per high power field; absent or rare large transformed cells on light microscopy; necrosis if present is focal
      • Grade II: EBV positive cells are 5 - 50 per high power field; few large transformed cells on light microscopy; necrosis is more commonly seen
      • Grade III: EBV positive cells are > 50 per high power field; large transformed cells are easily seen on light microscopy; large areas of necrosis are common
    Microscopic (histologic) images

    Contributed by Nicholas Joseph Dcunha, M.B.B.S., M.D. and Elanthenral Sigamani, M.B.B.S., M.D.

    Lymphocytic vasculitis with angioinvasion (lung)

    CD20 in tumor cells and CD3

    CD30 and EBV LMP1 IHC


    Lymphocytic vasculitis with angioinvasion (skin)

    CD20 and CD30 in tumor cells; CD3 and CD4 in reactive T cells

    Lymphocytic vasculitis with angioinvasion (CNS)

    CD20 and EBV LMP1 in tumor cells; CD3 in reactive T cells

    Positive stains
    Molecular / cytogenetics description
    Sample pathology report
    • Lung tissue, bronchoscopic biopsy:
      • Lymphomatoid granulomatosis, grade I, biopsies from left lung, upper and lower lobes (see comment)
      • Comment: The small reactive lymphoid cells are CD3 positive. The scattered large cells are positive for CD20 and CD79a and show faint membrane positivity for EBV LMP1. The MIB1 proliferation index is low (approximately 10 - 20%). Special stains for microorganisms (AFB TB, PASD, GMS) are negative.
      • Microscopic description: Dense interstitial and diffuse infiltrates of small lymphoid cells with round nuclei, clumped chromatin and scattered large lymphoid cells with round nuclei, visible nucleoli and moderate amounts of cytoplasm. Admixed with these are a few aggregates of histiocytes and occasional eosinophils. The lymphoid cells display angioinvasion with transmural infiltration by lymphocytes which adhere to the endothelium with focal luminal occlusion. There are extensive areas of necrosis with necrotic outlines of blood vessels. The adjacent lung tissue shows extensive type II pneumocyte hyperplasia.
    Differential diagnosis
    Board review style question #1
    What cell is neoplastic in a case of lymphomatoid granulomatosis?

    1. B lymphocytes
    2. Histiocytes
    3. NK cells
    4. T lymphocytes
    Board review style answer #1
    A. B lymphocytes

    Comment Here

    Reference: Lymphomatoid granulomatosis
    Board review style question #2

    Which of these findings is essential for the diagnosis of lymphomatoid granulomatosis?

    1. Angioinvasion by tumor cells
    2. Granulomatous reaction
    3. Necrosis
    4. Reed-Sternberg-like cells
    Board review style answer #2
    A. Angioinvasion by tumor cells

    Comment Here

    Reference: Lymphomatoid granulomatosis

    Lymphomatoid papulosis

    Lymphoplasmacytic lymphoma
    Definition / general
    • Lymphoplasmacytic lymphoma (LPL) is a B cell neoplasm of small lymphocytes, plasmacytoid lymphocytes and plasma cells
    • Usually involves bone marrow, sometimes lymph nodes and spleen
    • Does not fulfill the criteria for any other small B cell lymphoid neoplasm
    • Waldenström macroglobulinemia (WM): lymphoplasmacytic lymphoma involving the bone marrow associated with IgM monoclonal paraprotein of any concentration in the blood
    Essential features
    • Lymphoplasmacytic lymphoma
      • Spectrum of lymphocytes, plasmacytoid lymphocytes and plasma cells > 10% bone marrow infiltration
      • Flow cytometry: monoclonal B cells with typical immunophenotype and monoclonal plasma cells
      • Monoclonal IgM paraprotein by serum and urine protein electrophoresis (SPEP / UPEP), minority with IgA or IgG paraprotein
    • MYD88 L265P mutation present or detection of CXCR4 somatic mutation (Virchows Arch 2016;468:259)
    • Waldenström macroglobulinemia
      • Serum IgM of any level
      • Bone marrow involvement by small B cell lymphoma with plasmacytic differentiation
      • Exclusion of other types of small B cell lymphoma
    Terminology
    • Lymphoplasmacytic lymphoma
    • Waldenström macroglobulinemia
    ICD coding
    • ICD-O:
      • 9671/3 - lymphoplasmacytic lymphoma
      • 9761/3 - Waldenström macroglobulinemia
    • ICD-11: 2A85.4 - lymphoplasmacytic lymphoma
    Epidemiology
    Sites
    • Bone marrow
    • Some cases involving blood, lymph node and extranodal sites manifest with splenomegaly, hepatomegaly or adenopathy, or have cutaneous, gastrointestinal, lung and central nervous system (Bing-Neel syndrome) involvement
    Pathophysiology
    • Genetic / molecular factors may contribute to pathogenesis of lymphoplasmacytic lymphoma
      • Evidence of familial clustering in ~20% of patients
    • MYD88 mutation in > 90% of cases (Blood 2014;123:1637)
      • Gene at 3p22.2
        • Most common mutation c794T>C results in switching of leucine to proline at codon 265 (pL265P)
        • Protein MYD88 plays significant role in Toll-like receptor and interleukin 1 (IL1) receptor signaling
        • Gain of function mutation product activates downstream transcription protein complex nuclear factor κ light chain enhancer of activated B cells (NFkB)
        • NFkB promotes cell proliferation and survival
        • MYD88 mutation is also present in significant proportion (50%) of IgM monoclonal gammopathy of unknown significance (IgM MGUS)
        • Present in ~30% of nongerminal center type diffuse large B cell lymphoma
        • Present in > 50% of primary cutaneous diffuse large B cell lymphoma, leg type
        • In many diffuse large B cell lymphoma at immune privileged sites (e.g., central nervous system, testicle)
      • 6q deletion in 40 - 60% of lymphoplasmacytic lymphoma cases
        • Most common recurrent cytogenetic abnormality
        • Tumor suppressor genes B lymphocyte induced maturation protein 1 (PRDM1) and TNFα induced protein 3 (TNFAIP3) are deleted
    • Other cytogenetic abnormalities identified (Blood 2014;123:1637)
      • Trisomy 18, 13q deletion, 17p (TP53) deletion, trisomy 4, trisomy 12 and 11q (ATM) deletion are seen in various proportions
    • CXCR4 mutation in ~30% of lymphoplasmacytic lymphoma cases and 20% of IgM MGUS (Blood 2016;128:827)
      • CXC chemokine receptor type 4 (CXCR4) plays critical role in lymphoplasmacytic lymphoma cells homing to bone marrow
        • Almost always coexists with MYD88 mutation
        • Mutation in CXCR4 gene is linked to disease progression and drug resistance
      • ARID1A mutations in 17% of cases
      • JAK-STAT signaling pathway may be involved
      • MicroRNA-9, miRNA-155 and miRNA-206 abnormalities may contribute to pathogenesis of lymphoplasmacytic lymphoma
    • Chronic immune stimulation
      • Lymphoplasmacytic lymphoma / Waldenström macroglobulinemia risk is elevated among individuals with autoimmune disorders, particularly Sjögren syndrome and autoimmune hemolytic anemia
      • Lymphoplasmacytic lymphoma / Waldenström macroglobulinemia risk is elevated in persons with hepatitis, HIV infection and rickettsiosis
    • Predisposition
      • MGUS of IgM type is associated with estimated annual progression rate of 1.5% to lymphoplasmacytic lymphoma
        • IgM MGUS is precursor lesion to lymphoplasmacytic lymphoma
        • MYD88 mutation is identified in 50% of IgM MGUS cases
    Etiology
    • Postgerminal center memory B cell is likely cell of origin
    • Hepatitis C is associated in some series (Clin Lymphoma Myeloma Leuk 2020;20:e195)
      • Hepatitis C associated lymphoplasmacytic proliferations are nonprogressive and may be similar to monoclonal B cell lymphocytosis
      • Treating hepatitis C with antivirals may lead to regression to the lymphoplasmacytic proliferation
    • Autoimmune disorders: associated with an increased risk of occurrence
    Clinical features
    • 20 - 30% of patients are asymptomatic at diagnosis
    • Anemia related symptoms: fatigue, shortness of breath and chest pain
    • Thrombocytopenia or acquired von Willebrand disease related tendency
    • Constitutional symptoms, including weight loss and night sweats
    • Splenomegaly and adenopathy
      • High concentration of IgM may form aggregates and bind water
      • IgM may cause red blood cell aggregation (rouleaux formation) and increase red cell internal viscosity
      • Results in circulatory disturbances
      • Most common symptoms include oronasal bleeding, headache, visual disturbances due to retinal bleeding and dizziness
    • Hyperviscosity
      • 15 - 30% of patients with Waldenström macroglobulinemia have hyperviscosity (in particular when IgM level reaches ≥ 6 g/dL) (Br J Haematol 2017;177:717)
      • Worse clinical scenarios could occur when IgM ≥ 5 g/dL
    • Hyperviscosity syndrome
      • Due to IgM's large size and quantity
      • Symptoms
        • Visual impairment (distended and tortuous retinal veins with hemorrhage and exudates)
        • Neurologic: slowing blood flow
        • Bleeding: IgM binding of clotting factors and cryoglobulinemia causing Raynaud phenomena and cold urticaria
    • The following findings can be associated with bone marrow infiltration, cryoprecipitation, autoimmune or nonautoimmune antibody activity
      • Cytopenia
      • Cold agglutinin hemolytic anemia (10% of cases) (IgM antibodies that bind at < 37 °C)
      • Peripheral neuropathy
    • Primary amyloidosis
    • No lytic bone lesion distinguishes it from plasma cell myeloma
    • May transform to diffuse large B cell lymphoma with Reed-Sternberg-like cells or immunoblasts
    • Reference: Blood Rev 2015;29:301
    Diagnosis
    • Diagnostic criteria for Waldenström recommended by Second International Workshop (Semin Oncol 2003;30:110)
      • IgM monoclonal gammopathy of any concentration
        • Bone marrow infiltration by small lymphocytes, plasmacytoid lymphocytes or plasma cells
        • Combined paratrabecular and interstitial pattern of bone marrow infiltration
        • Frequently associated with reactive mast cells and hemosiderin laden macrophages
        • Lymph node involvement by LPL usually in a diffuse pattern, loaded with monotonous small lymphoid, plasmacytoid lymphocytes and plasma cells
    • Essential and desirable diagnostic criteria
      • Essential
        • Bone marrow infiltration by > 10% small lymphocytes with plasmacytoid or plasma cell differentiation
        • Immunophenotype of LPL cells: IgM+, CD19+, CD20+, CD22+, CD25+, CD10-, CD23-, CD103-, variable CD138
      • Desirable
        • Detection of MYD88 (NP_002459.2:p.L265P) (93 - 97% of LPL / WM)
        • Detection of CXCR4 somatic mutation
        • Serum electrophoresis and immunofixation showing presence of monoclonal Ig
    • International Consensus Committee 2022 (Blood 2023;141:437)
      • Diagnosis of lymphoplasmacytic lymphoma can be rendered in cases of abnormal lymphoplasmacytic aggregates in the bone marrow and evidence of clonal plasma cells and clonal B cells, even when the aggregates are < 10% of the trephine biopsy (Blood 2023;141:437)
      • Immunophenotype
        • Positive: IgM, CD19, CD20, CD22, CD79a, CD25 and CD38 frequent expression
        • Negative: CD5, CD10, CD103, CD23; however, CD23 expression is not uncommon in some cases
        • CD5 and CD10 positive but BCL6 negative in a minority of cases
        • Plasma cells in lymphoplasmacytic lymphoma
          • Positive: CD138, CD19, CD45 and sometimes MUM1 but can be negative
          • Can be PAX5 positive
    Laboratory
    • IgM monoclonal paraprotein is commonly present in serum, rarely IgG or IgA
      • Lymphoplasmacytic lymphoma
        • IgM paraprotein not required for diagnosis
        • IgA or IgG paraprotein alone or coexists with IgM in serum (rare)
      • Waldenström macroglobulinemia
        • IgM paraprotein required for diagnosis
        • No cutoff level for IgM in serum
    • Other abnormal tests with Waldenström macroglobulinemia
      • Elevated erythrocyte sedimentation rate (ESR), cytopenia / anemia
      • Elevated serum levels of lactate dehydrogenase (LDH) and beta 2 microglobulin
    • Reference: Blood 2015;126:721
    Prognostic factors
    • Indolent clinical course but incurable
    • Adverse prognostic factors (International Prognostic Scoring System) (Blood 2009;113:4163)
      • > 65 years old
      • Hemoglobin < 11.5 g/dL
      • Platelet count < 100 x 109/L
      • Monoclonal IgM > 7 g/dL
      • Increased beta 2 microglobulin > 3 mg/L
    • Overall survival varies from 5 - 11 years depending on the study; median survival is 5 years, 40% survive 10 years or more or die of unrelated causes (Blood 2009;113:4163, Am J Hematol 2013;88:60)
      • Low risk (≤ 65 years, 0 - 1 risk factor) 5 year survival rate of 87%
      • High risk (> 65 years, > 2 risk factors) 5 year survival rate of 36%
    • Transformation to large cell lymphoma can occur (13%) (Blood Cancer J 2015;5:e394, Am J Clin Pathol 2003;120:246)
    • Relapse: inevitable, can occur years after initial treatment
    • Main disease related causes of death
      • Disease progression
      • Transformation to high grade lymphoma
      • Complications of therapy
    Case reports
    • 43 year old woman with headaches; MRI revealed an abnormal homogeneously enhancing mass in the cerebellopontine angle (Medicine (Baltimore) 2016;95:e4627)
    • 53 year old woman diagnosed with relapsed lymphoplasmacytic lymphoma complicated by Bing-Neel syndrome who was successfully managed with tirabrutinib (Int J Hematol 2022;115:585)
    • 60 year old man with fatigue, sensory neuropathy and lab findings of anemia and elevated albumin globulin gap (Case Rep Hematol 2019;2019:4075960)
    • 65 year old man who presented with fatigue and decreased renal function was diagnosed with acute kidney injury due to direct infiltration by lymphoplasmacytic lymphoma secreting IgG paraproteins (Medicine (Baltimore) 2022;101:e29449)
    • 69 year old man presented with altered mental status and was diagnosed with intravascular large B cell lymphoma secondary to lymphoplasmacytic lymphoma (Int J Clin Exp Pathol 2015;8:3339)
    Treatment
    • No standardized treatment
    • Asymptomatic patients: watch and wait
    • Therapy initiated in patients with
      • Constitutional symptoms: fever, weight loss, night sweats, fatigue
      • Hyperviscosity
      • Severe neuropathy
      • Amyloidosis
      • Symptomatic cryoglobinemia
      • Cold agglutinin disease
      • Evidence of transformation
    • Treatment regimen / options (Leukemia 2014;28:1799, J Clin Oncol 2009;27:120)
      • Single agent
      • Combination therapy
        • Monoclonal antibodies
        • Proteasome inhibitors
        • Immunomodulatory agents
        • Signaling pathway inhibitors
        • Histone deacetylase
      • First line treatment
        • Anti-CD20 antibody in combination with a purine analogs or alkylating agent
        • Rituximab in combination with thalidomide or bortezomib and dexamethasone
        • Rituximab with bendamustine
      • Plasmapheresis: reduce circulating IgM in hyperviscosity patients
      • Splenectomy: chemoresistant patient
    • Stem cell transplant (Blood 2015;126:721)
      • Autologous stem cell transplantation is feasible, safe and associated with significant cytoreduction in relapsed or refractory patient
      • Allogeneic stem cell transplantation is used only in patients with advanced and refractory disease for whom no other options are available
    • Investigational therapies (Blood 2015;126:721)
      • Agents target MYD88, CXCR4, BCL2, CD27 / CD70 signaling
      • Novel proteasome inhibitors
      • Chimeric antigen receptor modified T cell therapy
    • Please follow the consensus treatment recommendation from the tenth International Workshop for Waldenström macroglobulinemia (Lancet Haematol 2020;7:e827)
    • Preferred treatment options
      • Bendamustine plus rituximab
      • Bortezomib, dexamethasone and rituximab
      • Cyclophosphamide, dexamethasone and rituximab
      • Ibrutinib (with or without rituximab)
    • Treatment recommendations
      • Avoid bortezomib and vincristine in patients with neuropathy
      • Avoid carfilzomib in patients with cardiac disease or patients who are older than 65 years
      • Avoid nucleoside analogues in patients who are candidates for stem cell transplantation
      • Consider delaying rituximab if serum IgM concentrations are > 4000 mg/dL
      • Consider ofatumumab in patients who are intolerant to rituximab
    • Other treatment options
      • Acalabrutinib
      • Carfilzomib, dexamethasone and rituximab
      • Fludarabine and rituximab
      • Ixazomib, dexamethasone and rituximab
      • R-CHOP
      • R-CVP
      • Rituximab
      • Ofatumumab
    Microscopic (histologic) description
    • Blood
      • Often shows lymphocytosis with a spectrum of lymphocytes, plasmacytoid lymphocytes and plasma cells
      • Rouleaux formation of red blood cells is routinely seen
      • Cold agglutinin or cryoglobulin may be present
    • Bone marrow (Arch Pathol Lab Med 2013;137:580)
      • Involvement in almost all cases
      • Aspirate is the most helpful sample in morphologic diagnosis
        • Predominantly small lymphocytes with a variable number of plasmacytoid lymphocytes and plasma cells
        • Mast cells are typically increased; most prominent within particles on aspirate smears
      • Core biopsy (Am J Surg Pathol 2005;29:1549, Am J Clin Pathol 2015;143:797)
        • Most common pattern is the combination of paratrabecular and nonparatrabecular lymphoid nodules
        • Less common patterns include paratrabecular or intrasinusoidal with or without interstitial infiltrate
        • Pseudointranuclear (Dutcher bodies) and intracytoplasmic inclusions (Russell bodies) are most prominent on core biopsy section
        • Amyloid deposition may be present within vessel walls or interstitial deposits
    • Lymph node (Arch Pathol Lab Med 2013;137:580)
      • 2 main patterns of nodal involvement
        • Classic pattern
          • Subtotal architecture effacement
          • Retention of small primary or enlarged reactive follicles
          • Patent dilated sinuses
          • Interfollicular areas contain monomorphous infiltrate of small lymphocytes, plasmacytoid lymphocytes and plasma cells
          • Dutcher bodies, increased number of mast cells and hemosiderin deposition may be present
        • Polymorphous pattern
          • Complete architectural effacement
          • Nodular to diffuse polymorphous infiltrate
            • Small lymphocytes
            • Plasmacytoid lymphocytes
            • Plasma cells
          • Greater numbers of large transformed cells
          • Clusters of epithelioid histiocytes may be conspicuous
            • Termed polymorphous immunocytoma
        • Extracellular immunoglobulin in the form of amyloid may be present
    • Spleen (Am J Surg Pathol 2003;27:1104)
      • Not well described
      • Older published series and review
        • Presence of nodular and diffuse infiltrates of lymphoplasmacytic cells involving red pulp
        • Spectrum of lymphoma cells analogous to bone marrow
        • Sparing of white pulp with absence of marginal zone growth pattern
    Microscopic (histologic) images

    Contributed by Ling Zhang, M.D.

    Increased lymphoid cells

    Round eccentrically located nuclei

    Condensed chromatin, inconspicuous nucleoli

    Small mature lymphoid cells

    Intermingled plasma cells


    Lymphoid cells negative for CD3

    Diffuse CD20 positivity

    Focal plasma cell positivity (CD138)

    Immunoglobulin M positivity

    Positive stains
    Negative stains
    Flow cytometry description
    Flow cytometry images

    Contributed by Ling Zhang, M.D. and Caroline An, M.D.
    Kappa clonal population of B cells

    Kappa clonal population of B cells

    CD38+ / CD138+ plasma cells

    CD38+ / CD138+ plasma cells

    Lambda restriction

    Lambda restriction

    Plasma cells express CD45

    Plasma cells express CD45

    Presence of CD19

    Presence of CD19


    Presence of CD20 expression

    Presence of CD20 expression

    B cells expressing both CD19 and CD20

    B cells expressing both CD19 and CD20

    B cells are lambda monoclonal

    B cells are lambda monoclonal

    B cells show variable expression for CD38

    B cells show variable expression for CD38

    Molecular / cytogenetics description
    • MYD88 mutation in > 90% of cases; increasingly used in diagnostic workup
    • CXCR4 mutation; not routinely tested
    • Cytogenetic study is not required for diagnosis; when done, 6q deletion is reported in 40 - 50% of patients
    • IGH gene rearranged
    • Biased VH3 and VH3-23 usage
    • Reference: Blood 2014;123:1637
    Molecular / cytogenetics images

    Contributed by Ling Zhang, M.D.

    ISH clonal kappa light chain expression

    Negative lambda light chain expression

    Sample pathology report
    • Bone marrow, right posterior iliac crest, aspirate and biopsy:
      • Peripheral blood: The red blood cells are normocytic and normochromic with mildly decreased hemoglobin level (11.8 g/dL). The reticulocyte count is adequate. There is no overt rouleaux formation. The white blood cell count is within normal range. Lymphocytes are not increased; however, few atypical lymphocytes are seen, which are mostly small in size, with slightly irregular nuclear contour and mature chromatin or plasmacytoid morphology. Platelets are mildly decreased in number with normal morphology. 
      • Hypercellular marrow with extensive involvement by lymphoplasmacytic lymphoma (see comment)  
      • Aspirate smear: The Wright stained bone marrow aspirate smears are cellular, with adequate spicules included for evaluation. Megakaryocytes are decreased in number, when identified, showing normal morphology. Myeloid and erythroid precursors show progressive maturation but are relatively decreased in number. The estimated M:E ratio is 4:1. The cellularity is composed of mostly atypical lymphocytes (84% of differential count). These atypical lymphocytes are predominantly small in size, with oval or slightly irregular nuclear contours, mature chromatin and small amount of basophilic cytoplasm. Rare atypical lymphocytes are medium in size, with plasmacytoid differentiation. Plasma cells account for 4.8% of the differential count. Scattered mature mast cells are present in the background.  
      • Core biopsy and cell clot: The bone marrow core biopsy includes normal appearing trabecular bone and marrow, adequate for morphologic evaluation. The marrow cellularity is very high, up to 95%. Megakaryocytes are essentially normal in number with normal morphology. Myeloid and erythroid precursors are markedly decreased in number. The marrow is diffusely infiltrated by atypical lymphocytes, accounting for approximately 80% of total cellularity, a small subset of which shows plasmacytoid differentiation. Mature plasma cells are identified. Iron stain shows adequate stainable storage iron. Reticulin stain highlights mild reticulin fibrosis. The clot section contains a few small cellular particles including predominantly atypical lymphoid cells, with findings similar to the core biopsy.    
      • Immunohistochemical study: Immunohistochemical stains and in situ hybridization stains are performed on the core biopsy with adequate controls. CD20 and PAX5 highlight a diffuse infiltration of neoplastic B cells, occupying 80% of total cellularity. CD3 highlights scattered interstitial T lymphocytes, ranging from 2% to 5%. CD138 highlights scattered plasma cells, focally forming small clusters, accounting for approximately 5% of the total cells. By in situ hybridization (ISH), the plasma cells are positive for kappa light chain and negative for lambda light chain.
    Differential diagnosis
    • IgM monoclonal gammopathy of undetermined significance (IgM MGUS):
      • All 3 criteria must be met
        • Serum IgM monoclonal protein < 3 gm/dL
        • Bone marrow lymphoplasmacytic infiltration < 10%
        • No evidence of anaemia, constitutional symptoms, hyperviscosity, lymphadenopathy or hepatosplenomegaly that can be attributed to the underlying lymphoproliferative disorder
    • IgG and IgA MGUS:
    • Chronic lymphocytic leukemia (CLL) / small lymphocytic lymphoma:
      • B cell lymphoma composed of small mature forms
      • Often coexpresses dim CD20, CD5 and CD23 by flow cytometry
      • Frequent nodal presentation with generalized lymphadenopathy, vaguely nodular proliferation with identifiable pseudoproliferation centers; the latter is lacked in LPL / WM
      • Frequent peripheral involvement, mature lymphocytosis with clumped chromatin (soccer ball chromatin pattern), > 5,000/uL neoplastic B cells, CLL phenotype is required to diagnose CLL
      • Usually negative for MYD88 mutation
    • Marginal zone lymphoma (MZL):
      • B cell lymphoma with monocytoid or plasmacytoid differentiation
      • Immunophenotypically similar to LPL / WM, triple negative for CD5, CD10 and CD23
      • Bone marrow involvement: mostly interstitial and intrasinusoidal infiltration pattern
      • Lymph node, spleen and extranodal involvement might not be easy to distinguish it from LPL / WM
      • Presence of MYD88 L265P mutation favors LPL / WM over MZL
      • Plasmacytoid differentiation with Dutcher bodies is less frequently in MZL than in LPL
    Board review style question #1

    Which of the following immunoglobulins is most commonly seen in lymphoplasmacytic lymphoma?

    1. IgA
    2. IgD
    3. IgE
    4. IgG
    5. IgM
    Board review style answer #1
    E. IgM. More than 90% of patients with LPL / WM show an overt elevated IgM level. Answer A is incorrect because non-IgM LPL / WM, mainly IgM or IgA, is reported in ~5% of cases. Answer D is incorrect because concurrent IgM and IgG LPL is rarely reported. Answers B and C are incorrect because IgD and IgE LPL / WM might occasionally exist but have not been well reported.

    Comment Here

    Reference: Lymphoplasmacytic lymphoma
    Board review style question #2
    What somatic mutation is associated with lymphoplasmacytic lymphoma and can aid in diagnosis?

    1. BCL2
    2. BRAF
    3. KRAS
    4. MYD88 L265P
    Board review style answer #2
    D. MYD88 L265P. MYD88 is considered the driver mutation and detected in 93 - 97% of patients with LPL / WM. The particular mutation leads to constitutive activation of NFkB pathway. While answer D is correct, be cautious, as the presence of MYD88 is not 100% specific for LPL / WM. The mutation has also been found in monoclonal gammopathy of unknown significance (MGUS), diffuse large B cell lymphoma, activated B cell (DLBCL ABC) type, etc. Integrating with other clinical and pathologic results is necessary for an accurate diagnosis. Answers A - C are incorrect because other mutations are infrequently reported in LPL / WM and are not helpful for diagnosis.

    Comment Here

    Reference: Lymphoplasmacytic lymphoma

    MALT-marginal zone
    Definition / general
    Essential features
    Terminology
    • Low grade B cell lymphoma of MALT
    • MALToma
    ICD coding
    • ICD-O: 9699/3 - marginal zone B cell lymphoma, NOS
    • ICD-10: C88.4 - extranodal marginal zone B cell lymphoma of mucosa associated lymphoid tissue (MALT lymphoma)
    Epidemiology
    Sites
    Pathophysiology
    • Somatic hypermutation and rearrangements in immunoglobulin heavy chain variable (IGHV) genes associated to cell surface and chemokine receptors of chronic inflammatory / infectious processes play an important role in the lymphomagenesis (Leuk Lymphoma 2012;53:1032, Blood 2016;127:2082, F1000Res 2018;7:406, Best Pract Res Clin Haematol 2018;31:57, World J Gastrointest Oncol 2022;14:153, Best Pract Res Clin Haematol 2017;30:13, Appl Immunohistochem Mol Morphol 2021;29:56)
      • Most common mutated IGH VH families by site:
        • Stomach: IGH VH3-30 and IGH VH3-23
        • Ocular adnexa: IGH VH4-34
        • Skin: IGH VH1-69 and IGH VH4-59
        • Lung: IGH VH3 and IGH VH4
        • Salivary gland: IGH VH1-69
      • In all types of MZL, both canonical and noncanonical NFκB activation can mediate the activation, neoplastic transformation and maintenance of the neoplastic lymphocytes
        • Antigen stimulation of BCR and TCR, followed by phosphorylation of CARD11, BCL10 and MALT1
          • Interaction with TRAF6 and NFκB essential modulator
          • TNFAIP3 (A20) performs a negative regulation of the whole pathway and due to mutations and deletions in this gene, the pathway may hyperactivate
      • Cases of gastric MALT lymphoma related to Helicobacter pylori infection may have upregulation of cyclin A2 and loss of PTEN
        • Same mechanism is described in diffuse large B cell lymphoma (DLBCL)
    • Recurrent CCR6 mutations impair its internalization and consequently the neoplastic cells are more resistant to apoptosis and more likely to have malignant transformation (Haematologica 2022 Feb 10 [Epub ahead of print])
      • HBD2, CCL20 and HD5 are ligands that could be expressed in epithelial cells
    Etiology
    Clinical features
    Diagnosis
    • Gastric MALT lymphoma (Best Pract Res Clin Gastroenterol 2014;28:1069)
      • Endoscopic mapping of the stomach is recommended due to the nonspecific presentation
        • Minimum of 10 biopsies of visible lesions and no specified number of normal mucosae
        • No standard protocol for follow up
          • Usually, biopsies are performed every 3 - 6 months for the first 2 years of follow up
    Laboratory
    • Laboratory abnormalities are not common (Best Pract Res Clin Haematol 2018;31:57, Br J Haematol 2022;196:1353)
      • Elevated serum β2 microglobulin (~30%)
      • Monoclonal gammopathy (~30%)
        • Association with advanced stage and bone marrow involvement
      • Elevated serum immunoglobulin paraprotein (~20 - 30%)
        • Most common types are IgM κ, IgM L and IgG L
        • Associated with advanced age, bone marrow involvement, B symptoms, advanced stage of disease, elevated serum β2 microglobulin, multiple sites of involvement, nodal involvement, high MALT International Prognostic Index (IPI)
      • Iron deficiency anemia (~10%)
      • Elevated serum lactate dehydrogenase (LDH) (~10%)
    Radiology description
    • For the detection of lesions, the value of 18F fluorodeoxyglucose positron emission tomography is controversial; it seems to show more concordant results for lesions located in tissues with low and homogeneous physiologic uptake (Blood 2016;127:2082, Cancers (Basel) 2022;14:750)
    Prognostic factors
    • Factors associated with worse outcomes: MALT IPI (Leuk Lymphoma 2013;54:1891, Best Pract Res Clin Haematol 2018;31:57, Blood 2017;130:1409, Front Oncol 2021;11:681689)
      • Low (0), intermediate (1) and high (≥ 2) risk according to the presence of the following factors:
        • Advanced age (≥ 70 years)
        • Elevated LDH
        • Ann Arbor Stage III / IV
      • Other factors that may be associated with worse outcome: relapse at the original or distant sites, elevated serum immunoglobulin paraproteinemia and bone marrow involvement
    • Localized forms of MALT lymphoma are usually early stage (IE / IIE) of disease and have similar outcomes compared with localized nodal MZL (Leuk Lymphoma 2013;54:1891)
    • Transformation to DLBCL may occur in up to ~20% of cases and is associated with a worse prognosis (Leuk Lymphoma 2012;53:1032, Blood 2016;127:2082, Br J Haematol 2021;193:369)
      • In cases of gastric MALT lymphoma, it is associated with hepatic involvement, advanced stage disease and hepatitis C infection
      • Genetic factors associated with transformation: mutation and allelic loss of p53, hypermethylation of p15 and p16, and p16 deletion
      • Cases of typical MALT lymphoma with different methylation profiles, MALT lymphoma with increased large cells and transformation to DLBCL
    • Gastrointestinal MALT lymphoma may have inferior outcome when compared with nongastrointestinal localization (Leuk Lymphoma 2013;54:1891, Best Pract Res Clin Gastroenterol 2014;28:1069)
    • Advanced stage (III / E / IVE) may occur in ~30% of patients and recurrence may happen in extranodal and nodal sites (Leuk Lymphoma 2012;53:1032)
    • 5 year overall survival (OS) by affected site (Leuk Lymphoma 2013;54:1891):
      • Central nervous system: 100%
      • Thyroid gland: 95%
      • Skin: 91%
      • Oral cavity: 88%
      • Bone and soft tissue: 86%
      • Ocular adnexa: 84%
      • Breast: 79%
      • Lung and respiratory tract: 77%
      • Colon: 72%
      • Stomach: 71%
      • Urinary tract: 65%
    • Median 10 year recurrence free survival: 57% (Blood 2016;127:2082)
    Case reports
    • 12 year old girl with gastric MALT lymphoma presenting with iron deficiency anemia and gastric perforation (BMC Pediatr 2019;19:63)
    • 37 year old woman with primary cutaneous marginal zone lymphoma (PCMZL) presented as a rosacea-like lesion (JAAD Case Rep 2020;6:902)
    • 56 year old woman with primary colonic MALT lymphoma presenting as a single mass on hepatic flexure of the colon (Am J Case Rep 2017;18:491)
    • 73 year old woman with monoclonal gammopathy and primary oral cavity MALT lymphoma (BMC Oral Health 2021;21:597)
    • 74 year old woman with a previous history of tuberculosis presenting with a pulmonary opacity diagnosed as a primary pulmonary MALT lymphoma (Biomed Hub 2019;4:1)
    Treatment
    Microscopic (histologic) description
    • Atypical lymphoid infiltrate is located in the marginal zone of reactive follicles and extends to interfollicular or intrafollicular (follicular colonization) regions (Leuk Lymphoma 2012;53:1032, J Clin Pathol 2007;60:361)
    • Reactive follicles may be present and are helpful in the diagnosis (J Clin Pathol 2007;60:361)
    • Lymphoepithelial lesion (Leuk Lymphoma 2012;53:1032, Best Pract Res Clin Gastroenterol 2014;28:1069, J Clin Pathol 2007;60:361)
      • Aggregates of 3 or more marginal zone cells with distortion or destruction of the epithelium
        • Presence of individual cells in the epithelium is not sufficient to define a lymphoepithelial lesion
      • Eosinophilic degeneration is observed
      • Presence is not pathognomonic for MALT lymphoma and can be found in reactive conditions
    • Diagnostic clues for the diagnosis by site:
      • Orbit (PLoS One 2014;9:e104004):
        • Nodular or diffuse infiltrate composed by monocytoid, centrocyte-like and lymphoplasmacytic cells
      • Stomach and intestine (Best Pract Res Clin Gastroenterol 2014;28:1069, Am J Clin Pathol 2022;157:23):
        • Dense lymphoid infiltrate that replaces the lamina propria, associated or not with lymphoepithelial lesions
          • Lymphoepithelial lesions are not common in colorectal cases
      • Lung (J Clin Pathol 2007;60:361, Eur Respir J 2009;34:1408):
        • Lymphangitic pattern of infiltration with spread through bronchovascular structures, interlobular septa and visceral pleura
        • Lymphoepithelial lesion is seen on the bronchial or bronchiolar epithelium
        • Angiotropism, fibrosis and necrosis may be found
      • Salivary gland (Leuk Lymphoma 2012;53:1032):
        • Background of chronic sialadenitis
        • Atrophic acinar parenchyma
        • Monocytoid cells around epithelial / myoepithelial islands
      • Skin (Am J Clin Pathol 2020;154:428):
        • Dense lymphoid infiltrate with nodular or diffuse pattern
          • Admixture of small lymphocytes, lymphoplasmacytic and plasma cells
        • Grenz zone
      • Breast (Clin Lymphoma Myeloma Leuk 2019;19:244):
        • Diffuse or nodular neoplastic infiltrate of monocytoid cells admixed with reactive follicles
        • Lymphoepithelial lesions may be present
      • Thyroid (Eur Thyroid J 2020;9:11):
        • Lymphoepithelial lesions and epithelial ball
          • Neoplastic cells inside a thyroid follicular epithelium
        • Scattered hyperplastic germinal centers in the background
    • Transformation to DLBCL:
      • Large cells with centroblastic appearance disposed in clusters of more than 20 cells, or sheet proliferation
    Microscopic (histologic) images

    Contributed by Roberto N. Miranda, M.D.
    Salivary gland lymphoepithelial lesion

    Salivary gland lymphoepithelial lesion

    Cytokeratin and lymphoepithelial lesion

    Cytokeratin and lymphoepithelial lesion

    CD20 positivity in salivary gland lymphoepithelial lesion

    CD20 positivity in salivary gland lymphoepithelial lesion

    Gastric gland lymphoepithelial lesion

    Gastric gland lymphoepithelial lesion

    Thyroid gland MALT lymphoma

    Thyroid gland MALT lymphoma

    Primary cutaneous marginal zone lymphoma (PCMZL)

    PCMZL


    PCMZL subepidermal lesion

    PCMZL subepidermal lesion

    PCMZL - CD20 positive

    PCMZL - CD20 positive

    PCMZL - CD3 negative

    PCMZL - CD3 negative

    Lung BALT lymphoma Lung BALT lymphoma Lung BALT lymphoma

    Lung BALT lymphoma


    Lymphoepithelial lesion of the stomach

    Lymphoepithelial lesion of the stomach

    Stomach MALT lymphoma

    Stomach MALT lymphoma

    Lambda light chain restriction

    Lambda light chain restriction

    Kappa negativity

    Kappa negativity

    Breast MALT lymphoma

    Breast MALT lymphoma

    CD10 negativity in MALT lymphoma

    CD10 negativity in MALT lymphoma


    BCL6 negativity in MALT lymphoma

    BCL6 negativity in MALT lymphoma

    Cytology description
    • Monomorphic pattern (Leuk Lymphoma 2012;53:1032, J Clin Pathol 2007;60:361):
      • Monocytoid or centrocyte-like appearance: intermediate size, distinct cell borders, nuclei with regular contours and rim of clear cytoplasm
    • Polymorphic pattern (Leuk Lymphoma 2012;53:1032, J Clin Pathol 2007;60:361):
      • Lymphocytes with monocytoid, centrocyte-like (small to intermediate size, small irregular) nuclei are the most predominant component
      • Scattered cells with centroblastic-like appearance: large cells with dispersed chromatin
      • Plasma cells are frequent and close to the epithelium
    Positive stains
    Flow cytometry description
    Molecular / cytogenetics description
    • Hypermutation of immunoglobulin heavy chain variable genes in up to 75% of patients (Leuk Lymphoma 2012;53:1032, J Clin Pathol 2007;60:361)
    • B cell clonality may remain in cases with histological remission in up to 50% of patients (J Clin Pathol 2007;60:361)
      • May be associated with delay in achieve complete remission and probability to relapse
    • Recurrent chromosomal aberrations (Leuk Lymphoma 2012;53:1032, Blood 2016;127:2082, Am J Surg Pathol 2006;30:1546, Leuk Res 2020;95:106399):
      • Stomach: t(11;18)(q21;q21), t(1;14)(p22;q32) and trisomies 3, 7, 12 and 18
      • Ocular adnexa: t(11;18)(q21;q21), t(14;18)(q32;q21), t(3;14)(p14.1;q32), deletion 6p23 (TNFAIP3) and trisomies 3 and 18
      • Intestine: t(11;18)(q21;q21), t(1;14)(p22;q32) and trisomies 3, 12 and 18
      • Skin: t(14;18)(q32;q21), t(3;14)(p14.1;q32) and trisomies 3 and 18
      • Lung: t(11;18)(q21;q21), t(14;18)(q32;q21), t(1;14)(p22;q32) and trisomies 3, 12 and 18
      • Salivary gland: t(11;18)(q21;q21), t(14;18)(q32;q21) and trisomies 3, 7 and 18
      • Thyroid gland: t(3;14)(p14.1;q32) and trisomies 3 and 12
      • Breast: trisomies 3 and 18
    • Aneuploidy is associated with t(14;18)(q32;q21) / IGH-MALT1 and t(1;14)(p22;q32) / IGH-BCL10 (Am J Surg Pathol 2006;30:1546)
      • Both MALT1 and BCL10 are associated with NFκB activation
    • Somatic mutations in PIM1 and cMYC (World J Gastrointest Oncol 2022;14:153)
      • Usually missense, 46% of gastric MALT lymphomas and 30% of nongastric
    • Somatic mutations or deletions of PIK3CD, CREBBP, KMTDC, SPEN, TET2 and TNFRSF14 (F1000Res 2018;7:406, Haematologica 2019;104:e558)
    • Somatic mutations or deletions of TNFAIP3 (6p23) (F1000Res 2018;7:406, Haematologica 2019;104:e558)
      • Directly involved in NFκB activation
    • Tumor suppressor genes can be silenced in cases of MALT progression (F1000Res 2018;7:406)
      • CDKN2A, DAPK1 and CDH1
    Sample pathology report
    • Esophagus, endoscopic biopsy:
      • Squamous epithelium with no significant pathologic changes
    • Stomach (greater curvature), endoscopic biopsy of a mass:
      • Extranodal marginal zone lymphoma of mucosa associated lymphoid tissue (MALT) involving gastric mucosa (see comment)
    • Stomach (lesser curvature), endoscopic biopsy:
      • Antral mucosae with moderate chronic inactive gastritis
      • No intestinal metaplasia identified
      • Helicobacter pylori present
    • Duodenum, endoscopic biopsy:
      • Duodenal mucosa with no pathologic changes
    • Comment: According to clinical notes, patient is a 60 year old woman who presented with abdominal pain and anemia for the past 10 months. She underwent endoscopic examination that showed large ulcerated noncircumferential lesion on the greater curvature of the stomach, which was biopsied.
    • Histologic sections from the mass in the greater curvature show gastric mucosa with dense infiltration of the lamina propria composed predominantly of small to intermediate monocytoid lymphocytes, small lymphocytes (centrocyte-like), plasma cells and eosinophils. The monocytoid lymphocytes show clear cytoplasm, irregular nuclear contour and hyperchromatic chromatin. Lymphoepithelial lesion is extensively present.
    • Immunohistochemical studies were performed on the greater curvature lesion and the lymphoma cells were positive for CD20, PAX5, CD43, BCL2 and negative for CD3, CD5, CD10, BCL6 and SOX11. The lymphoma cells showed kappa light chain restriction. CD21 highlighted the follicular dendritic meshwork and Ki67 was ~20% on the neoplastic cells.
    • No morphologic findings to support infiltration by lymphoma in esophageal, gastric lesser curvature and duodenum biopsies.
    Differential diagnosis
    Board review style question #1
    Which of the following is true about extranodal marginal zone lymphoma of mucosa associated lymphoid tissue (MALT)?

    1. Association with infectious agents and autoimmune disorders is rarely described
    2. Bone marrow involvement is common and detected in up to 90% of patients
    3. NFκB pathway activation is a shared feature of several subtypes of MALT lymphoma
    4. Trisomies 3 and 18 are extremely rare in MALT lymphoma
    Board review style answer #1
    C. NFκB pathway activation is a shared feature of several subtypes of MALT lymphoma. There is an important overlap in the molecular signatures of different subtypes of MALT lymphoma, which involve activation of canonical and noncanonical NFκB pathway.

    Comment Here

    Reference: MALT - marginal zone
    Board review style question #2

    What is the characteristic immunophenotype of extranodal marginal zone lymphoma of mucosa associated lymphoid tissue (MALT)?

    1. CD20+ / BCL2- / CD10- / BCL6+ / CD43- / EBER-
    2. CD20+ / BCL2+ / CD10- / BCL6- / CD43+ / EBER-
    3. CD20+ / BCL2+ / CD10- / BCL6- / CD43- / EBER+
    4. CD20- / BCL2+ / CD10+ / BCL6- / CD43- / EBER+
    Board review style answer #2
    B. CD20+ / BCL2+ / CD10- / BCL6- / CD43+ / EBER-. MALT lymphoma cells have a B cell phenotype (CD19+, CD20+, PAX5+) and are usually positive for BCL2 and CD43 and negative for CD10 and BCL6. EBV is not associated with MALT lymphoma.

    Comment Here

    Reference: MALT - marginal zone

    MCL-aggressive variants
    Definition / general
    • 2 types: blastoid and pleomorphic
    • Blastoid: cells resemble lymphoblasts with high mitotic rate (> 20 - 30 mitoses /10 high power fields) (Blood 1997;89:1421)
    • Pleomorphic: large, pleomorphic cells with variable sizes, prominent nucleoli
    • Definition is based on the morphology
    Essential features
    • Associated with t(11;14)(q13;q32) or IGH/CCND1 and cyclin D1 overexpression
    • High proliferation index and unfavorable outcome
    Terminology
    • Blastoid variant
      • Mantle cell lymphoma (MCL), lymphoblastoid variant
      • Mantle cell lymphoma (MCL), blastic
    • Pleomorphic variant
      • Mantle cell lymphoma (MCL), large cell variant
      • Mantle cell lymphoma (MCL), anaplastic
      • Mantle cell lymphoma (MCL), nucleolated variant
    ICD coding
    • ICD-O: 9673/3 – mantle cell lymphoma
    Epidemiology
    Sites
    • Lymph node, spleen, bone marrow
    • Extranodal sites:
      • Gastrointestinal tract is the most common site
      • CNS: More frequently involved by blastoid / pleomorphic variant than classic variant
    • Peripheral blood:
      • Leukemic involvement at diagnosis
    Pathophysiology and etiology
    • CCND1 at 11q13 with IGH at 14q32
      • New CCND1 will reposition near the nucleoli (nucleolin and RNA polymerase II) (Blood 2014;123:2044)
      • Higher level of cyclin D1 RNA, when 3'-untranslated region is deleted (common in blastoid mantle cell lymphoma) (Blood 2007;109:4599)
    • Higher level of chromosomal aberration in blastoid / pleomorphic mantle cell lymphoma than in classic mantle cell lymphoma (Hum Pathol 2003;34:1022)
    • Frequent tetraploidy in blastoid / pleomorphic mantle cell lymphoma than in classic mantle cell lymphoma (Blood 1997;89:1421)
    • 8q24 abnormalities (MYC gene) in blastoid mantle cell lymphoma (Mod Pathol 2002;15:1266)
    • High level of p53 expression in a subset of blastoid mantle cell lymphomas (Blood 2017;130:1903)
    • Mutated NOTCH1 and NOTCH2: affects signaling pathway (Blood 2012;119:1963, Proc Natl Acad Sci U S A 2013;110:18250)
    • De novo or from classic mantle cell lymphoma progression
    Clinical features
    • Median age: sixth and seventh decades
    • Clinical picture resembles classic mantle cell lymphoma
    • Male predominance (M:F = 2 - 3:1)
    • Ann Arbor clinical stage III/IV in most cases (Leukemia 2001;15:1785)
    • Short duration of clinical response after chemotherapy (Leukemia 2001;15:1785)
    • Poorer overall survival compared with classic mantle cell lymphoma (Leuk Lymphoma 2016;57:1327)
    • More common B symptoms
    • Blastoid variant: often presents de novo
    • Pleomorphic variant: often history of classic mantle cell lymphoma
    • Generalized lymphadenopathy
    • Extranodal involvement is common
    • Leukemic forms of mantle cell lymphoma (Am J Clin Pathol 2002;117:246)
    Diagnosis
    • Based on clinical presentation (refer to Clinical features)
    • B symptoms
    • Tissue biopsy (lymph node / extranodal sites)
    • Cytology
    • Peripheral blood exam / bone marrow biopsy
    Laboratory
    • Anemia and thrombocytopenia
    • High LDH and β2 microglobulin
    • A monoclonal serum component (low level)
    • Atypical lymphocytosis
    • Reference: Cancer 1998;82:567
    Prognostic factors
    Case reports
    • 45 year old man with blastoid mantle cell lymphoma presenting in leukemic phase (J Clin Diagn Res 2017;11:ED16)
    • 65 year old man with blastic transformation of mantle cell lymphoma presenting with acute onset binocular diplopia and proptosis of the left eye (Ocul Oncol Pathol 2019;5:245)
    • 66 year old man with cyclin D1 negative blastoid mantle cell lymphoma exhibiting cleaved to bilobated cytomorphology (Blood 2017;129:2711)
    • 67 year old woman with triple hit blastoid mantle cell lymphoma presenting like acute leukemia (Blood 2017;129:2593)
    • 74 year old man with ascites, liver lesions and growing 5 cm soft tissue abdominal mass (Case #499)
    • A case of double hit mantle cell lymphoma (Am J Clin Pathol 2020;153:49)
    Treatment
    • Aggressive chemotherapy or stem cell transplantation
      • Ibrutinib with rituximab (IR) induction
      • Followed by R-HCVAD-methotrexate / ara-C
      • R-HyperCVAD used at many institutions
        • Rituximab, hyperfractionated cyclophosphamide, vincristine, doxorubicin, dexamethasone
        • Alternating with methotrexate and cytosine arabinoside (ara-C)
      • Venetoclax may be added in some trials
    • Stem cell transplant (allogenic versus autologous)
    • CNS prophylaxis
    • Reference: Am J Hematol 2019;94:710
    Gross description
    • Lymph node
      • Generalized enlargement
      • Homogenous tan cut surface
      • With or without nodules
      • Not distinctive
    • GI
      • Lymphomatoid polyposis
      • Multiple lymphoid polyps, small and large bowel
      • Ulcers, tumor masses and diffuse thickening of the mucosa
    • Spleen
      • Generalized micronodular pattern
      • Perivascular infiltration
    Frozen section description
    • Same criteria applied in microscopic evaluation
    • Cytogenetics, flow cytometry, preparation of imprints should be considered
    • The nuclear shape, size and chromatin structure are helpful in smears
    Microscopic (histologic) description
    • Blastoid variant
      • The growth pattern is usually diffuse
      • Less frequently nodular pattern
      • Starry sky appearance common in low power
      • Intermediate sized cells
      • Monotonous population
      • Immature chromatin
      • Scant cytoplasm
      • High mitotic rate > 20 - 30/10 high power fields
      • Resembles, in part, lymphoblastic lymphoma
    • Pleomorphic variant
      • Variable size / heterogenous cell population including large cells
      • Prominent nucleoli +/-
      • High mitotic rate often seen
      • At least 10 mitoses per 10 high power fields and often higher rates
      • Resembles, in part, diffuse large B cell lymphoma
      • Peripheral blood lymphocytes are intermediate sized with prominent nucleoli
      • Resemble prolymphocytes to variable degree
      • Multinucleation or cerebriform nuclei often seen
    Microscopic (histologic) images

    Contributed by Chi Young Ok, M.D.

    Monotonous immature cells

    PAX5 immunohisto-
    chemical stain

    CD5 immunohisto-
    chemical stain

    Proliferation index, Ki67

    Follicular dendritic cell meshwork


    Large cells with variable sizes

    Cyclin D1 immunohisto-
    chemical stain

    p53 pattern of staining

    Diffuse involvement of bone marrow



    Contributed by Patricia Tsang, M.D., M.B.A. (Case #499)
    H&E stained section of the abdominal wall mass H&E stained section of the abdominal wall mass H&E stained section of the abdominal wall mass

    Abdominal wall mass

    PAX5


    CD5

    Ki67

    Cyclin D1

    SOX11

    Cytology description
    • Blastoid variant
      • Monomorphic proliferation of intermediate sized to large lymphoid cells
      • Round to oval nuclear contours
      • Dispersed chromatin
      • Small / inconspicuous nucleoli
      • Scant, pale blue cytoplasm
    • Pleomorphic variant
      • Large, pleomorphic cells
      • More nuclear irregularity
      • More variation in size of cells
      • Vesicular, open chromatin
      • Prominent nucleoli often seen (Br J Haematol 1996;93:475)
    Cytology images

    Contributed by Chi Young Ok, M.D.

    Immature monotonous cells in cytology

    Peripheral smear description
    • Prolymphocytoid-like morphology (leukemic form of mantle cell lymphoma)
    • Blastoid morphology (leukemic form of blastoid mantle cell lymphoma)
    Peripheral smear images

    Contributed by Chi Young Ok, M.D.

    Lymphoma cells in peripheral blood

    Positive stains
    Negative stains
    Flow cytometry description
    • CD5+, CD19+, CD20+, CD22+, CD79b+, FMC7+
    • Monotypic Ig (high level expression of surface light chains)
    • CD11c variable, CD43 variable, CD200 variable
    • CD3-, CD10-, CD23-
    • Rare cases have atypical immunophenotype:
      • CD5- or CD10+, CD200+ or CD23+ (dim ~10%)
    Molecular / cytogenetics description
    • Karyotypes are often complex (≥ 3 abnormalities)
    • Tetraploid clones are more frequently found in pleomorphic (80%) and blastoid (36%) variants
    • Some abnormalities may be specifically involved in pathogenesis
      • Chromosome 17p deletions (TP53)
      • Chromosome 9p deletions (CDKN2A)
      • Chromosome 8q24 translocations or amplification (MYC)
      • Chromosome 3q27translocations (BCL6)
    • Expression abnormalities can be simplified into 2 general types by array CGH
      • Cell cycle dysregulation / impaired DNA repair (ATM, cyclin D1, TP53 mutations)
      • Impaired apoptosis (BIRC3)
    • IG genes are clonally rearranged
    • IGV genes are unmutated or minimally mutated in most cases
    • t(11;14)(q13;q32)
      • FISH is convenient; it can be performed on fixed tissue sections
      • Conventional cytogenetics if fresh material available
      • Most PCR assays detect 1 major breakpoint region in mantle cell lymphoma
    • Aggressive variants of mantle cell lymphoma
      • TP53 or p16 mutations are common
      • t(8;14)(q24;q32)(IGH/MYC) in a small subset of cases
      • Mutations in many genes
        • ATM (~ 40%), TP53 (~ 25%), CDKN2A (TP16) (10 - 20%)
    • Other mutations correlated with blastoid or pleomorphic variant
      • KMT2D/MLL2 (~ 10%), WHSC1 (~ 10%), BIRC3 (~ 5%)
      • NOTCH1 (~ 5%), NOTCH2 (~ 5%), MEF2B (~ 3%), MYD88 (~ 1%)
      • CARD11 (10%)
    Molecular / cytogenetics images

    Contributed by Chi Young Ok, M.D.

    Complex karyotype

    Deletion of TP53 gene

    Sample pathology report
    • Left retroperitoneal lymph node, needle biopsies:
      • Mantle cell lymphoma, diffuse pattern, blastoid variant (see comment)
      • Comment: Routinely stained histologic sections reveal needle shaped fragments of tissue with partial crush artifact infiltrated by a diffuse monotonous infiltrate with focal starry sky pattern composed of large cells with irregular nuclei, blastoid chromatin, small nucleoli and scant cytoplasm. There is single cell apoptosis and mitotic figures are increased (45/10 high power fields).
        We performed immunohistochemical studies using paraffin embedded tissue, which show that the neoplastic cells are positive for cyclin D1, SOX11 (diffuse nuclear) and p53 (> 90%); while negative for TDT. Ki67 demonstrates a proliferation index of 100%. PAX5/CD5 immunohistochemical cocktail shows that the neoplastic cells are positive for PAX5 without co-expression of CD5.
        Fluorescence in situ hybridization (FISH) studies for t(11;14) translocation and MYC rearrangements are in progress and will be reported separately.
        Flow cytometry immunophenotyping detected a lambda restricted B cell population positive for CD10, CD19, CD20, CD22, CD38, CD79b; while negative for CD5, CD11c, CD30, CD43, CD44, CD200 and kappa light chain.
      • Addendum:
        • Positive for IGH/CCND1 gene rearrangement
        • Negative for MYC gene rearrangement
    Differential diagnosis
    Board review style question #1
      In the blastoid variant of mantle cell lymphoma, which of the following is the correct option?

    1. Morphology is composed of cells with variable size and prominent nucleoli
    2. Most cases are CD5 negative and CD23 positive
    3. Most cases are Cyclin D1 and SOX11 positive
    4. Ki67 index is typically below 20%
    Board review style answer #1
    C. Most cases are Cyclin D1 and SOX11 positive

    Comment Here

    Reference: Mantle cell lymphoma, aggressive variant
    Board review style question #2

      This is the concurrent fine needle aspiration flow cytometry of the lymph node in a 70 year old man with lymphadenopathy. Cytology smears show intermediate to large lymphoid cells in a necrotic background. What is the most probable preliminary diagnosis before further work ups on the excised lymph node?

    1. Nodal marginal zone lymphoma
    2. Follicular lymphoma, grade 3A
    3. Mantle cell lymphoma
    4. Chronic lymphocytic lymphoma
    Board review style answer #2
    C. Mantle cell lymphoma

    Comment Here

    Reference: Mantle cell lymphoma, aggressive variant

    MCL-classic
    Definition / general
    • Clinically aggressive, mature B cell lymphoma
    • Small to medium sized lymphoid cells (monomorphic, except in pleomorphic variant)
    Essential features
    • Associated with t(11;14)(q13;q32) or IGH/CCND1 and cyclin D1 overexpression
    Terminology
    ICD coding
    • ICD-O: 9673/3 – mantle cell lymphoma
    • ICD-10: C83.1 – mantle cell lymphoma
    Epidemiology
    • 7% of B cell lymphomas (Blood 1997;89:3909)
    • ~10% of non-Hodgkin lymphomas
    • Age adjusted incidence: 0.6/100,000 person years
    • Median age: 68 years
    • M:F = 2 - 3:1
    Sites
    • Lymph node, bone marrow (50 - 90%), peripheral blood (20 - 70%), spleen (50%), liver (20%) (Am J Clin Pathol 1996;106:196, Am J Clin Pathol 1997;108:302)
    • Frequent extranodal site involvement : gastrointestinal tract, Waldeyer ring, lungs, pleura, skin, CNS
    • Extranodal involvement without lymphadenopathies: 4 - 15%
    Pathophysiology
    Etiology
    • Pregerminal center origin (SOX11+, unmutated IGHV)
    • Postgerminal center origin, a subset (SOX11-, hypermutated IGHV)
    • Reference: Cancer Res 2012;72:5307
    Clinical features
    • 70%: stage IV disease (at presentation)
    • Two subtypes:
      • One more aggressive with SOX11+, nodal presentation
      • One more indolent with SOX11- disease, leukemic presentation, and non-nodal disease
    • Generalized lymphadenopathies
    • Hepatomegaly / splenomegaly
    • Peripheral blood:
      • Atypical lymphoid cells: present virtually in all cases by flow cytometry (Cancer 2007;109:2473)
      • Leukemic involvement at diagnosis: 20 - 70% of patients
    • Multiple intestinal polyps (lymphomatous polyposis)
    • Progress to blastoid / pleomorphic variant
      • In relapse (22%)
    Diagnosis
    • Tissue biopsy (lymph node / extranodal sites): monomorphic proliferation of small to intermediate sized B cells with overexpression of cyclin D1 or SOX11
    • t(11;14)(q13;q32) IGH/CCND1
    • Reference: Cancer Res 2012;72:5307
    Laboratory
    • Anemia and thrombocytopenia: 10 - 40%
    • High LDH and β2 microglobulin: 50%
    • A monoclonal serum component (low level): 10 - 30%
    • Atypical lymphocytosis
    • Reference: Cancer 1998;82:567
    Prognostic factors
    Case reports
    • 60 year old man with mantle cell lymphoma involving the prostate with features of granulomatous prostatitis (Int J Surg Pathol 2012;20:610)
    • 63 year old man with Hodgkin and Reed-Sternberg cells in a recurrent mantle cell lymphoma (Appl Immunohistochem Mol Morphol 2019;27:8)
    • 64 year old man with right conjunctival mass as an initial presentation of mantle cell lymphoma (BMC Res Notes 2012;5:671)
    • 69 year old man with a history of systemic nodal mantle cell lymphoma who presented with subcutaneous nodules on his lower extremities (J Cutan Pathol 2019;46:538)
    • A case of mantle cell lymphoma showing cyclin D1 overexpression by immunohistochemistry and a t(11;12)(q13;p11.2) by conventional cytogenetic studies (Hum Pathol 2017;64:207)
    Treatment
    • Fit for high dose therapy
      • Hyper CVAD chemotherapy or R-DHAP / R-CHOP with autologous stem cell transplantation
    • Fit for conventional dose therapy
      • Rituximab (R)-CHOP or R-bendamustine
    • Unfit for conventional dose therapy
      • R-bendamustine or R-lenalidomide
    • Indolent disease (low risk)
      • Watch and wait
    • New biologic agents
      • Ibrutinib (Bruton tyrosine kinase inhibitor)
      • Idelalisib (phosphoinositide-3 kinase inhibitor)
      • Bortezomib (proteasome inhibitors)
      • Temsirolimus (mTOR inhibitors)
      • Lenalidomide (immunomodulatory)
    • References: F1000Res 2018;7:1136, Best Pract Res Clin Haematol 2018;31:99
    Gross description
    • Lymph node
      • Generalized enlargement
      • Homogenous tan cut surface
      • With or without nodules
      • Not distinctive
    • Spleen
      • Generalized micronodular pattern
      • Perivascular infiltration
    • GI
      • Lymphomatoid polyposis
      • Multiple lymphoid polyps, small and large bowel
      • Ulcers, tumor masses and diffuse thickening of the mucosa (Histopathology 1995;26:413)
      • Note: Microscopic infiltration without gross lesion is common (Cancer 2003;97:586)
    Frozen section description
    • Same criteria applied in microscopic evaluation
    • Cytogenetics, flow cytometry, preparation of imprints should be considered
    • The nuclear shape, size and chromatin structure are helpful in smears
    • Reference: Jaffe: Hematopathology, 2nd Edition, 2016
    Microscopic (histologic) description
    • Diffuse > nodular > mantle zone growth patterns
      • Nodal (> 50% nodular), diffuse growth pattern (< 50% nodular)
    • Small to medium monomorphic lymphoid neoplasm
    • Irregular nuclear border, clumped chromatin and inconspicuous nucleoli
    • No proliferation centers
    • Centroblasts, immunoblasts or paraimmunoblasts are absent
    • Hyalinized vessels
    • Epithelioid histiocytes
    • Variants
      • Aggressive variants
        • Blastoid: lymphoblast-like in appearance, monomorphism
          • >20 - 30 mitoses per 10 high power fields
          • Resemble lymphoblastic lymphoma
        • Pleomorphic: large cells with irregular nuclear border, cerebriform nuclei, multinucleation, lack of monomorphism
          • Prominent nucleoli and abundant pale cytoplasm
          • Resemble DLBCL
      • Other variants
        • Small cell: small round lymphocytes with more clumped chromatin
          • Resemble CLL
        • Marginal zone-like: abundant pale cytoplasm
          • Resembling marginal zone or monocytoid B cells
        • Lymphoplasmacytic differentiation, some cases (Am J Surg Pathol 2006;30:954)
    • Follicular dendritic cell (FDC) meshwork
      • Nodular pattern
        • Primary follicle-like pattern
        • Germinal center-like pattern
      • Diffuse pattern
    • Bone marrow
      • Nodular, interstitial or paratrabecular or combination
    • Peripheral blood (see below)
      • Similar spectrum seen in tissue sample
      • Nucleoli are sometimes more prominent
    • Spleen
      • White pulp nodules involved (enlarged)
      • Variable involvement of the red pulp
      • Residual naked germinal centers
      • Tumor cells: similar monotonous morphology
      • Some cases may show a marginal zone-like area (Histopathology 1998;33:230)
    • GI
    • Relapse
      • Loss of a mantle zone growth pattern
      • Increase in nuclear size
      • Pleomorphism and chromatin dispersal
      • Increase in mitotic activity and Ki67
      • Cases that are blastoid at diagnosis may relapse with classic morphology (Histopathology 2013;62:334)
    Microscopic (histologic) images

    Contributed by Chi Young Ok, M.D.

    Monotonous population of small lymphoid cells

    Follicular localization of lymphoid cells

    Spleen involvement

    Cyclin D1 expression


    Mantle zone pattern

    Pink histiocytes

    Cyclin D1 and SOX11 in mantle zone pattern


    Ki67 in mantle zone pattern

    Bone marrow involvement

    Follicular dendritic cell meshwork

    Cytology description
    • A monotonous population of small to intermediate sized lymphoid cells
    • Nuclear clefts with fine stippled chromatin
    • Inconspicuous nucleoli
    • Distinct pale or basophilic cytoplasm
    • Reference: Jaffe: Hematopathology, 2nd Edition, 2016
    Cytology images

    Contributed by Chi Young Ok, M.D.

    Small to intermediate centrocytes in cytology

    Peripheral smear description
    Peripheral smear images

    Contributed by Chi Young Ok, M.D.

    Atypical mature cell in peripheral blood

    Positive stains
    Negative stains
    Flow cytometry description
    Flow cytometry images

    Contributed by Chi Young Ok, M.D.

    Aberrant B cell population with immunophenotype: CD5+, CD19+, CD20+, CD38+, lambda+, CD10-, CD23-, CD200- and kappa-

    Molecular / cytogenetics description
    • IG genes are clonally rearranged
    • IGHV genes are unmutated or minimally mutated in most cases
    • t(11;14)(q13;q32)
      • FISH is convenient because it can be performed on fixed tissue sections
      • Conventional cytogenetics if fresh material available
      • Most PCR assays usually detect 1 major breakpoint region in mantle cell lymphoma
    • Cyclin D2 rearrangements by FISH in some of cyclin D1(-) cases
    • Secondary chromosomal aberrations
      • Gains of 3q26 (in 31 - 50%)
      • 7p21 (16 - 34%),
      • 8q24 (MYC, 16 - 36%)
      • 13q31 (24%)
      • Losses of 1 p13-31 (29 - 52%)
      • 6q23-27 (TNFAIP3, 23 - 38%)
      • 9p21 (CDKN2A which codes for p16 INK4a and p14 ARF, 18 - 31%)
      • 11q22-23 (ATM, 21 - 59%)
      • 13q11-13 (22 - 55%)
      • 13q14-34 (43 - 51%)
      • 17p13 (TP53, 21 - 45%) (Blood 1999;93:436, Blood 2005;105:4445, Oncogene 1992;7:1401)
    Molecular / cytogenetics images

    Contributed by Chi Young Ok, M.D.

    Abnormal FISH result

    Abnormal karyotype result

    Sample pathology report
    • Inguinal lymph node, left, needle biopsies:
      • Mantle cell lymphoma, diffuse pattern, classic variant (see comment)
      • Comment: Histologic sections show needle core shaped lymphoid tissue. Lymph node architecture is entirely effaced by malignant lymphoma. The lymphoma demonstrates a diffuse pattern and the lymphoma cells are intermediate sized. Lymphoma cells have irregular nuclear contours, relatively closed chromatin, inconspicuous nucleolus and moderate to large amount of cytoplasm. Scattered epithelioid histiocytes with eosinophilic cytoplasm (“pink histiocytes”) are seen in the background. No sheets of large cells or necrosis are seen. Mitotic figures are rarely present.
        Flow cytometric analysis using concurrent cytologic sample demonstrates clonal B cells with surface kappa light chain expression, positive for CD5, CD19, CD20, CD22 ,CD38, CD43 (partial), CD44, CD79b, while they are negative for CD3, CD4, CD8, CD10, CD11c, CD23, CD30, CD34, CD56 and CD200. The clonal B cells comprise approximately 62.4% of total analyzed cellular events and 70.7% of the lymphoid population.
        Immunohistochemical studies were performed on formalin fixed, paraffin embedded tissue. Lymphoma cells are positive for CD20, PAX5, cyclin D1 and SOX11. They are negative for CD3 and p53. Ki67 proliferation rate is estimated at 10 - 20%.
        In summary, morphologic, flow cytometric and immunohistochemical studies are diagnostic of mantle cell lymphoma, classic variant.
    Differential diagnosis
    Board review style question #1
      Which of the following translocations is characteristic of mantle cell lymphoma?

    1. t(14;18)
    2. t(11;18)
    3. t(11;14)
    4. t(8;14)
    Board review style answer #1
    Board review style question #2

    Plot 1

    Plot 2

    Plot 3

    Plot 4


      A 74 year old man has presented with an absolute lymphocytosis (6 - 7 x 103/μL) for about 6 months. You are screening the peripheral blood with a preliminary flow cytometry that is provided in the first three plots. You decide to extend the flow cytometry markers. Which combination of markers in the last plot (plot 4) will more likely represent CLL phenotype rather than MCL phenotype?

    1. CD79a / CD45
    2. CD43 / CD200
    3. CD79a / FMC7
    4. CD5 / CD79a
    Board review style answer #2

    MCL-leukemic nonnodal
    Definition / general
    • Mantle cell lymphoma involving the peripheral blood, bone marrow, occasionally spleen and gastrointestinal tract but not lymph nodes
    Essential features
    • Must lack significant lymphadenopathy (< 1 cm on physical exam and not detected via imaging / CT scans)
    • t(11;14)(q13;q32) = fusion of CCND1 and IGH genes
    • Positive for Cyclin D1 (also known as BCL1) and negative for SOX11
    Terminology
    • Leukemic mantle cell lymphoma
    • Nonnodal leukemic mantle cell lymphoma (NN-L-MCL)
    • Monoclonal asymptomatic lymphocytosis, cyclin D1 positive (MALD1)
    ICD coding
    • ICD-10: C83.10 - mantle cell lymphoma, unspecified site
    Epidemiology
    • M > F = ~2:1
    • Mean age = ~60 years old
    Sites
    • Peripheral blood, bone marrow and occasionally spleen or gastrointestinal tract
    • Circulating cells may reversibly infiltrate extranodal sites and typically remain in the mantle zones of follicles, overlapping with in situ mantle cell lymphoma
    Pathophysiology
    • t(11;14)(q13;q32), fusion of CCND1 gene and IGH promoter increases production of the Cyclin D1 protein, regulator of cyclin dependent kinase (CDK4) and CDK6 which controls the G1 / S cell cycle transition (J Clin Invest 2012;122:3416)
    Etiology
    • Cell of etiology thought to be peripheral B cell of inner mantle zone
    Clinical features
    • Typically asymptomatic at presentation
    Laboratory
    • Mild leukocytosis and mild absolute lymphocytosis
    • 10 - 20% patients will have a monoclonal protein
    Prognostic factors
    Case reports
    Treatment
    • May be observed until progression, then treated as typical mantle cell lymphoma with combination chemotherapy, rituximab and subsequent stem cell transplant depending on age and performance status (Curr Oncol Rep 2018;20:79)
    Microscopic (histologic) description
    • Diffuse interstitial infiltrate or small scattered nonparatrabecular lymphoid aggregates
    • Small to intermediate size lymphoid cells with scant cytoplasm, round to slightly irregular nuclear contours and mature chromatin
    • Some cases will show dispersed chromatin and indistinct nucleoli
    Microscopic (histologic) images

    Contributed by Nicholas Nowacki, M.D.

    Lymphoid infiltrate

    Small lymphoma cells

    Marrow aspirate


    CD3 highlights T cells

    CD5 positive

    CD20 positive

    Cyclin D1 positive

    SOX11 negative

    Cytology description
    • Variable morphologic appearance
      • Majority (~70%) show chronic lymphocytic leukemia-like morphology = small with scant cytoplasm, round nuclear contours and condensed chromatin (Cancer Res 2010;70:1408)
      • Fewer (~30%) with typical mantle cell lymphoma, centrocyte-like morphology = small to medium sized lymphoid cells with scant cytoplasm, irregular nuclear contours, dispersed chromatin and indistinct nucleoli
    Peripheral smear description
    • Same as cytology description
    Peripheral smear images

    Contributed by Nicholas Nowacki, M.D.

    Peripheral blood

    Positive stains
    Negative stains
    Flow cytometry description
    • Typical immunophenotype:
      • Positive: CD20 (moderate to bright), surface light chain restriction (moderate to bright), CD5 (subset), CD79b, FMC7
      • Negative: CD10, CD103, CD123, CD11c
    Flow cytometry images

    Contributed by Nicholas Nowacki, M.D.

    Aberrant CD5 coexpression

    CD20 moderate expression

    Surface light chain restriction

    Molecular / cytogenetics description
    • t(11;14)(q13;q32), fusion of CCND1 and IGH genes
    • Most cases have somatic hypermutation / IgVH gene hypermutated (Clin Cancer Res 2014;20:1007)
    • Typically few additional cytogenetic abnormalities and much less likely to have a complex karyotype than classic mantle cell (Leukemia 2012;26:1895)
    Sample pathology report
    • Bone marrow, biopsy, aspirate and peripheral blood smear:
      • Mantle cell lymphoma involving a normocellular marrow with preserved trilineage hematopoiesis (see comment)
      • Comment: The overall findings are consistent with marrow involvement by mantle cell lymphoma. Given the lack of SOX11 staining, the possibility of the leukemic nonnodal mantle cell lymphoma variant should be considered. By definition, patients with this variant must lack significant lymphadenopathy (< 1 cm). The presence of lymphadenopathy would argue for the classic variant of mantle cell lymphoma. Clinical correlation and correlation with cytogenetic / FISH studies, including t(11;14)(q13;q32) IGH-CCND1, is recommended.
    Differential diagnosis
    Board review style question #1

    What is the most likely immunophenotype for leukemic nonnodal mantle cell lymphoma?

    1. CD20+, CD5+, CD10+, CD103-, CD123-, Cyclin D1+, SOX11+
    2. CD20+, CD5+, CD10-, CD103+, CD123+, Cyclin D1+, SOX11-
    3. CD20+, CD5+, CD10-, CD103-, CD123-, Cyclin D1+, SOX11+
    4. CD20+, CD5+, CD10-, CD103-, CD123-, Cyclin D1+, SOX11-
    5. CD20+, CD5-, CD10-, CD103-, CD123-, Cyclin D1-, SOX11+
    Board review style answer #1
    D. CD20+, CD5+, CD10-, CD103-, CD123-, Cyclin D1+, SOX11-

    Comment here

    Reference: Nonnodal mantle cell lymphoma
    Board review style question #2
    A 60 year old woman was noted to have a mild absolute lymphocytosis on routine labs. A bone marrow biopsy was performed and based on the morphologic and immunophenotypic features a diagnosis of leukemic nonnodal mantle cell lymphomais is suspected. If this is correct, which cytogenetic abnormality should be detected on cytogenetic / FISH analysis?

    1. deletion 17p and t(8;14)(q24;q32) IGH-MYC
    2. t(8;14)(q24;q32) IGH-MYC
    3. t(11;14)(q13;q32) IGH-CCND1
    4. t(14;18)(q32;q231) IGH-BCL2
    5. trisomy 12
    Board review style answer #2
    C. t(11;14)(q13;q32) IGH-CCND1

    Comment here

    Reference: Nonnodal mantle cell lymphoma

    Marginal zone-nodal
    Definition / general
    • Primary nodal mature B cell lymphoma derived from postgerminal center B cell
    • Morphological and immunophenotypical overlap with extranodal marginal zone lymphoma (EMZL) of mucosa associated lymphoid tissue (MALT) and splenic marginal zone lymphoma (SMZL)
    Essential features
    • Relatively rare, indolent low grade B cell lymphoma derived from postgerminal center B cell
    • Patients present with localized or generalized peripheral lymphadenopathy
    • Perifollicular and interfollicular lymphoid infiltrate that surrounds variably preserved germinal centers; diffuse pattern of infiltration can also be seen
    • Infiltrate composed of monocytoid lymphoid cells, admixed with scattered large transformed B cells, positive for pan-B cell markers with light chain restriction and negative for CD5 and CD10
    • Overall favorable prognosis with chemoimmunotherapy as mainstay of treatment
    ICD coding
    • ICD-0: 9699/3 - marginal zone B cell lymphoma, NOS
    Epidemiology
    Sites
    Etiology
    Clinical features
    • Localized or generalized peripheral lymphadenopathy (Br J Haematol 2007;136:301)
    • Bone marrow is involved in < 50% of patients; peripheral blood rarely involved
    • B symptoms rare
    • Clinical and radiologic evaluation needs to be performed to rule out lymph node involvement by EMZL and SMZL
    • Pediatric nodal marginal zone lymphoma (NMZL): predominantly young males, with median age of 16 years (M:F = 20:1); localized head and neck lymphadenopathy (Am J Surg Pathol 2003;27:522)
    • Reference: Best Pract Res Clin Haematol 2017;30:92
    Diagnosis
    • Biopsy of affected lymph node
    Laboratory
    Prognostic factors
    • Indolent lymphoma with 5 year overall survival of 60 - 70% (Eur J Haematol 2009;83:165)
    • Factors associated with worse prognosis include advanced age, high clinical stage and B symptoms
    • Pediatric NMZL: excellent prognosis and long term survival (Am J Surg Pathol 2003;27:522)
    Case reports
    Treatment
    • Watch and wait strategy in patients with low tumor burden and no clinical symptoms
    • Rituximab combined with chemotherapy is the mainstay of treatment (Ann Oncol 2020;31:17)
    • Radiation is a consideration in localized disease
    • Pediatric NMZL: conservative therapy (Am J Surg Pathol 2003;27:522)
    Microscopic (histologic) description
    • Typically nodular, perifollicular and interfollicular lymphoid infiltrate that surrounds variably preserved germinal centers (Am J Clin Pathol 2009;132:39)
    • Follicular colonization may be seen
    • Some cases show diffuse pattern of combined nodular and diffuse patterns
    • Neoplastic infiltrate contains marginal zone B cells (monocytoid) with abundant pale cytoplasm, admixed with scattered large transformed B cells; large cells cannot be seen in sheets
    • Fraction of cases that show increased large cells appears to be relatively high
    • Variable degree of plasma cell differentiation can be seen
    • Eosinophilia may be seen
    • Some cases have increased number of large B cells (Histopathology 2006;48:162)
    • Pediatric NMZL: pattern of follicular hyperplasia with extension of mantle zone cells into germinal centers mimicking progressive transformation of germinal centers (Am J Surg Pathol 2003;27:522)
    Microscopic (histologic) images

    Contributed by Anamarija M. Perry, M.D.
    Lymph node with nodal marginal zone lymphoma (NMZL) Lymph node with nodal marginal zone lymphoma (NMZL) Lymph node with nodal marginal zone lymphoma (NMZL)

    Lymph node with nodal marginal zone lymphoma (NMZL)

    Atypical infiltrate Atypical infiltrate

    Atypical infiltrate

    Lymph node with pediatric NMZL

    Lymph node with pediatric NMZL


    CD20 stain

    CD20 stain

    CD10 stain

    CD10 stain

    BCL2 stain

    BCL2 stain

    Ki67 stain

    Ki67 stain

    PAX5 stain in pediatric NMZL

    PAX5 stain in pediatric NMZL

    IgD stain in pediatric NMZL

    IgD stain in pediatric NMZL

    Virtual slides

    Images hosted on other servers:
    NMZL

    NMZL

    Pediatric NMZL

    Pediatric NMZL

    Positive stains
    Negative stains
    Flow cytometry description
    Molecular / cytogenetics description
    Sample pathology report
    • Lymph node, left neck, excisional biopsy:
      • Morphologically consistent with nodal marginal zone lymphoma (see comment)
      • Comment: Correlate with clinical, laboratory and radiologic findings.
    Differential diagnosis
    • Extranodal marginal zone lymphoma (EMZL):
      • Occasionally involves lymph nodes and shows overlapping morphologic and immunophenotypic features with NMZL
      • Typically, well preserved reactive follicles with mantle zone are seen with expanded marginal zone by monocytoid cells
      • Clinical and radiologic correlation is needed to rule it out
    • Splenic marginal zone lymphoma (SMZL) (Blood 2016;127:2072):
      • Clinically, patients present with splenomegaly and bone marrow involvement
      • Clinical and radiologic correlation is needed
      • Occasionally, SMZL can involve lymph nodes and show overlapping features with NMZL
    • Lymphoplasmacytic lymphoma (LPL) (Hematol Oncol Clin North Am 2019;33:639):
      • Lymph node architecture usually partially preserved with open sinuses
      • Infiltrate is composed of small lymphocytes, plasmacytoid cells and plasma cells with frequent Dutcher bodies
      • Bone marrow is involved in LPL and patients have monoclonal protein (IgM); therefore, correlation with laboratory findings and bone marrow biopsy is recommended
      • MYD88 mutation is consistently seen in LPL, while it has been reported in rare cases of NMZL
    • Marginal zone hyperplasia:
      • Expanded marginal zone seen in some reactive conditions
      • Difficult to distinguish from NMZL bases on morphology alone
      • Identification of clonal B cell population favors the diagnosis of NMZL
    • Follicular lymphoma (FL) with marginal zone differentiation:
      • Neoplastic nodules show cells with monocytoid appearance having abundant pale cytoplasm
      • In the centers of neoplastic nodules, lymphoma cells frequently show more typical FL appearance
      • Lymphoma cells are CD10 and BCL6 positive and coexpressing BCL2, although monocytoid component could show weak / absent CD10
      • Additional useful immunostains - HGAL, LMO2 and MEF2B are positive in FL, while MNDA and IRTA1 are positive in NMZL
    • Small lymphocytic lymphoma (SLL):
      • Occasional cases have perifollicular infiltration mimicking NMZL
      • Infiltrate in SLL is monotonous with small lymphoid cells with clumped chromatin and scant cytoplasm
      • Proliferation centers with prolymphocytes and paraimmunoblasts are seen
      • Lymphoma cells are positive for CD5, CD23 and LEF1
    Board review style question #1

    A 60 year old woman presents with cervical and axillary lymphadenopathy and history of night sweats for the past 3 months. Cervical lymph node is excised and infiltrate is shown in the image above. Flow cytometry showed clonal B cell population and was positive for CD20 and CD19 with kappa light chain restriction. The population was negative for CD5, CD10 and CD23. Which of the following genetic abnormalities can occasionally be found in this lymphoma?

    1. DUSP22 rearrangement
    2. IGH-BCL2 rearrangement
    3. IGH-CCND1 rearrangement
    4. MYD88 mutation
    5. t(2;5)(p23;q35); ALK-NPM1
    Board review style answer #1
    D. MYD88 mutation

    Comment Here

    Reference: Nodal marginal zone lymphoma
    Board review style question #2


    A 13 year old boy presents with cervical lymphadenopathy. Cervical lymph node is excised and infiltrate is shown in the images above. Molecular studies were performed on the excisional biopsy and showed clonal immunoglobulin gene rearrangement. Which of the following is true about this lymphoma?

    1. Approximately 50% of cases transform to diffuse large B cell lymphoma
    2. It is more common in women
    3. Long term prognosis is poor
    4. Most cases show TNFRSF14 mutation
    5. Patients typically present with head and neck lymphadenopathy
    Board review style answer #2
    E. Patients typically present with head and neck lymphadenopathy

    Comment Here

    Reference: Nodal marginal zone lymphoma

    Mediastinal gray zone lymphoma (pending)
    [Pending]

    Molecular overview (pending)
    Table of Contents
    Molecular overview
    Molecular overview

    Monomorphic epitheliotropic intestinal
    Definition / general
    • Monomorphic epitheliotropic intestinal T cell lymphoma (MEITL), previously known as type II enteropathy associated T cell lymphoma, is a primary intestinal mature T cell lymphoma that is considered to arise from intestinal intraepithelial T lymphocytes, is usually positive for CD8 and CD56 and lacks association with celiac disease
    Essential features
    • Aggressive primary T cell lymphoma with a poor clinical outcome, with an average of 6 - 10 months overall survival (Leukemia 2013;27:1688)
    • Histologically, it is monomorphic small to medium sized T cells expressing CD3, CD8, CD56 and cytotoxic markers (TIA1 or granzyme B)
    • Predominately expresses gamma / delta T cell receptors
    • Usually has complex cytogenetic abnormalities; mutations in STAT5B, SETD2, JAK3, GNAI2 and CREBBP are common; aberrant overexpression of SYK has been described (Blood Adv 2020;4:4769)
    Terminology
    • Previously known as type II enteropathy associated T cell lymphoma (EATL) or enteropathy associated T cell lymphoma type II
    ICD coding
    • ICD-10: C86.2 - enteropathy type (intestinal) T cell lymphoma
    Epidemiology
    Sites
    Pathophysiology
    • Neoplastic T cells are generally thought to be derived from the intestinal intraepithelial lymphocytes, with a phenotype of CD8+, CD56+ and megakaryocyte associated tyrosine kinase (MATK), which appears to have a pathogenetic role in MEITL (Leukemia 2013;27:1688)
    • Hypothesized that altered T cell receptor (TCR) signaling contributes to the occurrence of MEITL
      • Aberrant overexpression of SYK due to hypomethylation of the SYK promoter was reported, causing abnormal proliferation of the lymphoma cells (Mod Pathol 2018;31:505)
    • Studies also showed that chromosome alterations may be pathogenetic for MEITL, including chromosomal copy number variations, gains of the chromosomal region 9q33-q34 and an amplification of chromosome locus 8q24, resulting in neoplastic cell proliferation (Mod Pathol 2015;28:1286)
    • Characterized by activating mutations of the JAK-STAT pathway, with mutations of STAT5B being the most common
      • Other commonly mutated genes include JAK3, GNAI2, CREBBP and SETD2, which are involved in the pathogenesis of MEITL (Nat Commun 2016;7:12602)
    • Whole genome sequencing confirmed the previous studies of cytogenetic and molecular mutations involved in MEITL, including CREBBP, STAT5B, SETD2, GNAI2, JAK3 and AXSL3 (Blood Adv 2020;4:4769)
    Etiology
    • Unknown
    Clinical features
    • Usually nonspecific clinical symptoms, including a change in bowel habits, chronic abdominal pain (86%), chronic diarrhea (33%), weight loss and complications including intestinal obstruction / perforation (33%) and bleeding (17%); B symptoms are common (48%) (Ann Hematol 2019;98:2541)
    • Endoscopically or grossly, the tumor is manifested with mucosal granular forming ulcers, nodules, plaques, diffuse mucosal thickening, strictures or large masses (single or multifocal)
    • Patients usually do not manifest with malabsorption or celiac disease
    Diagnosis
    • Diagnosis requires clinicopathological correlation
    • In patients without a history of celiac disease presenting with chronic abdominal pain, diarrhea and other related symptoms, abdominal imaging studies are warranted, including abdominal ultrasound, CT with or without contrast, MRI and endoscopy
    • Endoscopy usually shows fine mucosal granularity and diffuse thickening of the mucosa with semicircular shallow ulcerations, while abdominal MRI or CT show thickening of bowel wall and mesenteric lymphadenopathy
    • Tissue biopsy demonstrates monomorphic neoplastic lymphocytes with aberrant T cell phenotype, positive for CD3, CD8, CD56, TIA1 and T cell receptor gamma or delta
    Laboratory
    • Important to rule out celiac disease by demonstrating negativity for endomysial antibodies, HLA DQ2 or HLA DQ8
    Radiology description
    • Abdominal computed tomography (CT) sometimes can demonstrate a mass wrapping around the small bowel, with thickening of the intestinal wall or distension of the small bowel due to obstruction and ascites
    • Free air under the diaphragm can be seen if perforated
    • Mesenteric lymphadenopathy usually indicates neoplastic spreading (Hematology 2018;23:10)
    Radiology images

    Contributed by Xiaoling Guo, M.D., Ph.D. and Yanhua Wang, M.D.
    CT abdomen and pelvis

    CT abdomen and pelvis

    Prognostic factors
    • Overall, poor prognosis; estimated average survival is about 6 - 10 months and a median survival of 7 months due to lack of specific targeted therapy (Am J Hematol 2012;87:663)
    Case reports
    Treatment
    • Commonly used regimens include surgical tumor resection, CHOEP (cyclophosphamide, doxorubicin, vincristine, etoposide, prednisone), CHOP14 and CHOP21 (cyclophosphamide, doxorubicin, vincristine, prednisone), dose adjusted EPOCH (etoposide, prednisone, vincristine, cyclophosphamide, doxorubicin), hyper-CVAD (hyperfractionated cyclophosphamide, vincristine, doxorubicin, dexamethasone) / R-MTX-Ara-C (rituximab, methotrexate, cytarabine) and stem cell transplant (J Natl Compr Canc Netw 2016;14:1067)
    Gross description
    • Grossly, the findings vary depending the tumor stage, from edematous or granular mucosa, to diffuse thickening of the intestinal wall, to a mass with or without ulceration
    • Mass can be sub centimeter to a few centimeters in size
    Gross images

    Contributed by Xiaoling Guo, M.D., Ph.D. and Yanhua Wang, M.D.

    Ulcerated mass

    Cross section of the mass

    Microscopic (histologic) description
    • Neoplastic lymphocytes are relatively monotonous, intermediate in size, with round or slightly irregular nuclear contours, dispersed chromatin, inconspicuous nucleoli and scant rim of pale cytoplasm
    • Prominent epitheliotropism as well as transmural infiltrate are characteristic; only few inflammatory cells are noted in the background, unless associated with mucosal ulceration
    • Reference: Cureus 2020;12:e10021
    Microscopic (histologic) images

    Contributed by Xiaoling Guo, M.D., Ph.D. and Yanhua Wang, M.D.

    Uninvolved mucosa

    Mucosa overlying the mass

    Monomorphic transmural lymphocytic infiltrate

    CD3

    CD7


    CD8

    Granzyme B

    TIA1

    CD20

    Ki67

    Positive stains
    • CD3, CD8, CD56 and TIA1
    • Other cytotoxic markers are variably expressed
    • CD20 (approximately 20% of MEITLs aberrantly express CD20; however, other pan B cell markers are negative) (Pathology 2020;52:128)
    Negative stains
    Flow cytometry description
    Molecular / cytogenetics description
    • Extra signals for MYC at 8q24 are commonly seen (J Clin Exp Hematop 2018;58:102)
    • Gains at 9q34.3 by FISH and copy number analysis were seen in > 75% of cases and other aberrations include gains at 1q32.3, 4p15.1, 5q34, 7q34, 8p11.23, 9q22.31, 9q33.2, 8q24 (MYC locus) and 12p13.31 and losses of 7p14.1 and 16q12.1 (Mod Pathol 2015;28:1286)
    • 63% of cases had mutations in STAT5B when examined by whole exome sequencing (Leukemia 2016;30:1311)
    • JAK3 and GNAI2 are also mutated in some cases, while SETD2 mutation was reported > 90% of cases (Nat Commun 2016;7:12602)
    • Spleen tyrosine kinase (SYK) was overexpressed in MEITL, which was caused most likely by hypomethylation of the SYK promoter (Mod Pathol 2018;31:505)
    Sample pathology report
    • Distal small intestine, resection:
      • Monomorphic epitheliotropic intestinal T cell lymphoma (MEITL) with gamma / delta phenotype (see comment)
      • No histomorphologic features of celiac disease in uninvolved adjacent nonneoplastic mucosa
      • Comment: According to the clinical records, patient had no history of celiac disease. Microscopic examination revealed diffuse and dense monomorphic small to medium sized atypical T cells with transmural infiltration and notable absence of necrosis. The immunohistochemical stains demonstrated aberrant T cells positive for CD3, CD7, CD8, CD56, granzyme B, TIA1, TCR delta, T-bet and CD20 (subset). The aberrant cells were negative for CD4, CD2, CD5, CD30, CD57, GATA3, TCR beta, pSTAT3, CD79a and PAX5. Proliferation index (Ki67) was approximately 90%. EBV small RNA in situ hybridization (EBER) was negative. Overall, the clinical history, histomorphology and immunophenotype support the diagnosis of monomorphic epitheliotropic intestinal T cell lymphoma (MEITL).
    Differential diagnosis
    Board review style question #1

    Which of the following best describes the features of the small intestine displayed in the above figure?

    1. Can be associated with celiac disease
    2. Commonly seen in Caucasian population
    3. Most common phenotype is CD3+, CD8+, CD56+ and TCRδγ+
    4. Monomorphic T cells in lymphoma with indolent clinical outcome
    5. Patients usually manifest symptoms of malabsorption
    Board review style answer #1
    C. Most common phenotype is CD3+, CD8+, CD56+ and TCRδγ+. The figure displays small intestine mucosa with a dense infiltrate of epithelium by monomorphic small to intermediate sized lymphocytes, consistent with monomorphic epitheliotropic T cell lymphoma (MEITL). This is a primary intestinal lymphoma that is distinguished from enteropathy associated T cell lymphoma (EATL) because it is not associated with celiac disease or malabsorption, it is more common in Asian populations and has an aggressive behavior with a poor outcome. Phenotypically, MEITL is usually CD3+, CD8+, CD56+ and TCRδγ+ and aberrantly expresses CD20 in 20% of patients.

    Comment Here

    Reference: Monomorphic epitheliotropic intestinal T cell lymphoma
    Board review style question #2
    Which of the following is helpful in the diagnosis of monomorphic epitheliotropic intestinal T cell lymphoma (MEITL), rather than enteropathy associated T cell lymphoma (EATL)?

    1. Expression of megakaryocyte associated tyrosine kinase (MATK) is downregulated in MEITL and upregulated in EATL
    2. Gains in chromosome 8q24 involving MYC are seen in a high proportion of cases of MEITL but not EATL
    3. HLA DQ2 or HLA DQ8 alleles are commonly seen in MEITL, not EATL
    4. T cell receptor expression in MEITL is usually alpha / beta more than gamma / delta, while in EATL, gamma / delta expression is more than alpha / beta expression
    5. T cell receptor expression is relatively the same in MEITL and EATL
    Board review style answer #2
    B. Gains in chromosome 8q24 involving MYC are seen in a high proportion of cases of MEITL but not EATL. The cytogenetic and molecular features associated with MEITL include gains of 8q24, overexpression of MATK and expression of TCRδγ. In contrast, EATL is characterized by expression of HLA DQ2 or HLA DQ8 alleles as well as the expression of TCRαΒ (Semin Diagn Pathol 2021 [Epub ahead of print]).

    Comment Here

    Reference: Monomorphic epitheliotropic intestinal T cell lymphoma

    Mycosis fungoides

    Mycosis fungoides variants (pending)
    [Pending]

    NK large granular lymphocytic leukemia
    Definition / general
    • Chronic lymphoproliferative disorders of NK cells (CLPD-NK, provisional entity) are characterized by a persistent (> 6 months) increase in peripheral blood NK cells (usually ≥ 2×109/L) without a clearly identified cause (WHO 2008)
    Terminology
    • Chronic NK cell lymphocytosis, chronic NK-LGL LPD, NK cell lineage granular lymphocyte proliferative disorder, NK cell LGL lymphocytosis, indolent large granular NK cell LPD
    Epidemiology
    • Rare and heterogeneous; difficult to distinguish between reactive and neoplastic without highly specialized techniques
    • Predominantly adults (median age 60 years)
    • No gender predominance, no racial or genetic predisposition
    Etiology
    • Autoimmune disorders or viral infections may be accompanied by a transient increase in circulating NK cells
    • Viral stimulus postulated as an activation factor leading to selection of NK cell clones (no evidence of direct NK cell infection observed so far)
    • May have genetic susceptibility related to haplotypes containing higher numbers of KIR
    Sites
    • Predominantly peripheral blood and bone marrow
    Clinical features
    • Most patients asymptomatic
    • May have systemic symptoms; variable cytopenias (mainly neutropenia and anemia)
    • Rarely lymphadenopathy, hepatosplenomegaly and skin lesions
    • Sometimes associated with solid tumors, hematologic neoplasms, vasculitis, splenectomy, neuropathy and autoimmune disorders
    Case reports
    • 65 year old man with a preexisting, untreated indolent NK cell lymphoproliferative disorder evolving into an aggressive extranodal NK cell lymphoma (Am J Surg Pathol 2005;29:1540)
    Treatment
    Treatment and prognosis:
    • Usually indolent clinical course over a prolonged period; occasional disease progression with increasing lymphocytosis and worsening cytopenias (Br J Haematol 1999;106:960)
    • Cytopenias, recurrent infections, comorbidity and cytogenetic abnormalities may indicate worse prognosis
    • Rare cases of spontaneous regression or transformation to an aggressive NK cell disorder
    Postulated normal counterpart
    • Mature NK cell
    Microscopic (histologic) description
    • Peripheral blood: NK cells of intermediate size with moderate amounts of slightly basophilic cytoplasm and coarse azurophilic granules, round nuclei, condensed chromatin
    • Bone marrow: intrasinusoidal and interstitial infiltration by cells with modest amounts of pale cytoplasm and small, minimally irregular nuclei; difficult to identify without immunostains
    Positive stains
    Negative stains
    Flow cytometry images

    Images hosted on other servers:

    Characteristic immunophenotype

    Immunophenotypic comparison

    Molecular / cytogenetics description
    • Usually normal karyotype with no Ig or TCR gene rearrangements
    • May be able to use X chromosome inactivation to prove clonality in female patients

    Nodal PTCL with a TFH phenotype
    Definition / general
    • Mature T cell lymphoma with a T follicular helper (TFH) phenotype, defined by the expression of CD4 and at least 2 (ideally 3) TFH markers, primarily involving lymph nodes
    • Must not fulfill the diagnostic criteria for other defined mature T cell lymphoma entities, including other nodal TFH lymphomas (Leukemia 2022;36:1720, Blood 2016;127:2375)
    Essential features
    • Aggressive, mature T cell lymphoma
    • Expression of CD4 and at least 2, preferably 3, T follicular helper markers (such as PD-1, CD10, BCL6, CXCL13 and ICOS)
    • Part of a spectrum of T cell lymphomas with TFH phenotype, which also includes nodal TFH cell lymphoma, angioimmunoblastic type (nTFHL-AI), formerly angioimmunoblastic T cell lymphoma and nodal TFH cell lymphoma, follicular type (nTFHL-F), formerly known as follicular T cell lymphoma
    • Other mature T cell lymphomas must be excluded
    • Frequent mutations in RHOA and epigenetic regulators such as TET2 and DNMT3A
    Terminology
    • Current terminology
      • Nodal T follicular helper (TFH) cell lymphoma, not otherwise specified in the 5th edition of the World Health Organization Classification of Hematolymphoid Tumors: Lymphoid Neoplasms (WHO HAEM5) (Leukemia 2022;36:1720)
      • Follicular helper T cell lymphoma, NOS in the International Consensus Classification of Mature Lymphoid Neoplasms (ICC 2022) (Mod Pathol 2022;35:306)
    • Historical names
      • These cases were classified initially within peripheral T cell lymphoma, not otherwise specified (PTCL, NOS) in the 4th edition of the WHO classification (WHO 2008)
      • Posteriorly renamed as nodal PTCL with a TFH phenotype in the revised 4th edition of the WHO classification (WHO 2017), where it was grouped with angioimmunoblastic T cell lymphoma (AITL) and T follicular lymphoma under the provisional umbrella category of nodal lymphomas of TFH origin (Blood 2016;127:2375)
    ICD coding
    • ICD-O: 9702/3 - nodal TFH cell lymphoma, NOS
    • ICD-11: 2A90 & XH6SR1 - mature T cell lymphoma, specified types, nodal or systemic & nodal peripheral T cell lymphoma with T follicular helper phenotype
    Epidemiology
    Sites
    • nTFHL, NOS is characterized by disseminated disease similar to nTFHL-AI (Leukemia 2022;36:1720)
    • Nearly all cases involve lymph nodes
    Pathophysiology
    • Proposed origin from TFH cells (a T cell subset essential for germinal center reactions and B cell specialization)
    • nTFH, NOS has a gene expression profile more comparable to nTFH-AI than PTCL, NOS (Blood 2016;127:2375, Haematologica 2017;102:e148)
    • Similarly, the mutational landscape resembles that of nTFH-AI
    • Physiologically, RHOA mediates migration and polarity of T cells
      • RHOA also functions in thymocyte development and activation of pre-T cell receptor (pre-TCR) signaling in thymocytes
      • G17V RHOA protein is considered to be a loss of function mutant on RhoA signaling pathway
      • Physiologic functions of RHOA are presumably abrogated in the G17V mutant, as the protein cannot be converted to the active GTP bound form
      • Its mutation may drive naive CD4+ T cells to differentiate to TFH cells and facilitate neoplasia
      • TET2 and DNMT3A are considered early mutations in TFH cells (Leukemia 2018;32:694)
    Clinical features
    • Lymphadenopathy with typical advanced stage disease at presentation
    • Similar bone marrow involvement to nTFHL-AI and PTCL, NOS (Blood 2021;137:2161)
    Diagnosis
    • Must not fulfill the required histopathological criteria for nTFHL, angioimmunoblastic type (nTFHL-AI) such as extrafollicular follicular dendritic cell expansion or high endothelial venule hyperplasia or nTFHL, follicular type (nTFHL-F), which displays a follicular growth pattern
    • Desirable clonal TCR gene rearrangement or mutation involving RHOA p.G17V
    • Staging performed according to the Lugano modification of the Ann Arbor staging system (J Clin Oncol 2014;32:3059)
    Laboratory
    • nTFH, NOS has been linked to Coombs positive hemolytic anemia and polyclonal hypergammaglobulinemia but not as strongly as nTFH-AI (Blood 2021;137:2161)
    Prognostic factors
    • Similar overall survival (OS) and progression free survival (PFS) to nTFHL-AI but better than PTCL, NOS, based on limited data (Haematologica 2017;102:e148)
    • Male gender, an Eastern Cooperative Oncology Group (ECOG) performance status of 2 or higher and thrombocytopenia are related to poor OS (Haematologica 2017;102:e148)
    Case reports
    Treatment
    • Cyclophosphamide, doxorubicin, vincristine and prednisone (CHOP) or CHOP-like chemotherapy is used in most patients
    • Ifosfamide, mesna, carboplatin, etoposide (ICE) gemcitabine based chemotherapy is less commonly used
    • Epigenetic modifiers, such as histone deacetylase inhibitors (HDACi) and DNA methyltransferase inhibitors, have shown a better treatment response rate than other PTCLs in the relapsed setting (Blood 2018;132:2305, Blood Adv 2020;4:4640, Am J Surg Pathol 2019;43:1282)
    Microscopic (histologic) description
    • Lymph node architecture is effaced by a tumor cell rich infiltration of varying sized lymphoid cells (typically medium to large sized)
    • 2 recognized patterns: diffuse infiltration or proliferation around follicles in a paracortical T zone infiltration
    • Lack of a prominent polymorphic inflammatory background, high endothelial venule hyperplasia or follicular dendritic cell meshwork proliferation; however, some of these features typical of nTFH-AI may be present (Am J Surg Pathol 2016;40:1249)
    • Secondary B cell proliferations, unlike nTFH-AI, have not been thoroughly studied (Leukemia 2022;36:1720)
    • Lymphoepithelioid (Lennert morphology) may be encountered (Am J Surg Pathol 2007;31:1695)
    Microscopic (histologic) images

    Contributed by Catalina Amador, M.D.
    Effacement of nodal architecture

    Effacement of nodal architecture

    Medium to large sized lymphocytes

    Medium to large sized lymphocytes

    Clear cell morphology

    Clear cell morphology

    Absent expansion of follicular dendritic cell meshworks Absent expansion of follicular dendritic cell meshworks

    CD21 and CD23: absent expansion of follicular dendritic cell meshworks


    CD20 negative for B cell markers

    CD20 negative for B cell markers

    Positive membranous expression of CD4

    Positive membranous expression of CD4

    ICOS membranous positivity

    ICOS membranous positivity

    PD1 membranous positivity

    PD-1 membranous positivity

    BCL6 nuclear positivity

    BCL6 nuclear positivity

    Positive stains
    Negative stains
    Molecular / cytogenetics description
    • Clonal TCR gene rearrangement in most cases
    • Similar gene expression profile to nTFH-AI and normal TFH cells (Blood 2007;109:4952)
    • Mutational profiles of nPTCL-TFH overlap with nTFH-AI, except for IDH2, which is restricted to nTFH-AI
    • Mutations in RHOA
      • Mutations in G17V locus specific to TFH lymphomas
      • Loss of RHOA GTPase activity
    • Up to 60% frequency in nTFHL, NOS, similar to nTFH-AI (Blood 2021;137:2161)
    • Mutations in epigenetic regulators
      • Nonspecific for TFH lymphomas; seen in other T cell lymphomas and hematolymphoid neoplasms
      • TET2
        • Loss of function mutations
        • 50 - 75% frequency in nTFH, NOS (comparable to 75% in nTFHL-F and 50 - 80% in nTFHL-AI)
      • DNMT3A
        • Loss of function mutations
        • 7 - 10% frequency (compared with 25% in nTFHL-F and 20 - 40% in nTFHL-AI)
    • Mutations in TCR signaling pathway genes
      • Not specific, as they are also seen in other T cell lymphomas
      • PLC gamma: up to 10% and FYN < 5%
      • CD28 is absent as it seems restricted to nTFHL-AI (Blood Adv 2020;4:4640)
    • Cytogenetic abnormalities
      • Copy number gains in chromosomes 5 and 7 occur with a similar frequency to nTFH-AI (~10%) (Blood 2021;137:2161)
    • Negative serologic or molecular evidence of HTLV-1
    Sample pathology report
    • Right inguinal lymph node, excisional biopsy:
      • Lymph node with nodal T follicular helper cell lymphoma, not otherwise specified (WHO HAEM5) / follicular helper T cell lymphoma, not otherwise specified (see comment)
      • Comment: The overall morphological features, including the expression of CD4 and 3 T follicular helper cell markers (ICOS, PD-1 and BCL6) by the neoplastic cells, support the diagnosis of a TFH lymphoma. The diagnosis of nTFH, NOS is favored over angioimmunoblastic T cell lymphoma due to the lack of polymorphism, expanded follicular dendritic cell meshworks and high endothelial venules. Given the patient's origin, adult T cell leukemia / lymphoma was also considered, given that these cases can express TFH markers. However, the serology study for HTLV-1 yielded negative results, excluding this possibility. T cell receptor gamma gene rearrangement studies were positive and the next generation sequencing panel revealed mutations in RHOA, TET2 and DNMT3, supporting the morphologic findings.
      • Microscopic description: The biopsy shows sections of lymph node with effaced architecture by an abnormal lymphoid infiltrate with a diffuse growth pattern. The neoplastic cells are composed of a spectrum of small, intermediate and large lymphoid cells with minimally associated inflammatory background. Immunohistochemistry shows that the neoplastic cells express CD3 and CD4, PD-1, BCL6 and ICOS and are negative for CD20, CD8, CD10, ALK and EBER in situ hybridization. CD30 is positive in 30% of the neoplastic cells. CD21 highlights the absence of expanded follicular dendritic cell meshworks. Ki67 proliferation is 50%. EBER in situ hybridization is negative.
    Differential diagnosis
    Board review style question #1


    Which option describes characteristic features of nodal peripheral T cell lymphomas with T follicular helper phenotype?

    1. Increased expression of CD21 and CD23
    2. MYC and BCL2 gene rearrangements
    3. Polymorphic inflammatory background with expanded follicular dendritic cell meshwork
    4. Predominance in the young population
    5. Proliferation around follicles in a T zone infiltration pattern of the paracortex
    Board review style answer #1
    E. Proliferation around follicles in a T zone infiltration pattern of the paracortex. nTFHL, NOS characteristically presents with 2 recognized patterns: diffuse infiltration or proliferation around follicles in a paracortical T zone infiltration. Answer A is incorrect because CD21, CD23, CD35 or clusterin do not highlight expanded follicular dendritic cell meshworks. Answer B is incorrect because MYC and BCL2 gene rearrangements are characteristic of double hit high grade B cell lymphomas. Answer D is incorrect because this entity predominates in the elder population. Answer C is incorrect because this finding is characteristic of nodal TFH cell lymphoma, angioimmunoblastic type (nTFHL-AI).

    Comment Here

    Reference: Nodal PTCL with a TFH phenotype
    Board review style question #2
    Which option describes a typical feature of nodal peripheral T cell lymphomas with T follicular helper phenotype?

    1. Expression of CD8 and TdT
    2. Formation of well defined aggregates surrounded by numerous small IgD+ mantle zone B cells arranged into large irregular nodules
    3. Indolent progression with a low incidence of disseminated disease
    4. Lack of TCR gene rearrangements
    5. RHOA and TET2 gene mutations
    Board review style answer #2
    E. RHOA and TET2 gene mutations. Detection of RHOA p.G17V mutation is observed in up to 60% of nTFHL, NOS and is a useful adjunct, as it supports the diagnosis of a nTFHL over PTCL, NOS. TET2 loss of function mutations have a 50 - 75% frequency in nTFH, NOS, comparable to 75% in nTFHL-F and 50 - 80% in nTFHL-AI. Answer A is incorrect because CD8 and TdT are mature T cell lymphomas, hence TdT is characteristically negative. CD8 is typically not expressed in these tumors but the expression does not exclude diagnosis. Answer B is incorrect because these findings describe nodal TFH cell lymphoma, follicular type (nTFHL-F). Answer C is incorrect because these tumors typically present as advanced disease. Answer D is incorrect because clonal TCR gene rearrangement is detected in most cases.

    Comment Here

    Reference: Nodal PTCL with a TFH phenotype

    Nodular lymphocyte predominant B cell lymphoma / nodular lymphocyte predominant Hodgkin lymphoma
    Definition / general
    Essential features
    • LP cells initially arise within altered follicles
    • Background small B cells are numerous in early phases and decrease with time
    • LP cells rosetted by CD4+, PD-1+ T cells
    • Number of background T cells increases over time, along with the loss of the nodular pattern
    • Diffuse pattern resembles T cell / histiocyte rich large B cell lymphoma
    • LP cells display a germinal center B cell phenotype: CD20, PAX5, OCT2 and BCL6 are positive, negative for CD10
    • CD30 is variably positive and if positive, usually weak
    • Most cases negative for CD15
    Terminology
    • Hodgkin paragranuloma (obsolete)
    ICD coding
    • ICD-10: C81.0 - nodular lymphocyte predominant Hodgkin lymphoma
    • ICD-10: C81.00 - nodular lymphocyte predominant Hodgkin lymphoma, unspecified site
    • ICD-10: C81.01 - nodular lymphocyte predominant Hodgkin lymphoma, lymph nodes of head, face and neck
    • ICD-10: C81.02 - nodular lymphocyte predominant Hodgkin lymphoma, intrathoracic lymph nodes
    • ICD-10: C81.03 - nodular lymphocyte predominant Hodgkin lymphoma, intra-abdominal lymph nodes
    • ICD-10: C81.04 - nodular lymphocyte predominant Hodgkin lymphoma, lymph nodes of axilla and upper limb
    • ICD-10: C81.05 - nodular lymphocyte predominant Hodgkin lymphoma, lymph nodes of inguinal region and lower limb
    • ICD-10: C81.06 - nodular lymphocyte predominant Hodgkin lymphoma, intrapelvic lymph nodes
    • ICD-10: C81.07 - nodular lymphocyte predominant Hodgkin lymphoma, spleen
    • ICD-10: C81.08 - nodular lymphocyte predominant Hodgkin lymphoma, lymph nodes of multiple sites
    • ICD-10: C81.09 - nodular lymphocyte predominant Hodgkin lymphoma, extranodal and solid organ sites
    • ICD-10: C81 - Hodgkin lymphoma
    Epidemiology
    Sites
    Pathophysiology
    Etiology
    Clinical features
    • Usually presents as localized disease; usually stage I / II (70 - 80%); 20% may present with advanced stage disease (Oncologist 2009;14:739)
    • Advanced stage disease treated as an aggressive B cell lymphoma (Blood 2017;130:472)
    • Presents with peripheral lymphadenopathy; rarely mediastinal or bone marrow involvement; often long history of localized disease (Am J Surg Pathol 2004;28:489)
    • M > F
    • Wide age range; may present in children
    • B symptoms are uncommon (5 - 10%)
    • Late recurrences more common than in classic Hodgkin lymphoma (Cancer 2010;116:631)
    Diagnosis
    • Biopsy of the involved lymph node with immunohistochemistry
    • LP cells in a background of small lymphocytes admixed with histiocytes
    • LP cells: CD45+, CD20+, PAX5+ OCT+, BCL6+, CD15-, CD30 variable; Subset positive for IgD, mainly in young males
    • Background: small B cells and T cells positive for CD4, PD-1 and partial CD57
    • Variant histological patterns have prognostic relevance (Am J Surg Pathol 2003;27:1346)
    • Expanded follicular dendritic cell meshwork within nodular areas; follicular dendritic cell meshwork lost with diffuse growth
    • With progression to diffuse growth pattern, histology resembles T cell / histiocyte rich large B cell lymphoma
    Prognostic factors
    Case reports
    Treatment
    Gross description
    • Involved lymph node is enlarged
    • Firm, fleshy mass
    Microscopic (histologic) description
    • Usually total replacement of nodal architecture by expansive vague nodules of small lymphocytes with sparse, relatively large tumor cells with multilobulated or round nucleus, thin nuclear membrane, finely granular chromatin and variable small nucleoli (popcorn cells) (Semin Hematol 2016;53:190, Clin Lymphoma Myeloma 2009;9:206)
    • May have rim of normal lymph node
    • Large cells are called LP cells; previously known as lymphocytic and histiocytic cells
    • LP cells are admixed with numerous small B lymphocytes, epithelioid histiocytes and CD21+ follicular dendritic cells
    • Often PD-1+, T cell rosettes surrounding LP cells (Am J Surg Pathol 2008;32:1252, Blood 2017;129:802)
    • Scant eosinophils, plasma cells or fibrosis
    • Prominent sclerosis is unusual but may occur in older lesions
    • Small germinal centers are rare between the nodules and usually present only at the compressed edge of normal lymph node tissue
    • No / rare classic Reed-Sternberg (RS) cells, although LP cells may occasionally resemble Reed-Sternberg cells / variants
    • No well formed granulomas, although epithelioid histiocytes may be prominent
    • Diffuse patterns has background of reactive T cells
    • 6 immunoarchitectural patterns have been described (Am J Surg Pathol 2003;27:1346):
      • Typical - B cell rich nodular
      • Serpiginous interconnected B cell rich
      • Variant - extranodular LP cells
      • T cell rich nodular
      • Diffuse T cell rich
      • Diffuse B cell rich
    • Variant patterns are associated with disease recurrence and transformation (Br J Haematol 2018;181:403, Br J Haematol 2014;167:238)
    • Nodular lymphocyte predominant Hodgkin lymphoma and T cell histiocyte rich large B cell lymphoma are considered to be parts of the same disease spectrum (PLoS One 2013;8:e78812, Br J Haematol 2015;169:415)
    • Syncytial variant is rarely reported (Am J Surg Pathol 2009;33:1725)
    • Bone marrow involvement: 3% of cases; involvement by large B cells (< 10% of all cases) in background of T cells and histiocytes
    Microscopic (histologic) images

    Contributed by Elaine S. Jaffe, M.D., Jayalakshmi Balakrishna, M.D. and Lauren B. Smith, M.D.

    Vaguely nodular architecture

    Rare large neoplastic cells

    LP cells or popcorn cells

    Neoplastic cells positive for CD20

    T cell rich variant with rare small B cells in the background


    Neoplastic cells positive for CD20

    OCT2 stain is helpful in highlighting the neoplastic cells

    Neoplastic cells positive for PAX5

    Neoplastic cells positive for BCL6

    IgD expressing LP cells


    T cell rosettes around LP cells

    CD4 T cells predominate in the background

    T cells express PD-1

    PD-1 positive T cells form rosettes around LP cells

    Expanded follicular dendritic cell meshwork


    Effaced lymph node architecture

    Lymphocyte predominant cell (LP cell / popcorn cell)

    LP cells express CD20

    Positive stains
    Negative stains
    Molecular / cytogenetics description
    Sample pathology report
    • Lymph node, right cervical, excision:
      • Nodular lymphocyte predominant Hodgkin lymphoma (see comment and synoptic report)
    Differential diagnosis
    Board review style question #1

    A 20 year old man presents with an enlarged painless cervical lymph node he has had for the past 6 months. An excisional biopsy is shown in the image. The neoplastic cells are positive for CD45 and CD20 and are negative for CD15 and CD30. Which of the following is true about this entity?

    1. Background is rich in cytotoxic T cells
    2. Background lacks small B cells
    3. Neoplastic cells are OCT2 positive
    4. Neoplastic cells are PAX5 negative
    Board review style answer #1
    C. The neoplastic cells are OCT2 positive

    Comment here

    Reference: Nodular lymphocyte predominant Hodgkin lymphoma
    Board review style question #2
    What is the immunophenotype of T cells in the microenvironment of nodular lymphocyte predominant Hodgkin lymphoma?

    1. CD3+ / CD4+ / PD-1+
    2. CD3+ / CD8+ / CD57+
    3. CD3+ / CD8+ / CD57-
    4. CD3+ / CD8+ / PD-1+
    Board review style answer #2

    Nodular lymphocyte predominant B cell lymphoma / nodular lymphocyte predominant Hodgkin lymphoma
    Definition / general
    Essential features
    • LP cells initially arise within altered follicles
    • Background small B cells are numerous in early phases and decrease with time
    • LP cells rosetted by CD4+, PD-1+ T cells
    • Number of background T cells increases over time, along with the loss of the nodular pattern
    • Diffuse pattern resembles T cell / histiocyte rich large B cell lymphoma
    • LP cells display a germinal center B cell phenotype: CD20, PAX5, OCT2 and BCL6 are positive, negative for CD10
    • CD30 is variably positive and if positive, usually weak
    • Most cases negative for CD15
    Terminology
    • Hodgkin paragranuloma (obsolete)
    ICD coding
    • ICD-10: C81.0 - nodular lymphocyte predominant Hodgkin lymphoma
    • ICD-10: C81.00 - nodular lymphocyte predominant Hodgkin lymphoma, unspecified site
    • ICD-10: C81.01 - nodular lymphocyte predominant Hodgkin lymphoma, lymph nodes of head, face and neck
    • ICD-10: C81.02 - nodular lymphocyte predominant Hodgkin lymphoma, intrathoracic lymph nodes
    • ICD-10: C81.03 - nodular lymphocyte predominant Hodgkin lymphoma, intra-abdominal lymph nodes
    • ICD-10: C81.04 - nodular lymphocyte predominant Hodgkin lymphoma, lymph nodes of axilla and upper limb
    • ICD-10: C81.05 - nodular lymphocyte predominant Hodgkin lymphoma, lymph nodes of inguinal region and lower limb
    • ICD-10: C81.06 - nodular lymphocyte predominant Hodgkin lymphoma, intrapelvic lymph nodes
    • ICD-10: C81.07 - nodular lymphocyte predominant Hodgkin lymphoma, spleen
    • ICD-10: C81.08 - nodular lymphocyte predominant Hodgkin lymphoma, lymph nodes of multiple sites
    • ICD-10: C81.09 - nodular lymphocyte predominant Hodgkin lymphoma, extranodal and solid organ sites
    • ICD-10: C81 - Hodgkin lymphoma
    Epidemiology
    Sites
    Pathophysiology
    Etiology
    Clinical features
    • Usually presents as localized disease; usually stage I / II (70 - 80%); 20% may present with advanced stage disease (Oncologist 2009;14:739)
    • Advanced stage disease treated as an aggressive B cell lymphoma (Blood 2017;130:472)
    • Presents with peripheral lymphadenopathy; rarely mediastinal or bone marrow involvement; often long history of localized disease (Am J Surg Pathol 2004;28:489)
    • M > F
    • Wide age range; may present in children
    • B symptoms are uncommon (5 - 10%)
    • Late recurrences more common than in classic Hodgkin lymphoma (Cancer 2010;116:631)
    Diagnosis
    • Biopsy of the involved lymph node with immunohistochemistry
    • LP cells in a background of small lymphocytes admixed with histiocytes
    • LP cells: CD45+, CD20+, PAX5+ OCT+, BCL6+, CD15-, CD30 variable; Subset positive for IgD, mainly in young males
    • Background: small B cells and T cells positive for CD4, PD-1 and partial CD57
    • Variant histological patterns have prognostic relevance (Am J Surg Pathol 2003;27:1346)
    • Expanded follicular dendritic cell meshwork within nodular areas; follicular dendritic cell meshwork lost with diffuse growth
    • With progression to diffuse growth pattern, histology resembles T cell / histiocyte rich large B cell lymphoma
    Prognostic factors
    Case reports
    Treatment
    Gross description
    • Involved lymph node is enlarged
    • Firm, fleshy mass
    Microscopic (histologic) description
    • Usually total replacement of nodal architecture by expansive vague nodules of small lymphocytes with sparse, relatively large tumor cells with multilobulated or round nucleus, thin nuclear membrane, finely granular chromatin and variable small nucleoli (popcorn cells) (Semin Hematol 2016;53:190, Clin Lymphoma Myeloma 2009;9:206)
    • May have rim of normal lymph node
    • Large cells are called LP cells; previously known as lymphocytic and histiocytic cells
    • LP cells are admixed with numerous small B lymphocytes, epithelioid histiocytes and CD21+ follicular dendritic cells
    • Often PD-1+, T cell rosettes surrounding LP cells (Am J Surg Pathol 2008;32:1252, Blood 2017;129:802)
    • Scant eosinophils, plasma cells or fibrosis
    • Prominent sclerosis is unusual but may occur in older lesions
    • Small germinal centers are rare between the nodules and usually present only at the compressed edge of normal lymph node tissue
    • No / rare classic Reed-Sternberg (RS) cells, although LP cells may occasionally resemble Reed-Sternberg cells / variants
    • No well formed granulomas, although epithelioid histiocytes may be prominent
    • Diffuse patterns has background of reactive T cells
    • 6 immunoarchitectural patterns have been described (Am J Surg Pathol 2003;27:1346):
      • Typical - B cell rich nodular
      • Serpiginous interconnected B cell rich
      • Variant - extranodular LP cells
      • T cell rich nodular
      • Diffuse T cell rich
      • Diffuse B cell rich
    • Variant patterns are associated with disease recurrence and transformation (Br J Haematol 2018;181:403, Br J Haematol 2014;167:238)
    • Nodular lymphocyte predominant Hodgkin lymphoma and T cell histiocyte rich large B cell lymphoma are considered to be parts of the same disease spectrum (PLoS One 2013;8:e78812, Br J Haematol 2015;169:415)
    • Syncytial variant is rarely reported (Am J Surg Pathol 2009;33:1725)
    • Bone marrow involvement: 3% of cases; involvement by large B cells (< 10% of all cases) in background of T cells and histiocytes
    Microscopic (histologic) images

    Contributed by Elaine S. Jaffe, M.D., Jayalakshmi Balakrishna, M.D. and Lauren B. Smith, M.D.

    Vaguely nodular architecture

    Rare large neoplastic cells

    LP cells or popcorn cells

    Neoplastic cells positive for CD20

    T cell rich variant with rare small B cells in the background


    Neoplastic cells positive for CD20

    OCT2 stain is helpful in highlighting the neoplastic cells

    Neoplastic cells positive for PAX5

    Neoplastic cells positive for BCL6

    IgD expressing LP cells


    T cell rosettes around LP cells

    CD4 T cells predominate in the background

    T cells express PD-1

    PD-1 positive T cells form rosettes around LP cells

    Expanded follicular dendritic cell meshwork


    Effaced lymph node architecture

    Lymphocyte predominant cell (LP cell / popcorn cell)

    LP cells express CD20

    Positive stains
    Negative stains
    Molecular / cytogenetics description
    Sample pathology report
    • Lymph node, right cervical, excision:
      • Nodular lymphocyte predominant Hodgkin lymphoma (see comment and synoptic report)
    Differential diagnosis
    Board review style question #1

    A 20 year old man presents with an enlarged painless cervical lymph node he has had for the past 6 months. An excisional biopsy is shown in the image. The neoplastic cells are positive for CD45 and CD20 and are negative for CD15 and CD30. Which of the following is true about this entity?

    1. Background is rich in cytotoxic T cells
    2. Background lacks small B cells
    3. Neoplastic cells are OCT2 positive
    4. Neoplastic cells are PAX5 negative
    Board review style answer #1
    C. The neoplastic cells are OCT2 positive

    Comment here

    Reference: Nodular lymphocyte predominant Hodgkin lymphoma
    Board review style question #2
    What is the immunophenotype of T cells in the microenvironment of nodular lymphocyte predominant Hodgkin lymphoma?

    1. CD3+ / CD4+ / PD-1+
    2. CD3+ / CD8+ / CD57+
    3. CD3+ / CD8+ / CD57-
    4. CD3+ / CD8+ / PD-1+
    Board review style answer #2

    PTLD-classic Hodgkin
    Definition / general
    • Lymphoproliferation that fulfills the diagnostic criteria for classic Hodgkin lymphoma and arises in a patient after solid organ or hematopoietic stem cell transplant
    • Classification is unaltered in the 2022 International Consensus Classification of Mature Lymphoid Neoplasms (Blood 2022;140:1229)
    • In the 5th edition of the World Health Organization Classification of Hematolymphoid Tumors (WHO HAEM), the entity is classified based on a 3 part nomenclature for lymphomas that arise in the setting of immune deficiency / dysregulation (Leukemia 2022;36:1720)
    Essential features
    • Fulfills the diagnostic criteria for classic Hodgkin lymphoma
    • Arises in a patient with a history of solid organ or allogeneic hematopoietic stem cell transplant
    • Almost always Epstein-Barr virus (EBV)+
    • Very rare; < 5% of all posttransplant lymphoproliferative disorders (PTLD)
    ICD coding
    • ICD-O: 9650/3 - Hodgkin lymphoma, NOS
    Epidemiology
    Sites
    Pathophysiology
    Etiology
    Clinical features
    • Can include lymphadenopathy, splenomegaly, fever, weight loss, night sweats or pancytopenia
    • Patients have a history of solid organ or hematopoietic cell transplant
    Diagnosis
    • Diagnosis is made on lymph node biopsy (most common), with biopsy of extranodal tissue (less common) and on bone marrow biopsy (rarely)
    Prognostic factors
    • There are an insufficient number of cases to draw meaningful conclusions
    Case reports
    Treatment
    Microscopic (histologic) description
    • Resembles mixed cellularity subtype of classic Hodgkin lymphoma
    • Effaced architecture or interfollicular pattern
    • Variable interstitial fibrosis; less commonly with broad bands of sclerosis or capsular thickening
    • Typical Hodgkin Reed-Sternberg cells are seen scattered in a mixed inflammatory background of eosinophils, histiocytes, lymphocytes and plasma cells
      • Hodgkin Reed-Sternberg cells are large cells with irregular nuclear contours (occasionally bilobed) and frequently with prominent nucleoli
    Microscopic (histologic) images

    Contributed by Cade Arries, M.D.

    Bone marrow

    Bone marrow, CD30

    Bone marrow, CD15

    Bone marrow, EBER


    Lymph node

    Lymph node


    Lymph node, CD15

    Lymph node, CD30

    Lymph node, PAX5

    Lymph node, EBER

    Positive stains
    Flow cytometry description
    Molecular / cytogenetics description
    • Recurrent cytogenetic / oncogene abnormalities are unknown at this time
    Sample pathology report
    • Lymph node, cervical, excisional biopsy:
      • Classic Hodgkin lymphoma posttransplant lymphoproliferative disorder, EBV positive (see comment)
      • Comment: The morphologic and immunohistochemical findings fulfill the criteria for classic Hodgkin lymphoma. In this patient with a history of solid organ transplant, the findings are compatible with classic Hodgkin lymphoma posttransplant lymphoproliferative disorder. The Reed-Sternberg cells / variants are EBV+.
    Differential diagnosis
    Board review style question #1

    A 45 year old man with a history of renal transplant presented with lymphadenopathy. A lymph node biopsy was performed. A representative image is shown. The large atypical cells are positive for CD30 and CD15 with dim PAX5 nuclear positivity and dim and variable CD20 expression. CD3 and CD45 are negative. What process is suggested by these morphologic and immunohistochemical findings?

    1. Classic Hodgkin lymphoma
    2. Diffuse large B cell lymphoma
    3. Florid follicular hyperplasia
    4. Polymorphic proliferation
    Board review style answer #1
    A. Classic Hodgkin lymphoma. The morphologic and immunohistochemical features are diagnostic of classic Hodgkin lymphoma. This is a rare disorder in the posttransplant setting and almost always shows EBV positivity by EBER in situ hybridization in the Reed-Sternberg cells / variants. In the context of solid organ transplantation, the posttransplant status and the presence or absence of viral association is noted in the interpretation. The exact nomenclature depends on the classification system used.

    Comment Here

    Reference: PTLD-classic Hodgkin
    Board review style question #2
    Which of the following is expected to be positive in the Hodgkin / Reed-Sternberg cells of classic Hodgkin lymphoma type posttransplant lymphoproliferative disorder (PTLD)?

    1. ALK1
    2. CD3
    3. CD45
    4. EBER in situ hybridization
    Board review style answer #2
    D. EBER in situ hybridization. The Hodgkin / Reed-Sternberg cells of classic Hodgkin lymphoma type posttransplant lymphoproliferative disorder are almost always EBV+ by EBER in situ hybridization. The other stains listed are negative.

    Comment Here

    Reference: PTLD-classic Hodgkin

    PTLD-polymorphic
    Definition / general
    • Polymorphic posttransplant lymphoproliferative disorders (P-PTLDs) are a heterogenous group of hematolymphoid proliferations composed of a polymorphic population of immunoblasts, plasma cells and small to intermediate sized lymphoid cells that efface normal tissue architecture and do not meet criteria for the diagnosis of any well established form of lymphoma recognized in immunocompetent hosts (Blood 2016;127:2375)
    Essential features
    • Occurs in the posttransplant setting
    • Effacement of normal tissue architecture
    • Presence of a polymorphic lymphoid infiltrate with a complete spectrum of maturation and variable number of transformed cells / immunoblasts
    • Variable resolution following the cessation of immunosuppressive therapy
    • Frequently EBV related (Lancet 2006;367:233, Jaffe: Hematopathology, 2nd Edition, 2017)
    Terminology
    • Polymorphic posttransplant lymphoproliferative disorder (P-PTLD)
    ICD coding
    • ICD-O: 9971/1 - posttransplant lymphoproliferative disorder, NOS
    Epidemiology
    • PTLDs develop in approximately 2% of all transplant patients and are more common in liver, lung, intestinal and multivisceral transplant recipients
    • P-PTLD accounts for the minority of PTLDs in older individuals
    • P-PTLD is a much more common form of PTLD in children, related to primary EBV infection
    • Occurs across a wide age range without sex predilection (Lancet 2006;367:233, Jaffe: Hematopathology, 2nd Edition, 2017)
    Sites
    • Can occur nearly anywhere throughout the body, including both nodal and extranodal sites and within the graft or in native tissue
    • Most common extranodal sites are the gastrointestinal tract, lung and liver
    Etiology
    • Majority of PTLDs following solid organ transplantation are of host origin, while those following bone marrow transplant are more often derived from the donor
    • P-PTLDs are caused by expansion of clonal lymphoid or plasma cells, which are frequently infected by EBV and inadequately controlled by the immune system of an immunosuppressed host
    • Most P-PTLDs demonstrate a type III latency pattern, with fewer demonstrating a type II latency pattern and still fewer demonstrating a type I latency pattern
    • Genetic predisposition likely also plays a role in the risk for development of PTLD, as evidenced by increased and early onset disease in patients with certain interferon gamma and human leukocyte antigen (HLA) polymorphisms
    • PTLDs are thought to exist on a spectrum beginning with an expanded EBV infected polyclonal population that develops increasingly pure clonal populations, resulting in the development of P-PTLD and monomorphic PTLD (World J Transplant 2016;6:505, Cancers (Basel) 2020;12:328)
    Clinical features
    • Variable frequency but generally considered to comprise the minority of PTLDs in adults
    • More common in children, often following primary EBV infection
    • Historically, 80% of PTLDs were reported to occur within the first year; however, recent studies suggest an interval of several years on average
    • May present as multiple mass-like lesions with symptomatology related to site of involvement (i.e. obstructive symptoms with an intestinal mass)
    • Reduction in immunosuppression leads to regression in a variable number of cases; some cases progress and warrant treatment as overt lymphoma (Cancer Treat Res 2015;165:305)
    Prognostic factors
    • Good response to reduced immunosuppression is associated with superior outcome
    • PTLD development following solid organ transplant generally fares better than after bone marrow transplant
    • PTLD limited to the allograft is associated with better outcome
    • CNS involvement, multiorgan involvement and organ dysfunction predict poor response to reduced immunosuppression
    • Other supposed prognostic factors in P-PTLD are a subject of debate:
      • Cases with demonstrable clonality are purported by some to be more resistant to reduction of immunosuppression
      • Absence of EBV appears to be associated with a more aggressive clinical course according to some studies (Exp Hematol Oncol 2017;6:26)
    Case reports
    Treatment
    Gross description
    • These lesions can have a nearly normal gross appearance or appear as a mass forming lesion with variable cut surface, depending on the histologic composition of the proliferation
    Frozen section description
    • Outright diagnosis of P-PTLD should not be made on frozen section examination
    • Histologic appearance will be similar to that described for permanent sections below, although lymphocyte atypia can often be more striking in frozen sections
    • Better to describe an atypical lymphoid infiltrate and defer to permanent sections for diagnosis
    • In this setting, tissue can be triaged for flow cytometric studies (Jaffe: Hematopathology, 2nd Edition, 2017)
    Microscopic (histologic) description
    • Distinction of P-PTLD from nondestructive lesions hinges on the presence of effacement of normal tissue architecture
    • Unlike most lymphomas, there is characteristically a full range of B cell maturation, including immunoblasts, plasma cells and small to intermediate sized lymphoid cells
    • Necrosis, cells resembling Hodgkin or Reed-Sternberg cells, and numerous mitoses may be present
    • Relatively more monomorphic areas may be present, suggesting a continuum between P-PTLD and monomorphic PTLD (Jaffe: Hematopathology, 2nd Edition, 2017)
    Microscopic (histologic) images

    Contributed by Daniel Cassidy, M.D. and Jennifer Chapman, M.D.
    Architectural distortion

    Architectural distortion

    Polymorphic lymphoid infiltrate

    Polymorphic lymphoid infiltrate

    Full spectrum of maturation

    Full spectrum of maturation

    Composition of lymphoid infiltrate

    Composition of lymphoid infiltrate

    Polymorphic infiltrate

    Polymorphic infiltrate


    Variably sized B cells

    Variably sized B cells

    PAX5 variable cell size

    PAX5 variable cell size

    Variably sized T cells

    Variably sized T cells

    Scattered immunoblasts

    Scattered immunoblasts

    EBER positivity

    EBER positivity

    Cytology description
    • Heterogeneous population of cells including small to intermediate sized lymphoid cells, which may have irregular nuclear contours, plasma cells and a variable number of immunoblasts with more open chromatin and prominent, centrally located nucleoli
    • Variable amount of necrotic debris and large cells with features of Hodgkin or Reed-Sternberg cells can be seen
    • Reference: Diagn Cytopathol 1997;16:489
    Immunohistochemistry & special stains
    • Admixture of B cells (expressing B cell markers including CD19, CD20, PAX5, etc.) and T cells (expressing T cell markers including CD2, CD3, CD5, CD7, etc.)
    • Light chain restriction may be present and does not exclude this diagnosis, although it should be noted if present
    • Variable or even strong CD30 expression can be seen; however, these CD30+ large cells are CD20 positive and CD15 negative, unlike classic Hodgkin cells
    • Most cases are positive for EBER by in situ hybridization
    • Ki67 nuclear proliferation index may be brisk (Jaffe: Hematopathology, 2nd Edition, 2017)
    Flow cytometry description
    • In many cases, a demonstrable monotypic B cell or plasma cell population is present
    Molecular / cytogenetics description
    • Clonally rearranged IGH / IGK / IGL genes frequent, albeit less prominent than in monomorphic PTLD cases
    • EBV terminal repeat analysis - most sensitive method for clonal populations in EBV+ cases
    • Tumors at different sites may be either clonally distinct or related
    • No significant T cell clone should be present
    • Cytogenetic studies may demonstrate abnormalities, such as trisomy 9 or trisomy 11, in some cases
    • BCL6 mutations occur in up to half of P-PTLD
    • P-PTLD does not segregate from nongerminal center monomorphic PTLD by gene expression profiling (Am J Clin Pathol 1994;101:590, Genes Chromosomes Cancer 2006;45:313, Hematol Oncol 2008;26:199)
    Sample pathology report
    • Left colon, biopsy:
      • EBV positive posttransplant lymphoproliferative disorder, favor polymorphic type (see comment)
      • Comment: This patient is a 6 year old female status post orthotopic liver transplant, now with nodularities identified in the colon. Biopsies show colonic mucosa within which the lamina propria is abnormally expanded by a heterogeneous population of hematolymphoid cells, including small and large, transformed lymphoid cells, plasma cells, neutrophils, immunoblasts and eosinophils. Histologic features of graft versus host disease are not seen.
      • By immunohistochemistry, CD20 identifies scattered and loosely aggregated B cells, which are heterogeneous in appearance, ranging from small to large. Large (transformed) CD20 positive B cells are few and are not present in sheets. CD3 immunostain confirms the presence of an associated population of small T cells. BCL6 is positive in scattered large B cells. Plasma cells are few and appear polytypic based on kappa and lambda in situ hybridization. EBER is detected in the scattered large and transformed lymphoid cells.
      • The above histologic and immunophenotypic features support the presence of a posttransplant lymphoproliferative disorder. Based on the architectural distortion observed in the biopsy, we would classify this PTLD as polymorphic type. We have also considered the possibility of an infectious mononucleosis type PTLD; however, we do not favor this interpretation due to the presence of architectural distortion within the colonic mucosa and lack of diffuse EBER expression, the latter being seen more frequently in infectious mononucleosis type PTLD.
    Differential diagnosis
    • Reactive inflammatory infiltrate:
      • While the distinction is blurred in patients with a history of transplant, EBER negativity should raise question in regard to the diagnosis of P-PTLD
      • EBER ISH positivity or the presence of a mass forming lesion often favor a diagnosis of P-PTLD
    • Nondestructive posttransplant lymphoproliferative disorder - infectious mononucleosis-like type:
      • Differential diagnosis, particularly at tonsillar site
      • IM-like PTLD usually should not have overt architectural effacement
        • At tonsillar site, florid IM-like PTLD may be indistinguishable from P-PTLD
        • At this site, as long as changes are in keeping with diagnosis of IM in immunocompetent host, a diagnosis of IM-PTLD is favored
    • Classic Hodgkin lymphoma posttransplant lymphoproliferative disorder:
      • Differential diagnosis given the presence of large, sometimes Hodgkin-like cells in P-PTLD
      • Most effective way to make distinction is an abnormal immunophenotype of HL cells, including CD15+, CD30+ and CD45-
        • A variable number of cases may be CD15-, however, further complicating differential diagnosis with P-PTLD
      • Of note, CHL-PTLD Hodgkin-like cells more commonly express B cell markers than CHL
      • Background of CHL-PTLD is more sparse in B cells and rich in T cells than P-PTLD tends to be
      • In CHL, Hodgkin cells are histologically and immunophenotypically distinct from background lymphoid cells
      • In P-PTLD, there is a spectrum of B cells, ranging from small to immunoblastic and Hodgkin-like, which may show EBV expression, in contrast to EBV being largely restricted to the Hodgkin Reed-Sternberg cells in CHL or CHL-PTLD
    • Monomorphic posttransplant lymphoproliferative disorder:
    Board review style question #1
    Which of the following features would be most helpful in favoring a diagnosis of polymorphic posttransplant lymphoproliferative disorder over infectious mononucleosis-like posttransplant lymphoproliferative disorder?

    1. Architectural effacement
    2. CD30 positive large B cells
    3. EBER ISH positivity
    4. Hodgkin-like cells
    Board review style answer #1
    A. Architectural effacement. Architectural effacement is one of the key distinguishing histologic features between P-PTLD and IM-PTLD, as other morphologic features tend to overlap significantly. Of note, the distinction between P-PTLD and IM-PTLD at tonsillar sites can be exceptionally difficult, if not impossible, on morphologic grounds alone, as florid IM at this site can cause architectural distortion. While a prominent clonal B cell population favor a diagnosis of P-PTLD in this setting, small clonal peaks have been described in IM-PTLD of unknown significance.

    Comment Here

    Reference: PTLD-polymorphic
    Board review style question #2

    A 73 year old man status post renal transplant 3 years prior for end stage renal disease secondary to chronic, uncontrolled hypertension presents with an ulcerated lesion involving the oral mucosa. Histologic sections of the lesion are shown above. The large cells are positive for CD30, CD20, CD45 and EBER ISH and negative for CD15. Which of the following is the best diagnosis according to the current WHO classification schema?

    1. Classic Hodgkin lymphoma posttransplant lymphoproliferative disorder
    2. EBV positive mucocutaneous ulcer
    3. Monomorphic posttransplant lymphoproliferative disorder
    4. Polymorphic posttransplant lymphoproliferative disorder
    5. Reactive immunoblastic proliferation
    Board review style answer #2
    B. EBV positive mucocutaneous ulcer. Based on the current WHO classification, PTLDs with clinical and morphologic features consistent with EBV MCU should be classified as such.

    Comment Here

    Reference: PTLD-polymorphic

    Pediatric nodal marginal zone lymphoma
    Definition / general
    • Rare pediatric non-Hodgkin lymphoma, with characteristic morphologic and clinical features and a typically excellent prognosis
    Essential features
    • Rare pediatric nodal lymphoma, more common in males
    • Typically localized to head and neck lymph nodes with excellent prognosis
    • Histologically, expanded marginal zones and interfollicular areas with expanded follicles
    • Polymorphous proliferation of monocytoid, centrocyte-like and plasmacytoid cells
    ICD coding
    • ICD-O: 9699/3 - marginal zone B cell lymphoma, NOS
    Epidemiology
    Sites
    • Majority of patients have localized (stage I) head and neck lymphadenopathy
    • Inguinal / femoral lymph nodes involved in up to 20% of cases (Adv Anat Pathol 2017;24:128)
    Pathophysiology
    Etiology
    • Unknown
    Clinical features
    Diagnosis
    Laboratory
    Prognostic factors
    Case reports
    Treatment
    Microscopic (histologic) description
    • Partial to complete effacement with sinusoidal obliteration (Adv Anat Pathol 2017;24:128)
    • Markedly expanded marginal zones with interfollicular infiltration of lymphoma cells
    • Diffuse areas often present (Virchows Arch 2016;468:141)
    • Often large follicles with extension of mantle zone into germinal centers, resembling progressive transformation of germinal centers; follicular colonization is sometimes seen (Pediatr Blood Cancer 2020;67:e28416)
    • Lymphoid infiltrate is polymorphous including monocytoid cells (small to medium sized, round nuclei, moderate cytoplasm), centrocyte-like cells (irregular nuclei, scant cytoplasm) and plasma cells (Adv Anat Pathol 2017;24:128)
    Microscopic (histologic) images

    Contributed by Kathryn Gibbons, M.D.

    Lymph node with pediatric nodal marginal zone lymphoma

    Atypical infiltrate


    CD20 stain

    PAX5 stain

    CD43 stain

    IgD stain

    Virtual slides

    Images hosted on other servers:

    Pediatric nodal marginal zone lymphoma

    Positive stains
    Negative stains
    Flow cytometry description
    Molecular / cytogenetics description
    Sample pathology report
    • Cervical lymph node, excisional biopsy:
      • Pediatric nodal marginal zone lymphoma
      • Histologic description: Sections show lymph node with effaced architecture by large nodules / follicles with markedly expanded marginal zones. Focally, there are large follicles that appear transformed with extension of mantle zones into germinal centers. Lymphoid infiltrate consists predominantly of small monocytoid appearing cells and scattered plasma cells.
      • Immunohistochemical stains show the atypical lymphoid cells to be positive for CD20, PAX5, CD43 and BCL2. They are negative for CD10 and BCL6. IgD highlights mantle zone expansion. Ki67 proliferative index is low (< 10%) in the atypical infiltrate.
      • Molecular studies are positive for clonal immunoglobulin heavy chain (IGH) gene rearrangement.
    Differential diagnosis
    Board review style question #1


    An 18 year old man presents with localized nontender cervical lymphadenopathy. Excisional biopsy was performed and representative images are shown above. Flow cytometry showed a lambda light chain restricted B cell population, while molecular analysis shows a clonal IGH rearrangement. What is the most likely diagnosis?

    1. Adult marginal zone lymphoma
    2. Atypical marginal zone hyperplasia
    3. Pediatric nodal marginal zone lymphoma
    4. Pediatric type follicular lymphoma
    5. Progressive transformation of germinal centers
    Board review style answer #1
    C. Pediatric nodal marginal zone lymphoma. The images show a lymph node with architecture effaced by atypical lymphoid proliferation composed of large follicles and extension of mantle zones into the germinal centers, mimicking progressive transformation of germinal centers. On higher magnification, the lymphoid infiltrate expands the marginal zone and extends into the interfollicular area. Flow cytometry and molecular studies support a lymphoma diagnosis over a benign process. Given the clinical history of a young male with localized nontender cervical lymphadenopathy, morphologic findings, in conjunction with ancillary studies, are most consistent with pediatric nodal marginal zone lymphoma.

    Comment Here

    Reference: Pediatric nodal marginal zone lymphoma
    Board review style question #2
    Which panel of immunohistochemical stains is most consistent with pediatric marginal zone lymphoma?

    1. CD20+, CD10+, BCL6+, BCL2+
    2. CD20+, CD43+, CD10-, BCL6-, BCL2+
    3. CD20+, CD43-, CD10-, BCL2-, CD5-
    4. CD20-, CD3+, PD-1+
    5. CD20+, CD10+, BCL6+, BCL2-
    Board review style answer #2
    B. CD20+, CD43+, CD10-, BCL6-, BCL2+. In pediatric nodal marginal zone lymphoma, the lymphoma cells are neoplastic B cells, positive for CD20, CD43 and BCL2. They are not derived from a germinal center and are negative for CD10 and BCL6. Therefore, answer B is the best choice. Answer A is consistent with lymphoma of germinal center origin. The stains in answer C are nonspecific. Stains in answer D are supportive of a T cell process. The stains in answer E can be seen in reactive follicular hyperplasia, among other diagnoses.

    Comment Here

    Reference: Pediatric nodal marginal zone lymphoma

    Pediatric type
    Definition / general
    • Localized neoplasm of germinal center B cells
    • Entirely follicular pattern
    • Lymph nodes of children and young adults
    Essential features
    • At least partial effacement of nodal architecture
    • Pure follicular proliferation
      • Any component of diffuse large B cell lymphoma (DLBCL) excludes pediatric type follicular lymphoma (PTFL)
    • Nodal disease
    • Stage I - II
    • BCL6 positivity
    • BCL2 negativity or weak positivity
    • High proliferative fraction (> 30%)
    • No BCL2, BCL6, IRF4 or aberrant IG rearrangement
    • No BCL2 amplification
    Terminology
    • Pediatric type nodal follicular lymphoma (PTNFL)
    ICD coding
    • ICD-O: 9690/3 - follicular lymphoma, NOS
    • ICD-10: C82.90 - follicular lymphoma, unspecified, unspecified site
    Epidemiology
    • 1 - 2% of all pediatric non-Hodgkin lymphoma
    • No upper age cutoff for diagnosis
    • Majority 5 - 25 years (< 40 years)
    • M:F = 10:1
    • Reference: Virchows Arch 2019;475:771
    Sites
    • Head and neck lymph node, most frequent
    • Inguinal and axillary nodes, less frequent
    Diagrams / tables

    Pediatric type follicular lymphoma versus follicular lymphoma usual type
    Pediatric type FL Usual type FL
    Age Young Old age (sixth decade)
    Stage Low (I - II) High (III - IV) in majority of cases
    Location Head and neck Variable
    Extranodal location Absent Present, variable
    Histology Grade 3 Grade 1 - 3
    BCL2 (IHC) Negative / dim Usually positive
    CD10 Positive (~100%) Positive (usually)
    Ki67 High Low (except for high grade)
    t(14;18) IGH-BCL2 Absent Present, up to 90%
    Monotypic B cells by flow cytometry Frequent Frequent
    Monoclonal IgH rearrangements Frequent Frequent
    BCL6 or MYC rearrangements Absent Variably present
    Genetic 1p36 loss Complex, variable
    Mutations TNFRSF14, MAP2K1, IRF8 (K66R) CREBBP, EZH2, KMT2D
    Prognosis Favorable Variable
    FL: follicular lymphoma; IHC: immunohistochemistry
    Clinical features
    • Young patients
    • Localized (stage I)
    • Commonly involves lymph nodes of the head and neck
    • Bone marrow involvement has not been reported
    • B symptoms are absent
    • Does not progress or recur after excision
    • References: Am J Surg Pathol 2013;37:333, Haematologica 2010;95:253
    Radiology description
    • Localized nature of the disease
    • Absence of mediastinal or intraabdominal lymph node involvement
    Prognostic factors
    • Excellent prognosis
    • 5 year survival of over 95%
    Case reports
    Treatment
    Gross description
    • Lymph node: vaguely nodular pattern in cut section with bulging of node parenchyma
    Frozen section description
    • Avoid specific diagnosis on frozen section due to requirement to examine submitted tissue completely to exclude a diffuse component and to confirm the absence of BCL2, BCL6 and MYC rearrangements
    • Cytogenetic analysis, flow cytometry, preparation of imprints should be considered
    Microscopic (histologic) description
    • At least partial effacement of nodal architecture
    • Entirely follicular pattern
    • Expansile or serpiginous follicles
    • Absence of diffuse component
    • Rim of residual normal lymph node architecture at the periphery
      • So called node in node appearance
    • No extracapsular extension
    • Attenuated mantle zones
    • Monotonous population of intermediate sized blastoid cells within neoplastic follicles imparting a high grade or starry sky histologic appearance
    • Blastoid cells comprising the neoplastic follicles are morphologically distinct from both centrocytes and centroblasts
    • Tingible body macrophages present
    • Lack of polarization of germinal center
    • Reference: Haematologica 2010;95:253
    Microscopic (histologic) images

    Contributed by L. Jeffrey Medeiros, M.D. and Mahsa Khanlari, M.D.

    Lymph node in 23 year old man

    IHC for CD20

    IHC for CD10


    IHC for BCL6

    IHC for BCL2

    IHC for IgD; attenuated and focally absent mantle zone

    Cytology description
    • 2 types of cytology:
      • Centroblastic cells
      • Monotonous, medium sized blastoid cells
        • Resemble centrocytes but are intermediate to large in size with more dispersed chromatin, high proliferative rate, frequent mitoses
        • Small nucleoli as compared with centroblasts
    • Reference: Am J Surg Pathol 2013;37:333
    Positive stains
    Negative stains
    Molecular / cytogenetics description
    Sample pathology report
    • Lymph node, left neck, excisional biopsy:
      • Pediatric type follicular lymphoma (see comment)
      • Comment: The patient is a 12 year old male presenting with isolated enlargement of left cervical lymph node. Other lymphadenopathy and extranodal masses as well as B symptoms are absent. Histologic sections show an excisional biopsy of lymph node demonstrating partial nodal architectural effacement. There is a rim of normal lymph node tissue in which reactive germinal centers are seen. Effaced areas of the lymph node are characterized by the presence of increased, irregularly shaped neoplastic follicles containing a monotonous population of intermediate sized lymphoid cells with blastic nuclear features. Mitotic activity is brisk and tingible body macrophages are present. A diffuse component is not identified. The lymph node is submitted entirely for histologic evaluation. By immunohistochemistry, the neoplastic cells are positive for CD20, CD10, BCL6 and HGAL and negative for CD3 and CD5. A subset of neoplastic cells express dim BCL2, but the majority are negative. The proliferative rate of lymphoid cells within neoplastic follicles is ~60%. FISH studies have been performed and are negative for the presence of BCL2, BCL6, MYC and IRF4 rearrangements. Next generation sequencing has been performed and has identified the presence of a TNFRSF14 mutation. The above histologic, immunophenotypic and cytogenetic features, in the appropriate clinical context, support the diagnosis of pediatric type follicular lymphoma.
    Differential diagnosis
    Board review style question #1
    Which of the following features is typical of pediatric type follicular lymphoma?

    1. High grade morphologic features indicating aggressive disease and poor prognosis
    2. Lack BCL2 gene rearrangements
    3. Lack monoclonal B cells
    4. Overexpress BCL2 protein
    Board review style answer #1
    B. PTFL, by definition, lack BCL2 gene rearrangements. These lymphoid proliferations do frequently contain monoclonal B cells and histologically appear high grade but they do not express BCL2 protein and are clinically low stage with excellent prognosis.

    Comment Here

    Reference: Pediatric type follicular lymphoma (PTFL)
    Board review style question #2

    A 23 year old man presented with a right cervical lymph node enlargement. Cut section of the lymph node is provided in images. By immiunohistochemical stain, the cells are positive for BCL6 in the expanded follicles (image 1) while negative for BCL2. PCR is performed and clonal immunoglobulin gene rearrangement is detected. Which of the following is true?

    1. Chance of recurrence after resection is high
    2. Diagnosis requires integration of morphology with lymphoma staging findings
    3. Frequent CREBBP mutation
    4. Ki67 proliferative index is low in cells highlighted by BCL6 in immunohistochemistry
    Board review style answer #2
    B. Diagnosis requires integration of morphology with lymphoma staging findings (Am J Surg Pathol 2013;37:333).

    Comment Here

    Reference: Pediatric type follicular lymphoma (PTFL)

    Peripheral T cell lymphoma, NOS
    Definition / general
    • Mature T cell lymphoma that does not meet WHO classification criteria for other mature T cell lymphoma entities
    Essential features
    • Mature T cell lymphoma that does not meet criteria for other T cell lymphomas in the current WHO classification system
    • Usually has aggressive clinical behavior
    • CD4 > CD8, CD30 variable (if positive, heterogenous staining), EMA-, ALK-, negative for T follicular helper markers (CD10, BCL6, PD1, CXCL13, etc.), EBER-
    Terminology
    • T cell lymphoma, NOS
    • Lymphoepithelioid lymphoma
    • Lennert lymphoma
    ICD coding
    • ICD-O: 9702/3 - Mature T cell lymphoma, NOS
    Epidemiology
    Sites
    • Most commonly presents with peripheral lymphadenopathy (87%) but can involve any site; extranodal disease present in 62% (Blood 2011;117:3402)
    • Most common extranodal sites: skin, gastrointestinal tract; lungs and central nervous system less common
    • Majority (69%) present with advanced stage disease (Blood 2011;117:3402)
    • Advanced stage disease associated with secondary involvement of marrow, liver, spleen and other extranodal sites
    • Can involve peripheral blood
    Pathophysiology
    • Not definitively established
    Etiology
    • Cell of origin: activated mature T cell (typically CD4+ memory T cell)
    Diagrams / tables

    Images hosted on other servers:

    A, B: monomorphic morphology
    characteristic of GATA3 subtype
    C, D: polymorphous morphology
    characteristic of TBX21 subtype

    Clinical features
    • B symptoms (35%) (J Clin Oncol 2006;24:2472)
    • Can be associated with eosinophilia, pruritis and hemophagocytic syndrome (rare)
    Diagnosis
    Laboratory
    Radiology description
    • Lymphadenopathy
    Prognostic factors
    • Stage and international prognostic index (IPI) score are the most well established prognostic factors
      • 5 year overall survival for low IPI is 50 - 59% versus 9 - 18% for high IPI
    • Other high risk prognostic features: bone marrow involvement, Ki67 > 80%, thrombocytopenia, GATA3 subtype by gene expression profiling or IHC (Blood 2014;123:2915)
    • Prognostically significant molecular subtypes evaluable by IHC (Blood 2019;134:2159):
      • GATA3 subtype (35% of PTCL, NOS):
        • ≥ 50% tumor cells positive for GATA3 or CXCR3 by IHC
        • Associated with monomorphic morphology with minimal inflammatory background
        • Very poor overall survival (5 year overall survival = 19%)
      • TBX21 subtype (58% of PTCL, NOS):
        • ≥ 20% tumor cells positive for TBX21 or CCR4 by IHC
        • Associated with polymorphous morphology
        • Longer overall survival than GATA3 subtype (5 year overall survival = 38%)
    Case reports
    Treatment
    • Induction: CHOP (cyclophosphamide, doxorubicin, vincristine and prednisone) or other anthracycline based regimens (CHOEP, EPOCH)
    • Consolidation: autologous stem cell transplantation for responders
    • Approved second line drugs for relapsed / refractory disease: pralatrexate, romidepsin, belinostat and brentuximab vedotin
    • Reference: Blood 2017;129:1103
    Gross description
    • Enlarged lymph node
    Microscopic (histologic) description
    • Lymph node:
      • Paracortical or diffuse infiltrate with effacement of the normal architecture by medium to large sized cells with pleomorphic nuclei, vesicular chromatin, prominent nucleoli and frequent mitoses
        • Clear cells and Reed-Sternberg-like cells can be seen
      • TBX21 subtype associated with a polymorphous inflammatory background composed of small lymphocytes, eosinophils, plasma cells, large B cells and epithelioid histiocytes (Blood 2019;134:2159)
      • GATA3 subtype associated with with monomorphic morphology with minimal inflammatory background (Blood 2019;134:2159)
      • Lymphoepithelioid variant (Lennert lymphoma)
        • Typically shows a diffuse (less commonly paracortical) infiltrate of small cells with subtle nuclear atypia (typically CD8+ with expression of cytotoxic markers), numerous epithelioid histiocytes which often form clusters and occasional immunoblasts
        • Correlates with TBX21 molecular subtype
        • May have better prognosis than other PTCL, NOS subsets
    • Skin:
      • Lymphoma cells infiltrate the dermis and subcutis; often form nodules which can undergo central necrosis
      • Epidermotropism, angiocentricity and adnexal involvement can be seen
      • Clinical correlation is needed to exclude mycosis fungoides, anaplastic large cell lymphoma, lymphomatoid papulosis
    Microscopic (histologic) images

    Contributed by Matthew M. Klairmont, M.D.

    PTCL, NOS

    PTCL, NOS involving the skin

    Cytology description
    • Broad cytologic spectrum, cannot diagnose based on cytology alone
    • Medium to large tumor cells with irregular, pleomorphic, hyperchromatic or vesicular chromatin; prominent nucleoli; numerous mitotic figures; polymorphous inflammatory background
    Cytology images

    Images hosted on other servers:

    Imprints of PTCL, NOS (lymphoepithelioid / Lennert variant)

    Peripheral smear description
    • Peripheral blood involvement can occur and presents with circulating medium to large tumor cells with nuclear atypia, vesicular chromatin and prominent nucleoli
    Positive stains
    Negative stains
    Flow cytometry description
    • Flow cytometry may be helpful in identifying aberrant loss of pan-T cell antigens but is typically not sufficient to classify tumor as PTCL, NOS
    Molecular / cytogenetics description
    • PCR demonstrates clonal rearrangement of TCR genes
    • Next generation sequencing shows recurrent alterations of TP53 (> 40%), TET2 (> 40%), RHOA (~35%), CDKN2A (~20%), CD28 (~20%), HLA-A (~20%), SETD1B (~20%) and DNMT3A (~10%), as well as numerous other less prevalent alterations (Leukemia 2019;33:2867)
    • Cytogenetics: typically shows a complex karyotype but specific recurrent abnormalities very uncommon
      • t(5;9)(q33;q22) / ITK-SYK associated with a follicular growth pattern (rare)
    Sample pathology report
    • Lymph node, left axillary, biopsy:
      • Peripheral T cell lymphoma, NOS (see comment)
      • Comment: H&E sections demonstrate effacement of the normal nodal architecture by a diffuse infiltrate of large, pleomorphic lymphoid cells with vesicular chromatin and prominent nucleoli. By paraffin immunohistochemistry, the tumor cells are positive for CD3, CD4, CD5 and CD30 (subset, variable intensity) while negative for CD20, CD7, CD8, CD15, TIA1, granzyme B, perforin, CD10, BCL6, PD1, CXCL13, CXCR5 and EBER ISH. PCR studies are positive for clonal TCR gene rearrangement. These findings are diagnostic for peripheral T cell lymphoma, NOS.
    Differential diagnosis
    • Classic Hodgkin lymphoma:
      • As PTCL, NOS can present with Reed-Sternberg-like cells and CD30 expression (as well as occasional coexpression of CD15), misclassification as classic Hodgkin lymphoma is an important pitfall to consider, particularly on limited needle biopsies
      • Careful immunophenotyping of the tumor cells to establish lineage is essential to avoid misclassification (Hodgkin / Reed-Sternberg cells may have aberrant T cell antigen expression but should be CD3 negative, though PTCL, NOS can also be CD3 negative)
      • Classic Hodgkin lymphoma should be CD43 negative and weakly PAX5 positive
      • PCR clonality assays can help resolve lineage (PTCL, NOS should show clonal rearrangement of TCR genes)
    • T cell lymphoma subtypes defined by WHO classification (Swerdlow: WHO Classification of Tumours of Haematopoietic and Lymphoid Tissues, 4th Edition, 2017):
      • ALK negative anaplastic large cell lymphoma:
        • ALK negative anaplastic large cell lymphoma: CD30 expression strong and uniform; hallmark cells; may be EMA positive
        • PTCL, NOS: CD30 expression, if present, only found in a subset of tumor cells and with variable intensity; EMA negative
      • T cell lymphomas with a T follicular helper phenotype (angioimmunoblastic T cell lymphoma, nodal peripheral T cell lymphoma with T follicular helper phenotype and follicular T cell lymphoma):
        • Expression of at least 2 or 3 of the following markers precludes the classification of PTCL, NOS: CD10, BCL6, PD1, CXCL13, CXCR5, CD278 and SAP
      • Adult T cell leukemia / lymphoma:
        • Adult T cell leukemia / lymphoma associated with positive HTLV-1 serology or demonstration of HTLV-1 integration in tumor cells
        • CD25 positive in adult T cell leukemia / lymphoma but variable in PTCL, NOS
        • PTCL, NOS often has polymorphous inflammatory background
      • Involvement by mycosis fungoides:
        • Clinical history and staging critical to separating nodal involvement by mycosis fungoides from PTCL, NOS
    Board review style question #1

    A 60 year old man presents with diffuse lymphadenopathy. Lymph node biopsy shows complete effacement of the normal lymph node architecture by a diffuse infiltrate of medium to large pleomorphic lymphoid cells with vesicular chromatin and prominent nucleoli. Immunostains show the tumor cells are positive for CD3, CD5 and CD4 while negative for CD8, CD7, CD20, ALK. Which of the following findings would support the diagnosis of PTCL, NOS?

    1. Coexpression of BCL6, PD1 and CD10
    2. EMA positivity
    3. Heterogenous CD30 positivity
    4. Strong and uniform CD30 positivity
    Board review style answer #1
    C. Heterogenous CD30 positivity

    1. PTCL, NOS should not coexpress multiple T follicular helper (TFH) markers. Expression of > 2 TFH markers in addition to CD4 (PD1, CD10, BCL6, CXCL13, CXCR5, CD278, SAP) would suggest nodal peripheral T cell lymphoma with a TFH phenotype.
    2. EMA should be negative in PTCL, NOS but can be expressed in ALCL.
    3. Heterogenous CD30 positivity can be a feature of PTCL, NOS. Strong and uniform CD30 positivity in this case would support the diagnosis of ALK negative anaplastic large cell lymphoma.
    4. Strong and uniform CD30 positivity in this case would support a diagnosis of ALK negative anaplastic large cell lymphoma. CD30 is positive in a subset of PTCL, NOS but the staining is heterogeneous and of variable intensity.

    Comment Here

    Reference: Peripheral T cell lymphoma, not otherwise specified
    Board review style question #2
    A 65 year old woman presents with lymphadenopathy. Lymph node biopsy shows effacement of the normal lymph node architecture by a diffuse infiltrate of medium to large pleomorphic tumor cells with vesicular chromatin and prominent nucleoli. By paraffin immunohistochemistry, the tumor cells are positive for CD3, CD8, CD5, PD1, granzyme B and CD30 (subset, variable intensity) while negative for CD4, CD7, ALK, perforin, EMA, CD10, BCL6, CXCR5 and CXCL13. EBER ISH shows rare positivity in small background lymphocytes with benign cytology. CD21 shows focal areas with small residual follicular dendritic cell meshworks. The best classification for this lesion is

    1. ALK negative anaplastic large cell lymphoma
    2. Angioimmunoblastic T cell lymphoma (AITL)
    3. Nodal peripheral T cell lymphoma with TFH phenotype
    4. Peripheral T cell lymphoma, NOS
    Board review style answer #2
    D. Peripheral T cell lymphoma, NOS

    1. ALK negative anaplastic large cell lymphoma would have strong and uniform CD30 expression.
    2. Angioimmunoblastic T cell lymphoma (AITL) is typically CD4+ and expresses > 2 TFH markers (this case only appears to express one, PD1). Further, AITL characteristically has expanded follicular dendritic cell meshworks which can be visualized with a CD21 immunostain.
    3. Like AITL, there must be expression of > 2 TFH markers to qualify for the classification of nodal peripheral T cell lymphoma with TFH phenotype.
    4. PTCL, NOS is the best classification for this T cell lymphoma which does not meet the criteria for any other category of mature T cell lymphoma. Although PTCL, NOS is typically CD4+ / CD8-, it can present with any pattern of CD4 / CD8 expression.

    Comment Here

    Reference: Peripheral T cell lymphoma, not otherwise specified

    Plasmablastic lymphoma
    Definition / general
    • Aggressive large B cell lymphoma with plasmablast / immunoblast morphologic features with plasma cell differentiation, usually in association with immunodeficiency
    Essential features
    • Aggressive lymphoma with unfavorable prognosis (Am J Surg Pathol 2004;28:736)
    • Distinct entity, according to the 2017 WHO classification
    • Predominantly occurs in extranodal sites (most common in oral cavity)
    • Commonly occurs in young adults with HIV infection or iatrogenic immunosuppression (Am J Surg Pathol 2004;28:736, Histopathology 2007;51:774)
    • Pathogenesis is not well understood
    • Requires histologic, immunophenotypic and clinical correlation for definitive diagnosis
    • Monomorphic sheets of intermediate to large cells with plasmablastic / immunoblastic features, prominent nucleoli and positivity for plasmacytic markers (Am J Surg Pathol 2004;28:736)
    Terminology
    • Plasmablastic lymphoma (PBL); subtype of diffuse large B cell lymphoma with sufficient distinctive features that is now considered a distinct entity
    ICD coding
    • ICD-O: 9735/3 - plasmablastic lymphoma
    Epidemiology
    • Mostly occurs in adults with HIV infection or with iatrogenic immunosuppression
    • In a 10 year study, most patients were men with a median age at presentation of 50 years (Am J Hematol 2019;94:E127)
    Sites
    • First described in the oral cavity (Blood 1997;89:1413)
    • Presents frequently in extranodal sites, especially of the head and neck region
    • Other common sites include skin, gastrointestinal tract and bone
    • Nodal involvement is generally infrequent but is seen in 30% of posttransplant cases (Blood 2015;125:2323)
    Pathophysiology
    • Derived from post germinal center B cells (plasmablast)
    Etiology
    • Not well understood
    • Epstein-Barr virus (EBV) positivity is observed in 75% of cases (Am J Surg Pathol 2014;38:875)
    • Iatrogenic immunosuppression (e.g., posttransplant) and HIV positive patients
    • Few cases are reported in immunocompetent individuals (J Med Case Rep 2011;5:168)
    Clinical features
    • Mass forming lesion in extranodal sites, typically in the context of HIV infection, iatrogenic immunosuppression and posttransplant setting
    • Disease is commonly disseminated and higher stage at the time of presentation (stage 3 - 4) with bone marrow involvement (75% of HIV positive patients and 50% of posttransplant patients) (Blood 2015;125:2323)
    • Elevated serum paraproteins may be detected
    Diagnosis
    • Requires correlation with clinical features to exclude plasmablastic plasma cell myeloma, including the presence of serum monoclonal proteins, SPEP / IFX studies and surveillance for lytic lesions
    • Diagnosis can be complicated by overlapping morphologic and phenotypic features with plasmablastic plasma cell myeloma with extramedullary involvement
    • MYC translocation is observed in approximately 50% of PBL cases (Am J Surg Pathol 2010;34:1686)
    • EBV positivity is present in approximately 75% of cases but may be rarely detected in myeloma (Am J Surg Pathol 2014;38:875)
    • Diffuse C-MYC expression by immunohistochemistry and high Ki67 proliferative index (70 - 90%) favors PBL over myeloma (Histopathology 2007;51:774, Blood 1997;89:1413)
    Laboratory
    • Serology for HIV
    • Paraprotein levels may be elevated
    • Monoclonal spike should not be present (Mod Pathol 2005;18:806)
    Radiology description
    Prognostic factors
    Case reports
    Treatment
    • Newer treatment modalities using V-EPOCH (bortezomib with etoposide, prednisone, vincristine, cyclophosphamide and doxorubicin) have shown increased survival (Br J Haematol 2019;184:679)
    • Despite newer treatments, prognosis remains unfavorable
    Gross description
    • No distinct or specific features on gross examination
    • Soft tissue mass can be well circumscribed or irregular
    • Cut surface is usually gray-tan to yellowish in color, with a fleshy appearance
    Frozen section description
    • Definitive diagnosis of plasmablastic lymphoma should be avoided at the time of frozen section, as definitive diagnosis requires phenotyping and clinical correlation
    • Descriptive diagnosis, such as lymphoma with plasmablastic features or lymphoma with features suggestive of plasmablastic lymphoma, allows for adequate workup and definitive diagnosis
    • Findings are similar to those seen on permanent sections; they consist of monomorphic sheets of intermediate to large sized cells with round eccentric nuclei
    Microscopic (histologic) description
    • 2 morphologic variants recognized: monomorphic PBL and PBL with plasmacytic differentiation
      • Monomorphic PBL:
        • Consist mostly of immunoblastic cells
      • PBL with plasmacytic differentiation:
        • Cells with plasmacytic differentiation / plasmablasts that are intermediate to large in size with round eccentric nuclei, prominent nucleoli and bluish cytoplasm (Crane: Survival Guide to Lymph Node Pathology, 2021)
          • Seen more frequently in the setting of HIV infection
          • Oral, nasal and paranasal sinuses are commonly affected
          • Anaplastic / plasmablastic plasma cell myeloma is an important differential that needs to be ruled out
            • Lytic bone lesions and monoclonal serum immunoglobulins can be seen in both
            • Bone marrow is not frequently involved in plasmablastic lymphoma
            • History of immune deficiency or EBER positivity by in situ hybridization is useful in establishing a diagnosis of plasmablastic lymphoma
            • MYC translocation will favor plasmablastic lymphoma
    • Mitotic figures are increased
    • Apoptotic cells and tingible body macrophages may be identified
    Microscopic (histologic) images

    Contributed by Julio Poveda, M.D.
    Intermediate to large sized cells Intermediate to large sized cells

    Intermediate to large sized cells

    CD138

    CD138

    CD20

    CD20

    MUM1

    MUM1

    Virtual slides

    Contributed by Genevieve M. Crane, M.D., Ph.D.

    Palate tumor

    Cytology description
    • Sheets of monomorphic cells of intermediate to large sized lymphoid cells with plasmacytic differentiation and immunoblastic or plasmablastic morphology
    • Large cells with abundant basophilic cytoplasm; large nucleus with prominent nucleolus
    Positive stains
    Negative stains
    • CD20, CD45 and PAX5 (typically negative but may show weak expression in a subset of cells)
    • HHV8
    • T cell antigens are usually negative but may show aberrant expression (Mod Pathol 2018;31:718)
    Flow cytometry description
    Electron microscopy description
    • Electron microscopy is not routinely undertaken for diagnosis
    • Case studies that have incorporated electron microscopy have shown (Exp Mol Pathol 2011;90:85)
      • Euchromatic nuclei with 1 or more nucleoli
      • Numerous ribosomes
      • Scant strands of rough endoplasmic reticulum
    Molecular / cytogenetics description
    • Genetic alterations in MYC gene are found in approximately 50% of cases (Mod Pathol 2017;30:85)
      • MYC translocation, gain and overexpression at mRNA level
      • Seen more often in EBV positive tumors
    • PRDM1α gene mutations (missense mutations) are associated with PRDM1 / Blimp1 expression (Mod Pathol 2017;30:85)
    • Clonal rearrangements in immunoglobulin genes
      • Clonal IGH rearrangement is present
    • More recent studies that used whole genome sequencing (WES) have found multiple mutations (Nat Commun 2021;12:5183)
      • NRAS: 31% of cases, mostly missense mutations
      • NOTCH signaling pathway: 26%
      • STAT3: 25% of cases, clustered in the SH2 domain
        • 47% of HIV positive patients
        • 10% of HIV negative patients
    Molecular / cytogenetics images

    Images hosted on other servers:
    Landscape using WES

    Landscape using WES

    Sample pathology report
    • Anterior hard palate, biopsy:
      • Plasmablastic lymphoma (see comment)
      • Comment: Histologic sections consist of squamous lined mucosa diffusely involved by lymphoma. The lymphoma is seen in sheets and composed of large atypical plasmablasts. Apoptotic figures and brisk mitotic activity are seen.
      • Immunohistochemical studies show that lymphoma cells are positive for MUM1, CD138 (variable), CD38 (variable), CD30 (variable) and C-MYC (~10%), while negative for CD20, PAX5, CD3 and HHV8. Lymphoma cells are lambda light chain restricted as detected by in situ hybridization (ISH). Kappa light chains are decreased (by ISH). Kappa light chains are markedly decreased (by ISH). EBER (by ISH) is diffusely positive. Ki67 proliferation index is approximately 100%.
    Differential diagnosis
    Board review style question #1

    What is the diagnosis of this hard palate lesion in a 54 year old man with HIV?

    1. Diffuse large B cell lymphoma
    2. Plasmablastic lymphoma
    3. Angioimmunoblastic T cell lymphoma
    4. Monomorphic epitheliotropic T cell lymphoma
    5. Mantle cell lymphoma
    Board review style answer #1
    B. Plasmablastic lymphoma. The patient is a 54 year old man with history of HIV who underwent biopsy of a hard palate mass. The lymphoma is present as sheets of large atypical cells with prominent nucleoli. Immunohistochemical studies showed that lymphoma cells were positive for MUM1, CD138, CD38, CD30 and C-MYC (~10%) and negative for CD20 (therefore, choices A and E are eliminated), PAX5, CD3 (choices C and D are eliminated), ALK (ALK positive large B cell lymphoma would be ruled out) and HHV8 (primary effusion lymphoma is ruled out). Lymphoma cells are lambda light chain restricted as detected by in situ hybridization (ISH). Kappa light chains are decreased (by ISH). EBER (by ISH) is diffusely positive. The history, site, morphologic appearance and immunohistochemical findings support a diagnosis of plasmablastic lymphoma.

    Comment Here

    Reference: Plasmablastic lymphoma
    Board review style question #2
    Which of the following stains is usually positive in plasmablastic lymphoma?

    1. ALK
    2. CD3
    3. CD20
    4. EBER
    5. HHV8
    Board review style answer #2
    D. EBER by in situ hybridization can be seen in other entities such as large B cell lymphoma and primary effusion lymphoma; however, the other stains are negative in plasmablastic lymphoma.

    Comment Here

    Reference: Plasmablastic lymphoma
    Board review style question #3
    Which of the following gene rearrangements can be seen in plasmablastic lymphoma?

    1. BCL2
    2. BCL6
    3. IGH
    4. MALT1
    5. MYC
    Board review style answer #3
    E. MYC translocation is seen in approximately 50% of cases.

    Comment Here

    Reference: Plasmablastic lymphoma

    Polymorphic lymphoproliferative disorders arising in immune deficiency / dysregulation
    Definition / general
    • Lymphoproliferative disorders (LPDs) arising in patients with immune deficiency or dysregulation are usually Epstein-Barr virus (EBV) driven and composed of a heterogeneous lymphoid cell infiltrate with a variable number of B cells that efface the architecture of involved tissues
    • New standardized nomenclature for reporting this histological diagnosis is based on an integrated approach and consists of 3 parts
      • Histological diagnosis (polymorphic lymphoproliferative disorder)
      • Viral association
      • Immune deficiency / dysregulation setting
    Essential features
    • Polymorphous lymphoproliferative disorder arising in any immune deficiency / dysregulation setting
    • Typically associated with EBV infection
    • Histologically, there is effacement of the architecture of the involved tissue by a mixed inflammatory process and absence of sheets of monotonous cells
    Terminology
    • Encompasses the previous polymorphic posttransplant lymphoproliferative disorders, other iatrogenic immunodeficiency associated lymphoproliferative disorders, among others such as lymphomatoid granulomatosis (LYG) grade 1 and 2 in IDD settings (except immunosenescence) and lymphoid proliferations in the HIV setting (although usage of these terms is not recommended)
    • Not recommended: polymorphic lymphoma
    ICD coding
    • ICD-O:
      • 9971/1 - posttransplant lymphoproliferative disorder, NOS
      • 9971/3 - polymorphic posttransplant lymphoproliferative disorder, NOS
    • ICD-10: D47.Z1 - posttransplant lymphoproliferative disorder (PTLD)
    • ICD-11: 2B32.3 - polymorphic posttransplant lymphoproliferative disorder
    Epidemiology
    Sites
    Pathophysiology
    • First step in the development of polymorphic LPDs is a disruption in T cell immune surveillance that results from an immunodeficient status
    • In the setting of EBV positive LPDs, associated proteins promote cellular proliferation and immune evasion
      • EBV is transmitted by oral transfer; EBV shedding can also be detected in cervical secretions of the gynecologic tract
      • EBV latent membrane proteins LMP2A (that mimics the B cell receptor signaling) and LMP1 (that mimics CD40 signaling) activate NFκB, inducing cell proliferation (N Engl J Med 1998;338:1413, N Engl J Med 2004;350:1328)
      • EBV nuclear antigen (EBNA1) plays a role in DNA regulation, transcription and immune evasion, while EBNA2 regulates viral and cellular genes
      • EBV can present as multiple latency patterns depending on which proteins are expressed
        • Latency patterns II (LMP1+ / EBNA2-) and III (LMP1+ / EBNA2+) are the most common patterns associated with immune deficiency / dysregulation associated LPDs
    • In the setting of EBV negative LPDs, multiple theories have been proposed
    Etiology
    • Any immunodeficiency setting, typically associated with EBV infection
    Diagrams / tables

    Images hosted on other servers:
    Relationship between immunodeficiency-associated lymphoproliferative disorders  in 4th edition and 5th edition of the WHO

    Immunodeficiency
    associated LPDs
    in WHO-HAEM4R
    and WHO-HAEM5

    3 part nomenclature for immunodeficiency-associated LPDs

    3 part nomenclature
    for immunodeficiency
    associated LPDs

    Clinical features
    Diagnosis
    Laboratory
    • Elevated lactate dehydrogenase (LDH) levels have been associated with a poor response to treatment and overall survival (Blood 2006;107:3053, Am J Transplant 2006;6:569)
    • EBV serology (Am J Transplant 2009;9:S87)
      • Its most important role in the posttransplant setting is the determination of pretransplant donor and recipient EBV serostatus for posttransplant lymphoproliferative disorder risk assessment
      • In immunocompromised patients, serology is an unreliable diagnostic tool due to delayed or absent humoral response
        • If these patients are receiving blood products, the passive transfer of antibodies is another important drawback to consider
    • EBV viral load can be used to monitor therapeutic response (Am J Transplant 2008;8:1016, Transplantation 1998;66:1641, Transplantation 2002;74:656)
    Radiology description
    • Depending on the involved site, imaging studies may show localized or diffuse lymphadenopathy, organomegaly or a mass lesion
    Prognostic factors
    • There are no well defined histopathologic, immunophenotypic or molecular prognostic / predictive factors within the group of polymorphic LPDs
    • Staging with PET / CT imaging should be done following the Lugano staging system (J Clin Oncol 2014;32:3059)
    • Lymphadenopathy resolves in up to 20% of cases with no therapy
    • Clinical progression has been reported in 38% of patients, with 10% progressing to DLBCL (Blood 2011;117:4726)
    • Improvement may be seen upon discontinuation of immunosuppression in the posttransplant setting or in therapy related immunodeficiency (Med Oncol 2009;26:1)
    • In the posttransplant setting, factors that have been associated with lack of response to decreased immunosuppression include elevated LDH levels, organ dysfunction, multiorgan involvement, advanced age, bulky disease and older age of presentation (Am J Transplant 2011;11:336, Transplantation 2001;71:1076)
    Case reports
    • 26 year old woman presented with pharyngeal foreign body sensation after peripheral blood stem cell transplant (PBSCT) (Int J Clin Exp Med 2014;7:1904)
    • 36 year old HIV infected man presented with acute abdominal pain (EJHaem 2021;2:562)
    • 67 year old man status post-renal transplant presented with multiple allograft parenchymal cystic lesions (Am J Transplant 2012;12:245)
    Treatment
    Gross description
    • Lesions can present as a mass with ulceration or necrosis
    Microscopic (histologic) description
    • Histologic features include architectural destruction and a mixed infiltrate of cells, including B cells at different stages of maturation such as immunoblasts, small lymphoid cells, plasmacytoid cells, plasma cells (Am J Clin Pathol 2017;147:129, Blood 2011;117:4726)
    • Intermixed T cells and histiocytes are commonly seen
    • Immunoblasts may show Hodgkin / Reed-Sternberg (HRS)-like morphology
    • Sheets of monotonous cells are not present
    • Necrosis may be seen and angiocentric / angiodestructive polymorphous infiltrate may be seen in lymphomatoid granulomatosis type lesions
    Microscopic (histologic) images

    Contributed by Miguel Gonzalez-Mancera, M.D.

    Heterogeneous infiltrate

    Heterogeneous infiltrate

    Gastrointestinal biopsy


    CD3

    CD20

    CD68

    MUM1

    Kappa light chain


    Lambda light chain

    EBV EBER

    EBV EBER

    EBV EBER

    LMP1

    Positive stains
    Negative stains
    • Not relevant to the diagnosis
    Flow cytometry description
    • Not essential for diagnosis; mixed lymphoid population expected to be seen
    Molecular / cytogenetics description
    Sample pathology report
    • Rectum, biopsy:
      • Polymorphic lymphoproliferative disorder, EBV+, in the setting of HIV infection (see comment)
      • Comment: The patient's history of HIV / AIDS with proctitis and lymphadenopathy is noted. The current specimen shows a polymorphic lymphoid infiltrate composed of small cells, plasma cells, immunoblasts and large, atypical cells. Occasional Reed-Sternberg-like cells are seen. EBV EBER is positive in many cells and plasma cells are polytypic. Immunoglobulin heavy chain and kappa light chain gene rearrangement studies are negative.
    Differential diagnosis
    Board review style question #1

    An excisional lymph node biopsy from a patient with rheumatoid arthritis on methotrexate shows the histologic findings seen in the image above. EBV EBER is positive in many cells of varying sizes. What is the most appropriate diagnostic interpretation?

    1. Diffuse large B cell lymphoma, EBV+, iatrogenic methotrexate related
    2. Infectious mononucleosis
    3. Polymorphic lymphoproliferative disorder, EBV+, iatrogenic methotrexate related
    4. Reactive follicular hyperplasia
    Board review style answer #1
    C. Polymorphic lymphoproliferative disorder, EBV+, iatrogenic methotrexate related. These findings are consistent with polymorphic lymphoproliferative disorder (LPD) arising in immune deficiency / dysregulation. Polymorphic LPDs in the posttransplant setting are among the most commonly seen.

    Comment Here

    Reference: Polymorphic lymphoproliferative disorders arising in immune deficiency / dysregulation
    Board review style question #2
    A 40 year old woman post-renal transplant presents with iliac lymphadenopathy. Excisional biopsy is consistent with polymorphous posttransplant lymphoproliferative disorder. Which of the following histologic features is characteristic of this entity?

    1. Florid reactive follicles with prominent germinal centers
    2. Mixed population of plasma cells, histiocytes, variably sized lymphocytes and immunoblasts effacing normal lymphoid architecture
    3. Necrotizing granulomatous lymphadenitis with palisading histiocytes
    4. Sheets of immunoblasts diffusely replacing normal architecture
    Board review style answer #2
    B. Mixed population of plasma cells, histiocytes, variably sized lymphocytes and immunoblasts effacing normal lymphoid architecture. In polymorphic LPDs, these are the usual histologic findings.

    Comment Here

    Reference: Polymorphic lymphoproliferative disorders arising in immune deficiency / dysregulation

    Primary CNS lymphoma
    Definition / general
    • Primary central nervous system (CNS) lymphoma occurs first in the brain, eye, spinal cord or leptomeninges, without systemic involvement and not arising in the setting of immunodeficiency
    • Majority are high grade diffuse large B cell lymphomas, with only rare cases representing T cell lymphomas, low grade B cell lymphomas or Burkitt lymphoma
    • This topic addresses primary diffuse large B cell lymphoma of the CNS and does not include cases that are associated with immunodeficiency (see separate category of immunodeficiency associated lymphoproliferative disorders)
    Essential features
    • Rare aggressive large B cell lymphoma occurs predominantly in elderly individuals, with an overall worse prognosis than systemic diffuse large B cell lymphoma
    • Composed of large, atypical basophilic lymphocytes with perivascular cuffing and frequent necrosis
    • Positive for CD45 and B cell markers and typically positive for BCL6, MUM1 and BCL2
    • Unlike CNS lymphomas associated with HIV infection, primary CNS lymphoma is not associated with Epstein-Barr virus infection
    Terminology
    • Previously known as reticulum cell sarcoma, diffuse histiocytic lymphoma or lymphomatosis cerebri (no longer recommended)
    • Primary CNS lymphoma is used interchangeably with primary diffuse large B cell lymphoma of the CNS
    • Excluded from this category are immunodeficiency associated lymphomas (see separate immunodeficiency associated topic)
    ICD coding
    • ICD-10: C83.8 - other nonfollicular lymphoma
    • ICD-10: C72.9 - malignant neoplasm of central nervous system, unspecified
    Epidemiology
    • Accounts for < 1% of all non-Hodgkin lymphoma cases and 2 - 3% of all CNS tumors (Cancer 2017;123:4314)
    • Overall incidence rate is < 1 cases per 100,000 population
    • Rates among men and women aged 65+ years have increased in recent decades (1992 - 2011), although the reason for the increase is unclear
    • Among immunocompetent individuals (HIV uninfected, nontransplant recipients), the median age at diagnosis among a U.S. population is 67 (range 52 - 77 years) with a similar male to female ratio (Br J Haematol 2016;174:417)
    Sites
    Pathophysiology
    • Not definitively understood
    • Central nervous system is an immune privileged site and the lymphoma itself may have genetic and phenotypic features that protect the tumor from ongoing immune surveillance (Virchows Arch 2020;476:647)
    • Neoplastic cells may arise from lymphocytes normally residing in the immune privileged milieu of the CNS or from lymphocytes that transform to a malignant clone outside the CNS and then home to the CNS and escape normal immune regulation in the immune privileged site (Virchows Arch 2020;476:647, Curr Oncol Rep 2015;17:60)
    Etiology
    • Unknown in immunocompetent patients
    • Viruses, including Epstein-Barr virus, do not play a role
    • Rising rates in elderly individuals (HIV uninfected, nontransplant recipients) may be related to increasing use of immunosuppressive medications to treat autoimmune conditions in the elderly (Br J Cancer 2011;105:1414)
    Clinical features
    • Symptoms vary depending on the CNS site involved and commonly include cognitive or behavioral changes, focal neurologic deficits and symptoms of increased intracranial pressure (Cancer 2017;123:4314)
    • Seizures are relatively uncommon and patients typically do not present with B symptoms (fever, night sweats, weight loss)
    • Ocular manifestations may present as decreased acuity, blurry vision or floaters (Hematol Oncol Clin North Am 2019;33:597)
    Diagnosis
    • Contrast enhanced MRI of the brain is the neuroimaging modality of choice for diagnosis; however, the diagnosis must be confirmed by review of sampled tumor tissue or fluid (Br J Haematol 2019;184:348)
    • Stereotactic needle biopsy is the gold standard for diagnosis and classification of CNS lymphoma (Neurooncol Pract 2019;6:415)
    • Cerebrospinal fluid (CSF) cytology and flow cytometry may be used when biopsy is not possible or to evaluate for leptomeningeal involvement
    Radiology description
    • On MRI, there is an isointense to hypointense lesion on T1 weighted images with homogenous gadolinium contrast enhancement
    • Lesions are usually solitary, involving the deep white matter and periventricular in location
    • In immunocompetent individuals, the lesions typically do not show hemorrhage, calcifications, necrosis or ring enhancement (Hematol Oncol Clin North Am 2019;33:597)
    Radiology images

    Images hosted on other servers:
    Brain MRI with contrast enhancing lesion

    Brain MRI with contrast enhancing lesion

    Solid and enhancing nodules on MRI

    Prognostic factors
    • Overall poor prognosis with considerably worse prognosis than systemic diffuse large B cell lymphoma
    • Age and performance status are the most important prognostic factors
    • In the Memorial Sloan Kettering Cancer Center prognostic model, only age and performance status were independent prognostic factors
    • The prognostic model from the International Extranodal Lymphoma Study Group includes age and performance status as well as serum lactate dehydrogenase level, CSF protein concentration and involvement of deep structures of the brain (Cancer 2017;123:4314)
    • In patients treated with methotrexate based regimens, BCL6 expression may be a predictor of improved progression free survival (Br J Haematol 2014;165:640)
    Case reports
    Treatment
    • High dose intravenous methotrexate based regimen is the first line induction therapy
    • Consolidation therapy may include conventional chemotherapy, radiotherapy, high dose chemotherapy with autologous stem cell transplantation or their combinations (Transl Oncol 2019;12:523)
    Gross description
    • From postmortem examination, tumors have a variable macroscopic appearance, including variable demarcation from the surrounding parenchyma
    • Lesions may be firm, homogenous, centrally necrotic, brownish, gray-tan or yellow with areas of hemorrhage (Clin Neuropathol 2008;27:13)
    Gross images

    Images hosted on other servers:

    Cerebral lymphoma
    spreading across
    the corpus callosum

    Frozen section description
    • Histologic features seen on frozen section are similar to those seen on permanent
    • Intraoperative cytology is an important component of intraoperative evaluation (Neuropathology 2016;36:313)
    Intraoperative frozen / smear cytology images

    Contributed by Genevieve M. Crane, M.D., Ph.D. and Lena Young, D.O.

    Squash preparation

    Frozen section

    Frozen section control

    Touch prep of
    monomorphic
    basophilic cells with
    crush artifact

    Touch prep

    Intraoperative squash preparation

    Microscopic (histologic) description
    • Characteristic angiocentric growth pattern with cuffs of neoplastic lymphocytes within and around blood vessels (Clin Neuropathol 2008;27:13)
    • Diffuse infiltration of brain parenchyma by small clusters or individual cells; necrosis is common
    • Accompanied by astrocytic and microglial activation and may be accompanied by a reactive inflammatory infiltrate
    • Neoplastic B cells are intermediate to large and frequently have vesicular chromatin and prominent nucleoli
    • Corticosteroid administration prior to biopsy may induce dramatic tumor shrinkage with increased macrophages and cellular debris
    Microscopic (histologic) images

    Contributed by Elizabeth Courville, M.D. and Lena Young, D.O.

    Perivascular infiltrate of atypical lymphoid cells

    Diffuse infiltrate
    of intermediate
    to large
    mononuclear cells

    Cytology of mononuclear cells

    Perivascular cuffing by neoplastic lymphocytes

    Infiltrate of neoplastic intermediate to large lymphocytes with background inflammation and necrosis


    Pleomorphic lymphocytes infiltrating brain

    Immunohisto-
    chemistry for B and T cells

    ISH shows lambda restriction

    Cytology description
    • Neoplastic lymphocytes in the cerebrospinal fluid are dispersed cells, increased in size compared to background reactive lymphocytes and frequently show irregular nuclear contours, abnormal chromatin and prominent nucleoli
    • Mostly discohesive groups of monomorphic, medium to large basophilic lymphoid cells with prominent smearing artifact and background reactive glial cells
    • Touch preparations show discohesive, large, atypical basophilic cells with a high nucleus to cytoplasm ratio and open chromatin with immature (lymphoblast) features; in post treatment tumors, only necrotic cells or reactive T cells may remain
    • Reference: Asian J Neurosurg 2013;8:195
    Negative stains
    • CD38, CD138 (plasma cell markers)
    • CD10 (< 10% are positive); CD10 positivity is seen more commonly in systemic diffuse large B cell lymphoma so should prompt careful search for systemic disease (Hematol Oncol 2014;32:57)
    • EBV positivity suggests underlying immunodeficiency
    • Pertinent negative stains include CD30, ALK1, HHV8, as they are helpful in the evaluation of mimicker neoplasms including systemic large B cell lymphoma, anaplastic large cell lymphoma and HHV8 associated lymphoproliferative disorders respectively
    Flow cytometry description
    • Specimen is not always sent for flow cytometry from stereotactic needle core biopsies due to limited material; flow cytometry of cerebrospinal fluid can aid in the diagnosis when needle core biopsy is not possible or to evaluate for leptomeningeal involvement
    • Flow cytometric immunophenotyping can identify abnormal B cell populations showing light chain restriction
    • References: J Exp Clin Cancer Res 2016;35:128, Cytometry B Clin Cytom 2018;94:928
    Molecular / cytogenetics description
    • MYC or BCL2 gene rearrangements are rare; BCL6 gene rearrangements are frequent (Blood Adv 2019;3:3953)
    • Oncogenic gain of function mutations in MYD88 (MYD88 L265P) are frequent (Blood 2016;127:869)
    Sample pathology report
    • Brain, biopsy:
      • Diffuse large B cell lymphoma (see comment)
      • Comment: In the absence of systemic disease, the morphologic and immunophenotypic findings are consistent with a diagnosis of primary diffuse large B cell lymphoma of the central nervous system. Clinical correlation is required, such as excluding an immunodeficiency associated lymphoproliferative disorder.
    Differential diagnosis
    • Systemic malignant lymphoma (diffuse large B cell lymphoma or other) involving the CNS:
      • CD10 expression by the lymphoma should strongly raise the possibility
      • Prior history and clinical workup is required to exclude systemic lymphoma involving the CNS
    • Lymphoma associated with HIV infection:
      • EBV positivity suggests this
      • Clinical workup is required
    • Posttransplant lymphoproliferative disorder (PTLD):
      • EBV positive
      • History of immunosuppression after allograft
    • High grade glioma:
      • Lacks the diffuse monomorphic lymphocyte infiltrate, neuronal morphology and IHC staining
      • CD45 negative
    • Infection including toxoplasma or cerebral abscess:
      • Occasionally multiloculate enhancement, generally encapsulated
      • Suppurative with mixed acute and chronic inflammation
    • Metastatic melanoma or carcinoma:
      • Cohesive groups of malignant cells that incite desmoplasia, CD45 negative, keratin or S100 positive, clinical correlation or history of malignancy
    • Demyelinating lesions:
      • May not enhance on imaging
      • Generally demarcated
      • Histiocytic as opposed to lymphoid infiltrate
    • Cerebral infarct:
      • Ischemic, "red dead" neurons
      • Slight to no lymphoid infiltrate
      • Histiocytes predominate
    Board review style question #1

    A 73 year old woman presents with progressive confusion and lethargy. Imaging shows a solitary lesion in the parietal lobe. A brain biopsy is performed and intraoperative touch preparations show discohesive, monomorphic, basophilic cells with crush artifact. Permanent sections show perivascular cuffing by intermediate to large mononuclear cells with scant cytoplasm, vesicular chromatin and prominent nucleoli. By immunohistochemical stains, the mononuclear cells are positive for CD20, BCL6 and MUM1 and negative for CD10 and EBV. What is the most likely diagnosis?

    1. Glioblastoma multiforme (GBM)
    2. Metastatic melanoma
    3. Metastatic neuroendocrine carcinoma
    4. Primary central nervous system lymphoma
    Board review style answer #1
    D. Primary central nervous system lymphoma. Based on the provided description, primary CNS lymphoma is the most likely diagnosis. This entity stains with B cells markers and is typically positive for MUM1 and negative for CD10. The entity primary CNS lymphoma excludes lymphomas associated with HIV infection or lymphomas arising after transplant. In contrast to the CNS lymphomas in those clinical scenarios, primary CNS lymphoma is almost always EBV negative.

    Comment Here

    Reference: Primary CNS lymphoma
    Board review style question #2
    Which of the following immunohistochemical stains is typically negative in the neoplastic lymphocytes in primary central nervous system (CNS) lymphoma?

    1. BCL6
    2. CD10
    3. CD20
    4. IRF4 / MUM1
    Board review style answer #2
    B. CD10. Less than 10% of cases of primary CNS lymphoma are positive for CD10. CD10 positivity is seen more commonly in systemic diffuse large B cell lymphoma so should prompt careful search for systemic disease. The other stains listed are typically positive in primary CNS lymphoma.

    Comment Here

    Reference: Primary CNS lymphoma

    Primary cutaneous CD4+ small / medium T cell lymphoproliferative disease
    Definition / general
    • Primary cutaneous CD4+ small / medium T cell lymphoproliferative disorder affects primarily upper dermis and is composed of a predominance of small to medium sized pleomorphic CD4+ T cells that presents as a solitary skin lesion without evidence of an underlying cutaneous lymphoma, such as mycosis fungoides, that is characterized by longstanding patches or plaques
    • By definition, the clinical course is benign
    Essential features
    • Solitary plaque or nodule on the face, neck or upper trunk
    • Slow growing
    • Epidermotropism only focal
    • Small / medium pleomorphic CD4+ T cells with T follicular helper (TFH) markers
    • Heterogenous background of hematolymphoid cells with hyperplastic B cells
    • Low proliferation rate by Ki67 (5 - 20%)
    • Large cells ≤ 30% of total infiltrating cells
    Terminology
    • Primary cutaneous CD4+ small / medium T cell lymphoma (no longer used due to benign clinical course)
    ICD coding
    • ICD-O: 9709/1 - primary cutaneous CD4+ small / medium T cell lymphoproliferative disorder
    Epidemiology
    • Rare, approximately 2% of all cutaneous T cell lymphomas / lymphoproliferative disorders
    • Usually adults or elderly; children rarely affected
    • M = F
    Sites
    • Solitary plaque or nodule on face, neck, upper trunk
    • Lower extremity, multifocal involvement is rare
    Etiology
    • Skin homing CD4+ T cells with follicular T helper phenotype
    Clinical features
    • Usually asymptomatic
    • Usually a solitary, red-purple, slow growing skin lesion commonly located on the face, neck or upper trunk
    • Rarely can present as multiple lesions; however, patches or plaques diagnostic of mycosis fungoides exclude the diagnosis
    Diagnosis
    • Tissue biopsy with complementary immunohistochemical stains, T cell receptor genes clonality studies and clinical correlation
    Prognostic factors
    • Excellent prognosis
    • Spontaneous remission following biopsy has been reported
    • Local recurrences are rare
    Case reports
    • 30 year old Chilean man with a 4 month history of multifocal, intermittently itchy, erythematous to violaceous nodules and plaques on the face (Clin Case Rep 2019;7:2405)
    • 36 year old woman with a progressively growing purplish erythematous nodule on the left nasal ala (An Bras Dermatol 2019;94:99)
    • 56 year old man with an 8 month history of multifocal cutaneous nodules, first appearing on his right leg with gradual progression to lesions on his left arm and bilateral legs (Cureus 2020;12:e8534)
    Treatment
    • Can be successfully treated with any of the following:
      • Intralesional steroids
      • Surgical excision
      • Radiotherapy
    Gross description
    • Solitary skin colored / red-purple plaque or nodule
    Frozen section description
    • Frozen sections should not be performed in these lesions
    Microscopic (histologic) description
    • Dense, diffuse or nodular dermal hematolymphoid infiltrates with extension into the subcutaneous tissue
    • Focal epidermotropism may be present but should be limited; prominent epidermotropism should not be seen
    • Small / medium sized pleomorphic CD4+ T cells are frequent
    • Large pleomorphic cells may be present but should be < 30% of all cells
    • Polymorphic background, including small CD8+ T cells, B cells, plasma cells and histiocytes including multinucleated giant cells
    • Ki67 proliferation index is low (5 - 20%)
    Microscopic (histologic) images

    Contributed by Jennifer Chapman, M.D.
    Dense dermal infiltrate

    Dense dermal infiltrate

    Predominantly small to medium sized lymphocytes

    Predominantly small to medium sized lymphocytes

    Scattered large cells

    Scattered large cells

    Neoplastic T cells showing strong CD3 expression

    Most cells show strong CD3 expression


    Neoplastic T cells showing strong CD4 expression

    Most cells show strong CD4 expression

    Neoplastic T cells showing strong PD-1 expression

    Numerous cells show strong PD1 expression

    Neoplastic T cells showing strong ICOS expression

    Subset of abnormal cells show strong ICOS expression

    Small CD8+ T cells

    Small CD8+ T cells

    Cytology description
    • Pleomorphic population of T cells: small to medium sized and occasional large lymphoid cells, which may have irregular nuclear contours
    • Large cells < 30% of all cells in the infiltrate
    • Heterogenous background of small lymphocytes, plasma cells, histiocytes and occasionally multinucleated giant cells
    Immunohistochemistry & special stains
    • Positive reactivity with T cell markers: CD3, CD5 and CD4, as well as T follicular helper markers BCL6 (variable), PD-1, ICOS and CXCL13
    • Loss of pan T cell markers is uncommon except for loss of CD7
    • Negative stains: CD8, CD30, CD10 and EBER
    Flow cytometry description
    • Abnormal CD4+ T cell population showing dim or negative expression of 1 or more pan T cell markers, negative for CD10; CD4:CD8 ratio may be normal due to the abundance of reactive CD8+ T cells
    • B cells with polytypic expression of kappa and lambda
    Molecular / cytogenetics description
    • Clonally rearranged TCR genes in most cases
    • Specific genetic abnormalities have not been described
    Sample pathology report
    • Face, left central malar cheek, biopsy:
      • Atypical CD4+ T cell infiltrate consistent with primary cutaneous CD4+ small / medium T cell lymphoproliferative disorder in an appropriate clinical context (see comment)
      • Comment: The patient is a 21 year old woman who recently developed a left cheek lesion. Histologic sections show skin punch biopsy with a dense dermal hematolymphoid infiltrate. The infiltrate is vaguely nodular and diffuse in distribution and is composed of mostly small to medium sized lymphocytes with scattered large, occasionally pleomorphic lymphoid cells. Clusters of small lymphoid cells and plasma cells are also present. Epidermotropism is focal. There is no diffuse growth of large cells.
      • Immunohistochemical stains highlight predominantly small to medium sized cells and occasional large, pleomorphic cells that are positive for the T cell markers CD3 and CD5, as well as for the T follicular helper markers BCL6, CXCL13 and PD1 (subset) and negative for CD10. The majority of the T cells present are CD4+ with relatively fewer CD8+ cells. In comparison to CD4+ T cells, CD8+ cells are small and cytologically unremarkable. CD20+ and CD79a+ B cells arranged in vague aggregates are present. B cells are small and mature in appearance. Kappa and lambda light chain assessment by in situ hybridization highlights few scattered, polytypic plasma cells. CD30 stains few, scattered, variably sized lymphoid cells. Concurrent polymerase chain reaction (PCR) for TRB and TRG demonstrated monoclonal T cell receptor gene rearrangements.
      • The overall findings are those of an atypical CD4+ T cell proliferation with histopathologic features consistent with those of primary cutaneous CD4+ small / medium T cell lymphoproliferative disorder. If this patient is presenting only with a solitary, slow growing lesion on the face, this clinical presentation would support this diagnosis. However, if the patient has multiple lesions or a recurring rash, other differential diagnostic possibilities, such as mycosis fungoides, should be considered. Clinical correlation is therefore required.
    Differential diagnosis
    Board review style question #1
    Which of the following features would be most helpful in favoring a diagnosis of primary cutaneous CD4+ small / medium T cell lymphoproliferative disorder over angioimmunoblastic lymphoma involving the skin?

    1. Clinical history of localized single lesion
    2. Destructive architecture
    3. High degree of cytologic atypia in CD4+ T cells
    4. Immunophenotype positive for PD1 and CXCL13
    5. Polymorphic nature of infiltrate, with small lymphocytes, plasma cells and histiocytes
    Board review style answer #1
    A. Clinical history of localized single lesion. Although angioimmunoblastic T cell lymphoma in primary sites (nodal) shows characteristic histologic features, including prominent arborizing vessels and expanded follicular dendritic cell meshworks, secondary involvement of cutaneous sites may not show these diagnostic features. The immunophenotype (answer D), degree of cytologic atypia (answer C) and polymorphic nature of associated nonneoplastic cells (answer E) is similar in angioimmunoblastic T cell lymphoma involving skin and CD4+ small / medium T cell lymphoproliferative disorder.

    Comment Here

    Reference: Primary cutaneous CD4+ small / medium T cell lymphoproliferative disorder
    Board review style question #2

    A 75 year old man presents with a 3 cm nodule on his left neck. He reports he previously had small red plaque-like lesions in the same area and on his upper back 2 years ago, which were treated with steroids with minimal improvement. Biopsy of the nodule demonstrated abnormal small to intermediate in size CD4+ T cells showing epidermotropism and scattered CD30+ large cells. What is the most likely diagnosis?

    1. Angioimmunoblastic T cell lymphoma
    2. CD4+ small / medium T cell lymphoproliferative disorder
    3. Mycosis fungoides
    4. Peripheral T cell lymphoma, NOS
    5. Primary cutaneous anaplastic large cell lymphoma
    Board review style answer #2
    C. Mycosis fungoides. The history of multiple, red plaque-like lesions unresolved following steroid treatment now presenting as a mass / nodule with epidermotropic atypical T cells is most likely consistent with mycosis fungoides, tumor stage. This clinical presentation is not consistent with that of CD4+ small / medium T cell lymphoproliferative disorder (answer B); that is characterized by the presence of a single lesion. The isolated cutaneous presentation would not be typical for angioimmunoblastic T cell lymphoma (answer A). Primary cutaneous anaplastic large cell lymphoma (answer E), characterized by the presence of sheets of large cells that express CD30, is excluded as a diagnostic possibility based on the description of only scattered CD30+ large cells. Since the histologic and clinical features are consistent with mycosis fungoides, this diagnosis would be favored as a more specific lymphoma classification over peripheral T cell lymphoma, NOS involving skin (answer D).

    Comment Here

    Reference: Primary cutaneous CD4+ small / medium T cell lymphoproliferative disorder

    Primary cutaneous CD8+ aggressive epidermotropic cytotoxic T cell lymphoma
    Definition / general
    Essential features
    ICD coding
    • ICD-O: 9709/3 - Cutaneous T cell lymphoma, NOS
    Sites
    Clinical features
    Diagnosis
    • Combination of clinical, histopathological and immunohistochemical features (J Am Acad Dermatol 2012;67:748)
    • Diagnostic criteria (J Am Acad Dermatol 2012;67:748)
      • Constant features
        • Aggressive clinical course with few weeks evolution
        • Widespread ulcerated papules, plaques and tumors without precursor lesion
        • Epidermotropism and nodular or diffuse dermal pleomorphic T cell infiltrate
        • CD8+ / CD4-
      • Variable features
        • Pyoderma gangrenosum-like, pagetoid reticulosis-like and annular erythematous scaling patches
        • Spongiosis, blisters and necrosis
        • Deep lymphoid infiltrate, angiocentrism and angiodestruction
        • Variable expression of CD2, CD7, CD15, CD45 and CD56
    Prognostic factors
    Case reports
    • 16 year old boy with primary cutaneous aggressive epidermotropic CD8+ T cell lymphoma with cerebral infiltration and treated with cord blood stem cell transplant (SCT) (Intern Med 2018;57:2051)
    • 19 year old woman with primary cutaneous aggressive epidermotropic CD8+ T cell lymphoma simulating pyoderma gangrenosum (Clin Exp Dermatol 2009;34:e261)
    • 29 year old woman with primary cutaneous aggressive epidermotropic CD8+ T cell lymphoma and homozygous mutation in SAMHD1 (JAAD Case Rep 2015;1:227)
    • 47 year old man with primary cutaneous aggressive epidermotropic CD8+ T cell lymphoma and complete durable remission after autologous and allogeneic hematopoietic stem cell transplantation (JAAD Case Rep 2017;3:196)
    • 61 year old man with primary cutaneous aggressive epidermotropic CD8+ T cell lymphoma and history of psoriasis (Ann Dermatol 2018;30:255)
    • 68 year old man with lupus-like initial clinical presentation (Indian Dermatol Online J 2019;10:298)
    • 72 year old man with primary cutaneous aggressive epidermotropic CD8+ cytotoxic T cell lymphoma of the ear (J Laryngol Otol 2007;121:503)
    Treatment
    • No definite successful therapy available
    • Multiagent chemotherapy: CHOP (cyclophosphamide, hydroxydorubicin, vincristine and prednisone), CHOEP (cyclophosphamide, hydroxydorubicin, vincristine, etoposide and prednisone), hyper-CVAD (hyper-cyclophosphamide, vincristine, doxorubicin, dexamethasone, methotrexate and cytarabine) and MACOP-B (methotrexate, doxorubicin, cyclophosphamide, vincristine, prednisone and bleomycin) (J Am Acad Dermatol 2012;67:748, Mod Pathol 2017;30:761)
      • Unsatisfactory and usually partial response
    • Total skin electron beam therapy with or without oral retinoid (e.g. bexarotene) (J Am Acad Dermatol 2012;67:748, J Am Acad Dermatol 2010;62:300)
      • Partial response and some cases with complete clearance
    • Allogeneic or autologous stem cell transplant (SCT) (J Am Acad Dermatol 2012;67:748, Mod Pathol 2017;30:761, J Am Acad Dermatol 2010;62:300)
      • Setting of unresponsive or relapse after multiagent chemotherapy or early use of combined intensive chemotherapy and SCT
    Microscopic (histologic) description
    Microscopic (histologic) images

    Contributed by Roberto N. Miranda, M.D.
    Skin epidermotropism and dermal infiltrate

    Skin epidermotropism and dermal infiltrate

    Epidermotropism with intraepidermal cluster

    Epidermotropism with intraepidermal cluster

    Lymphoma at the dermoepidermal junction

    Lymphoma at the dermoepidermal junction

    Large pleomorphic cells

    Large pleomorphic cells

    CD3 positivity

    CD3 positivity

    CD4 negativity of epidermotropic cells

    CD4 negativity of epidermotropic cells


    CD7 positivity of epidermotropic cells

    CD7 positivity of epidermotropic cells

    CD8 positivity of epidermotropic cells

    CD8 positivity of epidermotropic cells

    TIA1 positivity of lymphoma cells

    TIA1 positivity of lymphoma cells

    Proliferation marker Ki67

    Proliferation marker Ki67

    Pagetoid reticulosis

    Pagetoid reticulosis

    Microabscess of Pautrier in pagetoid reticulosis

    Microabscess of Pautrier in pagetoid reticulosis


    CD3 in pagetoid reticulosis

    CD3 in pagetoid reticulosis

    CD8 in pagetoid reticulosis

    CD8 in pagetoid reticulosis

    βF1 pagetoid reticulosis

    βF1 pagetoid reticulosis

    Folliculotropism in CD8 epidermotropic TCL

    Folliculotropism in CD8 epidermotropic TCL

    Epidermotropism in CD8 epidermotropic TCL

    Epidermotropism in CD8 epidermotropic TCL

    Epidermotropic small lymphoma cells

    Epidermotropic small lymphoma cells


    Liver involvement by CD8 cytotoxic TCL

    Liver involvement by CD8 cytotoxic TCL

    CD3 positive lymphoma cells

    CD3 positive lymphoma cells

    CD8 positive lymphoma cells

    CD8 positive lymphoma cells

    Molecular / cytogenetics description
    Sample pathology report
    • Ulcerated plaque of the trunk, skin punch biopsy:
      • Primary cutaneous T cell lymphoma, most consistent with primary cutaneous CD8+ aggressive epidermotropic cytotoxic T cell lymphoma (see comment)
      • Comment: Section shows an adequate skin punch biopsy with intense epidermotropism, focal epidermal necrosis, acanthosis and spongiosis. There is an extensive, nodular and diffuse dermal infiltrate composed of variably sized, pleomorphic lymphocytes with irregular nuclear contours, hyperchromatic nuclei and scant cytoplasm. Immunohistochemical studies show that the abnormal lymphocytes are diffusely positive for CD3, CD7, CD8, CD45RA, TIA1, T cell receptor beta (βF1) and granzyme B. The atypical cells are negative for CD2, CD4, CD5, CD56, EBER, CD30, T cell receptor gamma / delta and ALK1. The Ki67 proliferation index is approximately 70%. According to clinical notes, the patient is a 63 year old man with history of hypertension. The patient presented to the department of dermatology with a 5 week history of ulcerated plaques, papules and nodules in the upper and lower limbs, trunk, head, neck, tongue and perioral area. A computed tomography (CT) showed multiple nodules in liver, without evidence of lymphadenopathy. The final diagnosis is most consistent with primary cutaneous CD8+ aggressive epidermotropic cytotoxic T cell lymphoma.
    Differential diagnosis
    Board review style question #1
      What immunophenotype is more commonly expressed by primary cutaneous CD8+ aggressive epidermotropic cytotoxic T cell lymphoma?

    1. CD3+ / CD4- / CD7- / CD8+ / CD30- / EBER-
    2. CD3+ / CD4- / CD7+ / CD8+ / CD30- / EBER+
    3. CD3+ / CD4+ / CD7+ / CD8+ / CD30- / EBER+
    4. CD3+ / CD4+ / CD7+ / CD8+ / CD30+ / EBER-
    Board review style answer #1
    A. CD3+ / CD4- / CD7- / CD8+ / CD30- / EBER-

    Comment Here

    Reference: Primary cutaneous CD8+ aggressive epidermotropic T cell lymphoma
    Board review style question #2
      What is a consistent feature for the diagnostic criteria of primary cutaneous CD8+ aggressive epidermotropic T cell lymphoma?

    1. Pyoderma gangrenosum-like
    2. Spongiosis, blistering and necrosis
    3. Variable expression of CD2, CD4, CD5, CD7 and CD8
    4. Widespread ulcerated papules, plaques and tumors without precursor lesion
    Board review style answer #2
    D. Widespread ulcerated papules, plaques and tumors without precursor lesion

    Comment Here

    Reference: Primary cutaneous CD8+ aggressive epidermotropic T cell lymphoma

    Primary cutaneous DLBCL-leg

    Primary cutaneous PTCL, NOS (pending)
    [Pending]

    Primary cutaneous acral CD8+ lymphoproliferative disorder
    Definition / general
    • Rare indolent lymphoproliferative disorder characterized by slow growing papules or nodules in acral sites with a dermal infiltrate of CD8+ T lymphocytes
    Essential features
    Terminology
    ICD coding
    • ICD-10: C84.A0 - cutaneous T cell lymphoma, unspecified, unspecified site
    Epidemiology
    Sites
    • Usually localized to the dermis in the ear (Br J Dermatol 2022;186:887)
    • Multiple sites of involvement may be present
    • Other sites may include the nose, hands, elbows and feet
    • No extracutaneous involvement is present
    Pathophysiology
    • Pathophysiology is not known
    Etiology
    • Cell of origin is activated mature T lymphocyte, CD3+ / CD8+ type
    Clinical features
    • Painful, slowly growing single papule or nodule localized in the ear
    • Always an indolent course (Blood 2016;127:2375)
    • Cure is expected after local excision or localized therapy in 70% of patients; persistent disease in 20%
    Diagnosis
    • Biopsy or excision
    Radiology description
    • Single lesion localized to dermis
    • No systemic dissemination
    • Rare multifocal
    Prognostic factors
    • Recurrences are rare and are limited to the dermis
    • Surgical excision or radiotherapy usually result in cure (70%) with an overall low relapse rate (20%) (Br J Dermatol 2022;186:887)
    • Relapsing / progressing disease more frequent in younger patients (Br J Haematol 2019;185:598)
      Case reports
      • 16 year old boy with an enlarging solitary lesion on the right lower eyelid of 1 month duration and presence of CD56 expression and high Ki67 proliferation (J Cutan Pathol 2021;48:1489)
      • 35 year old man and 45 year old woman with nasal involvement and similar clinical and histomorphological features (J Cutan Pathol 2010:37;977)
      • 52 year old man and 41 year old man with indolent CD8+ lymphoid proliferation of the ear (J Cutan Pathol 2011:38;209)
      • 53 year old woman with a 12 month history of an asymptomatic solitary right leg nodule with atypical BCL6 / GATA3 positivity (Am J Dermatopathol 2021;43:137)
      • 57 year old man with history of slowly growing ear nodule with signet ring features (Histopathology 2009;55:468)
      • 69 year old man with a 10 year history of a solitary slow growing plaque on the left ear and a 4 year history of an identical plaque on the right ear with presence of double positive CD8 / CD4 and associated with thymic carcinoma (J Cutan Pathol 2019;46:231)
      • 69 year old man with history of relapsing asymptomatic nodules confined to the left ear with extracutaneous involvement (J Cutan Pathol 2017;44:964)
      • 3 cases with largest review of the literature, including 52 additional cases (J Cutan Pathol 2016;43:125)
      • 3 cases with ear involvement and review of the literature (Am J Dermatopathol 2019;41:644)
      • First report defining the entity included 4 cases with ear involvement (Am J Surg Pathol 2007:31:1887)
      Treatment
      • Surgical excision or radiotherapy
      Microscopic (histologic) description
      • Dense, diffuse dermal infiltrate with sparing of the epidermis and presence of a thin grenz zone
      • Infiltrate is composed of medium to large size atypical lymphocytes (blast-like) with irregular nuclear membranes, small nucleoli and scant cytoplasm; signet ring cells can be present
      • No fat rimming, angiodestruction or necrosis is identified
      • Focal epidermotropism and exocytosis of atypical lymphocytes have been noted in rare cases
      • References: Arch Pathol Lab Med 2017;141:1469, Br J Dermatol 2022;186:887
      Microscopic (histologic) images

      Contributed by Roberto N. Miranda, M.D. and Alexandra Hristov, M.D.

      Dermal infiltrate with grenz zone

      Morphology of infiltrates

      Characteristic CD68 staining

      Cytotoxic immunophenotype

      Positive stains
      Negative stains
      Molecular / cytogenetics description
      • Clonal TCR gene rearrangements present by PCR testing in almost 100% of cases
      Sample pathology report
      • Skin, left ear, punch biopsies:
        • CD8+ positive cutaneous lymphoproliferative disorder (see comment)
        • Comment: The morphological features and immunohistochemical findings are most consistent with a low grade CD8+ cutaneous lymphoproliferative disorder. If the lesion is unique and localized to the ear, the findings are most consistent with primary cutaneous acral CD8+ lymphoproliferative disorder (Leukemia 2022;36:1720).
      Differential diagnosis
      • Hydroa vacciniforme-like lymphoma:
        • Can show a similar infiltrate with CD8+ cytotoxic immunophenotype but is an EBV+ T cell lymphoproliferative disease that occurs in children (mean age 8 years)
        • Atypical infiltrate is mainly located around adnexa and vessels with angiodestruction
      • Primary cutaneous CD8+ aggressive epidermotropic cytotoxic T cell lymphoma:
        • Features skin papules or nodules with an aggressive clinical course, characterized by rapid progression, ulceration, poor response to chemotherapy and a 5 year overall survival rate of 32% (Mod Pathol 2017;30:761)
        • Morphologically characterized by full thickness pagetoid epidermotropism
        • Occasional cases show predominant dermal infiltrates; however, this is associated with atrophic epidermis and infiltration of adjacent adnexal structures (Mod Pathol 2017;30:761)
        • Tumor cells display a high Ki67 proliferation index (more than 50%)
      • Primary cutaneous gamma delta T cell lymphoma:
        • Morphologically involves the subcutaneous tissue and commonly has epidermotropism or ulceration
        • Usually the tumor cells are CD4- / CD8-; however, a few cases are CD4- / CD8+
        • Characteristically this disease features CD56 and TCR gamma expression
        • Aggressive clinical course
      • Subcutaneous panniculitis-like T cell lymphoma:
        • Solitary or multiple deeply seated plaques or subcutaneous nodules
        • Only subcutis involved, morphologically lobular panniculitis with septal sparing; adipocyte rimming with fat necrosis
        • High Ki67 proliferation index within CD8+ atypical lymphocytes rimming the fat lobules
      • Mycosis fungoides, cytotoxic:
        • Rare variant of mycosis fungoides
        • Presents as other cases of MF, with generalized, scaly patches and plaques, ulcerated nodules
        • Predilection for buttocks and sun protected areas
        • Morphologically has epidermotropism and shows Pautrier microabscesses, as well as intraepidermal vesiculation
      • Primary cutaneous peripheral T cell lymphoma, unspecified:
        • Solitary or multiple ulcerating nodules
        • Medium to large sized cells with moderate to marked pleomorphism
        • Positive expression of T cell markers, including loss of antigens; may coexpress TIA1, granzyme B or perforin
        • Aggressive clinical behavior
      Board review style question #1

      Which of the following statements is correct regarding primary cutaneous acral CD8+ lymphoproliferative disorder?

      1. Characterized by aggressive course with progressive disease and resistance to therapy
      2. Epidermotropism is usually absent
      3. Loss of expression of pan-T cell markers CD2, CD5 and CD7 is rarely observed
      4. Neoplastic cells feature a Ki67 proliferation index that ranges from 50 to 90%
      5. Restricted to the skin of the ear
      Board review style answer #1
      B. Epidermotropism is usually absent

      Comment Here

      Reference: Primary cutaneous acral CD8+ lymphoproliferative disorder

      Primary cutaneous anaplastic large cell lymphoma

      Primary cutaneous follicle center lymphoma

      Primary cutaneous gamma delta
      Definition / general
      Essential features
      Terminology
      • Obsolete terms:
        • Histiocytic cytophagic panniculitis
        • Weber-Christian disease
      ICD coding
      • ICD-O: 9726/3 - primary cutaneous gamma delta T cell lymphoma
      • ICD-10: C84.A - cutaneous T cell lymphoma, unspecified
      Pathophysiology
      Etiology
      • T cell receptors (TCR) are responsible for antigen recognition (Clin Hematol Int 2022;4:1, Nat Commun 2020;11:1806, Front Immunol 2021;12:627139, Surg Pathol Clin 2021;14:177, Nat Rev Clin Oncol 2009;6:707, Blood 2003;101:3407)
        • TCRs are usually classified in 2 groups (alpha beta and gamma delta chains) based on the type of antigen binding site
          • Linked with CD3 subunits; crucial for TCR insertion and signaling
          • TCR alpha and TCR gamma: chromosome 14
          • TCR beta and TCR delta: chromosome 7
          • Double negative 3 (DN3) stage (CD4- / CD8- / CD44- / CD25+) is determinant for the differentiation of alpha beta or gamma delta subsets
        • Gamma delta T cells are < 5% of circulating T cells
          • More abundant in spleen (red pulp)
          • Less dependent on thymic maturation
          • Strong plasticity and regulation of macrophage homeostasis
          • Role in B cell maturation
          • Possible role in the initial phase of infection: innate and adaptive immunity
            • Nonprocessed major histocompatibility complex (MHC) proteins, lipid and direct bacterial antigens
            • Memory-like characteristics
        • TCR diversity
          • Rearrangement of several segments: C (constant), D (diversity), J (joining) and V (variable)
            • V1δ and V2δ: > 97% of gamma delta T cells
              • V1δ
                • Thymus, intestine and spleen
                • More common in cases arising in superficial layers of skin
                • Cytokines: IL10, IL2, IL4, IL17, IFN gamma and TNF alpha
              • V2δ
                • Peripheral blood and lymph node
                • More common in panniculitic cases
                • Cytokines: IL21, IL17, IL24, IFN gamma and TNF alpha
              • V3δ
                • Peripheral blood and liver
                • Cytokines: IL4, IL10, IL17, IFN gamma and TNF alpha
          • Addition of nucleotides to the rearranged genes
      Clinical features
      Diagnosis
      Laboratory
      • No specific laboratory findings (Clin Hematol Int 2022;4:1)
        • Elevated lactate dehydrogenase (LDH): up to 30%
        • Cytopenias
          • Isolated or associated with hemophagocytic lymphohistiocytosis
      Prognostic factors
      Case reports
      • 45 year old woman with an indolent primary cutaneous gamma delta T cell lymphoma mimicking lupus erythematous profundus (Front Oncol 2020;10:133)
      • 54 year old man with primary cutaneous gamma delta T cell lymphoma mimicking leukemia cutis by chronic lymphocytic leukemia (J Cutan Pathol 2022;49:1015)
      • 60 year old man with primary cutaneous gamma delta T cell lymphoma in complete remission after bendamustine monotherapy (JAMA Dermatol 2020;156:1029)
      • 60 year old man with predominantly epidermotropic primary cutaneous gamma delta T cell lymphoma with aggressive clinical behavior albeit bland histopathological findings (Am J Dermatopathol 2016;38:e147)
      • 64 year old woman with primary cutaneous gamma delta T cell lymphoma and dermatomyositis initially diagnosed as subcutaneous panniculitis-like T cell lymphoma (Dermatopathology (Basel) 2022;9:143)
      • 67 year old man with a previous history of rheumatoid arthritis in treatment with etarnercept and a diagnosis of primary cutaneous gamma delta T cell lymphoma (Acta Derm Venereol 2009;89:653)
      Treatment
      • No standard treatment and poor response to treatment (J Am Acad Dermatol 2021;85:1093, Clin Hematol Int 2022;4:1, Surg Pathol Clin 2021;14:177)
        • Localized disease
          • Radiation therapy
        • Disseminated disease
          • Immunotherapy
            • Brentuximab (for CD30 positive cases)
            • Mogamulizumab
            • Alemtuzumab
          • Multiagent chemotherapy with or without allogeneic stem cell transplantation
            • Cyclophosphamide, doxorubicin hydrochloride, vincristine and prednisone (CHOP)
            • Etoposide and CHOP (EPOCH)
            • Ifosfamide, carboplatin and etoposide (ICE)
            • Hyperfractionated cyclophosphamide, vincristine, doxorubicin and dexamethasone (hyper CVAD)
      Clinical images

      Contributed by Roberto N. Miranda, M.D.
      Ulcerated tumors

      Ulcerated tumors

      Erythematous plaques

      Erythematous plaques

      Microscopic (histologic) description
      Microscopic (histologic) images

      Contributed by Roberto N. Miranda, M.D.
      Diffuse infiltrate

      Diffuse infiltrate

      Epidermotropic infiltrate

      Epidermotropic infiltrate

      Fat rimming

      Fat rimming

      Cytologic atypia Cytologic atypia

      Cytologic atypia

      TCR alpha beta negativity

      TCR alpha beta negativity


      Pagetoid pattern Pagetoid pattern

      Pagetoid pattern

      Hemophagocytosis

      Hemophagocytosis

      CD3 positivity

      CD3 positivity

      CD4 loss

      CD4 loss

      CD4 negativity (CD4 loss)

      CD4 negativity (CD4 loss)


      CD7 negativity (CD7 loss)

      CD7 negativity (CD7 loss)

      CD8 positivity

      CD8 positivity

      TIA1 positivity

      TIA1 positivity

      TCR delta positivity TCR delta positivity

      TCR delta positivity

      TCR beta chain negativity

      TCR beta chain negativity

      Flow cytometry description
      Flow cytometry images

      Contributed by Roberto N. Miranda, M.D.
      TCR gamma delta positivity

      TCR gamma delta positivity

      TCR gamma delta positivity and TCR alpha beta chain negativity

      TCR gamma delta
      positivity and
      TCR alpha beta
      chain negativity

      Molecular / cytogenetics description
      Sample pathology report
      • Right thigh, proximal, skin punch:
        • Primary cutaneous gamma delta T cell lymphoma (see comment)
        • Comment: According to clinical notes, this is a 64 year old man with a history of multiple ulcerated nodules, plaques and tumors in the extremities of a very recent onset (past 2 months). The patient has a previous history of diabetes mellitus in regular treatment with metformin.
        • The histologic sections of the punch biopsy show an ulcerated lesion infiltrated by intense lymphoid infiltrate located in the epidermis, dermis and subcutaneous tissue. This infiltrate is composed predominantly of medium sized lymphocytes with irregular nuclear contours, clumped chromatin and scant eosinophilic cytoplasm. Mitotic figures, necrosis and hemophagocytosis are noted.
        • Immunohistochemical studies reveal the neoplastic cells are positive for CD2, CD3, CD56, granzyme B, perforin, TIA1 and TCR delta and negative for CD4, CD7, CD8, CD30, CCR4, EBER, TdT and TCR alpha beta. CD5 and CD7 present a partial loss in the neoplastic infiltrate.
        • Concurrent flow cytometry immunophenotype of the bone marrow and peripheral blood shows absent monoclonal T cell population. Clinical correlation and follow up is recommended.
      Differential diagnosis
      Board review style question #1

      Which of the following is true about primary cutaneous gamma delta T cell lymphoma?

      1. Frequently presents with an indolent clinical behavior
      2. Localized plaques and nodules are the most common form
      3. Lymph node and bone marrow involvement are uncommon
      4. Subcutaneous forms have a better prognosis
      Board review style answer #1
      C. Lymph node and bone marrow involvement are uncommon. Primary cutaneous gamma delta T cell lymphoma usually presents as rapidly progressing disseminated and ulcerated nodules, plaques and tumors. The predominantly epidermotropic form conveys a better prognosis.

      Comment Here

      Reference: Primary cutaneous gamma delta
      Board review style question #2

      Which of the following immunophenotypic patterns corresponds to primary cutaneous gamma delta T cell lymphoma?

      1. CD2+, CD3-, CD4+, CD8+, TIA1-, granzyme B-, TCRγδ+
      2. CD2+, CD3+, CD4+, CD8-, TIA1+, granzyme B-, TCRɑβ+
      3. CD2+, CD3-, CD4-, CD8+, TIA1-, granzyme B+, TCRγδ+
      4. CD2+, CD3+, CD4-, CD8-, TIA1+, granzyme B+, TCRγδ+
      Board review style answer #2
      D. CD2+, CD3+, CD4-, CD8-, TIA1+, granzyme B+, TCRγδ+. This is the most typical immunophenotype.

      Comment Here

      Reference: Primary cutaneous gamma delta

      Primary effusion lymphoma
      Definition / general
      • Rare type of diffuse large B cell lymphoma with lymphomatous effusions in pleural, pericardial and abdominal cavities but no tumor mass
      Terminology
      • Also called body cavity lymphoma
      Clinical features
      • Only one body cavity typically involved
      • Strongly associated with HHV8 and advanced HIV; usually EBV+
      • Also occurs in nonimmunosuppressed patients, often elderly
      • Rarely, extracavitary tumors with features of PEL are seen in GI tract, skin, lung, CNS and lymph nodes
      • Survival usually only months
      • Solid variant: very rare; affects GI tract, skin, lung, cerebrum; rarely nodal (Hum Pathol 2002;33:846); may represent a heterogeneous group of disorders
      Case reports
      Microscopic (histologic) description
      • Immunoblastic cells with abundant basophilic cytoplasm, round nucleus, single prominent nucleoli, perinuclear halo, variable nuclear pleomorphism with binucleated cells
      • Occasional large Reed-Sternberg like and anaplastic cells
      • Rarely is solid with plasmablastic features
      • Solid variant: large pleomorphic cells; nodal case resembled anaplastic large cell lymphoma
      Microscopic (histologic) images

      Contributed by Mario L. Marques-Piubelli, M.D. and Roberto N. Miranda, M.D. (Case #519)
      Extracavitary presentation of PEL

      Extracavitary presentation of PEL

      EBER

      EBER

      LANA-1

      LANA-1

      LANA-1 in an extracavitary presentation

      LANA-1 in an extracavitary presentation



      Images hosted on other servers:

      70 year old man with HHV8+, EBV+ pleural effusions

      Cytology images

      Contributed by Mario L. Marques-Piubelli, M.D. and Roberto N. Miranda, M.D. (Case #519)
      Diff-Quik

      Diff-Quik

      Wright-Giemsa

      Wright-Giemsa

      PEL Thinprep

      PEL Thinprep

      Positive stains
      Negative stains
      Molecular / cytogenetics description
      • Often required for diagnosis since negative for B cell stains
      • Clonal rearrangements and somatic mutations of Ig heavy chain
      • Clonal Epstein Barr virus and HHV8 genetic sequences
      • Also gain of sequences in chromosomes 12 and X (Arch Pathol Lab Med 2000;124:824)
      Differential diagnosis
      Additional references
      Board review style question #1
      Which of the following pair of viruses are associated with primary effusion lymphoma (PEL)?

      1. EBV and HCV
      2. EBV and HHV-8
      3. HHV-8 and COVID-19
      4. HHV-8 and HPV
      Board review style answer #1
      B. EBV and HHV-8

      Comment Here

      Reference: Primary effusion lymphoma
      Board review style question #2
      Which is the latency pattern of the Epstein-Barr virus found associated with primary effusion lymphoma (PEL)?

      1. Latency Pattern 1: EBER(+), EBNA-1(+), LMP-1(-)
      2. Latency Pattern 1: EBER(+), EBNA-1(+), LMP-1(+)
      3. Latency Pattern 3: EBER(+), EBNA-2(-), LMP-1(+)
      4. Latency Pattern 2: EBER(+), EBNA-2(-), LMP-1(+)
      Board review style answer #2
      A. Latency Pattern 1: EBER(+), EBNA-1(+), LMP-1(-)

      Comment Here

      Reference: Primary effusion lymphoma

      Primary follicular lymphoma-testis
      Definition / general
      • Primary follicular lymphoma of testis (PFLT) in children and adolescents is a subset of extranodal follicular lymphoma (FL)
      • Lymphoma composed of follicle center B cells that arise in testis
      • Follicular lymphoma involving testis as manifestation of systemic disease is excluded
      • Not recognized as specific variant in 2017 World Health Organization classification
      Essential features
      • Localized disease limited to the testis
      • Favorable course despite its aggressive histology
      • Systemic relapses are uncommon
      Terminology
      ICD coding
      • ICD-O: 9695/3 - Follicular lymphoma
      • ICD-10: C82.90 - Follicular lymphoma, unspecified
      Epidemiology
      • Primary non-Hodgkin lymphoma (NHL) of testis: 1% of all cases of non-Hodgkin lymphoma
      • Primary follicular lymphoma of testis is rare
      • Limited to single case reports or small case series
      • Most cases series are in children and young adults
      • Less common in adults
        • In adults non-Hodgkin lymphoma of the testis is mostly diffuse large B cell lymphoma (DLBCL)
      • Reference: Am J Surg Pathol 2007;31:1050
      Sites
      • Testis
      Pathophysiology / etiology
      Diagrams / tables

      Clinical features:
      Nodal follicular lymphoma Testicular follicular lymphoma
      Age (median) Adults and elder (sixth decade) Children and young adults
      Gender (M:F) Men and women Men only
      Affected sites Lymph nodes with extranodal spread Testicle and adnexa
      Symptoms Generalized lymphadenopathy Painless mass
      Stage (Ann Arbor) High (III - IV) in most cases IE

      Pathologic features:
      Nodal follicular lymphoma Testicular follicular lymphoma
      Gross appearance Discrete mass or complete effacement Discrete mass or diffuse involvement
      Histologic grade Grades 1 - 3 Grade 3

      Immunophenotype:
      Nodal follicular lymphoma Testicular follicular lymphoma
      CD10 Variable Variable
      BCL2 Usually positive Negative

      Molecular features:
      Nodal follicular lymphoma Testicular follicular lymphoma
      IGH-BCL2 Present, up to 90% Negative
      Clinical features
      • Painless discrete mass or diffuse testicular enlargement
      • May be mistaken with hydrocele (Cancer 1999;85:1626)
      • No peripheral lymphadenopathy
      • No hepatosplenomegaly
      • No B symptoms
      • Reported cases are predominantly at clinical stage of IE
      Radiology description
      • Ultrasonography:
        • Hypoechoic areas
        • Variably hypervascular
      Prognostic factors
      • Excellent prognosis for children and young adults with stage IE disease
        • Enrollment in Children’s Oncology Group may be important for developing future advances in treatment
      • Small number of adult cases with generally short follow up
      • Reference: J Pediatr Hematol Oncol 2004;26:104
      Case reports
      Treatment
      • Resection
      • With or without subsequent chemotherapy
      Gross description
      • Discrete mass or diffuse involvement of testis
      • Fleshy tan or pink distinct nodule
      • Either poorly or well demarcated
      • 1.2 - 4.0 cm
      • Reference: J Pediatr Hematol Oncol 2012;34:68
      Frozen section description
      • Same criteria applied in microscopic evaluation
      • When possible, spare tissue for cytogenetic analysis, flow cytometry immunophenotype; preparation of imprints is recommended
      Microscopic (histologic) description
      • Nodular pattern consistent with follicular lymphoma
      • Follicular lymphoma involves mainly testicular parenchyma and secondarily adnexa
        • Neoplastic follicles grow between tubuli
        • May completely replace testicular parenchyma
      • Reported cases are predominantly grade 3A
        • Large lymphocytes (centroblasts) predominate
        • Some small cleaved lymphocytes (centrocytes) are admixed
      • Occasionally, focal areas of diffuse growth support focal diffuse large B cell lymphoma
      • Prominent interstitial fibrosis (Am J Surg Pathol 2007;31:1050)
      • Mantle zone usually attenuated or lost around neoplastic follicles
      Microscopic (histologic) images

      Contributed by Roberto N. Miranda, M.D.

      Follicular lymphoma of testis

      Seminiferous tubuli and neoplastic follicle

      Large centroblasts of follicular lymphoma

      BCL6 IHC

      CD21 IHC

      DLBCL of testis

      Cytology description
      • Majority of cells: large noncleaved cells (centroblasts)
      • Multilobated large cells
      • May be an admixture of centroblasts and small centrocytes
      • Reference: Hum Pathol 2012;43:1514
      Positive stains
      • Pan-B cell antigens
      • Monotypic surface Ig
      • CD10 (variable) and BCL6 protein (variable)
        • Weakly expressed by cells in the interfollicular region
      • Ki67 / MIB1: high proliferation rate
      • p53 protein: often positive in primary follicular lymphoma of testis of adults
      • Reference: Arch Pathol Lab Med 2001;125:428
      Negative stains
      Molecular / cytogenetics description
      • Monoclonal IGH rearrangements
      • IGH-BCL2 fusion is uncommon based on PCR and FISH
      • With or without BCL6 translocations (Cancer 1999;85:1626)
      Sample pathology report
      • Right testicular tumor, orchiectomy:
        • Follicular lymphoma with a follicular pattern, grade 3A, with prominent sclerosis (see comment)
        • Comment: Histologic sections demonstrate testicular parenchyma almost completely involved by malignant lymphoma. The lymphoma consists of numerous neoplastic follicles, that are composed of large noncleaved cells (centroblasts) that average > 15 per high power field, admixed with small cleaved lymphocytes (centrocytes) supporting a diagnosis of follicular lymphoma grade 3A. The neoplastic cells surround and invade seminiferous tubulei displacing germ cells and Sertoli cells. Tumor cells inside of the seminiferous tubulei are also seen. Mitoses are easily found. Small areas of infarction are seen in the testicular parenchyma. The noninvolved testicular parenchyma is morphologically unremarkable.
      • Immunophenotypic studies show that the tumor cells are positive for CD10, CD20, CD45 (LCA) and BCL6 (less than 10%). The aberrant cells are negative for CD3, CD5, MUM1 and BCL2. CD21 highlights residual follicular dendritic cell networks in a subset of the neoplastic follicles. The proliferation index as determined by Ki67 is approximately 60%.
      • Fluorescence in situ hybridization (FISH) performed in the laboratory of cytogenetics did not detect rearrangement of the MYC gene. However, extra copies of the MYC gene consistent with an aneuploid cell population were detected.
      • FISH studies failed to detect the presence of of t(14;18) or IGH-BCL2 fusion.
      Differential diagnosis
      • Diffuse large B cell lymphoma (DLBCL):
        • Diffuse large B cell lymphoma is most common lymphoma involving testis
          • Primary follicular lymphoma of testis can be grade 3 and predominantly diffuse
        • Features supporting diffuse large B cell lymphoma over extranodal follicular lymphoma
          • Evidence of other disease sites in patients with systemic diffuse large B cell lymphoma
          • No areas of follicular pattern in primary diffuse large B cell lymphoma of testis
          • Diffuse large B cell lymphoma often CD10-, BCL6-
      • Reactive follicular hyperplasia:
        • Follicles show polarization
        • Surrounded by distinct mantle zones
        • Follicles have polymorphic appearance
        • Tingible body macrophages
        • B cell antigens+, CD10+, BCL6+, BCL2-
        • No evidence of monoclonal IGH rearrangements
      Board review style question #1
      Which of the following extranodal follicular lymphomas is predominantly high grade?

      1. Ocular adnexa
      2. Small intestine
      3. Stomach
      4. Testis
      Board review style answer #1
      Board review style question #2


      A 26 year old man has presented with a testicular mass without any previous history of infection or autoimmune disease. Monoclonal IGH rearrangement is detected on the fresh tissue submitted at the time of excision. Which of the following options is most likely correct about this patient or patient's pathology sample?

      1. IGH-BCL2 rearrangement is detected in cytogenetic studies
      2. Lymphoma has high grade feature
      3. Patient has generalized lymphadenopathy
      4. Patient's prognosis is poor despite therapy
      Board review style answer #2
      B. Lymphoma has high grade feature

      Comment Here

      Reference: Testicular follicular lymphoma

      Primary follicular lymphoma-testis

      Primary mediastinal
      Definition / general
      • Mature and clinically aggressive large B cell lymphoma
      • Probable thymic B cell origin
      • Arising in mediastinum but may involve other sites (lung)
      • Characteristic clinical, immunophenotypic, genetic and molecular features
      Essential features
      • Uncommon to arise outside of mediastinum (Am J Surg Pathol 2015;39:1322)
      • Occurs more frequently in young adults
      • Must be distinguished from systemic diffuse large B cell lymphoma with secondary mediastinal involvement, classic Hodgkin lymphoma and gray zone lymphoma with features intermediate between large B cell lymphoma and classic Hodgkin lymphoma
      • Genetic overlap based on gene expression profile between primary mediastinal large B cell lymphoma and classic Hodgkin lymphoma
      Terminology
      • Primary mediastinal clear cell lymphoma of B cell type (terminology no longer used)
      • Mediastinal diffuse large cell lymphoma with sclerosis (terminology no longer used)
      ICD coding
      • ICD-O: 9679 / 3 - mediastinal large B cell lymphoma
      • ICD-10: C85.20 - mediastinal (thymic) large B cell lymphoma, unspecified site
      • ICD-10: C85.29 - mediastinal (thymic) large B cell lymphoma, extranodal and solid organ sites
      Epidemiology
      Sites
      • Presents with localized anterosuperior mediastinal mass arising in thymic area
      • Regional supraclavicular and cervical lymph node involvement
      Etiology
      • Activation of JAK-STAT and NF-kB pathways
      • IL4R mutations (Blood 2018;131:2036)
      • Distinctive population of CD21- thymic B cells in the normal thymus are considered the normal counterpart for primary mediastinal large B cell lymphoma (Int J Cancer 1989;43:10)
      Clinical features
      • Symptoms: cough, dyspnea or superior vena cava syndrome (mass effect), effusions
      • Mass is often bulky (> 10 cm in 60 - 70% of patients)
      • Frequent direct extension invades adjacent structures, such as the lung, pleura and pericardium
        • Note that biopsy of lung may be the first diagnostic biopsy, thus this differential diagnosis must be considered when making a diagnosis of large B cell lymphoma in lung, particularly in young or female patients
      • Aside from direct extension into surrounding structures, extramediastinal involvement is unusual at the time of initial diagnosis (Am J Surg Pathol 2019;43:110, Am J Surg Pathol 2015;39:1322)
      • Bone marrow is not involved at the time of initial presentation; absence of systemic lymphadenopathy
      • At the time of relapse / progression, shows unusual predilection for involvement of kidney, adrenal gland, liver and central nervous system; may involve bone marrow
      Prognostic factors
      • 70 - 90% chance of cure with appropriate therapy (Br J Haematol 2019;185:25)
      • Poor prognostic factors: pleural or pericardial effusion, extension beyond mediastinum, B symptoms, advanced clinical stage, high serum lactate dehydrogenase, relapse within the first 18 months, unsatisfactory response to induction (15 - 20% of patients)
      • Outcomes for patients with relapsed or refractory primary mediastinal large B cell lymphoma are generally poor (J Clin Oncol 1997;15:1646)
      Case reports
      • 15 year old girl with rearrangements of MYC and BCL6 showed by fluorescent in situ hybridization (Am J Clin Pathol 2016;145:710)
      • 44 year old man presented with ileal perforation (BMJ Case Rep 2016 Jul 14;2016)
      • 54 year old Chinese man, a renal recipient with Hashimoto thyroiditis, presented with increased neck mass and progressive dyspnea (Int J Clin Exp Pathol 2015;8:5944)
      • 3 middle aged to elderly patients with nodal diffuse large B cell lymphoma with morphologic features of primary mediastinal large B cell lymphoma presenting in submandibular or supraclavicular lymph nodes; all 3 cases expressed cyclin D1 with copy number gains of CCND1 gene but without rearrangement (Am J Surg Pathol 2019;43:110)
      Treatment
      • DA-EPOCH+R: etoposide, prednisone, Oncovin (vincristine), cyclophosphamide, doxorubicin plus rituximab
        • Better than R-CHOP: rituximab, cyclophosphamide, vincristine, doxorubicin and prednisone
        • Note that the treatment regimen is different than that of diffuse large B cell lymphoma, NOS and classic Hodgkin lymphoma, thus appropriate identification of this distinct lymphoma has immediate therapeutic implications

      • Adult treatment protocol may need to be considered in children (Blood 2013;121:278)
      • Postchemotherapy PET evaluation may represent a tool to guide radiation therapy usage (Best Pract Res Clin Haematol 2018;31:241)
      • Disease management in relapse: intensive chemotherapy (DA-EPOCH+R) followed by an autologous stem cell transplantation (Br J Haematol 2019;185:25)
      Gross description
      • Mass is often bulky (usually > 10 cm)
      • Most frequently diagnosed based on core biopsy of mediastinal mass or by biopsy of involved lung in patients with mediastinal mass
        • Because mediastinal masses may be lymphomas, which are generally not resected, it is unusual to have a resection specimen of this tumor
      • Reference: Jaffe: Hematopathology, 2nd Edition, 2016
      Frozen section description
      • A definitive diagnosis should not be made at the time of frozen section given that required diagnostic features cannot be determined (immunophenotype, for example)
      • Diagnosis of lymphoma can be suspected if a population of intermediate to large sized and cytologically atypical lymphoid cells are seen
        • There is frequently associated sclerosis and may be necrosis, thus the differential diagnosis should include the possibility of classic Hodgkin lymphoma
      • Recommend that not all diagnostic tissue be frozen; save a portion of the sample for routine histology so that important morphologic features can be seen in permanent sections
      Microscopic (histologic) description
      • Characterized by a large spectrum of possible morphologic appearances
      • May mimic classic Hodgkin lymphoma or diffuse large B cell lymphoma, NOS
      • Polymorphic background associated with classic Hodgkin lymphoma is usually absent
      • Neoplastic cells are usually intermediate sized lymphoma cells present in a diffuse or clustered distribution
        • Usually more numerous tumor cells than in classic Hodgkin lymphoma
        • Nuclei are round to oval and may be hyperchromatic or vesicular
        • Clear cell change (abundant pale cytoplasm)
        • Prominent sclerosis (compartmentalizing fibrosis)
        • Frequently, at least a subset of lymphoma cells are pleomorphic and may resemble Reed-Sternberg cells
      • Reference: Hum Pathol 1999;30:178
      Microscopic (histologic) images

      Contributed by Jennifer Chapman, M.D.

      Intermediate to large sized cytologically atypical lymphoid cells

      Diffuse distribution of intermediate to large sized lymphoma cells

      CD19

      CD20


      CD23

      CD30

      Endobronchial biopsy

      Endobronchial biopsy

      Cytology description
      • Predominantly single, intermediate to large lymphoid cells
        • Round to oval nuclei
        • Smooth to irregular nuclear contours
        • One or more visible nucleoli
        • Scant to abundant cytoplasm (may be clear)
        • Oval / elongated nuclei due to fibrosis
      • In some cases, the atypical lymphoid cells show markedly lobulated nuclei and may resemble Reed-Sternberg cells
      • Background may contain connective tissue fragments admixed with single lymphocytes or groups of lymphocytes
      • Reference: Jaffe: Hematopathology, 2nd Edition, 2016
      Cytology images

      Contributed by Mahsa Khanlari, M.D.

      Papanicolaou

      Positive stains
      Negative stains
      Flow cytometry description
      • Discordance in B cell receptor and sIg
      • CD79a+ and CD19 / CD20+, while light chain Ig-, CD10-, CD15-
      • CD30 frequently expressed (Cytometry B Clin Cytom 2018;94:459)
      • Variable loss of HLA classes I and II (HLA-DR) molecules
      Molecular / cytogenetics description
      • Clonally rearranged immunoglobulin genes
      • Activation of JAK-STAT pathway (JAK2 amplification, STAT6 mutation, inactivating mutation of SOCS1 and PTPN1)
      • Activation of NF-κB pathway
        • Nuclear translocation of c-REL: target gene of tumor necrosis factor receptor associated factor 1 (TRAF1) (Am J Surg Pathol 2007;31:106)
          • Combination of nuclear c-REL with cytoplasmic TRAF1 reflects activation of NF-κB
          • Useful to distinguish primary mediastinal large B cell lymphoma from other subtypes of diffuse large B cell lymphoma
        • Loss of TNFAIP3 (A20) by coding sequence mutation (J Exp Med 2009;206:981)
          • A20 is a ubiquitin modifying enzyme that inhibits NF-κB activation in succession of TNF receptor and toll-like receptor induced signals
      • Immune privilege / evasion of immune surveillance (CIITA gene translocation leading to loss of MHC class II expression, PDL1 and PDL2 amplification and translocation)
      • Note: BCL2 and BCL6 genes usually not rearranged but high frequency of BCL6 gene mutations (~70%)
        • Rare / no translocations involving CCND1 and MYC
        • MAL gene (myelin and lymphocyte protein [MAL]) frequently expressed (70%) (Br J Haematol 2019;185:25)
      • Genomic landscape / comparative genomic hybridization (PLoS One 2015;10:e0139663):
        • 2p15 (REL / COMMD1)
        • 9p24 (JAK2, CD274: locus for PD1 ligand)
        • 16p13 (SOCS1, LITAF, CIITA)
        • 2p16 (MSH6)
        • 6q23 (TNFAIP3)
        • 9p22 (CDKN2A / B)
        • 20p12 (PTPN1)
      Sample pathology report
      • Lung, right upper lobe anterior tumor, endobronchial biopsies:
        • Diffuse large B cell lymphoma partially expressing CD23 and CD30, most consistent with primary mediastinal (thymic) large B cell lymphoma (see comment)
        • Comment:
          • The patient is a 35 year old female who presented with fatigue and was found to have an anterior mediastinal mass with extension into the upper lobe of the right lung.
          • Histologic sections consist of endobronchial biopsies of the lung mass and show respiratory mucosa that is involved by lymphoma. Lymphoma cells are present in a diffuse distribution and consist of intermediate to large sized neoplastic cells with round to ovoid nuclei, vesicular chromatin and occasionally prominent nucleoli. Mitotic figures and cellular apoptosis are increased. Associated granulation tissue is seen.
          • By immunohistochemistry, lymphoma cells are positive for CD20 (strong and diffuse), BCL2, CD30 (subset, heterogeneous intensity) and CD23 (subset) and are negative for CD10, BCL6 and CD3. The proliferative index based on Ki67 immunostain is 70%. MYC expression by immunohistochemistry is detected in 30% of tumor cell nuclei. In situ hybridization for EBER is negative.
          • The overall findings are those of diffuse large B cell lymphoma with histologic, immunophenotypic and clinical features compatible with designation as primary mediastinal (thymic) large B cell lymphoma.
      Differential diagnosis
      • Diffuse large B cell lymphoma, NOS (involving or arising in mediastinum):
        • Features favoring primary mediastinal large B cell lymphoma:
          • Young
          • F > M
          • Localized to mediastinum, absent systemic involvement on staging imaging
          • Intermediate sized cells with clear cytoplasm in a sclerotic stroma
          • Expression of CD23, nuclear NFKB (with cytoplasmic TRAF1) and MAL
      • Classic Hodgkin lymphoma:
        • Both express CD30 and have Reed-Sternberg cells
        • Features favoring primary mediastinal large B cell lymphoma:
          • Unlike classic Hodgkin lymphoma, primary mediastinal large B cell lymphoma expresses CD45, shows preservation of B cell program (expression of OCT2, BOB1, strong PAX5) and lacks strong expression of CD15
          • MAL expression (a small minority of cases of classic Hodgkin lymphoma [10 - 20%] express MAL) (Am J Clin Pathol 2006;125:776)
          • Polymorphic background that may be seen in classic Hodgkin lymphoma is absent in primary mediastinal large B cell lymphoma
      • Mediastinal gray zone lymphomas:
        • B cell lymphoma, unclassifiable, with features intermediate between diffuse large B cell lymphoma and classic Hodgkin lymphoma
        • Most frequent histologic finding is a variable microscopic appearance in different areas (some areas look like classic Hodgkin lymphoma, some areas look like primary mediastinal large B cell lymphoma)
        • Mediastinal gray zone lymphoma is characterized by a heterogenous immunophenotype among lymphoma cells in an individual case OR the immunophenotype of the lymphoma cells does not match the morphologic appearance (for example, cells look like classic Hodgkin lymphoma but have preserved B cell program; cells do not look like classic Hodgkin lymphoma but show classic Hodgkin lymphoma immunophenotype)
        • Variable expression of B cell antigens (primary mediastinal large B cell lymphoma consistently express the B cell antigens in homogenous pattern)
        • Although both mediastinal gray zone lymphomas and primary mediastinal large B cell lymphoma usually express CD30, coexpression of CD15 or presence of EBV (EBER positive), especially when associated with CD20 loss, favors the diagnosis of gray zone lymphoma (or classic Hodgkin lymphoma)
      Board review style question #1
      Which of the following genes / alterations is not considered part of the main landscape of primary mediastinal large B cell lymphoma genetic alteration?

      1. CARD11
      2. MAL
      3. PDL1
      4. PTPN1
      5. REL
      Board review style answer #1
      Board review style question #2

      Which of the following immunohistochemical markers is least likely to be positive in the mediastinal mass of a 26 year old woman with the above morphology?

      1. BOB1
      2. CD20
      3. CD30
      4. LMP1 (EBV)
      5. MUM1
      Board review style answer #2
      D. LMP1 (EBV). The image shows primary mediastinal large B cell lymphoma.

      Comment Here

      Reference: Primary mediastinal large B cell lymphoma

      Prolymphocytic leukemia
      Definition / general
      • Rare aggressive mature B cell neoplasm that represents less than ~1% of all leukemias
      • Characterized by the presence of large lymphoid cells (prolymphocytes) accounting for at least 55% of total circulating cells in the peripheral blood
      • Prolymphocytes are defined as large cells (> 2 erythrocytes) with clumped chromatin (but more open than in chronic lymphocytic leukemia [CLL]), a large single prominent vesicular nucleolus and abundant cytoplasm (Br J Haematol 2016;174:767)
      Essential features
      • Aggressive B cell neoplasm defined by large prolymphocytes accounting for 55% of total circulating cells in the peripheral blood
      • Main sites of involvement are peripheral blood, spleen and bone marrow
      • Marked absolute lymphocytosis (usually > 100 x 109/L)
      • Poor prognosis in general; the degree of treatment response is dictated by anemia, degree of lymphocytosis and molecular prognostic markers
      Terminology
      • Prolymphocytic leukemia, B cell type (B-PLL)
      ICD coding
      • ICD-O: 9833/3 - Prolymphocytic leukemia, B cell type
      Epidemiology
      Sites
      • Sites of involvement are peripheral blood, bone marrow and typically the spleen
      Pathophysiology
      Etiology
      • Very rare malignancy; no specific etiology or causal lesion associated with it
      Clinical features
      • Peripheral blood lymphocytosis usually > 100 x 109/L
      • Patients often present with constitutional symptoms and massive splenomegaly
      • Lymphadenopathy is typically absent or mild (Semin Oncol 2006;33:257)
      Diagnosis
      • Peripheral blood: CBC count, morphology review and flow cytometry
      • Bone marrow biopsy: morphology, flow cytometry and immunohistochemistry studies
      • Molecular and cytogenetic analysis have prognostic significance but are not necessary for a diagnosis
      Laboratory
      • Additional laboratory findings to the presence of marked absolute lymphocytosis in the CBC include anemia and thrombocytopenia; these are present ~50% of the time
      Radiology description
      • No specific radiology findings
      Prognostic factors
      Case reports
      Treatment
      • Rare cases that present as indolent disease and are asymptomatic can be followed with a watch and wait approach without immediate treatment; however, cases often progress rapidly and will require close monitoring
      • Chemotherapy is the treatment indicated; in the absence of 17p deletion or TP53 mutation, rituximab in association to FC (fludarabine, cyclophosphamide) or bendamustine is the recommended first line therapy (Hematology Am Soc Hematol Educ Program 2015;2015:361)
      • Chemotherapy for poor prognosis cases is different; in del(17p) or TP53 mutated tumors, treatment with alemtuzumab or more recently ibrutinib are used (Hematology Am Soc Hematol Educ Program 2015;2015:361, Br J Haematol 2017;179:501)
      Microscopic (histologic) description
      • Bone marrow involvement shows neoplastic infiltrates with a mixed nodular, interstitial or diffuse patterns composed of medium to large lymphoid cells
      • Spleen shows expanded white pulp with red pulp infiltration by medium to large neoplastic cells with ample cytoplasm, round or irregular nuclei and a central large, eosinophilic nucleolus
      • Lymph node involvement when present shows diffuse or nodular infiltration by neoplastic lymphoid cells without proliferation centers
      Microscopic (histologic) images

      Contributed by Richard K. Wood, M.D. and Dietrich Werner, M.D.

      Bone marrow involvement

      CD20 positivity in bone marrow

      CD79a positivity in bone marrow

      Cyclin D1 negativity in bone marrow

      Peripheral smear description
      • Marked absolute lymphocytosis with at least a 55% prolymphocyte count out of total circulating cells
      • Prolymphocytes:
        • Medium to large sized lymphoid cells with round or occasionally indented nuclei
        • Moderately condensed to open chromatin
        • Single large prominent central nucleolus
        • Moderate amount of basophilic cytoplasm
        • Reference: Br J Haematol 1974;27:7
      Peripheral smear images

      Contributed by Allam Shawwa, M.D.

      Prolymphocytes in peripheral blood

      Prolymphocytes in peripheral blood

      Positive stains
      Negative stains
      Flow cytometry description
      Flow cytometry images

      Case #120

      CD19 and CD20

      Kappa and lambda

      FMC7 and CD22

      Molecular / cytogenetics description
      Sample pathology report
      • Bone marrow, right posterior iliac crest, biopsy, aspirate and clot section:
        • B cell prolymphocytic leukemia (B-PLL) (see comment)
        • Comment: This is a 79 year old man presenting with an elevated CBC count of 153 x 109/L and an absolute lymphocyte count of 124 x 109/L. A manual peripheral blood smear count shows a prolymphocyte count of 74%.
      • Bone marrow aspirate:
        • Quality of specimen: particulate
        • Cellularity: hypercellular
        • Approx. M:E ratio: 4:1
        • Erythropoiesis: normoblastic and decreased
        • Granulopoiesis: all stages of maturation present and decreased
        • Megakaryocytopoiesis: normal
        • Lymphocytes: increased with two monomorphic populations noted. The first is characterized by medium sized cells with prominent central nucleoli and abundant cytoplasm. The second consists of small lymphocytes with densely packed chromatin and minimal cytoplasm.
        • Plasma cells: normal; not increased
      • Bone marrow aspirate differential:
        • Metamyelocytes: 3%
        • Bands: 7%
        • Seg. neutrophils: 10%
        • Eosinophils: 1%
        • Lymphocytes: 72%
        • Monocytes: 2%
        • Erythroblasts: 8%
        • Total cells counted: 500
      • Bone marrow biopsy:
        • A trephine biopsy with a length of 1.5 cm is submitted. The sample is adequate for examination. The specimen is hypercellular at approximately 70% cellularity. Trilineage hematopoiesis is decreased. Megakaryopoiesis is nondysplastic. Granulopoiesis is decreased but still shows a normal range of maturation and geographic distribution. There is a diffuse, interstitial and nodular infiltrate of lymphocytes consisting of two discrete populations. The first shows medium to large sized cells with prominent nucleoli and vesicular chromatin while the second consists of small, monomorphic lymphocytes with densely packed chromatin.
      • Ancillary testing:
        • Immunohistochemical evaluation of the larger cell population shows expression of CD20, CD19 and CD79a while CD5 and CD23 are not expressed. The population of smaller cells expresses CD20, CD19, CD5 and CD23. Both populations are negative for expression of CD3, SOX11 and cyclin D1.
        • The flow cytometry report shows two neoplastic cell populations surface light chain kappa restricted. The first larger population represents 69% of the sample and is positive for CD19, CD20 bright and kappa bright and is negative for CD5, CD10 and CD23. A second smaller population in 18% of the sample is positive for CD19, CD20 dim, CD5, CD23 and kappa dim.
        • Molecular and cytogenetic studies demonstrated deletion of 17p13. Translocation (11;14)(q13;q32) was not detected.
      Differential diagnosis
      • Mantle cell lymphoma:
        • Usually contains t(11;14)(q13;q32) translocation between IGH and CCND1 with positive expression of cyclin D1 and SOX11
      • T cell prolymphocytic leukemia:
        • Neoplastic cells show cytoplasmic projections on peripheral smear
        • Marrow involvement shows perivascular or diffuse patterns
        • Expresses T cell markers CD2, CD3, CD7, TCR, TCL1 and is negative for B cell markers
      • Chronic lymphocytic leukemia with increased prolymphocytes:
        • Small, monomorphic lymphocytes with tightly condensed chromatin with fewer than 55% prolymphocytes
        • Nearly all cases express CD5 and CD23
        • Bone marrow involvement shows proliferation centers with background small, monomorphic lymphocytes
      • Hairy cell leukemia variant:
        • May lack CD25 but typically expresses CD103
        • Typically demonstrate the characteristic cytoplasmic projections of HCL
      • Splenic marginal zone lymphoma:
        • Neoplastic cells show characteristic short polar villi and do not have prominent nucleoli
        • Splenic involvement by MZL shows prominent marginal zone
        • Marrow involvement may show follicular arrangement with surrounding marginal zone B cells
        • Molecular analysis shows in some cases NOTCH2 and NFκB mutations (not reported in B-PLL)
      • Follicular lymphoma:
        • Typically follicular pattern and often involving lymph nodes
        • Expresses CD10
      • Lymphoplasmacytic lymphoma:
        • Plasmacytoid appearance of neoplastic cells often accompanied by rouleaux
        • MYD88 mutation in > 90% of cases
      Board review style question #1

      You review the peripheral blood smear of a 74 year old man with a high WBC count of 117 x 109/L, anemia and thrombocytopenia. The majority of the cells (75%) have the appearance shown in the above image. Flow cytometry shows that these cells have the following immunophenotype: CD20+, CD19+, FMC7+, CD200+ and CD10-, CD5-, CD23-. What is your diagnosis?

      1. Acute myeloid leukemia
      2. B cell prolymphocytic leukemia
      3. Chronic lymphocytic leukemia
      4. Mantle zone lymphoma
      5. Splenic marginal zone lymphoma
      Board review style answer #1
      B. B cell prolymphocytic leukemia

      Comment Here

      Reference: Prolymphocytic leukemia
      Board review style question #2
      Which of the following findings is the required criteria for a diagnosis of B cell prolymphocytic leukemia?

      1. Anemia
      2. Bone marrow involvement
      3. Deletion of 17p13
      4. Lymphocytosis over 100 x 109/L
      5. Medium to large sized lymphocytes with central, large nucleoli that are greater than 55% of the total WBC
      Board review style answer #2
      E. Medium to large sized lymphocytes with central, large nucleoli that are greater than 55% of the total WBC

      Comment Here

      Reference: Prolymphocytic leukemia

      Pyothorax associated
      Definition / general
      • Large B cell lymphoma that arises in chronically inflamed tissue and is associated with EBV infection
      Essential features
      • Majority of cases associated with 20+ year history of pyothorax in patients treated with artificial pneumothorax for pulmonary tuberculosis or tuberculous pleuritis
      • Primary site of involvement is the pleural / pleural space, with or without involvement of nearby structures
      • Tumor cells are positive for CD20, CD79a and EBER but negative for CD10, BCL6 and HHV8
      • Aggressive disease with poor prognosis
      Terminology
      • Pyothorax associated lymphoma
      • Pyothorax associated diffuse large B cell lymphoma
      ICD coding
      • ICD-O: 9680/3 - malignant lymphoma, large B cell, diffuse, NOS
      Epidemiology
      • Majority of cases associated with 20+ year history of pyothorax in patients treated with artificial pneumothorax for pulmonary tuberculosis or tuberculous pleuritis
      • Mean age: 70 years old
      • Marked male predominance
      • References: Ann Oncol 2007;18:122, J Clin Oncol 2002;20:4255
      Sites
      Etiology
      • Important factors in the pathogenesis of this lymphoma include history of an artificial pneumothorax, EBV infection and cytokines (IL10, IL6) and reactive oxygen species produced in longstanding pyothorax
      Clinical features
      Diagnosis
      • Biopsy
      Laboratory
      • Leukocytosis without leukemic cells
      • Elevated lactate dehydrogenase (LDH)
      Prognostic factors
      • Aggressive, with 5 year overall survival rate of 20 - 35% (J Clin Oncol 2002;20:4255)
      • Patient achieving complete remission with chemotherapy or radiotherapy 5 year overall survival of 50%
      • Unfavorable prognostic factors:
        • Poor performance status
        • High serum LDH, alanine transaminase or urea
        • High clinical stage
      Case reports
      Treatment
      • Chemotherapy with or without radiotherapy
      Microscopic (histologic) description
      Positive stains
      Negative stains
      Molecular / cytogenetics description
      • Immunoglobulin genes clonally rearranged and hypermutated
      • Complex karyotypes
      • TP53 mutation (70% of cases) (Cancer Res 1998;58:1105)
      • Distinct gene expression profile from nodal diffuse large B cell lymphomas (Cancer Sci 2004;95:828)
        • Overexpression of IFI27: gene expressed by B cells by stimulation of interferon alpha
      Sample pathology report
      • Pleura, mass, resection:
        • Diffuse large B cell lymphoma associated with chronic inflammation
      Differential diagnosis
      • EBV+ diffuse large B cell lymphoma, NOS:
        • Patients present with extranodal masses without involvement of body cavities
      • Primary effusion lymphoma:
        • Occurs in setting of immunodeficiency (e.g., HIV infection)
        • Cytology can include immunoblastic, plasmablastic or anaplastic features
        • All cases associated with HHV8 infection
        • Pan B cell markers negative
      • Systemic lymphoma involving body cavity lining:
        • Generally, the patient will have a tissue based mass and stage IV disease
      Additional references
      Board review style question #1
      A 70 year old man presents with chest pain and a remote history significant for tuberculous pleuritis and artificial pneumothorax. Imaging reveals a nodular thickening of the right pleural. Biopsy was performed, showing sheets of large cells with central nucleoli, frequent mitotic activity and patchy necrosis. Molecular studies demonstrate immunoglobulin gene rearrangement and cytogenetics reveals a complex karyotype. Which of the following is true about this disease entity?

      1. Cells are positive for CD10 and BCL6
      2. Patients are usually female and immunocompromised
      3. This is a subset of primary effusion lymphoma
      4. Tumor cells are HHV8 positive
      5. Tumor cells demonstrate a type 3 EBV latency pattern
      Board review style answer #1
      E. Tumor cells demonstrate a type 3 EBV latency pattern

      Comment Here

      Reference: Pyothorax associated diffuse large B cell lymphoma

      Richter syndrome
      Definition / general
      Essential features
      • Progression of a chronic lymphocytic leukemia / small lymphocytic lymphoma into an aggressive lymphoma, most commonly diffuse large B cell lymphoma, with rare cases transforming into Hodgkin lymphoma (Hum Pathol 2016;55:108)
      • Common clinical presentation: sudden clinical deterioration, development of systemic symptoms, rapid lymph node enlargement and extranodal manifestations
      • Association with CDKN2A, NOTCH1, MYC, TP53 mutations (Curr Hematol Malig Rep 2014;9:294)
      • Poor overall prognosis: prognosis improved with clonally unrelated diffuse large B cell lymphoma (Blood 2011;117:3391)
      Terminology
      • Richter syndrome
      • Richter transformation
      ICD coding
      • ICD-10: C91.10 - chronic lymphocytic leukemia of B cell type not having achieved remission
      Epidemiology
      • Typically older adults
      • Occurs in approximately 2 - 10% of patients with chronic lymphocytic leukemia (Ann Hematol 2018;97:1859, Adv Clin Exp Med 2018;27:1683)
      • Richter syndrome of Hodgkin lymphoma type is a rare disease associated with EBV, occurring in 0.4 - 0.7% of chronic lymphocytic leukemia patients (Hum Pathol 2016;55:108, Med Hypotheses 2006;66:577)
      • May occur during various times after diagnosis, with some reports of chronic lymphocytic leukemia and Richter syndrome diffuse large B cell lymphoma type being diagnosed simultaneously
      • Incidence increased in patients undergoing heavy pretreatment
      Sites
      Pathophysiology
      Etiology
      • Risk factors for high grade transformation:
        • Clinical: advanced Rai stage at diagnosis and lymph node size > 3 cm, heavily pretreated chronic lymphocytic leukemia (Ann Hematol 2018 ;97:1859)
        • Increased risk of Richter syndrome associated with adverse prognostic factors, including del(11q), del(17p), unmutated immunoglobulin heavy chain variable region genes (IGHV) and a high expression of ZAP70, CD38 and CD49d at diagnosis
        • Increased risk of transformation with NOTCH1 mutation (Leukemia 2013;27:1100)
      Clinical features
      Diagnosis
      Laboratory
      • Elevated LDH level
      • Anemia, neutropenia and thrombocytopenia
      • Hypercalcemia with or without lytic bone lesion
      • Paraproteinemia in 44% of patients (Cancer 2005;103:216)
      Radiology description
      • Radiologic assessment demonstrates higher predictive value over clinical criteria
      • Computerized tomography (CT) scan helpful criteria (Ann Hematol 2018;97:1859)
        • Rapid disproportionate enlargement of a single group of lymph nodes
        • Inhomogeneous attenuation of lymph nodes, reflecting central necrosis
        • Evidence of extranodal involvement
      • PET / CT scan findings (Blood 2018;131:2761)
        • Evaluation of lymph nodes with SUVmax > 5 demonstrates high sensitivity but lower specificity for detecting Richter syndrome
        • Usefulness of PET / CT with its high negative predictive value (97 - 98%)
      Prognostic factors
      Case reports
      Treatment
      • Richter syndrome - diffuse large B cell lymphoma type (DLBCL RS):
        • R-CHOP or R-CHOP-like regimens (i.e. R-EPOCH) are widely used as first line treatment option (Blood 2018;131:2761)
        • Stem cell transplantation an available option in young, fit patients
        • Emerging role of immunotherapy, including BTK inhibitors (ibrutinib, acalabrutinib), BCL2 inhibitors (venetoclax), PD-1 receptor inhibitors (pembrolizumab), among others (Blood 2018;131:2761, Immunotargets Ther 2019;8:1)
      • Richter syndrome - Hodgkin lymphoma type (HS RS):
      Microscopic (histologic) description
      • Diffuse large B cell lymphoma type:
        • Diffuse effacement of lymph nodes or extranodal sites with sheets of large cells with centroblastic, immunoblastic or anaplastic morphology
        • Proposed criteria to distinguish from aggressive chronic lymphocytic leukemia (Immunotargets Ther 2019;8:1):
          • Presence of large B cells with nuclear sizes either equal to the nucleus of macrophages or more than twice that of a normal lymphocyte
          • Diffuse growth pattern of large cells
      • Hodgkin lymphoma type:
        • Presence of classic Reed-Sternberg cells in an appropriate polymorphous background of small T cells, histiocytes, eosinophils and plasma cells
      Microscopic (histologic) images

      Contributed by Shahbaz Khan, M.D. and Sepideh Nikki Asadbeigi, M.D.
      H&E of Richter syndrome H&E of Richter syndrome

      Diffuse large B cell lymphoma type

      Extranodal Richter syndrome Extranodal Richter syndrome

      Extranodal

      Peripheral smear description
      • Richter syndrome malignant cells are occasionally seen in the bone marrow and very rarely in the peripheral blood (J Hematol. 2014;3:86)
      • Morphology of transformed cells may show enlarged cells with pleomorphic nuclei, less condensed chromatin, prominent nucleoli and scant cytoplasm (J Clin Oncol 2006;24:2343)
      • Background circulating chronic lymphocytic leukemia cells present (small lymphocytes with condensed / clumped chromatin)
      Positive stains
      Negative stains
      • Diffuse large B cell lymphoma type:
        • Rare germinal center phenotype (CD10, BCL6 expression) (positive in 5 - 10% of cases)
        • EBV negative
      • Hodgkin type:
        • Reed-Sternberg cells: negative for CD20
      Flow cytometry description
      Molecular / cytogenetics description
      • Most molecular findings / knowledge based on research of diffuse large B cell lymphoma type Richter syndrome (findings in Hodgkin type Richter syndrome are less known)
      • Deregulation of cell cycle control, proliferation and damage to DNA repair and target genes via somatic mutations of TP53 (60 - 80%), CDKNA2 (30%) or MYC itself (30%) or by affecting their regulatory functions, e.g. NOTCH1 (30%) and MGA (10%)
      • 2 common genetic pathways responsible for transformation (Immunotargets Ther 2019;8:1)
        • TP53 and CDKNA2 mutations in over 50% of cases
        • NOTCH1 mutations and trisomy 12 in almost 33% of cases
      • Increased risk of Richter syndrome associated with adverse prognostic factors, such as del(11q), del(17p), unmutated immunoglobulin heavy chain variable region genes (IGHV) and a high expression of ZAP70, CD38 and CD49d
      • Cytogenetic analysis of Richter syndrome: predominantly complex karyotype
      Sample pathology report
      • Lymph node, right level II, excision:
        • Diffuse large B cell lymphoma, nongerminal center subtype (see comment)
        • Comment: In light of the patient's clinical history of chronic lymphocytic leukemia with a recent rapidly enlarging cervical lymph node, this lesion would be best classified as Richter transformation of the patient's chronic lymphocytic leukemia / small lymphocytic lymphoma to diffuse large B cell lymphoma.
      Differential diagnosis
      • Aggressive chronic lymphocytic leukemia (CLL):
        • Increase in size and proliferative activity of the CLL cells with confluent expansion of the proliferation centers in the lymph nodes (proliferation centers broader than a 20x field)
        • Proposed criteria to distinguish from aggressive CLL (Immunotargets Ther 2019;8:1):
          • Presence of large B cells with nuclear sizes either equal to the nucleus of macrophages or more than twice that of a normal lymphocyte
          • Diffuse growth pattern of large cells
        • Distinction requires pathologist expertise
      • De novo diffuse large B cell lymphoma (DLBCL):
        • Morphologically indistinguishable
        • PD-1 negative
        • Distinct molecular signature: typical recurrent mutations of de novo DLBCL affecting NFκB (e.g. CARD11, TNFAIP / A20, CD79A, CD79B, BCL6, BCL2, PRDM1 and EZH2) or genes associated with CLL chemorefractoriness and progression (e.g. BIRC3, MYD88, DDX3X, SF3B1 and RPS15) not seen in Richter syndrome DLBCL
      • Hodgkin-like lesion:
        • Hodgkin/Reed-Sternberg (HRS) cells in the setting of chronic lymphocytic leukemia (CLL) (HRS-CLL) and Hodgkin lymphoma variant of Richter transformation (CHL-RT) can show similar morphology (Hum Pathol 2016;55:108)
        • HRS-CLL has potential to progress to CHL
        • 27% of patients with HRS-CLL have a prior known diagnosis of CLL, as opposed to 73% of CHL-RT patient who had prior diagnosis of CLL
        • Cytologic features are typical of CLL in most cases of HRS-CLL with rare mixed inflammatory cells, in contrast to CHL-RT, which shows polymorphous inflammatory background
        • CHL-RT shows a moth-eaten or segregated pattern of CHL in background of CLL
        • Immunophenotype and EBV status of the HRS cells are similar in both HRS / CLL and CHL / RT cases (Hum Pathol 2016;55:108)
        • Both majority of HHR-CLL and CHL-RT patients are EBV+
      • Ibrutinib related pseudo Richter transformation:
        • Happens after a brief dose interruption of Ibrutinib or acute infection in CLL / SLL patients (Br J Haematol 2020;191:e22)
        • Presents with lymphadenopathy, lymphocytosis or progressive cytopenia and tissue biopsy shows DLBCL transformation; return of LDH and lymphocyte count to normal after treatment resumption with rebiopsy showing CLL / SLL with no DLBCL left
        • Large cells with the same immunophenotype as CLL / SLL
      Board review style question #1

      A 75 year old man with a history of CLL / SLL presents with a rapidly enlarging cervical lymph node. A representative microscopic section is seen above. Which factor is indicative of a favorable prognosis in this patient?

      1. Immunoglobulin heavy chain gene rearrangement of lymph node tissue is clonally unrelated to the concurrent CLL / SLL clone
      2. Laboratory work up showing platelet count of 75,000 / μL with normal WBC count
      3. PET / CT scan showing a 6 x 3 cm cervical lymph node with SUV of 7
      4. TP53 mutation in the biopsied tissue
      5. Positive PD-1 immunohistochemical expression in the biopsied tissue
      Board review style answer #1
      A. Immunoglobulin heavy chain gene rearrangement of lymph node tissue is clonally unrelated to the concurrent CLL / SLL clone

      Comment Here

      Reference: Richter syndrome
      Board review style question #2
      In cases of Richter syndrome of diffuse large B cell lymphoma subtype, which of the following is a common genetic alteration?

      1. JAK2
      2. CD79B
      3. TP53
      4. MYD88
      5. SF3B1
      Board review style answer #2
      C. TP53

      Comment Here

      Reference: Richter syndrome

      Sézary syndrome
      Definition / general
      Essential features
      Terminology
      • Erythrodermic CTCL includes all primary cutaneous lymphoma that evolve to erythroderma, such as Sézary syndrome (SS) and erythrodermic mycosis fungoides
      • SS was first described by Albert Sézary in 1938 (Lancet 2008;371:945, Eur J Cancer 2017;77:57)
      ICD coding
      • ICD-O: 9701/3 - Sézary syndrome
      • ICD-10: C84.1 - Sézary disease
      Pathophysiology / etiology
      • Mycosis fungoides and SS are thought to arise from chronic antigenic stimulation; early mycosis fungoides lesions show increased numbers of dendritic cells and upregulation of their antigen presenting cell (APC) ligands B7 and CD40 and their respective T cell costimulatory ligands CD28 and CD40L (J Am Acad Dermatol 2014;70:205.e1)
      • Both mycosis fungoides and SS have as cell of origin the skin resident CD45RO+ effector memory T cell (Lancet 2008;371:945)
      • Antigen presenting dendritic cells could maintain the survival and proliferation of clonal T cells: higher specific human leukocyte antigen (HLA) class II alleles than the general population (Lancet 2008;371:945)
      • Skin microbiomes (e.g. as Chlamydia spp) could play a role in the pathogenesis by the stimulation of clonality in T cells but it is still controversial (Lancet 2008;371:945)
      • Abnormalities in pathways: NFκB / JAK STAT activation, cell cycle dysregulation / apoptosis and DNA structural dysregulation affecting gene expression (Nat Genet 2015;47:1465)
      • Micro RNA expression profile detected differences between erythrodermic mycosis fungoides and SS (Acta Derm Venereol 2019;99:1148)
      • Chemokines receptors (Lancet 2008;371:945, Curr Hematol Malig Rep 2016;11:468, J Am Acad Dermatol 2011;64:352)
        • CCR4 and CCR10 play a role in the homing of malignant T cells to skin where they bind to ligands on endothelial cells, keratinocytes or Langerhans cells
          • CCL17 is a major CCR4 ligand and it is increased in the serum of patients with mycosis fungoides and SS
          • May explain the lower epidermotropism observed in SS compared with mycosis fungoides
        • Loss of CD26 promotes the inactivation of CXCL12, which is the CXCR4 ligand
      • T helper 2 (Th2) response is characteristic of SS as well as that of tumor stage mycosis fungoides; overexpression of CD47 in Sézary cells is under the influence of Th2 cytokines, such as IL-4, IL-7 and IL-13 (Blood Adv 2019;3:1145, Curr Hematol Malig Rep 2016;11:468, J Am Acad Dermatol 2011;64:352)
      Clinical features
      • Erythroderma is the key feature for diagnosis and is defined by involvement of > 80% of body surface and may range from mild to intense (J Am Acad Dermatol 2012;67:1189, Lancet 2008;371:945, Eur J Cancer 2018;93:47, Curr Hematol Malig Rep 2016;11:468)
        • Alopecia and plaques at initial clinical presentation are uncommon (systemic symptoms usually absent)
        • Nail dystrophy, blepharoconjunctivitis and ectropium can be present in advanced disease
        • Palms and sole usually show thickening, scales and fissures
        • Median duration of all dermatologic symptoms before diagnosis: 3.5 years
          • Erythroderma is present usually 1.7 years before diagnosis
        • Clinical presentation with erythroderma with < 1 x 109/L is defined as pre-Sézary and some patients progress to SS, while others follow a more indolent clinical course
      • Pruritus is often present (Lancet 2008;371:945)
      • Systemic symptoms usually absent (J Am Acad Dermatol 2012;67:1189)
      • Adenopathy is usually present (J Am Acad Dermatol 2012;67:1189)
      • Bone marrow involvement in approximately 20% of the cases (J Am Acad Dermatol 2012;67:1189)
      • Cases may be preceded by mycosis fungoides and should be designated as SS preceded by mycosis fungoides; these cases are different from de novo SS (Am J Hematol 2019;94:1027)
      Diagnosis
      • Characterized by the presence of erythroderma, generalized lymphadenopathy and clonal T cells (same clone in skin, lymph node and peripheral blood) in addition to 1 or more of the following criteria (Am J Hematol 2019;94:1027, Eur J Cancer 2018;93:47, Am J Clin Pathol 2011;136:944, Curr Hematol Malig Rep 2016;11:468, J Am Acad Dermatol 2011;64:352):
        • Absolute Sézary cell count ≥ 1 x 109/L (manual count or by flow cytometry)
        • Alternative diagnostic criteria: ≥ 40% of CD4+ / CD7- and ≥ 30% of CD4+ / CD26- T cells (flow cytometry)
          • TRBC1 is more sensitive and specific for detection of clonal T cells by flow cytometry
      • Expanded CD4+ T cell population with CD4:CD8 ≥ 10
        • It is reliant on the absolute CD8 count and the ratio may decrease after treatment
      • Loss of one or more T cell antigens
      • Demonstration of T cell clonality by southern blot or PCR based methods
      • Use of V beta antibodies allows quantification of the clone (obsolete)
        • Clonality by flow cytometry: constant chain of T cell receptor: TRBC1 (Int J Mol Sci 2021;22:1817)
        • Use of high throughput sequencing is more sensitive but not widely used
      • Cytogenetic demonstration of an abnormal clone
      • Bone marrow involvement is not a defined criteria for SS
      • Criteria for blood response (see staging table below):
        • Complete response (CR): B2 should change to B0
        • Partial response (PR): at least 50% reduction of tumor burden
        • Progressive disease (PD): B0 or B1 to B2 with an increase in absolute counts of ≥ 50% or B2 with an increase in absolute counts of ≥ 50% or loss of response with an increase in absolute counts ≥ 1x109/L and ≥ 50% from nadir
        • Relapse: increase in absolute counts of ≥ 1x109/L in the context of CR
      Staging / staging classifications
      • Staging of mycosis fungoides and SS is performed according to the International Society for Cutaneous Lymphomas (ISCL) and the European Organization for Research and Treatment of Cancer (EORTC) (Eur J Cancer 2017;77:57, Blood 2016;127:3142)

      Skin
      T1 Limited patches, papules or plaques covering < 10% of the skin surface:
      T1a: patch only
      T1b: plaque with or without patch
      T2 Patches, papules or plaques covering ≥ 10% of the skin surface:
      T2a: patch only
      T2b: plaque with or without patch
      T3 One or more tumors (≥ 1cm diameter)
      T4 Confluence of erythema covering ≥ 80% body surface area
      Node
      N0 No clinically abnormal peripheral lymph node
      N1 Clinically abnormal peripheral lymph nodes; histopathology Dutch grade 1 or NCI LN0-2
      N1a: clone negative
      N1b: clone positive
      N2 Clinically abnormal peripheral lymph nodes; histopathology Dutch grade 2 or NCI LN3
      N2a: clone negative
      N2b: clone positive
      N3 Clinically abnormal peripheral lymph nodes; histopathology Dutch grade 3 - 4 or NCI LN4; clone positive or negative
      Nx Clinically abnormal peripheral lymph nodes with no histological confirmation
      Visceral
      M0 No visceral organ involvement
      M1 Visceral involvement (must have pathological confirmation)
      Peripheral Blood (PB)
      B0 Absence of significant blood involvement: ≤ 5% Sézary cells in PB
      B0a: clone negative
      B0b: clone positive
      B1 Low blood tumor burden: > 5% Sézary cells in PB and does not meet criteria of B2
      B1a: clone negative
      B1b: clone positive
      B2 High blood tumor burden: ≥ 1000/µL Sézary cells in PB with clone positive

      • SS is defined as stage IVA1 (T1-T4, N0-N2, M0, B2) and above
      Prognostic factors
      Case reports
      Treatment
      Clinical images

      Contributed by Roberto N. Miranda, M.D.
      Prominent exfoliative lesion

      Prominent exfoliative lesion

      Prominent erythroderma

      Prominent erythroderma

      Gross images

      Contributed by Roberto N. Miranda, M.D.
      Central nervous system (CNS) involvement

      Central nervous system (CNS) involvement

      Microscopic (histologic) description
      • SS is histologically similar to mycosis fungoides but sometimes epidermotropism is not present (Lancet 2008;371:945, Curr Hematol Malig Rep 2016;11:468, J Am Acad Dermatol 2011;64:352)
        • May range from limited to superficial perivascular lymphocytic and eosinophilic dermatitis with or without spongiosis (atopic-like lesions) to lichenoid lymphocytic infiltrate
        • Pautrier microabscesses are not common
      • Peripheral blood (Lancet 2008;371:945)
        • Sézary cells: atypical lymphocytes of intermediate to large size and cerebriform nuclei
          • Similar cells, albeit few or rare may be found in healthy individuals or in patients with other inflammatory skin diseases
      • Lymph node (Am J Hematol 2019;94:1027, Cancer 1986;57:237)
        • Complete effacement of the nodal architecture by monotonous infiltrating population of Sézary cells
        • Large cell or blastoid morphology may occur in advanced stages.
      Microscopic (histologic) images

      Contributed by Roberto N. Miranda, M.D.
      Bone marrow involvement

      Bone marrow involvement

      CD3 positivity

      CD3 positivity

      Epidermotropic and lichenoid lymphoid infiltrate

      Epidermotropic and lichenoid lymphoid infiltrate

      Epidermotropic infiltrate

      Epidermotropic infiltrate


      Cerebriform cytology

      Cerebriform cytology

      Lymph node involvement

      Lymph node involvement

      Cytological atypia Cytological atypia

      Cytological atypia

      Cytology images

      Contributed by Roberto N. Miranda, M.D.
      Lymph node touch print

      Lymph node touch print

      Peripheral smear images

      Contributed by Roberto N. Miranda, M.D.
      Cerebriform lymphocytes in peripheral blood smear

      Cerebriform lymphocytes in peripheral blood smear

      Positive stains
      Negative stains
      Flow cytometry images

      Contributed by Roberto N. Miranda, M.D.
      Flow cytometry in SS

      Flow cytometry in SS

      Molecular / cytogenetics description
      Sample pathology report
      • Skin, shave biopsy of the arm:
        • Primary cutaneous T cell lymphoma, most consistent with Sézary syndrome (see comment)
        • Comment: Section shows an adequate skin shave biopsy with mild spongiosis and perivascular and lichenoid infiltrate in superficial dermis with mild epidermotropism. The infiltrate is composed of small lymphocytes, with irregular nuclear contours (cerebriform-like), hyperchromatic nuclei and scant cytoplasm. Immunohistochemical studies show that the abnormal lymphocytes are diffusely positive for CD3, CD4, CD5 and CD45RO and negative for CD2, CD7 and CD30. The concurrent complete blood count (CBC) reveals lymphocytosis (7 x 109/L) with abnormal Sézary cell count and the flow cytometry immunophenotype of the peripheral blood revealed CD4+ lymphocytosis (5 x 109/L) with low CD8+ count (0.8 x 109/L) and expanded CD4+ CD26- (63%) and CD4+ CD7- (58%). According to clinical notes, the patient is a 67 year old man who presented to the department of dermatology with a 1 month history of generalized erythroderma (90% of skin involvement) and inguinal and axillary bilateral lymphadenopathy. In summary, patient presented with erythroderma and skin is involved by cerebriform T lymphocytes with minimal epidermotropism. Peripheral blood is involved morphologically by cerebriform lymphocytes that by flow cytometry express CD3, CD4 and lack CD7 and CD26. These features support the diagnosis of Sézary syndrome.
      Differential diagnosis
      Board review style question #1

      Which of the following is true about Sézary syndrome (SS) shown above?

      1. Erythroderma is essential for the diagnosis
      2. Frequently presents as an indolent disease with favorable prognosis
      3. Lymph node involvement is uncommon
      4. Most common dermatologic lesion is plaque
      Board review style answer #1
      A. Erythroderma is essential for the diagnosis

      Comment Here

      Reference: Sézary syndrome
      Board review style question #2
      What is a consistent feature for the diagnostic criteria of Sézary syndrome (SS)?

      1. Absolute Sézary cell count ≥ 1 x 109/L
      2. Different T cell clones in skin, blood or lymph node
      3. Expanded CD8+ T cells with CD8:CD4 ≥ 10
      4. Loss of CD3 (T cell marker) expression
      Board review style answer #2
      A. Absolute Sézary cell count ≥ 1 x 109/L

      Comment Here

      Reference: Sézary syndrome

      Severe mosquito bite allergy (pending)
      [Pending]

      Splenic B cell leukemia / lymphoma, unclassifiable (pending)
      [Pending]

      Splenic diffuse red pulp
      Definition / general
      • Mature splenic B cell lymphoma characterized by diffuse infiltration of the splenic red pulp with small, monomorphous B lymphocytes
      • Bone marrow sinusoids and peripheral blood are often also involved acommonly with a villous morphology
      • Diagnosis should be restricted to those fulfilling major features; if in doubt, the terminology splenic B cell lymphoma / leukemia, unclassifiable is preferred
      Essential features
      • Provisional entity within the category of splenic B cell lymphoma / leukemia, unclassifiable, along with hairy cell leukemia variant (Swerdlow: WHO Classification of Tumours of Haematopoietic and Lymphoid Tissues, 4th Edition, 2017)
      • Indolent disease course, often diagnosed at clinical stage IV disease (involvement of spleen, bone marrow and peripheral blood)
      • Microscopically characterized by small to intermediate sized B lymphocytes diffusely invading the red pulp cords
        • Intrasinusoidal involvement can be seen on bone marrow biopsy
        • Peripheral blood smear reveals villous cytology
      • Important to differentiate from hairy cell leukemia, hairy cell leukemia variant and splenic marginal zone lymphoma
      Terminology
      • Provisional entity under splenic B cell lymphoma / leukemia, unclassifiable (WHO classification)
      • Splenic lymphoma with villous lymphocytes
      • SDRPL
      • SDRP SBCL
      ICD coding
      • ICD-O: 9591/3 - splenic diffuse red pulp small B cell lymphoma
      Epidemiology
      • Incidence:
        • Rare, < 1% of all non-Hodgkin lymphomas
        • 10% of splenic B cell lymphomas
      • Age: usually > 40 years (median 65.5 - 77 years)
      • Sex: M > F (ratio varies between 1.6 - 2.4) (Discov Med 2012;13:253)
      Sites
      Clinical features
      • Massive splenomegaly is seen in most cases
      • Typically stage IV disease (blood and bone marrow involvement)
      • Splenic hilar lymph nodes are frequently reported (Haematologica 2010;95:1122)
      • B symptoms (rare)
      • Erythematous and pruritic skin papules (10% of cases) (Haematologica 2010;95:1122)
      Diagnosis
      • Requires constellation of clinical features, peripheral blood smear, marrow and spleen histology, immunophenotyping and cytogenetics
      • Splenectomy may be is required for confirmation of diagnosis (Arch Pathol Lab Med 2014;138:1295)
      Laboratory
      Radiology description
      • Massive splenomegaly
      Prognostic factors
      • Indolent but incurable disease
      • Must differentiate from similar diseases, as it has a good prognosis but is often resistant to conventional chemotherapy that is effective for hairy cell leukemia, hairy cell leukemia variant and splenic marginal zone lymphoma (Blood Res 2018;53:74)
      • Cases with mutations in NOTCH1, MAP2K1 and TP53 have been reported to have shorter progression free survival (Am J Surg Pathol 2016;40:192)
      Case reports
      Treatment
      Gross description
      • Diffusely enlarged spleen with a homogenous beefy red-brown cut surface
      • Wedge shaped subcapsular infarcts (Blood Res 2018;53:74)
      Gross images

      Images hosted on other servers:

      Diffusely enlarged spleen

      Microscopic (histologic) description
      Cytology description
      • Small to medium, monomorphic lymphocytes with round oval nuclei and compact, clumped chromatin
      • Occasionally distinct, small nucleoli
      Peripheral smear description
      • Small villous lymphocytes with clumped chromatin and a smooth nuclear outline
      • Cytoplasm is basophilic with broad based cytoplasmic extensions in a polar distribution (Blood 2008;111:2253)
      Peripheral smear images

      Images hosted on other servers:

      Neoplastic villous lymphocytes

      Positive stains
      Flow cytometry description
      Molecular / cytogenetics description
      Differential diagnosis
      Splenic diffuse red pulp small B cell lymphoma Splenic marginal zone lymphoma Hairy cell leukemia Hairy cell leukemia variant
      Frequency of non-Hodgkin lymphoma 0.5% < 2% 2% 0.4%
      Anemia and thrombocytopenia Uncommon Present (autoimmune) Present Present
      Monocytopenia Absent Absent Present (characteristic) Absent (leukocytosis with lymphocytosis and normal monocyte count)
      Blood smear Broad based polar cytoplastic extensions (similar to splenic marginal zone lymphoma) Polar villi and inconspicuous nucleoli Reniform or oval nuclei, circumferential long villi, inconspicuous nucleoli Abundant circumferential villi and prominent nucleoli
      Spleen Diffuse involvement of red pulp with cord and sinusoid infiltration, blood lakes may be present, absence of white pulp involvement Micronodular pattern with white pulp replacement, marginal zone differentiation Diffuse infiltrate in red pulp cords, effaced white pulp, formation of blood lakes Red pulp involved, blood lakes uncommon, effaced white pulp with absent white pulp follicles.
      TRAP - Variable + -
      Bone marrow Predominantly intrasinusoidal infiltrate, interstitial and nodular infiltration have been reported; no lymphoid follicles have been reported Nodular interstitial and intrasinusoidal infiltrate; occasionally, tumor cells surround reactive follicles Dense infiltrate, fibrosis Interstitial and sinusoidal infiltrate
      Flow cytometry CD25-
      CD103-
      CD123-
      CD25+
      CD103 variable
      CD123-
      CD25+
      CD103+
      CD123+
      CD25-
      CD103+
      CD123-
      Immunohistochemistry Annexin A1-
      DBA-44+
      Annexin A1-
      DBA-44 variable
      Annexin A1+
      DBA-44+
      Annexin A1-
      DBA-44+
      Genetic mutation CCND3
      NOTCH1
      MAP2K1
      TP53
      NOTCH2
      VH1 / 2
      BRAF V600E (~100%) MAP2K1
      Cytogenetics Uncommon
      del 7q
      trisomy 18
      del 17p
      del 7q
      trisomy 3 (less common)
      del 13q
      del 7q (uncommon)
      del 17p
      Treatment Splenectomy Splenectomy
      rituximab
      Purine analogs
      rituximab
      Partial response to purine analogs
      Splenectomy
      Board review style question #1

      A 60 year old man presents with abdominal discomfort and generalized weakness. CT of the chest / abdomen / pelvis reveals massive splenomegaly. Splenectomy demonstrates a monomorphic diffuse infiltration of the red pulp. Peripheral blood smear is shown above. Immunohistochemistry is positive for CD20 and DBA-44 and negative for CD25, CD103, CD123 and Annexin A1. What is the most likely diagnosis?

      1. Hairy cell leukemia
      2. Hairy cell leukemia variant
      3. Splenic diffuse red pulp small B cell lymphoma
      4. Splenic marginal zone lymphoma
      Board review style answer #1
      C. Splenic diffuse red pulp small B cell lymphoma. While the 2017 revised WHO describes SDRPL as having a diffuse pattern of involvement of the red pulp with absence of the white pulp involvement (see table above), obliteration of the while pulp has been reported in the literature. As such, given the immunophenotype, a diagnosis of SDRPL is favored (Arch Pathol Lab Med 2014;138:1295, Haematologica 2010;95:1122, Discov Med 2012;13:253)

      Comment here

      Reference: Splenic diffuse red pulp small B cell Lymphoma
      Board review style question #2
      What is the most common treatment for splenic diffuse red pulp small B cell lymphoma?

      1. Chemotherapy
      2. Immunotherapy
      3. Radiation
      4. Splenectomy
      Board review style answer #2
      Board review style question #1

        What is the most common cytogenetic abnormality in splenic marginal zone lymphoma (SMZL)?

      1. Amp 22
      2. Del 7q
      3. Del 11q
      4. t (8;14)
      Board review style answer #1
      B. Deletion 7q is identified in up to 40% of splenic marginal zone lymphoma patients.

      Comment Here

      Reference: Splenic marginal zone B cell lymphoma
      Board review style question #2
        What is the immunophenotypic pattern of splenic marginal zone lymphoma (SMZL)?

      1. CD19+, CD20+, CD5-, CD23-
      2. CD19+, CD20+, CD5+, CD23+
      3. CD19+, CD20+, CD10+, BCL6+
      4. CD19+, CD20+, CD10+, BCL6+
      Board review style answer #2
      A. CD19+, CD20+, CD5-, CD23- is consistent with splenic marginal zone lymphoma.

      Comment Here

      Reference: Splenic marginal zone B cell lymphoma

      Staging-primary cutaneous
      Definition / general
      Essential features
      • Staging criteria for mycosis fungoides / Sézary syndrome (MF / SS) is different from non mycosis fungoides / Sézary syndrome lymphomas (including primary cutaneous B cell lymphomas [CBCL], other primary cutaneous T cell lymphomas [CTCL] and NK cell lymphomas) (Blood 2007;110:1713, Blood 2007;110:479)
      ICD coding
      • ICD-10:
        • C82.6 - cutaneous follicle center lymphoma
        • C83.3 - diffuse large B cell lymphoma, unspecified site
        • C84.0 - mycosis fungoides, unspecified
        • C84.1 - Sézary disease
        • C84.A0 - cutaneous T cell lymphoma, unspecified
        • C86.0 - extranodal NK / T cell lymphoma, nasal type
        • C86.3 - subcutaneous panniculitis-like T cell lymphoma
        • C86.6 - primary cutaneous CD30 positive T cell proliferations
        • C88.4 - extranodal marginal zone B cell lymphoma of skin associated lymphoid tissue (SALT lymphoma)
      Diagrams / tables

      Images hosted on other servers:

      Body regions

      Lymph node regions

      TNMB classification of MF / SS

      Lymph node grading

      TNM classification of non-MF / SS

      Primary tumor of skin (pT)
      • Mycosis fungoides and Sézary syndrome; this is the revised ISCL/EORTC staging of MF/SS (Blood 2007;110:1713)
        • pT1: involving < 10% of skin surface
          • pT1a: patch only
          • pT1b: plaque ± patch
        • pT2: involving > 10% of skin surface
          • pT2a: patch only
          • pT2b: plaque ± patch
        • pT3: 1 or more tumors > 1 cm in diameter
        • pT4: confluence of erythema covering > 80% of skin surface
      • Non mycosis fungoides / Sézary syndrome primary cutaneous lymphomas (Blood 2007;110:479)
        • pT1: solitary lesion
          • pT1a: < 5 cm diameter
          • pT1b: ≥ 5 cm diameter
        • pT2: multiple lesions limited to 1 body region or 2 contiguous body regions
          • pT2a: all disease encompassing a ≤ 15 cm diameter circular area
          • pT2b: all disease encompassing a > 15 and ≤ 30 cm diameter circular area
          • pT2c: all disease encompassing a > 30 cm diameter circular area
        • pT3: generalized skin involvement
          • pT3a: multiple lesions involving 2 noncontiguous body regions
          • pT3b: multiple lesions involving ≥ 3 body regions
      Regional lymph nodes (pN)
      • Mycosis fungoides and Sézary syndrome (Blood 2007;110:1713)
        • pN0: no clinically abnormal peripheral lymph nodes; biopsy not required
        • pN1: clinically abnormal peripheral lymph nodes; histopathology Dutch grade 1 or NCI LN0 - 2
          • pN1a: clone negative
          • pN1b: clone positive
        • pN2: clinically abnormal peripheral lymph nodes; histopathology Dutch grade 2 or NCI LN3
          • pN2a: clone negative
          • pN2b: clone positive
        • pN3: clinically abnormal peripheral lymph nodes; histopathology Dutch grades 3 - 4 or NCI LN4; clone positive or negative
        • pNx: clinically abnormal peripheral lymph nodes; no histologic confirmation
      • Non mycosis fungoides / Sézary syndrome primary cutaneous lymphomas (Blood 2007;110:479)
        • pN0: no clinically or pathologically abnormal lymph nodes
        • pN1: involvement of 1 peripheral lymph node region that drains an area of current or prior skin involvement
        • pN2: involvement of 2 or more peripheral lymph node regions or involvement of any lymph node region that does not drain an area of current or prior skin involvement
        • pN3: involvement of central lymph nodes

      Notes:
      • Mycosis fungoides and Sézary syndrome
        • Abnormal peripheral lymph node(s) indicates any palpable peripheral node that on physical examination is firm, irregular, clustered, fixed or 1.5 cm or larger in diameter
        • Node groups examined on physical examination include cervical, supraclavicular, epitrochlear, axillary and inguinal
        • Central nodes, which are not generally amenable to pathologic assessment, are not currently considered in the nodal classification unless used to establish N3 histopathologically
        • T cell clone is defined by PCR or Southern blot analysis of the T cell receptor gene
      • Non mycosis fungoides / Sézary syndrome primary cutaneous lymphomas
        • Definition of lymph node regions is consistent with the Ann Arbor system
          • Peripheral sites: antecubital, cervical, supraclavicular, axillary, inguinal femoral and popliteal
          • Central sites: mediastinal, pulmonary hilar, para-aortic, iliac
      Distant metastasis (pM)
      • Mycosis fungoides and Sézary syndrome (Blood 2007;110:1713)
        • pM0: no visceral organ involvement
        • pM1: visceral involvement with pathology confirmation
      • Non mycosis fungoides / Sézary syndrome primary cutaneous lymphomas (Blood 2007;110:479)
        • pM0: no evidence of extracutaneous non lymph node disease
        • pM1: extracutaneous non lymph node disease present
      Blood involvement (pB)

      Notes:
      • Sézary cells are defined as lymphocytes with hyperconvoluted cerebriform nuclei
      • If Sézary cells are not able to be used to determine tumor burden for B2, then 1 of the following modified ISCL criteria along with a positive clonal rearrangement of the TCR may be used instead:
        • Expanded CD4+ or CD3+ cells with CD4/CD8 ratio of 10 or more
        • Expanded CD4+ cells with abnormal immunophenotype including loss of CD7 or CD26
      • T cell clone is defined by PCR or Southern blot analysis of the T cell receptor gene
      Prefixes
      Not standardized for primary cutaneous lymphomas, therefore not used currently
      AJCC prognostic stage groups
      Mycosis fungoides / Sézary syndrome (Amin: AJCC Cancer Staging Manual, 8th Edition, 2017)
      Stage group IA:  T1   N0   M0   B0 - 1
      Stage group IB:  T2   N0   M0   B0 - 1
      Stage group IIA:  T1 - 2   N1 - 2   M0   B0 - 1
      Stage group IIB:  T3   N0 - 2   M0   B0 - 1
      Stage group III:  T4   N0 - 2   M0   B0 - 1
      Stage group IIIA:  T4   N0 - 2   M0   B0
      Stage group IIIB:  T4   N0 - 2   M0   B1
      Stage group IVA1:  T1 - 4   N0 - 2   M0   B2
      Stage group IVA2:  T1 - 4   N3   M0   B0 - 2
      Stage group IVB:  T1 - 4   N0 - 3   M1   B0 - 2
      Registry data collection variables
      • For MF/SS : peripheral blood involvement
      Histologic grade (G)
      Not done for primary cutaneous lymphomas
      Histopathologic type
      • Cutaneous T cell lymphomas
        • Mycosis fungoides
          • Folliculotropic MF
          • Pagetoid reticulosis
          • Granulomatous slack skin
        • Sézary syndrome
        • Primary cutaneous anaplastic large cell lymphoma
        • Subcutaneous panniculitis-like T cell lymphoma
        • Extranodal NK/T cell lymphoma, nasal type
        • Primary cutaneous gamma delta T cell lymphoma
        • Primary cutaneous CD8+ aggressive epidermotropic cytotoxic T cell lymphoma
        • Primary cutaneous acral CD8+ T cell lymphoma
        • Primary cutaneous CD4+ small / medium T cell lymphoproliferative disorder
        • Primary cutaneous peripheral T cell lymphoma, NOS
      • Cutaneous B cell lymphomas
        • Primary cutaneous marginal zone lymphoma
        • Primary cutaneous follicle center lymphoma
        • Primary cutaneous diffuse large B cell lymphoma, leg type
      Clinical images

      Contributed by Henry K. Wong, M.D., Ph.D.
      Mycosis fungoides, patch stage

      Mycosis fungoides, patch stage

      Mycosis fungoides, plaque stage

      Mycosis fungoides, plaque stage

      Mycosis fungoides, tumor stage Mycosis fungoides, tumor stage

      Mycosis fungoides, tumor stage

      Peripheral smear images

      Images hosted on other servers:

      Sézary cells, peripheral smear

      Videos

      Diagnosis and staging of cutaneous lymphoma

      Board review style question #1

      A 45 year old woman presents with new onset violaceous plaques and tumors on the lower leg and foot with rapid progression (see picture above). The lesions measure 3.2 cm and 4.9 cm in greatest dimension, respectively. No other abnormalities are found on physical exam. Biopsy of the lesions shows sheet of large, atypical cells that stain positively with CD20, MUM1 and BCL2. A diagnosis of diffuse large B cell lymphoma, leg type is made. What is the correct T stage of the disease?

      1. T1a
      2. T2a
      3. T3a
      4. T3b
      Board review style answer #1
      B. T2a. Multiple lesions of diffuse large B cell lymphoma, leg type are present on 2 contiguous regions of the body, hence, the T stage is T2a using the non mycosis fungoides / Sézary syndrome primary cutaneous lymphoma staging criteria.

      Comment Here

      Reference: Staging - primary cutaneous lymphoma
      Board review style question #2
      A 68 year old man presents with erythema of the entire back, chest, abdomen and bilateral extremities. On further inspection, there is a 1.8 cm raised ulcerated lesion present on the left upper thigh and multiple enlarged palpable lymph nodes in the inguinal region. Biopsy of the thigh lesion shows tagging of CD4 + T cells along the dermal epidermal junction and epidermotropism. The cells show loss of CD7 and CD26. What is the most likely diagnosis and correct T stage of the disease?

      1. Mycosis fungoides; stage T1a
      2. Mycosis fungoides; stage T2b
      3. Mycosis fungoides; stage T3
      4. Mycosis fungoides; stage T4
      Board review style answer #2
      D. Mycosis fungoides; stage T4. The erythema covers > 80% of body surface which is stage T4 according to the mycosis fungoides / Sézary syndrome primary cutaneous lymphoma staging criteria.

      Comment Here

      Reference: Staging - primary cutaneous lymphoma

      Subcutaneous panniculitis-like T cell lymphoma
      Definition / general
      Terminology
      • T cell lymphoma involving subcutaneous tissue
      ICD coding
      • ICD-10: C86.3 - subcutaneous panniculitis-like T cell lymphoma
      Etiology
      Clinical features
      Diagnosis
      • Association of clinicopathologic features and correct interpretation of histologic findings (JAMA Dermatol 2022;158:1167, J Clin Oncol 2015;33:1216)
        • Unrelated malignant and nonmalignant confounding histopathology can masquerade as subcutaneous panniculitis-like T cell lymphoma (SPTCL)
      Laboratory
      Radiology description
      Radiology images

      Contributed by Roberto N. Miranda, M.D.
      Subcutaneous uptake

      Subcutaneous uptake

      Prognostic factors
      Case reports
      • 26 year old man with a diagnosis of panniculitis-like T cell lymphoma with HAVCR2 mutation and previous history of lupus panniculitis (Intern Med 2023;62:1537)
      • 34 year old woman with liver failure and skin lesions as main manifestation of subcutaneous panniculitis-like T cell lymphoma (Ann Transl Med 2022;10:1408)
      • 39 year old man with a diagnosis of panniculitis-like T cell lymphoma presenting as an abscess following arthropod bite (Dermatol Online J 2022;28:3)
      • 42 year old man diagnosed with panniculitis-like T cell lymphoma in complete remission after 2 cycles of romidepsin (Case Rep Oncol 2022;15:1088)
      • 47 year old woman with a diagnosis of panniculitis-like T cell lymphoma with inverted FDG uptake pattern in 18F-FDG PET (Clin Nucl Med 2023;48:186)
      Treatment
      • No standard treatment
        • Immune modulatory treatments as single agents or combined (JAMA Dermatol 2022;158:1167, Surg Pathol Clin 2021;14:177)
          • Cyclosporine A, systemic steroids, bexarotene, low dose methotrexate, chlorambucil, azathioprine
          • Overall response rate (ORR): ~50%
        • Chemotherapy
          • CHOP: cyclophosphamide, doxorubicin, vincristine and prednisone
          • CHOP-like regimens
        • Radiation
          • Localized disease
        • Hematopoietic stem cell transplantation (Asia Pac J Clin Oncol 2023;19:27)
          • Refractory / relapsed or disseminated cases
      Clinical images

      Contributed by Roberto N. Miranda, M.D.
      Subcutaneous lesions

      Subcutaneous lesions

      Subcutaneous plaque

      Subcutaneous plaque

      Microscopic (histologic) description
      • Skin involvement (JAMA Dermatol 2022;158:1167, Surg Pathol Clin 2021;14:177, Histopathology 2013;62:1057, Blood Adv 2021;5:3919, J Clin Pathol 2015;68:954)
        • Epidermis and dermis are usually spared
          • Interface dermatitis, plasma cell aggregates, follicular plugging or mucin deposits may occur
          • Minimal involvement by lymphoma cells of deep dermis can be observed
        • Lobular infiltrate
          • Variable size (small to intermediate) lymphocytes with hyperchromatic and irregular nuclei and scant pale / clear cytoplasm
            • Subset of cases with large cell or pleomorphic morphology
          • Variable density of atypical lymphocytes involving lobules and sparing septa
            • Rimming adipocytes or lipid vacuoles
          • Background
            • Scattered reactive plasma cells and neutrophils
            • Absence of plasmacytoid dendritic cells aggregates (> 10 cells)
            • Reactive histiocytes, sometimes dense and forming granulomas or lipogranulomas
        • Necrosis may vary from mild to extensive
        • Karyorrhexis, mitotic figures and hemorrhage can be present
        • Hemophagocytosis may be detected in skin or other tissues (e.g., bone marrow)
      • Bone marrow is not involved by overt lymphoma cells but aggregates of small lymphocytes can be detected around adipocytes
      Microscopic (histologic) images

      Contributed by Roberto N. Miranda, M.D.
      Deep dermis and subcutaneous involvement

      Deep dermis and subcutaneous involvement

      Epidermis and reticular dermis sparing

      Epidermis and reticular dermis sparing

      Lobular distribution Lobular distribution

      Lobular distribution

      Acellular hyaline change

      Acellular hyaline change

      Adipocyte rimming

      Adipocyte rimming


      CD8 positivity

      CD8 positivity

      CD4 negativity

      CD4 negativity

      TIA1 positivity

      TIA1 positivity

      βF1 (TCR α / β) positivity

      βF1 (TCRα / β) positivity

      TIM-3 positivity

      TIM-3 positivity

      Molecular / cytogenetics description
      Sample pathology report
      • Skin, right thigh lesion, punch:
        • Subcutaneous panniculitis-like T cell lymphoma (see comment)
        • Comment: According to clinical notes, this is a 37 year old woman with a history of a solitary, tender and painless lesion in distal thigh (3 cm in the greater axis) that the patient noted wax and wane for several months.
        • Histologic sections of the specimen designated right thigh lesion demonstrate a punch skin biopsy that includes epidermis, dermis and subcutaneous adipose tissue. Epidermis is unremarkable. Deep dermis and subcutaneous adipose tissue show dense small to medium sized lymphoid cells, neutrophils and histiocytes. The lymphocytes show irregular nuclear contours, hyperchromatic chromatin and scant pale cytoplasm. The distribution is mainly involving the lobules with fat rimming and sparing the septa. Mitotic figures are frequent and karyorrhectic debris is present.
        • Immunohistochemical studies showed the atypical lymphocytes are positive for CD2, CD3, CD8, CD43, TIA1, perforin and TCRα-β (βF1). The abnormal lymphocytes are negative for CD4, CD7, CD30, CD56 and TCRγ / δ. Ki67 index is ~70% (variable, ranging from 40 - 100%). Epstein-Barr virus encoded small RNAs (EBER) in situ hybridization is negative.
        • Polymerase chain reaction to assess for clonality of T cell receptors gamma and delta chains clonal TCRβ (TRB) and polyclonal TCRγ (TRG) and TCRδ (TRD).
      Differential diagnosis
      Board review style question #1

      Which of the following is true about subcutaneous panniculitis-like T cell lymphoma?

      1. Frequently presents with an aggressive clinical behavior
      2. Localized or multiple subcutaneous nodules are the most common clinical presentation
      3. Lymph nodes are usually involved at initial diagnosis
      4. Somatic mutations of HAVCR2 are common
      Board review style answer #1
      B. Localized or multiple subcutaneous nodules are the most common clinical presentation. Subcutaneous panniculitis-like T cell lymphoma usually presents as localized or multiple subcutaneous nodules, usually in the legs, arms or trunk and rarely with lymph node involvement. Germline mutations of HAVCR2 are classically described as a predisposing factor in up to 85% of patients.

      Comment Here

      Reference: Subcutaneous panniculitis-like T cell lymphoma
      Board review style question #2

      Which of the following immunophenotypic pattern corresponds to subcutaneous panniculitis-like T cell lymphoma?

      1. CD3-, CD4-, CD8+, TIA1-, granzyme B+, TCRγδ+
      2. CD3-, CD4+, CD8+, TIA1-, granzyme B-, TCRγδ+
      3. CD3+, CD4-, CD8+, TIA1+, granzyme B+, TCRαβ+
      4. CD3+, CD4+, CD8-, TIA1+, granzyme B-, TCRαβ+
      Board review style answer #2
      C. CD3+, CD4-, CD8+, TIA1+, granzyme B+, TCRαβ+. The typical immunophenotype is CD3+, CD4-, CD8+, TIA1+, granzyme B+ and TCRαβ+. The picture shows positivity in the neoplastic cells and highlights rimming of adipocytes.

      Comment Here

      Reference: Subcutaneous panniculitis-like T cell lymphoma

      Systemic EBV+ T cell lymphoma of childhood
      Definition / general
      • Life threatening illness of children and young adults characterized by a clonal proliferation of EBV infected T cells with an activated cytotoxic phenotype (WHO 2008)
      • Increased frequency in Asians and Native Americans from Central and South America, and Mexico
      Terminology
      • Overlapping clinicopathologic features with aggressive NK cell leukemia
      • Historical terms: fulminant EBV+ T cell lymphoproliferative disorder of childhood, sporadic fatal infectious mononucleosis, fulminant hemophagocytic syndrome in children (Taiwan, Am J Pathol 1994;144:1219), fatal EBV associated hemophagocytic syndrome (Japan), severe chronic active EBV infection
      • Severe chronic active EBV infection: infectious mononucleosis-like syndrome persisting for at least 6 months and associated with high titers of VCA IgG and EA IgG but no association with malignancy, autoimmune disease or immunodeficiency
      Epidemiology
      • Most prevalent in Asia, primarily Japan and Taiwan; also reported in Mexico and rarely in West
      • Children and young adults, no sex predilection
      • Western version of chronic active EBV infection - progression to EBV-TLPD uncommon
      • Japanese version of chronic active EBV infection - more severe with high fever, hepatosplenomegaly, extensive lymphadenopathy, pancytopenia, higher viral copy numbers in peripheral blood T cells or NK cells, monoclonal proliferation, usually progresses to malignancy (Blood 2001;98:280)
      Sites
      • Systemic: liver, spleen, lymph nodes, bone marrow, skin, lung
      Etiology
      • Develops shortly after primary EBV infection or in the setting of chronic active EBV infection
      • Etiology unknown, but association with EBV and racial predisposition strongly suggest genetic immune defect
      Postulated normal counterpart
      • Cytotoxic CD8+ T lymphocytes or activated CD4+ T cells
      Clinical features
      • Previously healthy, then acute onset with fever and general malaise suggestive of viral syndrome, hepatosplenomegaly and liver failure and variable lymphadenopathy
      • Aphthous stomatitis, ulcerative gingivitis, vesiculopapular and papulonecrotic mucocutaneous eruptions
      • Very fulminant clinical course
      • Rapid progression with multiple organ failure, sepsis and death, usually occuring in days to weeks
      • Lab: pancytopenia, abnormal liver function tests, abnormal EBV serology with low or absent anti-VCA IgM antibodies
      • Imaging: interstitial pneumonia, basal ganglia calcifications, coronary aneurysms
      • Complications: hemophagocytic syndrome (Leuk Lymphoma 1995;19:401), coagulopathy, GI ulcers, coronary aneurysms, CNS involvement, multiorgan failure and sepsis
      Treatment
      • No standard treatment
      • Usually resistant to conventional chemotherapy; hematopoietic stem cell transplant has been introduced
      • Fulminant course in most cases with death within days or weeks
      • Subacute course of several months to a year also possible
      Microscopic (histologic) description
      • Infiltrating, usually small T cells with no significant atypia; may have pleomorphic medium to large lymphoid cells, irregular nuclei and frequent mitoses
      • Marked sinusoidal infiltration and hemophagocytosis in liver and spleen, splenic white pulp depletion, hepatic portal and sinusoidal infiltration, cholestasis, steatosis and necrosis
      • Lymph nodes: preserved architecture, open sinuses, variable sinus histiocytosis with erythrophagocytosis
      • Bone marrow: lymphohistiocytic hyperplasia with prominent erythrophagocytosis
      • Lung: necrosis and peribronchial infiltration, sometimes angiocentricity and angioinvasion
      • Skin: extension from epidermis to subcutis, necrosis, angiocentricity, angioinvasion, periappendageal infiltration, epidermal ulceration
      Microscopic (histologic) images

      Images hosted on other servers:

      Bone marrow

      Liver and lung

      Liver with CD3 and CD20

      Liver double staining with CD4 / EBER and CD8 / EBER


      Spleen

      Left: bone marrow, EBER1; right: liver, CD45 RO

      Positive stains
      Negative staining
      Molecular / cytogenetics description
      • Clonal TCR gene rearrangements
      • Clonal episomal form of EBV type A (wild type or the 30bp deleted product of LMP1) in all cases
      • No consistent chromosomal abnormalities, but often 6q deletion
      Molecular / cytogenetics images

      Images hosted on other servers:

      PCR for TCR gamma gene


      Systemic EBV+ T cell lymphoma of childhood
      Definition / general
      • An often rapid and fulminant disease within the spectrum of childhood Epstein-Barr virus (EBV) positive T cell lymphomas
      Essential features
      • Clinically characterized by systemic involvement, including the bone marrow, lymph nodes, spleen and lungs; most patients have a rapid, often fatal, clinical course
      • The neoplastic elements are EBV+ T cell lymphocytes identified in biopsies from end organ damage biopsy sites; EBV+ T cell lymphocytes predominantly have a cytotoxic immunophenotype
      • The neoplastic elements are mostly small lymphocytes without cytological atypia or effacement of the background tissue architecture
      • It occurs in pediatric and young adult immunocompetent patients and there is an overlap in symptoms, clinical outcomes, laboratory findings and histomorphological features with the other 2 EBV+ T / NK lymphoproliferative disorders of childhood: EBV+ hemophagocytic lymphohistiocytosis (EBV+ HLH) and T / NK systemic form chronic active EBV disease (T / NK sCAEBV)
        • These 3 entities represent different ends of a spectrum of end organ damage secondary to EBV infected hyperactivated T cell lymphocytes
      Terminology
      • No longer recommended
        • Fulminant T cell lymphoproliferative disease
        • Sporadic fatal infectious mononucleosis
        • Fatal infectious mononucleosis
        • Fulminant EBV+ T cell lymphoproliferative disorder of childhood
        • Lethal hemophagocytic lymphohistiocytosis
        • Fulminant hemophagocytic syndrome in children
        • Severe chronic active EBV disease (CAEBV)
      ICD coding
      • ICD-O: 9724/3 - systemic EBV positive T cell lymphoproliferative disease of childhood
      • ICD-10: C84.5 - other and unspecified T cell lymphomas
      • ICD-11: 2A90.4 - systemic Epstein-Barr virus positive T cell lymphoma of childhood
      Epidemiology
      Sites
      Pathophysiology
      Etiology
      Diagrams / tables
      N/A
      Clinical features
      • Symptoms include hepatosplenomegaly, fever, lymphadenopathy, malaise and upper respiratory tract symptoms
      • Often, there is rapid development of multiorgan failure, sepsis and death within a few days to a few months in immunocompetent patients
      • Proposed as a diagnosis of exclusion in EBV positive T / NK lymphoproliferative disorders of childhood that do not fulfill the criteria for T / NK sCAEBV or EBV positive HLH
      • References: Diagn Pathol 2021;16:48, Leuk Lymphoma 2017;58:53, Blood 2000;96:443
      Diagnosis
      • Histologic, immunohistochemical, molecular and cytogenetic analysis of involved tissues and clinical correlation
      Laboratory
      Radiology description
      N/A
      Radiology images
      N/A
      Prognostic factors
      Case reports
      Treatment
      Clinical images
      N/A
      Gross description
      N/A
      Gross images
      N/A
      Frozen section description
      N/A
      Frozen section images
      N/A
      Microscopic (histologic) description
      • More frequently, there is a mild expansion of small to medium sized T cell lymphocytes without cytological atypia or effacement of the background target tissue architecture
      • When involved, the spleen shows depletion of the white pulp, lymphocyte infiltration of the red pulp and prominent hemophagocytosis
      • When the liver is involved, findings include a sinusoidal and portal lymphocytic infiltrate; additional reported findings include cholestasis, steatosis and variable degrees of liver necrosis
      • Bone marrow shows a scattered to diffuse, medium / large atypical lymphocytic infiltrate and increased histiocytes and hemophagocytosis
      • Lymph nodes, when involved, usually show preserved architecture with open sinuses, expanded paracortical regions, sinus histiocytosis and hemophagocytosis (Pediatr Blood Cancer 2019;66:e27798, Blood 2000;96:443)
      • Few cases with diffuse sheets of atypical medium to large lymphocytes involving the liver, bone marrow, spleen, lung or lymph node have been reported
      • References: Diagn Pathol 2021;16:48, Leuk Lymphoma 2017;58:53, Blood 2000;96:443, Pediatr Blood Cancer 2019;66:e27798
      Microscopic (histologic) images

      Contributed by Yoon Kyung Jeon, M.D.
      Lymphocytic infiltrates in the liver

      Lymphocytic infiltrates in the liver

      SETBL in the liver

      SETBL in the liver

      Lymphocytic infiltrates in the liver

      Lymphocytic infiltrates in the liver

      Lymph node, atypical infiltrates

      Lymph node, atypical infiltrates


      Lymph node, atypical infiltrates Lymph node, atypical infiltrates Lymph node, atypical infiltrates

      Lymph node, atypical infiltrates

      Lymphocytic infiltrates in the liver

      Lymphocytic infiltrates in the liver

      Virtual slides
      N/A
      Cytology description
      N/A
      Cytology images
      N/A
      Peripheral smear description
      N/A
      Peripheral smear images
      N/A
      Positive stains
      Flow cytometry description
      N/A
      Flow cytometry images
      N/A
      Electron microscopy description
      N/A
      Electron microscopy images
      N/A
      Molecular / cytogenetics description
      Molecular / cytogenetics images

      Images hosted on other servers:
      PCR for TCR gamma gene

      PCR for TCR gamma gene

      Videos
      N/A
      Sample pathology report
      • Bone marrow core biopsy, touch imprints, aspirate and clot:
        • Hypercellular bone marrow with atypical EBV positive T cell lymphocytic infiltrates (see comment)
        • Comment: The T cell lymphocytic infiltrates feature aberrant loss of CD5 and concurrent T cell receptor rearrangement studies are consistent with a monoclonal T cell population. In the proper clinical context, these findings can be consistent with systemic EBV positive T cell lymphoma of childhood (SETBL).
      Differential diagnosis

      Table 1: Comparison of the clinical and laboratory findings between the entities included in the differential diagnosis of SETBL (adapted from Am J Clin Pathol 2024 Feb 12 [Epub ahead of print])
      Epstein-Barr virus associated hemolymphohistiocytosis (EBV HLH) Chronic active EBV disease of T / NK cell type systemic form (T / NK sCAEBV) Systemic EBV positive T cell lymphoproliferative disorder of childhood (SETBL) Nodal EBV positive T / NK cell lymphoma Aggressive NK cell leukemia
      Morphological features Small T cell lymphocytes with no definitive cytological atypia The infiltrates are predominantly composed by lymphocytes and plasma cells with no effacement or distortion of the background tissue and without morphological atypia; HRS-like cells may be present Variable expansion of lymphocytic infiltrates composed by small to medium sized T cells with no morphological atypia Effacement of the nodal architecture by medium to large lymphocytes with marked cytological atypia Medium sized atypical lymphocytes, frequently with prominent nucleoli
      Immunophenotype A subset of T cells are EBV and CD8 positive; loss of pan T cell markers may be observed The HRS-like cells are CD30 positive with variable expression of PAX5 and CD20; the EBV positive T cells are predominantly CD4 positive with loss of pan T cell markers (rare EBV+ CD8 T cells have been reported) Majority of cases are composed by CD8 positive neoplastic T cells (CD4+ cases are rare) A minimum of 30% EBV positive T cell lymphocytes is required; majority of the cases are composed by CD8 positive neoplastic cells Cytoplasmic CD3 expression and positive expression of CD2, CD56 and CD16; negative for CD57
      Anatomical sites of presentation Bone marrow and liver Liver, bone marrow and spleen Bone marrow, liver, lymph nodes, spleen Lymph nodes Peripheral blood and bone marrow
      Age at diagnosis Predominantly pediatric population Predominantly pediatric population Predominantly pediatric population Adult populations Peak incidence between 21 and 30 year old
      Relevant clinical features Meets diagnostic criteria for HLH; good responses to etoposide based regimens Persistent mononucleosis-like symptoms and high EBV titers for at least 3 months; good responses to systemic chemotherapy and hematopoietic stem cell transplantation is curative in most of pediatric patients; dismal clinical course in adult populations Rapid development of multiorgan failure and end organ damage including liver failure, hepatosplenomegaly and pancytopenia, with high mortality rates Aggressive clinical course and poor responses to current chemotherapeutic agents Fulminant clinical course with a median overall survival of < 2 months
      Relevant clinical features and prognosis Meets diagnostic criteria for HLH; overall good prognosis with good responses to etoposide based regimens Persistent mononucleosis-like symptoms and high EBV titers for at least 3 months; pediatric patients display good responses to systemic chemotherapy and hematopoietic stem cell transplantation is curative in most of pediatric patients; adult patients are characterized by dismal clinical course and poor prognosis Rapid development of multiorgan failure and end organ damage including liver failure, hepatosplenomegaly and pancytopenia, with high mortality rates and poor prognosis Aggressive clinical course, poor responses to current chemotherapeutic agents and bad prognosis Fulminant clinical course, median overall survival of < 2 months and dismal prognosis
      Additional references
      N/A
      Board review style question #1

      Which of the following features supports a diagnosis of systemic EBV+ T cell lymphoma of childhood (SETBL) over EBV+ hemophagocytic lymphohistiocytosis (EBV+ HLH)?

      1. EBER positivity in CD8+ T cells
      2. Lack of cytologic atypia
      3. Onset of disease in childhood
      4. Presence of a clonal cytogenetic abnormality
      Board review style answer #1
      D. Presence of a clonal cytogenetic abnormality. Clonal cytogenetic abnormalities are found in SETBL but not EBV+ HLH. Answer A is incorrect because both SETBL and EBV+ HLH show EBER positivity in CD8+ T cells in most cases. Answer B is incorrect because SETBL and EBV+ HLH show no cytologic atypia. However, the presence of cytologic atypia would favor a diagnosis of SETBL. Answer C is incorrect because SETBL and EBV+ HLH most commonly occur in children and young adults (Int J Clin Exp Pathol 2014;7:5738).

      Comment Here

      Reference: Systemic EBV+ T cell lymphoma of childhood
      Board review style question #2

      Which of the following is correct regarding EBV+ T cell lymphoproliferative neoplasms of childhood?

      1. An indolent clinical course characterizes EBV+ T cell lymphoproliferative neoplasms
      2. Diagnosis can be established only after documentation of T cell receptor clonality studies
      3. Distinction between systemic EBV+ T cell lymphoma (SETBL) and chronic active EBV disease of T / NK cell type, systemic form (T / NK sCAEBV) can be established solely on morphological analysis of the lymphocytic infiltrates
      4. These groups of diseases constitute a continuum with similar histological features and overlap in clinical and laboratory findings
      Board review style answer #2
      D. These groups of diseases constitute a continuum with similar histological features and overlap in clinical and laboratory findings. SETBL, T / NK sCAEBV and HLH EBV overlap in clinical features, are indistinguishable in histological features and can precede one another. Answer A is incorrect because these entities usually have an aggressive clinical course. Answer B is incorrect because T cell receptor clonality studies are not positive in all cases and its presence is not required to establish the diagnosis. Answer C is incorrect because the histological features between SETBL and T / NK sCAEBV are similar (Leuk Lymphoma 2017;58:53, Am J Clin Pathol 2024 Feb 12 [Epub ahead of print]).

      Comment Here

      Reference: Systemic EBV+ T cell lymphoma of childhood

      Systemic chronic active EBV disease
      Definition / general
      • Systemic chronic active Epstein-Barr virus (CAEBV) disease is a systemic Epstein-Barr virus (EBV) related T / NK cell lymphoproliferative disorder that occurs mostly in children and adolescents
      • Manifests as persistent clinical symptoms and high EBV viremia for more than 3 months with a protracted but potentially progressive clinical course
      Essential features
      • Rare chronic EBV related T / NK cell lymphoproliferative disorder that mainly affects pediatric patients of Asian or Hispanic descent
      • Systemic symptoms for at least 3 months, high EBV viremia and absence of immunodeficiency are all essential criteria for the diagnosis
      • Demonstration of EBV positive T or NK cells in affected tissues is also required for the diagnosis but affected tissues usually have preserved architectural features
      • Cases previously described as B cell type CAEBV have now been excluded from this category, given that they arise mostly in Western countries, affect older adults and usually arise in a background of known immunodeficiency
      Terminology
      • Chronic active EBV infection
      • Severe chronic active EBV infection
      • Chronic active EBV disease of T / NK cell phenotype
      • Chronic active EBV infection of T / NK cell type, systemic form
      • Epstein-Barr virus associated T / NK cell lymphoproliferative disease
      ICD coding
      • ICD-O: 9725/1 - systemic chronic active EBV positive disease (Int J Hematol 2023;118:568)
      • ICD-11: 2B0Y & XH6TZ4 - other specified primary cutaneous mature T cell or NK cell lymphomas and lymphoproliferative disorders & systemic EBV positive T cell lymphoproliferative disease of childhood
      Epidemiology
      Sites
      Pathophysiology
      • EBV infects mostly B lymphocytes and epithelial cells during primary infection; it can also infect T and NK cells but this process is less well understood
        • CD21 is a receptor used by EBV to infect host cells and it may be expressed in common lymphoid progenitors and precursor T cells (J Immunol 1991;146:865)
        • Infection of T lymphocytes and common lymphoid progenitors by EBV has been documented (Blood 1995;85:744, Adv Exp Med Biol 2018;1045:459)
        • Cell of origin studies and the presence of divergent phenotypes in CAEBV support the notion that EBV may infect a common lymphoid progenitor in this disease (Nat Microbiol 2019;4:404)
        • Cell to cell infection during an attempt to kill infected cells and acquisition of CD21 by NK cells through synaptic transfer have also been suggested as mechanisms of viral acquisition (J Immunol 2003;170:5993)
      • It is speculated that the evasion of host immunity by EBV infected T / NK cells is associated with specific human leukocyte antigens (HLA) and genetic background of some patients (Adv Exp Med Biol 2018;1045:459)
        • Some HLA alleles might not properly present EBV epitopes and thus impair viral clearance
        • Traits related to lymphomagenesis might also be codominantly expressed along with HLA-A26, one of the putative HLA alleles related to EBV T / NK lymphoproliferative disorder (LPD)
      • Evasion of host immune response is also facilitated by the latency type II program, which does not express the immunodominant antigens EBNA2 and EBNA3 (Adv Exp Med Biol 2018;1045:459)
      • Development and progression of clinically evident disease might be associated with the acquisition of driver gene mutations or epigenetic modifications
        • DDX3X and KMT2D mutations have been suggested to be present in the cell of origin models at diagnosis (Nat Microbiol 2019;4:404)
          • DDX3X encodes an RNA helicase and loss of function variants alter RNA processing and unwinding
          • KMT2D encodes a lysine methyltransferase that participates in chromatin remodeling targeting active chromatin sites involved in transcription and DNA repair (Nat Rev Cancer 2021;21:413)
      • Intragenic deletions in the EBV genome target mostly the BART microRNA clusters 1 and 2 (Nat Microbiol 2019;4:404)
        • The most affected microRNAs negatively regulate BZLF1, BRLF1 and BRLF14, which are proteins that function as transcriptional activators of immediate early genes associated with the lytic cycle
        • In the lytic cycle, there is a marked reduction of expression of MHC class 1 and class 2 as well as CD74, which may protect the infected cells from EBV specific cytotoxic T lymphocytes
        • BZLF1 can also interfere with the activity of CD4 positive T lymphocytes (PLoS Pathog 2011;7:e1002455)
      • Transcriptomic studies also imply inflammatory response and cell proliferation genes in the pathogenesis of CAEBV (Pathology 2020;52:111)
        • Elevated levels of proinflammatory cytokines may explain the relationship with hemophagocytic lymphohistiocytosis (HLH) (Blood 2018;131:2899)
      Etiology
      • Precise cause is unknown but an impaired control of EBV due to genetic polymorphisms is suspected
      • Association between HLA-A*26 and HLA-B*52 has been reported (Adv Exp Med Biol 2018;1045:459)
      Diagrams / tables

      Contributed by Elaine Jaffe, M.D. and João Víctor Alves de Castro, M.D.
      Proposed interrelatedness of EBV positive T / NK LPDs

      Proposed interrelatedness of EBV positive T / NK LPDs

      Clinical features
      • At presentation (Blood 2018;131:2899)
        • Fever
        • Liver dysfunction
        • Hepatosplenomegaly
        • Lymphadenopathy
      • Complications (Blood 2018;131:2899)
        • Hemophagocytic lymphohistiocytosis
        • Coronary artery aneurysms
        • Liver failure
        • Overt T cell lymphomas, including systemic EBV positive T cell lymphoma of childhood
      • Other possible features
        • Uveitis, interstitial pneumonia, hydroa vacciniforme-like eruptions (HV LPD) and hypersensitivity to mosquito bites (severe mosquito bite allergy)
      • Periorbital face and edema of the face in South American patients reported as hydroa vacciniforme is more consistent with CAEBV (Am J Dermatopathol 2015;37:20)
      Diagnosis
      • Infectious mononucleosis-like symptoms lasting for at least 3 months (essential)
      • High levels of EBV DNA in blood (essential)
      • Infiltration of tissues by EBV positive T / NK cells demonstrated by histology (essential)
        • B cell type is excluded by both the 5th edition of the World Health Organization Classification of Haematolymphoid Tumours (WHO-HAEM5) and the International Consensus Classification (ICC) 2022 (Virchows Arch 2023;482:227)
      • Exclusion of primary or acquired immunodeficiencies, history of transplant, previous malignancies and rheumatic diseases (essential)
      • References: Int J Hematol 2023;118:568, Leuk Lymphoma 2020;61:808, Virchows Arch 2023;482:227, Leukemia 2022;36:1720
      Laboratory
      • Elevation of liver enzymes
      • Hypoalbuminemia
      • Elevated bilirubin levels
      • Cytopenias
      • EBV DNA blood levels as measured by polymerase reaction in whole blood or peripheral blood mononuclear cells (preferred over plasma)
      • High EBV related IgG titers (viral capsid antigen [VCA] IgG and early antigen [EA] IgG) are often seen but not necessary for the diagnosis (Int J Hematol 2023;118:568)
      Radiology description
      • For cases with coronary artery involvement, contrast enhanced computed tomography (CT) angiography may show coronary artery dilation (Virol J 2020;17:166)
      Radiology images

      Images hosted on other servers:
      Muscle involvement and coronary artery dilation

      Muscle involvement and coronary artery dilation

      Prognostic factors
      Case reports
      Treatment
      • Standard treatment approach has not been established (Blood 2018;131:2899)
      • Treatment of hemophagocytic lymphohistiocytosis, mainly with the HLH-2004 protocol, is the most important once hemophagocytic lymphohistiocytosis is identified (Blood 2018;131:2899)
      • Diverse chemotherapy regimens are usually necessary, including high dose systemic corticosteroids or ganciclovir combined with either histone deacetylase inhibitors or bortezomib (Blood 2018;131:2899)
      • Hematopoietic stem cell transplant (HSTC) is the only curative approach (Blood 2018;131:2899)
        • Early diagnosis and transplant before disease progression might be critical for improving outcomes for these patients (Blood 2018;131:2899)
      Microscopic (histologic) description
      • Affected tissues frequently do not display morphologic features of malignancy (Am J Clin Pathol 2023;159:14)
      • Hemophagocytosis can be present in the liver, spleen and bone marrow of cases associated with hemophagocytic lymphohistiocytosis
      • Liver
        • Predominantly intrasinusoidal lymphocytosis
        • Hepatocytes may display features of viral hepatitis
      • Spleen
        • Mild expansion of the red pulp by lymphocytes arranged singly or in clusters
        • White pulp is often preserved
      • Lymph node
        • Interfollicular / paracortical infiltrate of mildly atypical EBV positive cells
        • Architecture is mostly preserved, with follicular and paracortical hyperplasia, open sinuses and a thin capsule
      • Bone marrow
        • Usually normal
        • Discrete interstitial and sinusoidal lymphocytosis may be seen with immunohistochemistry
      • Skin (in cases associated with hydroa vacciniforme lesions)
        • Dense periadnexal and perivascular dermal and subcutaneous infiltrate of small to medium lymphoid cells
        • Epidermal necrosis, ulceration and bullae formation
      • Other affected tissues (lung and heart): interstitial pneumonitis and viral myocarditis
      Microscopic (histologic) images

      Contributed by Elaine Jaffe, M.D. and João Víctor Alves de Castro, M.D.
      Spleen, red and white pulp

      Spleen, red and white pulp

      Lymphoid aggregate in the red pulp

      Lymphoid aggregate in the red pulp

      Hemophagocytosis

      Hemophagocytosis

      EBER positive T cells in spleen

      EBER positive T cells in spleen

      Ulcerative lesion

      Ulcerative lesion

      Perivascular and periadnexal involvement

      Perivascular and periadnexal involvement


      Follicular and paracortical hyperplasia

      Follicular and paracortical hyperplasia

      Paracortical hyperplasia Paracortical hyperplasia

      Paracortical hyperplasia

      Dense CD3 positive infiltrate

      Dense CD3 positive infiltrate

      TCR delta positive infiltrate

      TCR delta positive infiltrate

      EBER positive T cells in lymph node

      EBER positive T cells in lymph node


      CD3 in the red pulp

      CD3 in the red pulp

      CD8 positive T cells CD8 positive T cells

      CD8 positive T cells

      Peripheral smear description
      Positive stains
      Negative stains
      Flow cytometry description
      Flow cytometry images

      Images hosted on other servers:
      NK cells with abnormal phenotype in the bone marrow

      NK cells with abnormal phenotype in the bone marrow

      Molecular / cytogenetics description
      • Chromogenic in situ hybridization for EBV encoded RNA (EBER ISH) must be positive
        • Demonstration of EBER positivity in T cells by double staining with EBER / CD3 is advisable
      • T cell receptor clonality studies by fragment analysis are mostly monoclonal but can be oligoclonal or polyclonal (Ann Oncol 2009;20:1472)
      • Abnormal karyotype with 6q deletion has been reported (Int J Cancer 1997;71:943)
      • Next generation sequencing (NGS) studies are limited and mostly nondiscriminatory between CAEBV and other EBV positive T / NK cell lymphoproliferative disorders (Nat Microbiol 2019;4:404)
        • Somatic mutations are present in 28.75% of 80 cases
        • DDX3X is the most frequently mutated gene (87.5%, 14/16 patients)
        • Recurrent mutations also involved chromatin modifiers and epigenetic regulators (KMT2D, BCOR, BCORL1, TET2, KDM6A)
        • In contrast to extranodal NK / T cell lymphoma (ENKTL), TP53 mutations have not been identified so far (Cancers (Basel) 2021;13:1414)
      Sample pathology report
      • Spleen, splenectomy:
        • EBV positive T cell lymphoproliferative disorder, consistent with chronic active EBV disease of T cell type (see comment)
        • Comment: Histologic sections show a preserved splenic architecture. There are mildly atypical lymphoid cells predominantly in the red pulp, while the white pulp is unremarkable. Performed immunohistochemistry stains show that most of the lymphoid infiltrate in the red pulp is composed of T cells positive for CD3, with a slight decrease in CD5 expression. Most of the T cells are positive for CD8, TIA, granzyme B and perforin. Some T cells in the red pulp are also CD4 positive. In situ hybridization for EBV (EBER) is positive in numerous cells in the red pulp, with a distribution similar to the CD8 positive T cells. CD56 is negative. There is evidence of hemophagocytosis on H&E stained sections as well as by CD68 and CD163 immunohistochemistry. TCR clonality by fragment analysis showed a polyclonal pattern. In summary, the overall findings are consistent with chronic active EBV (CAEBV) disease of T cell type.
      Differential diagnosis
      Board review style question #1

      A 13 year old Hispanic boy presented with fever, generalized lymphadenopathy, hepatosplenomegaly and cutaneous papules for 4 months. He was previously healthy. After a thorough clinical and laboratory workup, it was shown that he has an elevation of liver enzymes up to 5 times the upper limit of normal range and an EBV viral load of 400,000 copies/mL. Screening for primary and secondary immunodeficiencies is negative. The above image shows the morphological findings of an axillary lymph node excisional biopsy. Which statement about this clinicopathological entity is correct?

      1. EBV positive cells by in situ hybridization (EBER) should also be positive for CD20 in this lymphoproliferative disorder
      2. High EBV viral load in this setting confirms the diagnosis of infectious mononucleosis
      3. Most of the affected patients are elderly and a presentation in childhood is highly unusual
      4. Preservation of the lymph node architecture, with paracortical and follicular hyperplasia, is not specific but is compatible with the diagnosis of (systemic) chronic active EBV disease (CAEBV)
      Board review style answer #1
      D. Preservation of the lymph node architecture, with paracortical and follicular hyperplasia, is not specific but is compatible with the diagnosis of (systemic) chronic active EBV disease (CAEBV). The affected tissues in CAEBV usually do not show prominent or specific morphological features and lymph nodes often have a pattern of mixed follicular and paracortical hyperplasia. Answer C is incorrect because CAEBV typically occurs in children and adolescents. Answer B is incorrect because a high EBV viral load is essential for the diagnosis of CAEBV and cannot be used to exclude infectious mononucleosis alone. Answer A is incorrect because the EBV positive cells must be of T or NK cell lineage; B cell type is excluded by current diagnostic criteria.

      Comment Here

      Reference: Systemic chronic active EBV disease

      T cell / histiocyte rich
      Definition / general
      • T cell / histiocyte rich large B cell lymphoma (THRLBCL) is a large B cell lymphoma in which there are scattered neoplastic B cells that compose < 10% of the total cells and are seen in a background of numerous reactive small T cells and histiocytes with no significant small B cell component
      • Subset of cases of THRLBCL are known to transform from nodular lymphocyte predominant Hodgkin lymphoma (NLPHL)
      • There is no significant difference in the diagnostic criteria for THRLBCL in the WHO 5th edition versus the International Consensus Classification (ICC)
      Essential features
      • Typically, an aggressive lymphoma that may involve several anatomic sites and may be resistant to chemotherapy
      • Often a diffuse growth pattern (Virchows Arch 2011;459:557)
      • Neoplastic cells compose < 10% of total cells and are often scattered and not in aggregates (Virchows Arch 2011;459:557)
      • Reactive cells compose > 90% of total cells with numerous small T cells and histiocytes but few reactive B cells
      • Neoplastic B cells are positive for pan B cell markers and CD45 but typically negative for CD15 and CD30
      Terminology
      • T cell rich B cell lymphoma
      • B cell lymphoma rich in T cells and simulating Hodgkin disease
      • Histiocyte rich / T cell rich large B cell lymphoma
      • T cell rich / histiocyte rich large B cell lymphoma
      • Histiocyte rich large B cell lymphoma
      • Secondary THRLBC has been used to designate cases of T cell / histiocyte rich large B cell lymphoma that has progressed from NLPHL
      ICD coding
      • ICD-O: 9688/3 - T cell / histiocyte rich large B cell lymphoma
      • ICD-11: 2A81.4 - T cell / histiocyte rich large B cell lymphoma
      Epidemiology
      • Usually middle aged adults or older, although rare pediatric cases are noted
      • Mild preferential involvement in men compared to women
      • In the United States, Black people are more often affected than White people
      • References: Blood 2022;140:1229, Leukemia 2022;36:1720
      Sites
      • Typically involves lymph nodes but is also often detected in bone marrow, spleen and liver with a high stage at diagnosis (stage III - IV)
      Pathophysiology
      • Cells of origin are germinal center B cells that show somatic hypermutation
      • Overlap between THRLBCL and NLPHL with some patients having both types of lymphoma either synchronously or metachronously (Pathology 2020;52:142)
      • Both tumors have some similarities in their immunophenotype
      • JUNB, DUSP2, SGK1, SOCS1 and CREBBP alterations have been detected in THRLBCL and in NLPHL variant (Haematologica 2019;104:330)
      • Gene expression profiles for THRLBCL and NLPHL are similar
      • This is thought to be from a proinflammatory host response with expression of interferons (such as IL4) and metal binding proteins (such as MT2A)
      Etiology
      • Unknown
      Diagrams / tables

      Contributed by Konrad Chan, M.D.
      NLPHL versus THRLBCL NLPHL versus THRLBCL

      NLPHL versus THRLBCL

      NLPHL versus THRLBCL

      CHL versus THRLBCL

      Clinical features
      • Patients often have B symptoms and are usually diagnosed at a high stage of disease (stage III - IV) unlike NLPHL, which is typically diagnosed at a lower stage (stage I - II)
      • International prognostic index (IPI) score is usually intermediate risk or high risk
      • Disease is often refractory to the chemotherapy regimens currently in use (Leuk Lymphoma 2019;60:3426)
      Diagnosis
      • Lymphadenopathy is often detected by clinical exam and by imaging studies with PET / CT scans looking for metabolically active lymph nodes
      • Subsequent excision of a mass or lymph node with morphology exam and immunohistochemistry is necessary for an accurate diagnosis
      • References: Blood 2022;140:1229, Leukemia 2022;36:1720
      Laboratory
      • Patients typically have elevated lactate dehydrogenase (LDH)
      Radiology description
      • PET / CT scans show hypermetabolic lymphadenopathy with elevated maximum standard uptake value (SUV max)
      • Total metabolic tumor volume is often more than that seen in diffuse large B cell lymphoma, not otherwise specified (DLBCL, NOS) and NLPHL; splenomegaly and hepatomegaly are also frequently seen
      • References: Blood 2022;140:1229, Leukemia 2022;36:1720
      Prognostic factors
      Case reports
      Treatment
      Gross description
      • Lymph nodes are typically enlarged with a homogenous gray appearance on cut surface and no nodularity
      Gross images

      Contributed by Mingyi Chen, M.D., Ph.D. (Case #317)
      Axillary node

      Axillary node

      Microscopic (histologic) description
      • Lymph node is either completely or partially effaced by lymphoma with a diffuse or vaguely nodular growth pattern (Virchows Arch 2011;459:557)
      • Scattered single neoplastic large B cells (< 10% of all cells) among a background of small reactive T cells and histiocytes
      • Near absence of small B lymphocytes in the background; no remnants of B cell follicles or clusters of small B lymphocytes
      • Large numbers of plasma cells and eosinophils are usually not present (Virchows Arch 2011;459:557)
      • Large neoplastic B cells are scattered and not seen in aggregates or sheets
      • Large neoplastic B cells vary in appearance and can look similar to centroblasts, immunoblasts, Hodgkin and Reed-Sternberg (HRS) cells or lymphocyte predominant (LP) cells (Virchows Arch 2011;459:557)
      • Diverse morphologic and immunophenotypic features (Am J Surg Pathol 2002;26:1458)
        • LP-like cells have polylobated and vesicular nuclei that look similar to kernels of popcorn and have been called popcorn cells
        • Centroblast-like cells have oval / round nuclei with vesicular chromatin and small basophilic nucleoli adjacent to the nuclear membrane
        • HRS-like (Hodgkin Reed-Sternberg cell-like) cells are large multinucleated cells with pleomorphic nuclei with prominent eosinophilic or amphophilic round / central nucleoli, sometimes binucleated
      • Neoplastic B cells are surrounded by clusters of bland nonepithelioid histiocytes that do not have grooved nuclei
      • Necrosis can be present
      • THRLBCL in the spleen can involve the white pulp with a micronodular pattern, sometimes in fibrosis
      • Involvement of the liver can be seen within portal areas
      • Bone marrow with THRLBCL can be paratrabecular or sinusoidal with a diffuse or nodular growth pattern
      Microscopic (histologic) images

      Contributed by Aaron Auerbach, M.D., M.P.H., Asmaa Gaber Abdou, M.D. and Nancy Youssef Asaad, M.D.
      Scattered large atypical cells

      Scattered large atypical cells

      Large neoplastic cells

      CD20 in a biopsy

      CD20 in a biopsy

      CD20 in an excision

      CD20 in an excision

      CD3+ small T cells

      CD3+ small T cells

      CD68+ histiocytes

      CD68+ histiocytes



      Contributed by Contributed by Mingyi Chen, M.D., Ph.D. (Case #317)

      CD3

      CD20

      PAX5

      CD30

      Cytology description
      • Cytology specimens will contain relatively rare large atypical lymphocytes in a background of more numerous small mature lymphocytes and cytologically bland histiocytes
      • Diagnosis of THRLBCL cannot be accurately made on cytologic specimens alone and requires a biopsy or excision as well as phenotyping by immunohistochemistry
      • References: Blood 2022;140:1229, Leukemia 2022;36:1720
      Positive stains
      Negative stains
      Flow cytometry description
      • Flow cytometry is typically not helpful due to the relative paucity of neoplastic B cells
      Molecular / cytogenetics description
      • IgH gene rearrangement studies can be detected by PCR, although the results may not show clonality due to the paucity of neoplastic B cells
      • T cell receptor (TCR) gene rearrangement should not be detected
      • References: Blood 2022;140:1229, Leukemia 2022;36:1720
      Videos

      NLPHL versus THRLBCL

      Sample pathology report
      • Lymph node, excision:
        • T cell / histiocyte rich large B cell lymphoma (see comment)
        • Comment: Histologic sections show a few cores of soft tissue involved by a diffuse proliferation of numerous small lymphocytes and histiocytes with occasional isolated large atypical cells scattered throughout the specimen. No significant plasma cell component or eosinophilia is appreciated. No normal appearing lymph node tissue is seen. No diffuse sheets of large atypical cells are identified.
        • The scattered large, atypical cells are B cells positive for CD20, CD45, CD79a, PAX5, OCT2, BCL6 and MUM1 and negative for CD3, CD4, CD5, CD8, CD10, CD15, CD21, CD23, CD30, CD68, PD-1, EMA, IgD and EBER ISH. The background small lymphocytes consist mostly of T cells positive for CD3, CD5 and CD45 with approximately equal numbers of CD8+ T cells and CD4+ T cells. The CD21 and CD23 stains are negative for follicular dendritic cell networks. The CD68 stain highlights numerous histiocytes, which show normal coexpression of CD4 and CD45. The Ki67 stain is positive in the large atypical B cells as well as in scattered background small lymphocytes and histiocytes.
        • The morphologic findings together with the immunophenotype are in keeping with a diagnosis of T cell / histiocyte rich large B cell lymphoma (THRLBCL). THRLBCL typically presents at an advanced Ann Arbor stage with frequent liver and spleen involvement.
      Differential diagnosis
      • Nodular lymphocyte predominant Hodgkin lymphoma (NLPHL):
        • Can be difficult to distinguish from THRLBCL, especially on small biopsy material (Pathology 2020;52:142)
        • Clinical presentation is less aggressive in NLPHL, which usually presents at low stage (stage I - II), usually as a solitary mass
        • Classical NLPHL has an indolent clinical course, while THRLBCL is an advanced stage aggressive disease
        • There are cases of NLPHL with a diffuse T cell rich THRLBCL-like morphology (pattern E) but unlike THRLBCL, these have evidence of residual follicular dendritic cell meshworks and a preponderance of follicular helper T cells, which form rosette-like structures around some of the large B cells
        • Presence of nodularity in NLPHL distinguishes it from THRLBCL (Pathology 2020;52:142)
        • Presence of numerous mantle zone B cells that are positive for CD20, CD23, IgD and TCL1 favors NLPHL over THRLBCL
        • T cells positive for follicular helper T cell markers (e.g., CD4+, CD57+, PD-1+, ICOS+, CXCL13+, MUM1+, BCL6+, BOB1+), which form rosette-like structures around the neoplastic B cells favors NLPHL over THRLBCL
        • Presence of expanded follicular dendritic cell meshwork positive for CD21, CD23 and CD35 favors NLPHL over THRLBCL (Pathology 2020;52:142)
        • CD163+ histiocytes more frequent in THRLBCL than NLPHL
      • EBV+ diffuse large B cell lymphoma:
        • EBV+ DLBCL can have scattered malignant cells similar to THRLBCL and the tumor cells can look like centroblasts, immunoblasts, Reed-Sternberg cell-like or LP cells
        • Presence of EBER+ large cells is diagnostic of EBV+ DLBCL as long as it does not fulfill the requirements of another lymphoma, such as classic Hodgkin lymphoma (CHL)
        • EBER+ cells exclude a diagnosis of THRLBCL
      • Classic Hodgkin lymphoma:
        • Mixed cellularity CHL and lymphocyte depleted CHL often have ample reactive histiocytes, like THRLBCL
        • Background cells often include eosinophils and neutrophils, unlike THRLBCL
        • Bands of sclerosis in nodular sclerosing CHL are not seen in THRLBCL
        • Neoplastic B cells are CD15+, CD30+, MUM1+ and CD45-, a phenotype different from THRLBCL
        • CHL is often negative or only weakly positive in B cell markers like CD20 and PAX5, whereas these markers are strongly expressed in THRLBCL
      • Peripheral T cell lymphoma:
        • Will have numerous T cells similar to THRLBCL but the neoplastic tumor cells are the atypical T cells often present in aggregates
        • Atypical cells will express 1 or more T cell markers
        • CD30 can be expressed in a subset of patients, unlike THRLBCL
        • Some cases will show a follicular helper T cell phenotype and express CD10, BCL6, PD-1 and ICOS
        • T cell receptor gene rearrangements should be monoclonal
      • Granulomatous lymphadenitis (GL):
        • Can sometimes resemble THRLBCL, especially on small biopsy material
        • Granulomatous lymphadenitis from infections can contain necrosis as well as neutrophils and special stains can be performed to look for microorganisms
        • Sarcoidosis will typically show well formed variably confluent noncaseating granulomas with some multinucleated giant cells and can involve multiple different organs
      • Reactive conditions (drugs, viral infection, autoimmune diseases):
        • Preserved architecture
        • Heterogeneous cytological appearance of immunoblasts
        • CD15- immunoblasts
        • EBER+ in both large and small lymphocytes (infectious mononucleosis)
      Board review style question #1

      The histologic image above is from an axillary lymph node biopsy taken from a 60 year old man with axillary lymphadenopathy. Which of the following features would favor a diagnosis of T cell / histiocyte rich large B cell lymphoma (THRLBCL) over nodular lymphocyte predominant Hodgkin lymphoma (NLPHL)?

      1. Background CD21+ follicular dendritic cell networks
      2. CD30 and CD15 expression in the large cells
      3. Numerous CD4+, PD-1+ small T cells
      4. Similar histologic features involving bone marrow
      5. T cell rosettes around the large cells
      Board review style answer #1
      D. Similar histologic features involving bone marrow. THRLBCL typically presents at a high stage with frequent bone marrow involvement. NLPHL, in contrast, usually presents at low stage with only rare bone marrow involvement. Answer A is incorrect because background CD21+ follicular dendritic cell networks are a feature of NLPHL, not THRLBCL. Answer B is incorrect because CD30 and CD15 expression is most frequently negative in the large cells of THRLBCL. Answer C is incorrect because numerous CD4+, PD-1+ small T cells are a feature of NLPHL, not THRLBCL. Answer E is incorrect because T cell rosettes of PD-1+ T cells are a feature of NLPHL, not THRLBCL.

      Comment Here

      Reference: T cell / histiocyte rich
      Board review style question #2
      Which one of the following statements regarding T cell / histiocyte rich large B cell lymphoma (THRLBCL) is true?

      1. Differential diagnosis of THRLBCL includes infection, peripheral T cell lymphoma and classic Hodgkin lymphoma
      2. Large cell component of THRLBCL is CD20+, OCT2+, BCL6+ and EBV+
      3. Peritumoral environment of THRLBCL contains prominent follicular dendritic cell networks and abundant reactive small B cells
      4. THRLBCL and NLPHL show similar clinical presentation and organ involvement
      5. THRLBCL typically shows T cell receptor gene rearrangements
      Board review style answer #2
      A. Differential diagnosis of THRLBCL includes infection, peripheral T cell lymphoma and classic Hodgkin lymphoma. Answer B is incorrect because EBV is negative in THRLBCL and similar EBV+ cases would be defined as an EBV+ DLBCL. Answer C is incorrect because THRLBCL lacks follicular dendritic cell networks and has abundant small T cells. Answer D is incorrect because THRLBCL is a more aggressive disease than NLPHL. NLPHL often only presents as a single lymph node, whereas THRLBCL can present with more than one site of involvement. Answer E is incorrect because THRLBCL is a clonal B cell lymphoma and would not be expected to show T cell receptor gene rearrangements.

      Comment Here

      Reference: T cell / histiocyte rich

      T cell large granular lymphocytic leukemia
      Definition / general
      • A chronic T cell lymphoproliferative disorder characterized by a clonal proliferation of mature cytotoxic T cells without an identified cause
      • Patients commonly present with cytopenia(s) and eventually require treatment
      Essential features
      • Indolent clonal proliferation of mature cytotoxic T cells
      • Neutropenia or anemia
      • Associated with autoimmune disorders
      • Commonly CD8+ T cells coexpressing NK cell associated markers
      • Characteristic intrasinusoidal involvement in bone marrow, spleen and liver
      • Approximately half of the patients carry a STAT3 / STAT5b mutation
      Terminology
      • T cell large granular lymphocytosis; T cell lymphoproliferative disease of granular lymphocytes
      ICD coding
      • ICD-O: 9831/3 - T cell large granular lymphocytic leukemia
      • ICD-10: C91.Z - other lymphoid leukemia
      Epidemiology
      • 2 - 5% of mature lymphocytic leukemias
      • M = F
      • Commonly occurs in elderly individuals with a median age of 60 years (Blood 2017;129:1082)
      Sites
      • Peripheral blood, bone marrow, spleen, liver
      Pathophysiology
      • Oligoclonal expansion of cytotoxic large granular T cells in response to antigen stimulation
      • Subsequent clonal proliferation of large granular T cells due to constitutive upregulation of cellular survival signals or downregulation of apoptotic pathways via a secondary event
      • STAT3 / STAT5b mutation as a secondary event (N Engl J Med 2012;366:1905, Blood 2012;120:3048)
      • Other secondary events include resistance to Fas / FasL mediated cell death, increased cell survival by interleukin 15 and platelet derived growth factor and activation of NFkB pathway (Blood 2018;131:2803)
      • Neutropenia and anemia are caused by direct cytotoxicity from the clonal T cells
      Etiology
      • Chronic antigen exposure from autoimmune disorders such as rheumatoid arthritis and Sjögren syndrome
      • Chronic antigen exposure due to viral infection such as human T lymphotropic virus
      Clinical features
      • Approximately 70% of patients have neutropenia or anemia
      • Approximately 30% of patients are asymptomatic but regular complete blood count reveals lymphocytosis and cytopenia(s) (Blood 2017;129:1082)
      • Splenomegaly in approximately 30% of patients
      • Approximately 30% of patients carry autoimmune disorders with rheumatoid arthritis being by far the most common (Blood 2017;129:1082)
      • Could be associated with other hematologic malignancies such as B cell lymphoma, myelodysplastic syndrome and aplastic anemia
      Diagnosis
      • Persistent (more than 6 months) increase in the number of peripheral blood large granular lymphocytes, usually 2-20 x 10⁹
      • Distinct T cell population with coexpression of one or more NK cell associated antigens (CD16, CD56 or CD57) and decreased CD2, CD5 or CD7 expression
      • Proof of T cell clonality by a molecular or flow cytometry study
      • Intrasinusoidal cytotoxic T cell infiltrates in bone marrow, spleen or liver (Blood 2002;99:268)
      • STAT3 or STAT5b mutation (N Engl J Med 2012;366:1905, Blood 2012;120:3048)
      Laboratory
      • Variable lymphocytosis (Blood 2017;129:1082)
      • Neutropenia occurs in approximately 85% of patients, anemia in 50% with pure red cell aplasia in 10% and thrombocytopenia in < 25%
      • Rheumatoid factor and antinuclear antibodies could be positive
      Radiology description
      • Splenomegaly in 30% of patients (Blood. 2017;129:1082)
      • Hepatomegaly < 30% of patients
      • Rarely lymphadenopathy
      Prognostic factors
      Case reports
      Treatment
      • The majority of patients eventually require treatment because of severe or symptomatic neutropenia or anemia
      • No standard therapy established
      • Immunosuppressive therapy (methotrexate, cyclophosphamide or cyclosporine) as initial agents
      Microscopic (histologic) description
      • Bone marrow biopsy does not show apparent abnormal lymphocytic infiltrates by hematoxylin and eosin stain; immunohistochemical studies reveal the characteristic linear / intrasinusoidal distribution of cytotoxic T cells (Blood 2002;99:268)
      • Spleen biopsy demonstrates linear arrays of cytotoxic T cells in the sinusoids and red pulp cords
      • Liver biopsy reveals linear arrays of cytotoxic T cells in the sinusoids
      Microscopic (histologic) images

      Contributed by Min Shi, M.D., Ph.D. and Dragan Jevremovic, M.D., Ph.D.

      T LGLL bone marrow involvement

      Intrasinusoidal CD3+ T LGLL

      Intrasinusoidal CD8+ T LGLL

      Intrasinusoidal granzyme B+ T LGLL

      Intrasinusoidal TIA1+ T LGLL


      T LGLL liver involvement

      CD8+ T LGLL in liver

      CD5- T LGLL in liver

      TIA1+ T LGLL in liver

      Peripheral smear description
      • Increased lymphocytes containing small to intermediate sized reticulated nuclei and fine to coarse azurophilic cytoplasmic granules
      Peripheral smear images

      Contributed by Min Shi, M.D., Ph.D. and Dragan Jevremovic, M.D., Ph.D.

      T LGLL in peripheral blood

      Positive stains
      Negative stains
      Flow cytometry description
      • Mature CD3 positive T cells, usually coexpress NK cell associated markers (CD16 and CD57), with variable expression of other pan T cell markers such as CD2, CD5, CD7; 25% of cases are KIR restricted
      • CD4- CD8+ T cell type being the most common, followed by TCRγδ+ T cell type, CD4+ CD8- T cell type, TCRαβ+ CD4- CD8- T cell type and rarely mixed phenotype (Hum Pathol 2018;81:96)
      Flow cytometry images

      Contributed by Min Shi, M.D., Ph.D. and Dragan Jevremovic, M.D., Ph.D.

      T LGLL flow cytometry

      Molecular / cytogenetics description
      Sample pathology report
      • Peripheral blood, bone marrow aspirate and biopsy, iliac crest:
        • T cell large granular lymphocytic leukemia involving 15% marrow cellularity with normal trilineage hematopoiesis
        • Peripheral blood smear: neutropenia; absolute lymphocytosis with increased large granular cells
        • Bone marrow aspirate and biopsy: adequate quality; normal M:E ratio; slightly hypercellular marrow with left shift granulopoiesis, otherwise normal hematopoiesis; slight interstitial lymphocytic infiltrates of small lymphocytes
        • Flow cytometric immunophenotyping: distinct T cell population (70% of gated lymphoid events, 30% of total analyzed events) with expression of CD2, CD3, CD5 (dim), CD7 (dim), CD8, CD16, CD57, CD94 (partial), CD158b (restricted); they do not express CD4, CD56, CD158a, CD158e, NKG2A and TCR gamma / delta
        • Molecular study for TCR gene rearrangement: positive for clonal TCR gene rearrangement
      Differential diagnosis
      Board review style question #1
      What is the most common immunphenotype of T cell large granular lymphocytic leukemia?

      1. CD2+ CD3- CD4- CD5- CD7+ CD8+ CD16+ CD56- CD57+
      2. CD2+ CD3+ CD4- CD5- CD7+ CD8+ CD16+ CD56- CD57+
      3. CD2+ CD3+ CD4+ CD5- CD7+ CD8+ CD16+ CD56- CD57+
      4. CD2- CD3+ CD4- CD5- CD7+ CD8+ CD16- CD56- CD57-
      5. CD2+ CD3- CD4- CD5- CD7+ CD8- CD16- CD56+ CD57-
      Board review style answer #1
      B. The most common immunophenotype of T cell large granular lymphocytic leukemia is that of mature CD3+ CD8+ T cells expressing NK cell associated markers such as CD16 and CD57 and variable pan T cell marker lost. A) Typical chronic lymphoproliferative disorder of NK cells immunophenotype. C) Rare T cell large granular lymphocytic leukemia immunophenotype. D) Peripheral T cell lymphoma immunophenotype without NK cell associated marker expression. E) Aggressive NK cell leukemia immunophenotype.

      Comment Here

      Reference: T cell large granular lymphocytic leukemia
      Board review style question #2
        Which genetic abnormality is most commonly identified in T cell large granular lymphocytic leukemia?

      1. STAT5b mutation
      2. NOTCH mutation
      3. TP53 mutation
      4. STAT3 mutation
      5. TCL1A rearrangement
      Board review style answer #2
      D. STAT3 mutation is the most common genetic mutation in T cell large granular lymphocytic leukemia which results in constitutive activation of JAK / STAT3 pathway and drives proliferation and survival of neoplastic cells.

      Comment Here

      Reference: T cell large granular lymphocytic leukemia

      T prolymphocytic leukemia

      T prolymphocytic leukemia
      Definition / general
      • T prolymphocytic leukemia (T PLL) is an aggressive, mature T cell leukemia, composed of small to medium sized mature T cells, usually with high white blood cell (WBC) count and widespread organ involvement
      • No significant changes in WHO 5 and ICC classifications (Leukemia 2022;36:1720, Blood 2022;140:1229)
      Essential features
      • Aggressive leukemia of mature T cells
      • High white blood cell count (lymphocytosis)
      • Usually CD4+
      • Usually TCL1 positive by immunophenotyping and TCL1 rearranged by FISH
      • Difficult to treat, poor prognosis
      Terminology
      • T cell chronic lymphocytic leukemia (small cell variant of T cell prolymphocytic leukemia)
      ICD coding
      • ICD-O: 9834/3 - prolymphocytic leukemia, T cell type
      • ICD-10: C91.6 - prolymphocytic leukemia of T cell type
      Epidemiology
      • Rare (2% of mature lymphocytic leukemias)
      • Adults and elderly (> 30 years, median age: 65 years)
      Sites
      • Widespread: peripheral blood, bone marrow, lymph nodes, spleen, liver, skin
      Pathophysiology
      • Combination of overexpression of TCL1 family of proteins (which stimulate AKT / protein kinase B driven proliferation) and functional deficit of ATM protein (Nat Commun 2018;9:697)
      Etiology
      • Unknown at this time
      • Higher risk in patients with ataxia-telangiectasia (germline ATM mutations)
      Clinical features
      • High tumor burden with high white blood cell count (median 50 - 60 x 109/L), bone marrow involvement (in 100% of patients) with cytopenias, hepatosplenomegaly, lymphadenopathy, skin and mucosal lesions, other organ involvement / dysfunction (Am J Hematol 2017;92:441)
      • Prominent constitutional symptoms
      • 20 - 30% present with inactive disease and progress to active disease within 1 - 2 years (criteria for progression: lymphocyte doubling time (LDT) of less than 6 months or lymphocyte count increase by > 50% in 2 months) (Blood 2019;134:1132)
      Diagnosis
      • Peripheral blood: morphology + flow cytometry with or without fluorescent in situ hybridization (FISH)
      • Bone marrow or solid tissue biopsy (lymph node, spleen, liver, skin, other): morphology + phenotyping (immunohistochemistry or flow cytometry) with or without FISH
      • Note: FISH not necessary if TCL1 overexpression in neoplastic T cells can be shown by IHC or flow (J Clin Pathol 2018;71:309)
      • Consensus criteria for diagnosis by the T PLL International Study Group (Blood 2019;134:1132)
      • All 3 major criteria or the first 2 major and 1 minor criteria are required for diagnosis
      • Major criteria:
        • 5 x 109/L cells of T PLL phenotype in peripheral blood or bone marrow
        • T cell clonality by molecular or flow cytometry methods
        • Abnormalities of 14q32 or Xq28 or expression of TCL1A/B or MTCP1
      • Minor criteria:
        • Abnormalities involving chromosome 11 (11q22.3; ATM)
        • Abnormalities in chromosome 8: idic(8)(p11), t(8;8), trisomy 8q
        • Abnormalities in chromosome 5, 12, 13, 22 or complex karyotype
        • Involvement of specific sites (spleen, effusions)
      Laboratory
      • Increased peripheral blood lymphocytes, often > 100 x 109/L, increased lactate dehydrogenase (LDH) and beta2 microglobulin (Ann Oncol 2017;28:1554)
      • Negative serology for HTLV1
      Radiology description
      • Prominent hepatosplenomegaly and widespread lymphadenopathy; moderate to high FEV on PET scan
      Prognostic factors
      • Overall poor prognosis; median survival with active disease is 1 - 2 years
      • Worse prognosis: pleural effusion, high LDH (> 1668 IU/l), low hemoglobin (< 9.3 g/dl), complex karyotype (Ann Oncol 2017;28:1554, Am J Hematol 2017;92:441)
      Case reports
      Treatment
      • Only for active disease
      • Standard treatment: alemtuzumab (anti-CD52) variable allogeneic bone marrow transplant
      • Experimental therapies with BCL2, JAK3 or HDAC inhibitors (Blood 2019;134:1132)
      Microscopic (histologic) description
      • Perivascular and diffuse tissue infiltrates of uniform small to medium sized lymphocytes
      • Red pulp involvement in the spleen
      Microscopic (histologic) images

      Contributed by Min Shi, M.D., Ph.D. and Dragan Jevremovic, M.D., Ph.D.

      Diffuse bone marrow involvement

      Splenic red pulp infiltrate

      Monotonous, paracortical lymphoid infiltrate

      Intermediate size, atypical lymphocyte


      CD3 positivity in bone marrow

      Monotonous T cell infiltrate

      TCL1A positivity

      Uniform strong TCL1A staining

      Peripheral smear description
      • Lymphocytosis with small to medium sized lymphocytes with cytoplasmic blebs, clumped chromatin and variably prominent central nucleolus
      • Small cell variant in 25% of cases: smaller cells without obvious nucleolus
      • Cerebriform variant in 5% of cases
      • Reference: Blood 2019;134:1132
      Peripheral smear images

      Contributed by Min Shi, M.D., Ph.D. and Dragan Jevremovic, M.D., Ph.D.

      Cerebriform variant

      Prolymphocytes in peripheral blood

      Small cell variant

      Positive stains
      Negative stains
      Flow cytometry description
      Flow cytometry images

      Contributed by Min Shi, M.D., Ph.D. and Dragan Jevremovic, M.D., Ph.D.

      T PLL flow cytometry

      Molecular / cytogenetics description
      • Clonal T cell receptor gene rearrangements (TRB@ and TRG@)
      • FISH is commonly used for diagnosis; T cell receptor locus rearranged with the TCL1 family of genes:
        • TCL1A / TCL1B rearrangement: inv(14)(q11;q32) in 80%, t(14;14)(q11;q32) in 10%
        • Rarely MTCP1 rearrangement t(X;14)(q28;q11)
        • Rarely negative for TCL1A / TCL1B or MTCP1 rearrangements (Blood 2019;134:1132)
      • Complex karyotype in 70 - 80%; abnormalities of chromosomes 6, 8, 12p, 17p
      • Mutations in ATM gene on 11q23 in 80 - 90%
      • Other mutations / alterations: TP53, JAK / STAT pathway genes IL2RG, JAK1, JAK3, STAT5B (Blood 2014;124:1460)
      Molecular / cytogenetics images

      Contributed by Min Shi, M.D., Ph.D. and Dragan Jevremovic, M.D., Ph.D.

      FISH for TCL1A separation

      Sample pathology report
      • Peripheral blood, bone marrow aspirate and biopsy, iliac crest:
        • T cell prolymphocytic leukemia, extensively involving peripheral blood and bone marrow, with decreased trilineage hematopoiesis.
        • Peripheral blood:
          • Complete blood count: hemoglobin 11.2 g/dL; red blood cell 3.55 x 1012/L; mean corpuscular volume 97.2 fL; red blood cell distribution width 15.2%; white blood cell 470.0 x 109/L; PLT 111 x 109/L
          • Cell percentage of total cells: neutrophils (4%), lymphocytes (95%), monocytes (1%)
          • Peripheral smear: lymphocytosis; small to intermediate lymphocytes with mature chromatin, prominent nucleoli, eccentric nuclei and moderate amounts of basophilic cytoplasm
        • Bone marrow aspirate and biopsy:
          • Aspirate quality: cellular
          • Biopsy quality: adequate
          • Myeloid to erythroid (M:E) ratio: normal, 3:1
          • Cellularity: hypercellular, 90%
          • Erythroid precursors: markedly decreased quantity, normal morphology
          • Myeloid precursors: markedly decreased quantity, normal morphology, blasts not increased
          • Megakaryocytes: markedly decreased quantity, normal morphology and distribution
          • Lymphocytes: abnormal (diffuse) infiltrates of small to intermediate sized cells present (90% of the total marrow cellularity)
          • Plasma cells: not increased
        • Ancillary studies:
          • Flow cytometry, bone marrow:
            • Blasts: not increased by CD45 / side scatter and CD34
            • B cells: no monotypic; normal expression pattern of CD19, CD10, surface kappa and lambda
            • T cells / NK cells: distinct T cell population
            • Express: CD4, CD2, CD3, CD5, CD7
            • Do not express: CD8, CD16, TCR gamma / delta, CD1a, nTdT, cMPO, cCD79a, cCD22
            • Estimated size: 95% gated lymphoid events; 87% total analyzed events
            • T cell lymphoma FISH, bone marrow: the result is abnormal and indicates 88.5% of nuclei had a rearrangement involving TCL1A; this observation may indicate a clone with inv(14) or t(14;14), which are common rearrangements in T cell prolymphocytic leukemia
      Differential diagnosis
      Board review style question #1
      What is the most common phenotype of T cell prolymphocytic leukemia (T PLL)?

      1. CD3-, CD19+, CD20+, CD5+, CD23+
      2. CD3+, CD4+, CD5+, CD7-, CD8-, CD26-
      3. CD3-, CD4+, CD8+, TdT+
      4. CD3+, CD4+, CD5+, CD7+, CD8-
      5. CD3+, CD4-, CD7-, CD8+, CD16+
      Board review style answer #1
      D. CD3+, CD4+, CD5+, CD7+, CD8-. The most common phenotype of T PLL is that of mature CD4+ T cells expressing pan T cell antigens, including CD7. Answer A is a common B CLL / SLL phenotype. Answer B is a common Sézary syndrome phenotype. Answer C is a common T lymphoblastic leukemia / lymphoma phenotype. Answer E is a common T cell large granulocytic lymphocyte leukemia phenotype.

      Comment Here

      Reference: T cell prolymphocytic leukemia
      Board review style question #2

      Which genetic abnormality is used to define T cell prolymphocytic leukemia (T PLL)?

      1. 17p (TP53) deletion
      2. CCND1 rearrangement
      3. MYC rearrangement
      4. TCL1 rearrangement
      5. TP53 mutations
      Board review style answer #2
      D. TCL1 rearrangement on chromosome 14 is the most common genomic alteration, which results in the overexpression of TCL1 protein and drives proliferation of neoplastic cells in T cell prolymphocytic leukemia.

      Comment Here

      Reference: T cell prolymphocytic leukemia

      T/NK cell disorders-general
      Definition / general
      • T/NK cell neoplasms are clonal tumors of mature and immature T cells or natural killer cells at various stages of differentiation (WHO 2008)
      • T and NK neoplasms are considered together because they are closely related and share immunophenotypic and functional properties
      Epidemiology
      • Precursor T lymphoid neoplasms (T-ALL / LBL): primarily diseases of children / young adults, 10 - 15% of acute lymphoblastic leukemia (ALL), 85 - 90% of lymphoblastic lymphoma (LBL), male predominance (Pediatr Blood Cancer 2008;51:489)
      • Mature T/NK cell neoplasms: relatively uncommon; 15% of non-Hodgkin lymphomas in U.S. / Western Europe; are usually aggressive with median survival of 10 - 30 months
      • Large international study (Blood 1997;89:3909): 12% of all NHL are T/NK cell (peripheral T cell lymphoma NOS - 25.9%, angioimmunoblastic T cell lymphoma - 18.5%)
      • Significant geographic and racial incidence variation: generally more common in Asia or Asian populations; adult T cell leukemia / lymphoma in endemic HTLV-I areas, enteropathy associated T cell lymphoma in those of Welsh and Irish descent
      • Overall incidence of T cell neoplasms in the U.S. is 2.6 cases / 100,000 persons / year (slight overall increase, mostly in the cutaneous T cell lymphoma category)
      Sites
      • Usually mirror the distribution of their normal cellular counterparts
      • Most T cell lymphomas are nodal; most NK cell lymphomas are extranodal
      • Lymphomas of the innate immune system: predominantly extranodal; pediatric and young adult (aggressive NK cell leukemia, systemic EBV+ T-LPD of childhood, hepatosplenic T cell lymphoma, mucosal / cutaneous γδ T cell lymphomas)
      • Lymphomas of the adaptive immune system: mainly nodal, usually adult patients
      Pathophysiology
      • Cells of the innate immune system (NK cells, CD3+ CD56+ T cells or NK-like cells, and γδ T cells): first line of defense, primitive response, do not require antigen presentation
      • Adaptive immune system: utilizes specificity and memory
      • T lymphocytes originate from a bone marrow precursor undergoing maturation in the thymus
      • Cortical thymocytes: positive for TdT, CD1a, CD3, CD5, CD7 (immature T cell phenotype); initially CD4 / CD8 double negative, then double positive, then either CD4 or CD8
      • Medullary thymocytes: phenotype similar to mature T cells (could be αβ or γδ); associated with a complete CD3 complex (γ, δ and ε chains)
      • γδ T cells: < 5% of all normal T cells, CD4 / CD8 double negative, usually lack CD5, a subpopulation expresses CD8, restricted distribution (splenic red pulp, intestinal epithelium, other epithelial sites)
      • Two categories of CD4+ T cells: Th1 (secrete IL-2 and interferon γ) and Th2 (secrete IL-4, 5, 6 and 10)
      • Unique subset of CD4+ T cells found in the germinal center: follicular T helper cells (TFH) - positive for BCL6, CD10, CD4, CD57, PD-1, CXCL13
      • Regulatory CD4+ T cells (Treg) with immune suppressing function: positive for CD25, FOXP3 (adult T cell leukemia / lymphoma is associated with marked immunosuppression)
      • NK cells: incomplete T cell receptor complex; positive for cytoplasmic CD3 (not surface), CD2, CD7, CD16, CD56, variably CD57, cytotoxic proteins, sometimes CD8
      Etiology
      • EBV associated: extranodal NK/T cell lymphoma, systemic EBV+ T cell lymphoproliferative disorder of childhood, Hydroa vacciniforme-like T cell lymphoma
      • HTLV-I: adult T cell leukemia / lymphoma
      Microscopic (histologic) description
      • 99% have diffuse lymphocytic infiltrate with numerous, evenly dispersed, ill defined, small clusters of epithelioid histiocytes, but these may actually be Hodgkin lymphoma or lymphoplasmacytic lymphoma
      • Peripheral T cell lymphomas are, by definition, composed of mature (not precursor) T cells; often misdiagnosed (Mod Pathol 2002;15:420)
      • Most relapses of nodal disease have similar histology, pattern of nodal involvement, immunophenotype (Am J Clin Pathol 2000;114:438)
      • Large B cells present in peripheral T cell disorders may be due to EBV (in immunodeficient patients) or represent clonal populations that develop into diffuse large B cell lymphoma (Am J Clin Pathol 2000;114:236, Am J Clin Pathol 2002;117:368)
      Immunohistochemistry & special stains
      • Aberrant T cell phenotype (immunophenotypic clusters exhibiting altered expression of T cell antigens relative to normal) usually implies clonality
      • Most common aberrant values are for CD3, CD7, CD5; CD2 is most stable; for CD7, deletion is common (Am J Clin Pathol 2001;116:512)
      • Rarely express B cell markers CD20 and CD79a (Mod Pathol 2001;14:105)
      • NK markers are CD11b, CD11c, CD16, CD56, CD57 and polyclonal CD3 (detects CD3 epsilon)
      • Recommended immunostains include T cell markers (CD3, CD43, CD45RO) and B cell markers (CD20)
      • Granzyme M (suggested role in the effector phase of the innate immune system): NK cells, CD3+ CD56+ T cells and γδ T cells (hepatosplenic T cell lymphoma, cutaneous γδ T cell lymphomas, most intestinal T cell lymphomas)
      Molecular / cytogenetics description
      • Must first establish that process is neoplastic
      • Only a few T cell neoplasms have specific genetic abnormalities: anaplastic large cell lymphoma (ALK), HSTL (iso7q)
      • Immunostaining or flow cytometry is useful for classification but usually cannot prove clonality
      • Requires T cell receptor gene rearrangement studies by PCR (can use paraffin)
      • Analysis of KIR receptors in NK cell proliferations is useful (NK cells do not rearrange the T cell receptor genes)

      Unusual morphologic patterns of follicular lymphoma
      Definition / general
      • Follicular lymphoma (FL) is a neoplasm composed of germinal center B cells (centrocytes and centroblasts in varying proportions), usually (but not always) distributed in a follicular pattern
      • Distinct biologic variants of FL have identifiable clinical and histopathologic features that help their recognition in clinical practice; examples include pediatric type FL, primary cutaneous follicular center cell lymphoma, primary testicular FL and duodenal type FL
      • Unusual morphologic patterns can also be seen in cases of FL and do not represent biologically distinct variants of FL; these FL may not be easily recognizable due to histopathologic overlap with other B cell lymphomas and are discussed in this chapter (see Table 1)
      Essential features
      • FL with Castleman-like changes:
        • Castleman-like changes with B cell markers, BCL2 and germinal center markers
      • FL with plasmacytic differentiation and IgG4 positive plasma cells:
        • FL with associated increased and monotypic plasma cells
      • FL with marginal zone (MZ) differentiation involving mucosa associated lymphoid tissue (MALT) sites:
        • Marginal zone differentiation
        • Monocytoid B cells
        • Lymphoepithelial lesions
        • Plasmacytic differentiation
      • FL negative for CD10, positive for MUM1 and with BCL6 abnormalities:
        • More frequent diffuse proliferation and grade 3B compared with conventional FL
        • Usually lack BCL2 rearrangement
        • MUM1 expression and BCL6 abnormalities (translocation, amplification)
      • EBV positive FL:
        • Morphology and immunophenotype similar to the conventional type of FL
        • EBER is positive in ~100% of cases
      • Floral variant of FL:
        • Irregular neoplastic follicles with expanded, prominent mantle zone lymphocytes penetrating the neoplastic follicles
      ICD coding
      • ICD-O: 9690/3 - follicular lymphoma, NOS
      • ICD-10: C82.90 - follicular lymphoma, unspecified, unspecified site
      Epidemiology
      Diagrams / tables

      Table 1
      Unusual morphologic patterns of FL
      FL with Castleman-like changes
      FL with plasmacytic differentiation with or without IgG4 positive plasma cells
      FL with marginal zone differentiation, typically involving MALT sites
      FL negative for CD10, positive for MUM1 with BCL6 abnormalities
      EBV positive FL
      Floral variant of FL
      Clinical features
      Diagnosis
      • Constitutional symptoms: fatigue, fever or night sweats, weight loss or recurrent infections
      • Diagnostic imaging: enlarged lymph nodes, nodules / lesions / infiltration in extranodal sites
      • Tissue biopsy / excision (lymph node / extranodal sites)
      • Laboratory tests: peripheral blood exam / bone marrow biopsy (especially in staging) / lumbar puncture
      • Reference: Am J Hematol 2018;93:296
      Prognostic factors
      Case reports
      • 32 year old man presented with tonsillar hypertrophy in laryngoscopy and underwent a low temperature plasma tonsillectomy (Diagn Pathol 2019;14:70)
      • 74 year old woman presented with a 4 year history of squamous cell carcinoma of the skin and regional lymphadenopathy; biopsy of the postauricular lymph node was performed (Virchows Arch A Pathol Anat Histopathol 1985;405:473)
      • 77 year old woman presented with left foot swelling attributed to an insect bite; Doppler ultrasound was negative for deep venous thrombosis but the study revealed left inguinal lymphadenopathy (Hum Pathol 2017;68:136)
      Treatment
      • More studies are needed to evaluate the response to the standard therapy for FL
      Frozen section description
      • Same histologic features (see Microscopic description)
      • Cytogenetics, flow cytometry, preparation of imprints should be considered
      Microscopic (histologic) description
      • FL with Castleman-like changes (Hum Pathol 2017;68:136):
        • Small and atrophic neoplastic follicles
        • Atretic and hyalinized germinal centers
        • Concentric mantle zones
        • Increased interfollicular vascularity
        • Penetrating hyalinized blood vessels
      • FL with plasmacytic differentiation and IgG4 positive plasma cells (Mod Pathol 2010;23:71):
        • FL with associated increased and monotypic plasma cells
        • B cells and plasma cells show the same light chain expression, thus supporting a common clonal relationship
          • 2 distinct types:
            • Predominantly interfollicular distribution of plasma cells, these cases show BCL2 rearrangements and are considered to be FL with plasmacytic differentiation
            • Predominantly intrafollicular plasma cell distribution; these cases tend not to show BCL2 rearrangements and some evidence supports that these cases may not represent neoplastic FL with plasmacytic differentiation but show biologic overlap with marginal zone lymphomas
            • Can lack CD10 expression but retain expression of BCL6 and other germinal center markers such as LMO2, HGAL, etc.
      • FL with marginal zone differentiation involving MALT sites:
        • Marginal zone differentiation variable, characterized by a perifollicular band of monocytoid B cells rimming the neoplastic follicles
        • Monocytoid B cells are represented by:
          • Round to slightly irregular nuclei
          • Moderate amount of clear cytoplasm
          • Pale appearance at low magnification
        • Lymphoepithelial lesions are prominent
        • Plasmacytic differentiation with monotypic interfollicular plasma cells
      • FL negative for CD10, positive for MUM1 and with BCL6 abnormalities (Blood 2007;109:3076):
        • More frequent diffuse proliferation compared with conventional FL
        • More frequent grade 3B in follicular foci
        • More frequently shows component of diffuse large B cell lymphoma (DLBCL)
        • Usually lacks BCL2 rearrangement
        • Frequently expresses MUM1 and cytoplasmic light chain
        • More frequent BCL6 abnormalities (translocation, amplification)
      • EBV positive FL:
        • Histologically and immunophenotypically indistinguishable from nodal FL
      • Floral variant of FL:
        • Neoplastic follicles are irregular in shape
        • Expanded, prominent mantle zone lymphocytes surround follicles
        • Mantle zone lymphocytes penetrate neoplastic follicles
      Microscopic (histologic) images

      Contributed by Jennifer Chapman, M.D.

      Castleman-like changes

      Castleman-like changes

      Floral variant


      Floral variant

      Follicular lymphoma negative for CD10 and expressing MUM1

      Cytology description
      • Variable mixture of centrocytes and centroblasts
        • Centrocytes
          • Small to large
          • Angulated nuclei
          • Dense chromatin
          • Scant cytoplasm
        • Centroblasts
          • Large cells with oval nuclei
          • Vesicular chromatin
          • 1 - 3 nucleoli
          • Moderate cytoplasm
          • > 3 times the size of lymphocytes
          • Differentiate from follicular dendritic cells
            • Large round nuclei
            • Dispersed, nearly clear chromatin
            • Single eosinophilic nucleolus
            • Indistinct cytoplasm
      • Absent / rare tingible body macrophages
      • Reference: Jaffe: Hematopathology, 2nd Edition, 2016
      Positive stains
      • FL with Castleman-like changes:
      • FL with plasmacytic differentiation and IgG4 positive plasma cells:
        • Immunophenotype similar to that of nodal FL
        • Plasmacytic differentiation: cytoplasmic light chain restriction among interfollicular neoplastic plasma cells
        • IgG4 positive plasma cells may be increased: > 100/high power field and IgG4:IgG ratio > 40%
      • FL with marginal zone differentiation involving MALT sites:
      • FL negative for CD10 and expressing MUM1 (Blood 2007;109:3076):
        • B cell markers
        • MUM1 / IRF4
        • BCL6 (54% of cases)
        • CD30 (variable)
        • BCL2 (59% of cases)
          • Less frequent BCL2 protein expression than conventional FL (~94% of cases)
        • CD21, CD23 and CD35 highlight the follicular dendritic cell meshwork in all CD10- MUM1+ FL in follicular foci
      • EBV positive FL (Am J Hematol 2019;94:E62):
        • Immunophenotype similar to the conventional type of FL
        • CD30 (subset)
        • LMP1 (75 - 90% of cases)
        • EBER (~100% of cases)
        • Most common latency pattern of EBV infection: type II (~92% of cases), followed by type I latency (~8% of cases)
      • Floral variant of FL:
      Negative stains
      Molecular / cytogenetics description
      • FL with Castleman-like changes:
      • FL with plasmacytic differentiation and IgG4 positive plasma cells (Mod Pathol 2010;23:71):
        • BCL2 rearrangements
          • Present in monotypic plasma cells with an interfollicular distribution
          • Absent in monotypic plasma cells with prominent intrafollicular distribution
      • FL with marginal zone differentiation involving MALT sites:
      • FL negative for CD10 and expressing MUM1 (Blood 2007;109:3076):
        • Frequently lacks IGH-BCL2 translocation
        • BCL6 gene abnormalities (translocation or amplification) in the majority of cases
        • BCL2 gene amplification (in ~50% of cases)
      • EBV positive FL:
      • Floral variant of FL:
        • t(14;18)(q32;q21) translocation
      Sample pathology report
      • Lymph node, cervical, excision:
        • Follicular lymphoma, low grade, with Castleman-like changes (see comment)
        • Comment: Sections of the lymph node show a follicular lymphocytic proliferation, with a dense, atypical lymphocytic infiltrate. In most lymph node areas, sections reveal the presence of lymphoid follicles containing predominantly small, slightly angulated lymphocytes having irregular nuclei. In many of the follicles, the central areas of the follicle are nearly fully occupied by histiocytic / dendritic cells, with surrounding mantle zones prominent, giving an appearance of atretic follicles and imparting a distinctly Castleman-like appearance. The interfollicular regions of the lymph node show scattered plasma cells, in which a variable degree of plasma cell enlargement is present. Eosinophilic staining collagenous material is present around blood vessels and extensively in the lymph node capsule.
        • Immunohistochemical studies show positive staining for CD20, PAX5, BCL2 and BCL6 and are negative for CD3, CD5, cyclin D1 and HHV8. Cyclin D1 highlights endothelial cells, as well as histiocytic / dendritic cells. CD35 highlights numerous follicular dendritic cell meshworks in the areas of follicle formation and highlights the sclerotic centers. Few CD138 positive plasma cells are identified, which are polytypic by kappa and lambda immunohistochemical and in situ hybridization study. Congo red stain is negative for birefringent material.
        • By in situ hybridization analysis for Epstein-Barr virus encoded RNA (EBER), the neoplastic cells are negative for EBER.
        • Fluorescence in situ hybridization (FISH) reports IGH-BCL2 fusion (25% nuclei), BCL6 rearrangement (30% nuclei) and no evidence of MYC rearrangement.
      Differential diagnosis
      • FL with Castleman-like changes (Hum Pathol 2017;68:136, Mod Pathol 2018;31:429):
        • Overlapping features with Castleman disease; however, features favoring FL include:
          • Increase in the number of follicles
          • Atypical follicles containing increased centrocytes
          • Presence of BCL6 positive lymphoid cells in atretic follicles
          • BCL2 positive, even if weak
          • Presence of t(14;18)(q32;q21)
      • FL with plasmacytic differentiation and IgG4 positive plasma cells:
        • Overlapping features with marginal zone lymphoma
        • Immunophenotype in neoplastic cells is similar to that of nodal FL
        • Distinction of these entities typically requires extensive immunophenotyping and FISH studies to assess for IGH-BCL2 translocation
      • FL with marginal zone differentiation involving MALT sites:
      • Floral variant of FL:
      Board review style question #1
      Which of the following follicular lymphomas or variants more commonly has BCL2 rearrangement?

      1. Follicular lymphoma with Castleman-like changes
      2. Large B cell lymphoma with IRF4 rearrangement
      3. Pediatric follicular lymphoma
      4. Testicular follicular lymphoma
      Board review style answer #1
      A. Follicular lymphoma with Castleman-like changes

      Comment Here

      Reference: Unusual morphologic patterns of follicular lymphoma
      Board review style question #2


      A 50 year old man presented with abdominal pain for 1 month. Physical examination and laboratory tests were within normal limits. An imaging study showed multiple masses, the largest one measuring 10.5 x 5.0 cm. The patient underwent an excisional biopsy of one of the lymph nodes. The cut section of the lymph node showed lymphoid nodules, some surrounding atrophic follicles. Flow cytometry showed an aberrant CD10+ B cell population, positive for cytoplasmic CD22 and CD79a and negative for surface or cytoplasmic immunoglobulin, CD5 and CD23. Fluorescence in situ hybridization (FISH) analysis showed an IGH-BCL2 fusion gene consistent with t(14;18)(q32;q21). Which of the following options is the best correct option in this case?

      1. Cyclin D1 is positive in lymphocytes in the lymphoid nodule
      2. IgD is is positive in lymphocytes in the lymphoid nodule
      3. Ki67 proliferation index is expected to be high in lymphoid nodules
      4. Lymphoid cells in lymphoid nodules are expected to be positive for CD10, CD79a, BCL2 and BCL6
      Board review style answer #2
      D. Lymphoid cells in lymphoid nodules are expected to be positive for CD10, CD79a, BCL2 and BCL6

      Comment Here

      Reference: Unusual morphologic patterns of follicular lymphoma

      WHO 2017 classification-B cell
      Definition / general
      • Note: this classification was replaced in 2022 and this topic will eventually be deleted
      • B cell lymphoma is the most prevalent type of lymphoma and includes the largest number of distinct diagnostic entities among lymphomas
      • 2016 revised World Health Organization (WHO) classification of lymphoid neoplasms incorporates new clinical and genetic data gleaned from modern technologies, e.g. next generation sequencing and FISH, since the previous classification system in 2008 (Expert Rev Hematol 2017;10:405)
      • Includes introduction of new lymphoma entities and updated nomenclature for certain entities to better reflect the clinical features
      Highlights of 2016 updated WHO classification
      • More refined subtyping of lymphoma entities based on international consensus enables more targeted therapeutic strategies (Curr Treat Options Oncol 2017;18:45)
      • Current WHO B cell lymphoma classification features the following major updates (Int J Lab Hematol 2017;39:14):
        • Significantly more detailed sorting of B lymphoblastic leukemia / lymphoma based on association with various recurrent genetic abnormalities
        • Lower limit of chronic lymphocytic leukemia cells in blood has been set at 5 × 10⁹/L, below which it may represent either peripheralized small lymphocytic lymphoma or the newly described monoclonal B cell lymphocytosis
        • Newly defined high grade B cell lymphomas, such as the so called double hit or triple hit lymphoma that includes MYC gene rearrangement detectable by FISH
          • Replaces the 2008 diagnostic entity of B cell lymphoma, unclassifiable, with features intermediate between DLBCL and Burkitt lymphoma
        • More refined subtyping of diffuse large B cell lymphomas (DLBCL)
          • Requires determination of DLBCL cell of origin (germinal center B cell type versus activated B cell type); immunohistochemical algorithm acceptable
          • New large B cell lymphoma entities, e.g. fibrin associated diffuse large B cell lymphoma and large B cell lymphoma with IRF4 rearrangement (provisional entity)
        • Renaming of in situ lesions from lymphoma to neoplasia to reflect the low clinical risk observed
          • From in situ follicular lymphoma to in situ follicular neoplasia
          • From in situ mantle cell lymphoma to in situ mantle cell neoplasia
        • Newly designated pediatric lymphoma entity or subtype, e.g. pediatric type follicular lymphoma and pediatric nodal marginal zone lymphoma (provisional subtype)
        • More detailed subtyping of plasma cell neoplasms, including those associated with paraneoplastic syndromes and 3 new myeloma variants
      • B cell neoplasms listed below are based on the WHO's 2016 updated classification of hematopoietic neoplasms (Blood 2016;127:2375)
        • New entities are in italics and provisional entities are marked with an asterisk
      Classification of B cell neoplasms (WHO 2016)
      Precursor B cell neoplasms:
      Mature (peripheral) B cell neoplasms:
    • Extranodal marginal zone B cell lymphoma of mucosa associated lymphoid tissue (MALT lymphoma)
    • Nodal marginal zone B cell lymphoma (MZL)
      • Pediatric type nodal MZL*
    • Follicular lymphoma
    • Pediatric type follicular lymphoma
    • Large B cell lymphoma with IRF4 rearrangement*
    • Primary cutaneous follicle center lymphoma
    • Mantle cell lymphoma
    • Diffuse large B cell lymphoma (DLBCL), NOS
      • Germinal center B cell type
      • Activated B cell type
    • T cell / histiocyte rich large B cell lymphoma
    • Primary DLBCL of the central nervous system
    • EBV positive DLBCL, NOS
    • EBV positive mucocutaneous ulcer*
    • DLBCL associated with chronic inflammation
    • Lymphomatoid granulomatosis
    • Primary mediastinal (thymic) large B cell lymphoma
    • Intravascular large B cell lymphoma
    • Primary cutaneous DLBCL, leg type
    • ALK+ large B cell lymphoma
    • Plasmablastic lymphoma
    • Primary effusion lymphoma
    • HHV8 associated lymphoproliferative disorders
    • Burkitt lymphoma
    • Burkitt-like lymphoma with 11q aberration*
    • B cell lymphoma, unclassifiable, with features intermediate between diffuse large B cell lymphoma and Burkitt lymphoma
    • High grade B cell lymphoma
    • B cell lymphoma, unclassifiable, with features intermediate between diffuse large B cell lymphoma and classical Hodgkin lymphoma
      Hodgkin lymphomas:
      Notes:
      • New disease entities or subtypes of an entity are in italics
      • Asterisk (*) denotes provisional entities or subtypes
    • Prior classification systems
      • Rappaport classification:
        • 1956, revised 1966
        • Developed before lymphocytes were classified as B and T cells
        • Includes well differentiated lymphocytic lymphoma, poorly differentiated lymphocytic lymphoma and histiocytic lymphoma (Cancer Res 1966;26:1082)
      • Lukes-Collins classification:
        • 1974; classified non-Hodgkin lymphomas as B cell, T cell, histiocytic and unclassifiable types (Cancer 1974;34:1488)
      • Working formulation:
        • 1982; classified as low, intermediate or high grade; nodular versus diffuse; small, large or mixed tumor cell size (Cancer 1982;49:2112)
      • Kiel classification:
      • REAL (Revised European American Lymphoma):
        • Integrated clinical, morphologic, immunohistochemical and molecular characteristics
        • Included non-Hodgkin lymphoma, lymphocytic leukemias, plasma cell neoplasms
        • Excludes histiocytic neoplasms
        • Tumors not classified as low grade / high grade since one entity could have both types (Blood 1994;84:1361)
      • World Health Organization (WHO) 2008 classification:
        • Based on worldwide consensus on lymphoma diagnosis and was focused on the recognition of distinct diseases, using a multidisciplinary approach
        • Refined the definitions of well recognized diseases
        • Identified new entities and variants
        • Recognized provisional entities due to insufficient evidence to recognize them as distinct diseases at the time
        • Acknowledged the presence of borderline B cell lymphoma lesions, e.g. those with features intermediate between DLBCL and Burkitt and those with features intermediate between DLBCL and classic Hodgkin (Blood 2011;117:5019)
      Board review style question #1
      Which of the following is true regarding the 2016 WHO classification of B cell lymphoid neoplasms?

      1. Provisional lymphoma entities, such as large B cell lymphoma with IRF4 rearrangement, are those from the previous classification that are either transitioning to be merged with another entity or expected to be phased out
      2. Determination of cell of origin for diffuse large B cell lymphoma is required and can be performed by immunohistochemistry
      3. B lymphoblastic leukemia / lymphoma is considered to be a single entity in order to facilitate and simplify clinical trial enrollment of patients
      4. By definition, monoclonal B cell lymphocytosis has a lower limit of 5 × 10⁹/L cells in the blood
      5. High grade double hit B cell lymphoma characteristically includes TP53 and BCR-ABL gene rearrangements
      Board review style answer #1
      B. Determination of cell of origin for diffuse large B cell lymphoma is required and can be performed by immunohistochemistry. This lymphoma entity is divided into germinal center B cell type and activated B cell type.

      Comment Here

      Reference: WHO classification - B cell

      WHO 2022 & ICC-B cell
      Definition / general
      • 2 new classification systems for B cell lymphoid neoplasms have emerged in 2022:
      • Evolution from the 2016 WHO revised 4th edition (WHO HAEM4R) classification system to the new version reflects advancements in genomic profiling and evidence based clinical data
      • Updates include newly defined subtypes, more encompassing umbrella terms, deletion of old entities and modified nomenclature
      • Putative new entities with limited data are designated as provisional in WHO HAEM4 and ICC but no provisional designation exists in WHO HAEM5 (see tables 1 and 2)
      Major updates
      • Certain newly added categories are specific to only one classification system (table 1), e.g.:
        • Newly included nonneoplastic tumor-like lesions in WHO HAEM5 include IgG4 related disease and 2 additional types of Castleman disease (unicentric and idiopathic multicentric)
        • EBV+ polymorphic B cell lymphoproliferative disorder, NOS is a new entity under ICC to describe EBV+ B cell lesions with altered nodal architecture that are not diagnostic of lymphoma
        • Multiple myeloma (MM) is categorized by ICC into MM with recurrent genetic abnormalities (based on interphase FISH) and MM, NOS
      • B prolymphocytic leukemia (B PLL) is no longer defined as an entity in WHO HAEM5 based on the notion that it represents prolymphocytoid transformation of various small B cell lymphoma entities (Ann Diagn Pathol 2021;54:151790)
        • Under WHO HAEM5, previous cases of B PLL are categorized as one of the following entities:
          • Mantle cell lymphoma with IGH::CCND1
          • CD5 positive nonmantle B cell neoplasm with elevated prolymphocytes in peripheral blood or bone marrow
          • Splenic B cell lymphoma / leukemia with prominent nucleoli
        • ICC continues to recognize B PLL as a neoplasm after excluding transformation from other B cell entities
      • Splenic B cell lymphoma / leukemia with prominent nucleoli (SBLPN):
        • Corresponds to hairy cell leukemia (HCL) variant (rare, provisional entity) in WHO HAEM4R and ICC
        • Newly created terminology under WHO HAEM5 reflects its biologic distinction from typical hairy cell leukemia (Expert Rev Hematol 2021;14:355)
        • Includes previous cases of CD5 negative B PLL under WHO HAEM4R
      • Marginal zone lymphomas:
        • WHO HAEM5 and ICC both recognize pediatric nodal marginal zone lymphoma (pNMZL) as a separate entity from nodal marginal zone lymphoma (NMZL)
        • WHO HAEM5 and ICC both recognize primary cutaneous marginal zone lymphoma / lymphoproliferative disorder as distinct from extranodal marginal zone lymphoma of MALT of other sites
      • Follicular lymphoma:
        • WHO HAEM5 recognizes 3 subsets of follicular lymphoma based on BCL2 gene rearrangement:
          • Classic follicular lymphoma (cFL) with BCL2 rearrangement
          • Follicular large B cell lymphoma (FLBL) without BCL2 rearrangement
          • Follicular lymphoma with uncommon features (uFL) without BCL2 rearrangement
          • Histologic grading of cFL is no longer mandatory
        • ICC proposes a new provisional entity based on absence of BCL2 rearrangement
          • BCL2 rearrangement negative, CD23 positive follicle center lymphoma (provisional): often diffuse
          • Continues with grading of follicular lymphoma with emphasis on 3A and 3B
        • ICC recognizes testicular follicular lymphoma as a new entity that is distinct from pediatric type follicular lymphoma
      • New large B cell lymphoma entities:
        • Fibrin associated large B cell lymphoma (both WHO HAEM5 and ICC)
        • Fluid overload associated large B cell lymphoma (WHO HAEM5) overlaps with HHV8 and EBV negative primary effusion based lymphoma (provisional entity in ICC); distinct from primary effusion lymphoma that is HHV8 associated
      • Primary large B cell lymphomas of immune privileged site:
        • New umbrella term in WHO HAEM5 encompassing diffuse large B cell lymphoma (DLBCL) of the following extranodal sites: CNS, vitreoretina compartment and testis
        • ICC maintains that each should remain as distinct entities to facilitate further research
      • Nodular lymphocyte predominant Hodgkin lymphoma (NLPHL):
        • Known as NLPHL in WHO HAEM4R, it has been renamed under ICC as nodular lymphocyte predominant B cell lymphoma (NLPBL), in recognition of its overlap with T cell / histiocyte rich large B cell lymphoma (Pathology 2020;52:142)
        • Under WHO HAEM5, the terminology remains unchanged and continues to list NLPHL under the family of Hodgkin lymphoma to avoid potential disruption of clinical trials
        • However, WHO HAEM5 also recognizes it may be more accurately called nodular lymphocyte predominant B cell lymphoma (based on the neoplastic cells having a functional B cell program) and also accepts this term in preparation for future adoption
        • Importance of different morphologic growth patterns (variant histology) in relation to clinical behavior is recognized by both ICC and WHO HAEM5 (Am J Surg Pathol 2003;27:1346)
      • High grade B cell lymphomas (HGBCL):
        • Previously a single entity, HGBCL with MYC and BCL2 or BCL6 rearrangement (double / triple hit lymphomas) has been categorized into 2 entities under WHO HAEM5:
          • B cell lymphomas with dual MYC and BCL2 rearrangements are considered a homogenous entity and have been renamed as diffuse large B cell lymphoma / high grade B cell lymphoma with MYC and BCL2 rearrangements (DLBCL / HGBCL MYC / BCL2)
          • B cell lymphomas with MYC and BCL6 rearrangements are biologically distinct from those with MYC / BCL2 rearrangements and are designated as DLBCL, NOS or HGBCL, NOS based on cytomorphology
        • ICC recognizes 2 subtypes of HGBCL double hit:
          • HGBCL with MYC and BCL2 rearrangements (with or without BCL6), an aggressive lymphoma of germinal center B cell origin
          • HGBCL with MYC and BCL6 rearrangements, a provisional entity that has shown heterogeneous biology based on relatively small sample sizes (J Hematol Oncol 2022;15:26)
      • Burkitt lymphoma:
        • WHO HAEM5 recommends subtyping Burkitt lymphomas based on EBV status rather than epidemiologic context
        • Under ICC, previous rare cases of TdT positive Burkitt lymphomas with IG::MYC translocation should be designated as B lymphoblastic leukemia / lymphoma with MYC rearrangement
      • Burkitt-like lymphoma with 11q aberration:
        • Provisional under WHO HAEM4R; resembling Burkitt lymphoma but lacking MYC rearrangement
        • Under WHO HAEM5: renamed as high grade B cell lymphoma with 11q aberration
        • Under ICC: renamed as large B cell lymphoma with 11q aberration, in recognition of its genetic profile being closer to DLBCL than Burkitt and occasional cases with predominantly large cell morphology
      • Lymphoplasmacytic lymphoma (LPL):
        • MYD88 p.L265P mutation (> 90%)
        • CXCR4 mutations (30 - 40%), associated with drug resistance and increased bone marrow disease (J Clin Oncol 2020;38:1198)
        • WHO HAEM5 recognizes 2 subsets of LPL:
          • IgM LPL / Waldenström macroglobulinemia
          • Non-Waldenström macroglobulinemia (includes IgG / IgA cases, nonsecretory LPL and IgM LPL without bone marrow involvement)
      • IgM monoclonal gammopathy of undetermined significance (MGUS):
        • ICC recognizes 2 distinct subtypes of IgM MGUS:
          • IgM MGUS of plasma cell type (precursor of MM)
          • IgM MGUS, NOS (may show MYD88 mutation or monoclonal B cells)
      • Monoclonal gammopathy of renal significance (MGRS):
        • WHO HAEM5 defines MGRS as a new entity
        • ICC considers MGRS as a clinical description of the underlying MGUS rather than a distinct entity
      • Cold agglutinin disease or primary cold agglutinin disease:
        • A new diagnostic category in both WHO HAEM5 and ICC
        • Clonal B cell proliferation usually of IgM isotype, distinct from IgM MGUS and LGL and lacking MYD88 L265P mutation (Clin Adv Hematol Oncol 2020;18:35)
      • Plasma cell neoplasms with associated paraneoplastic syndrome:
        • WHO HAEM5 recognizes a new syndrome: adenopathy and extensive skin patch overlying a plasmacytoma (AESOP) (Blood Cancer J 2022;12:58)
      • Lymphoid proliferations / lymphomas with immune deficiency or dysregulation:
        • WHO HAEM4R: categorization based on underlying etiology (primary immune disorders, HIV, posttransplant status and iatrogenic agents / immunosuppressants)
        • WHO HAEM5: new integrated approaches with 3 components
          • Histologic diagnosis (hyperplasia, polymorphic lymphoproliferation, mucocutaneous ulcer or lymphoma)
          • Oncologic viral association (EBV or KSHV / HHV8)
          • Immune deficiency / dysregulation (HIV, inborn error of immunity, posttransplant, autoimmune, iatrogenic or immune senescence)
        • ICC: retains posttransplant lymphoproliferative disorders (PTLD) as a subgroup of iatrogenic lymphoid lesions with distinct clinical management
      • B lymphoblastic leukemia / lymphoma (B ALL) (see table 2):
        • WHO HAEM5 uses shorter nomenclature consisting of gene names of molecular drivers but excluding the cytogenetic changes that appeared in the WHO HAEM4R nomenclature (e.g., B ALL with BCR::ABL1 fusion in WHO HAEM5 is equivalent to B ALL with t(9;22)(q34;q11.2); BCR-ABL1 in WHO HAEM4R)
        • Under WHO HAEM5, B ALL with BCR::ABL1-like features is no longer considered a provisional entity as in WHO HAEM4R
        • 2 new entities are recognized in WHO HAEM5:
          • B ALL with ETV6::RUNX1-like features (distinct from B ALL with ETV6::RUNX1 fusion)
          • B ALL with TCF3::HLF fusion (associated with aggressive behavior)
        • No major reclassification of most B ALL entities in WHO HAEM5
      WHO (2016), WHO (2022) and ICC (2022)

      Table 1: Mature B cell entities - comparison of 3 classification systems
      WHO HAEM4R WHO HAEM5 ICC
      Tumor-like lesions with B cell proliferation
      Not included Reactive B cell rich lymphoid proliferations that can mimic lymphoma
      Not included IgG4 related disease
      Not included Unicentric Castleman disease
      Not included Idiopathic multicentric Castleman disease
      Multicentric Castleman disease KSHV / HHV8 associated multicentric Castleman disease Multicentric Castleman disease
      Not included EBV positive polymorphic B cell lymphoproliferative disorder, NOS
      Neoplastic and preneoplastic small lymphocytic proliferation
      Chronic lymphocytic leukemia / small lymphocytic lymphoma (CLL / SLL) Chronic lymphocytic leukemia / small lymphocytic lymphoma (CLL / SLL) Chronic lymphocytic leukemia / small lymphocytic lymphoma (CLL / SLL)
      Monoclonal B cell lymphocytosis
      • CLL type
      • Atypical CLL type
      • Non-CLL type
      Monoclonal B cell lymphocytosis
      • Low count or clonal B cell expansion
      • CLL / SLL type
      • Non-CLL / SLL type
      Monoclonal B cell lymphocytosis
      • CLL type
      • Non-CLL type
      B prolymphocytic leukemia (B PLL) Not a distinct entity but heterogeneous B prolymphocytic leukemia (B PLL)
      Lymphoplasmacytic lymphoma
      • Waldenström macroglobulinemia
      Lymphoplasmacytic lymphoma
      • Waldenström macroglobulinemia
      • Non-Waldenström macroglobulinemia
      Lymphoplasmacytic lymphoma
      • Waldenström macroglobulinemia
      Marginal zone lymphoma
      Nodal marginal zone lymphoma Nodal marginal zone lymphoma Nodal marginal zone lymphoma
      Pediatric nodal marginal zone lymphoma* Pediatric nodal marginal zone lymphoma Pediatric nodal marginal zone lymphoma*
      Extranodal marginal zone lymphoma of mucosa associated lymphoid tissue (MALT) Extranodal marginal zone lymphoma of mucosa associated lymphoid tissue (MALT) Extranodal marginal zone lymphoma of mucosa associated lymphoid tissue (MALT)
      Not included; part of extranodal marginal zone lymphoma of MALT Primary cutaneous marginal zone lymphoma Primary cutaneous marginal zone lymphoproliferative disorder
      Splenic B cell lymphoma / leukemia
      Splenic marginal zone lymphoma Splenic marginal zone lymphoma Splenic marginal zone lymphoma
      Hairy cell leukemia Hairy cell leukemia Hairy cell leukemia
      Hairy cell leukemia variant* Splenic B cell lymphoma / leukemia with prominent nucleoli (distinct from HCL and includes all previous CD5 negative B PLL) Hairy cell leukemia variant*
      Splenic diffuse red pulp small B cell lymphoma* Splenic diffuse red pulp small B cell lymphoma Splenic diffuse red pulp small B cell lymphoma*
      Follicular lymphoma
      In situ follicular neoplasia In situ follicular B cell neoplasm In situ follicular neoplasia
      Follicular lymphoma (FL) Follicular lymphoma (grading of cFL not mandatory)
      • Classic follicular lymphoma (cFL)
      • Follicular large B cell lymphoma (FLBL)
      • Follicular lymphoma with uncommon features (uFL)
      Follicular lymphoma (continue grading FL with emphasis on 3A and 3B)
      • Follicular lymphoma
      • BCL2 rearrangement negative, CD23 positive follicle center lymphoma*
      Pediatric type follicular lymphoma Pediatric type follicular lymphoma Pediatric type follicular lymphoma
      Not included Testicular follicular lymphoma
      Duodenal type follicular lymphoma Duodenal type follicular lymphoma Duodenal type follicular lymphoma
      Primary cutaneous follicle center lymphoma Primary cutaneous follicle center lymphoma Primary cutaneous follicle center lymphoma
      Mantle cell lymphoma
      In situ mantle cell neoplasia In situ mantle cell neoplasia In situ mantle cell neoplasia
      Mantle cell lymphoma Mantle cell lymphoma Mantle cell lymphoma
      Leukemic nonnodal mantle cell lymphoma Leukemic nonnodal mantle cell lymphoma Leukemic nonnodal mantle cell lymphoma
      Aggressive lymphomas transformed from low grade B cell lymphomas
      Not included Transformations of indolent B cell lymphomas
      Large B cell lymphoma
      Diffuse large B cell lymphoma (DLBCL), NOS Diffuse large B cell lymphoma, NOS Diffuse large B cell lymphoma, NOS
      EBV positive mucocutaneous ulcer* EBV positive mucocutaneous ulcer EBV positive mucocutaneous ulcer
      EBV positive diffuse large B cell lymphoma, NOS EBV positive diffuse large B cell lymphoma EBV positive diffuse large B cell lymphoma, NOS
      Diffuse large B cell lymphoma associated with chronic inflammation Diffuse large B cell lymphoma associated with chronic inflammation Diffuse large B cell lymphoma associated with chronic inflammation
      Primary large B cell lymphoma of the central nervous system Primary large B cell lymphoma of immune privileged sites (new umbrella term for DLBCL arising in the CNS, vitreoretina and testis) Primary diffuse large B cell lymphoma of central nervous system
      Not included Primary diffuse large B cell lymphoma of testis
      Primary cutaneous diffuse large B cell lymphoma, leg type Primary cutaneous diffuse large B cell lymphoma, leg type Primary cutaneous diffuse large B cell lymphoma, leg type
      Intravascular large B cell lymphoma Intravascular large B cell lymphoma Intravascular large B cell lymphoma
      ALK positive large B cell lymphoma ALK positive large B cell lymphoma ALK positive large B cell lymphoma
      Plasmablastic lymphoma Plasmablastic lymphoma Plasmablastic lymphoma
      Large B cell lymphoma with IRF4 rearrangement Large B cell lymphoma with IRF4 rearrangement Large B cell lymphoma with IRF4 rearrangement
      Primary mediastinal large B cell lymphoma Primary mediastinal large B cell lymphoma Primary mediastinal large B cell lymphoma
      B cell lymphoma, unclassified with features intermediate between DLBCL and classic Hodgkin lymphoma Mediastinal gray zone lymphoma (cases without mediastinal involvement are classified as DLBCL, NOS) Mediastinal gray zone lymphoma
      Not included Fibrin associated large B cell lymphoma Fibrin associated large B cell lymphoma
      Not included Fluid overload associated large B cell lymphoma (previously included in primary effusion lymphoma) HHV8 and EBV8 negative primary effusion based lymphoma*
      T cell / histiocyte rich large B cell lymphoma T cell / histiocyte rich large B cell lymphoma T cell / histiocyte rich large B cell lymphoma
      See Hodgkin lymphomas See Hodgkin lymphomas Nodular lymphocyte predominant B cell lymphoma (renamed from nodular lymphocyte predominant Hodgkin lymphoma)
      Lymphomatoid granulomatosis Lymphomatoid granulomatosis Lymphomatoid granulomatosis
      KSHV / HHV8 associated B cell lymphoid proliferation / lymphoma
      Primary effusion lymphoma Primary effusion lymphoma Primary effusion lymphoma
      HHV8 positive diffuse large B cell lymphoma, NOS HHV8 positive diffuse large B cell lymphoma HHV8 positive diffuse large B cell lymphoma, NOS
      HHV8 positive germinotropic lymphoproliferative disorder KSHV / HHV8 positive germinotropic lymphoproliferative disorder HHV8 positive germinotropic lymphoproliferative disorder
      High grade B cell lymphomas
      High grade B cell lymphoma, NOS High grade B cell lymphoma, NOS High grade B cell lymphoma, NOS
      High grade B cell lymphoma with MYC and BCL2 or BCL6 rearrangements Diffuse large B cell lymphoma / high grade B cell lymphoma with MYC and BCL2 rearrangements (previous high grade B cell lymphoma with MYC and BCL6 rearrangements is designated as DLBCL, NOS) High grade B cell lymphoma with MYC and BCL2 rearrangements
      High grade B cell lymphoma with MYC and BCL6 rearrangements*
      Burkitt lymphoma Burkitt lymphoma (EBV status supersedes epidemiologic subtyping) Burkitt lymphoma
      Burkitt-like lymphoma with 11q aberration* High grade B cell lymphoma with 11q aberration Large B cell lymphoma with 11q aberration*
      Hodgkin lymphomas
      Classic Hodgkin lymphoma
      • Nodular sclerosis
      • Lymphocyte rich
      • Mixed cellularity
      • Lymphocyte depleted
      Classic Hodgkin lymphoma
      • Nodular sclerosis
      • Lymphocyte rich
      • Mixed cellularity
      • Lymphocyte depleted
      Classic Hodgkin lymphoma
      • Nodular sclerosis
      • Lymphocyte rich
      • Mixed cellularity
      • Lymphocyte depleted
      Nodular lymphocyte predominant Hodgkin lymphoma Nodular lymphocyte predominant Hodgkin lymphoma Renamed as nodular lymphocyte predominant B cell lymphoma; categorized as non-Hodgkin lymphoma
      Plasma cell neoplasms and entities with paraproteins
      Solitary plasmacytoma of bone Solitary plasmacytoma of bone Solitary plasmacytoma of bone
      Extraosseous plasmacytoma Extraosseous plasmacytoma Extraosseous plasmacytoma
      Plasma cell myeloma Plasma cell myeloma Multiple myeloma (MM), NOS
      MM with recurrent genetic abnormality
      • MM with CCND family translocation
      • MM with MAF family translocation
      • MM with NSD2 family translocation
      • MM with hyperdiploidy
      Plasma cell neoplasm with associated paraneoplastic syndrome
      • POEMS
      • TEMPI
      Plasma cell neoplasm with associated paraneoplastic syndrome
      • POEMS
      • TEMPI
      • AESOP (new syndrome)
      IgM monoclonal gammopathy of undetermined significance IgM monoclonal gammopathy of undetermined significance IgM monoclonal gammopathy of undetermined significance
      • IgM MGUS, plasma cell type
      • IgM MGUS, NOS
      Non-IgM monoclonal gammopathy of undetermined significance Non-IgM monoclonal gammopathy of undetermined significance Non-IgM monoclonal gammopathy of undetermined significance
      Not included Monoclonal gammopathy of renal significance Not a separate entity; clinical descriptor of the underlying diagnosis (e.g., MGUS)
      Not included Cold agglutinin disease Primary cold agglutinin disease
      Primary amyloidosis Immunoglobulin related (AL) amyloidosis Immunoglobulin light chain (AL) amyloidosis
      Localized AL amyloidosis
      Light chain and heavy chain deposition disease Monoclonal immunoglobulin deposition disease (renamed) Light chain and heavy chain deposition
      Mu heavy chain disease Mu heavy chain disease Mu heavy chain disease
      Gamma heavy chain disease Gamma heavy chain disease Gamma heavy chain disease
      Alpha heavy chain disease Alpha heavy chain disease Alpha heavy chain disease
      Lymphoid proliferations / lymphomas with immune deficiency or dysregulation
      Nondestructive PTLD Hyperplasias arising in immune deficiency / dysregulation Plasmacytic hyperplasia PTLD
      Florid follicular hyperplasia PTLD
      Infectious mononucleosis PTLD
      Polymorphic PTLD Polymorphic lymphoproliferative disorders arising in immune deficiency / dysregulation (new term that includes various etiologies) Polymorphic PTLD
      Other iatrogenic immunodeficiency associated lymphoproliferative disorders Other iatrogenic immunodeficiency associated lymphoproliferative disorders
      Monomorphic PTLD Lymphomas arising in immune deficiency / dysregulation (new umbrella term that includes monomorphic PTLD, lymphomas associated with HIV infection, etc.) Monomorphic PTLD
      Classic Hodgkin lymphoma PTLD Classic Hodgkin lymphoma PTLD
      Lymphomas associated with HIV infection
      Lymphoproliferative diseases associated with primary immune disorders Inborn error of immunity associated lymphoid proliferations and lymphomas
      Note: asterisk (*) denotes a provisional entity


      Table 2: Precursor B cell neoplasms - comparison of WHO 4th and 5th edition classification systems
      WHO HAEM4R WHO HAEM5
      B lymphoblastic leukemia / lymphoma, NOS B lymphoblastic leukemia / lymphoma, NOS
      B lymphoblastic leukemia / lymphoma with hyperdiploidy B lymphoblastic leukemia / lymphoma with high hyperdiploidy
      B lymphoblastic leukemia / lymphoma with hypodiploidy B lymphoblastic leukemia / lymphoma with hypodiploidy
      B lymphoblastic leukemia / lymphoma with iAMP21 B lymphoblastic leukemia / lymphoma with iAMP21
      B lymphoblastic leukemia / lymphoma with t(9;22)(q34;q11.2); BCR-ABL1 B lymphoblastic leukemia / lymphoma with BCR::ABL1 fusion
      B lymphoblastic leukemia / lymphoma, BCR-ABL1-like* B lymphoblastic leukemia / lymphoma with BCR::ABL1-like features
      B lymphoblastic leukemia / lymphoma with t(12;21)(p13.2;q22.1); ETV6-RUNX1 B lymphoblastic leukemia / lymphoma with ETV6::RUNX1
      Not included B lymphoblastic leukemia / lymphoma with ETV6::RUNX1-like feature
      B lymphoblastic leukemia / lymphoma with t(1;19)(q23;p13.3); TCF3-PBX1 B lymphoblastic leukemia / lymphoma with TCF3::PBX1 fusion
      Not included B lymphoblastic leukemia / lymphoma with TCF3::HLF fusion
      B lymphoblastic leukemia / lymphoma with t(v;11q23.3); KMT2A rearranged B lymphoblastic leukemia / lymphoma with KMT2A rearrangement
      B lymphoblastic leukemia / lymphoma with t(5;14)(q31.1;q32.1); IGH / IL3 B lymphoblastic leukemia / lymphoma with IGH::IL3 fusion
      B lymphoblastic leukemia / lymphoma with other defined genetic abnormalities B lymphoblastic leukemia / lymphoma with other defined genetic abnormalities
      Note: asterisk (*) denotes a provisional entity
      Microscopic (histologic) images

      Contributed by Patricia Tsang, M.D., M.B.A.

      Cold agglutinin disease (CAD)

      CAD lymphoid aggregate

      CAD (CD20)

      CAD (CD3)

      Board review style question #1

      Which of the following is a characteristic of splenic B cell lymphoma / leukemia with prominent nucleoli as defined in the WHO classification of hematolymphoid neoplasms, 5th edition (WHO HAEM5)?

      1. Classified as an immature B cell lymphoid neoplasm
      2. Considered as a subtype of diffuse large B cell lymphoma originating from the spleen
      3. Corresponds to hairy cell leukemia variant under the WHO classification, 4th edition
      4. Shares a similar genetic profile with hairy cell leukemia
      5. Tumor cells characteristically coexpress CD5
      Board review style answer #1
      C. Corresponds to hairy cell leukemia variant under the WHO classification, 4th edition. This entity is a mature B cell neoplasm that is biologically distinct from both hairy cell leukemia and diffuse large B cell lymphoma and typically lacks CD5 coexpression.

      Comment Here

      Reference: WHO HAEM5 and ICC-B cell
      Board review style question #2
      Which of the following is true regarding B cell or plasma cell neoplasms as defined by the International Consensus Classification (ICC) system?

      1. B cell prolymphocytic leukemia is the counterpart of Richter transformation of small lymphocytic lymphoma
      2. Lymphoplasmacytic lymphoma can manifest clinically as primary cold agglutinin disease
      3. Multiple myeloma with recurrent genetic abnormality is a newly added category
      4. Nodular lymphocytic predominant B cell lymphoma is renamed from nodular sclerosis Hodgkin lymphoma
      5. Testicular follicular lymphoma is a provisional subtype of pediatric type follicular lymphoma
      Board review style answer #2
      C. Multiple myeloma with recurrent genetic abnormality is a newly added category in the International Consensus Classification system.

      Comment Here

      Reference: WHO HAEM5 and ICC-B cell

      WHO 2022 & ICC-T / NK cell
      Definition / general
      • 2 new classification systems for T / NK cell lymphoid neoplasms emerged in 2022 and evolved from the previous WHO revised 4th edition of 2016 (WHO HAEM4R)
      • Putative new entities with limited data are designated as provisional in the WHO HAEM4R and ICC while no provisional designation exists in WHO HAEM5 (see table 1)
      • Updates include newly defined entities, more encompassing umbrella terms, deletion of old entities and modified nomenclature
      Major updates
      • Certain newly incorporated entities are unique to one or the other of the updated classifications (see table 1)
        • Tumor-like lesions with T cell predominance (Kikuchi-Fujimoto disease, indolent T lymphoblastic proliferation and autoimmune lymphoproliferative syndrome) are included only in WHO HAEM5
        • Primary cutaneous peripheral T cell lymphoma, NOS is a newly introduced term in WHO HAEM5 to describe rare cases of cutaneous T cell lymphoma that do not fit into existing entities
        • Type II refractory celiac disease is recognized as an entity in ICC as a precursor of enteropathy associated T cell lymphoma (EATL)
      • New entities have been added concurrently to WHO HAEM5 and ICC
        • Indolent NK cell lymphoproliferative disorder (LPD) of the gastrointestinal (GI) tract, previously thought to be reactive, is newly recognized as neoplastic by WHO HAEM5 and ICC
        • Previously included as a variant of peripheral T cell lymphoma, not otherwise specified (PTCL, NOS), EBV positive nodal T and NK cell lymphoma is a distinct entity in WHO HAEM5, that is equivalent to primary nodal EBV positive T / NK cell lymphoma in ICC (provisional entity)
      • Breast implant associated anaplastic large cell lymphoma (ALCL): upgraded from provisional status (WHO HAEM4R) to a definite entity (J Clin Oncol 2020;38:1102)
      • Certain T / NK cell neoplastic entities have new nomenclature
        • Indolent T cell LPD of the GI tract has been renamed as indolent T cell lymphoma of the GI tract (WHO HAEM5) or indolent clonal T cell LPD of the GI tract (ICC) due to its potential for disease dissemination, associated morbidity and monoclonal nature
        • Category of nodal lymphomas of T follicular helper (TFH) origin is now unified into a single entity comprising 3 subtypes with new nomenclature
          • Angioimmunoblastic T cell lymphoma (WHO HAEM4R) is now nodal TFH cell lymphoma, angioimmunoblastic type (WHO HAEM5) or follicular helper T cell lymphoma, angioimmunoblastic type (ICC)
          • Follicular T cell lymphoma (provisional, WHO HAEM4R) is now nodal TFH cell lymphoma, follicular type (WHO HAEM5) or follicular helper T cell lymphoma, follicular type (ICC)
          • Nodal peripheral T cell lymphoma with TFH phenotype (provisional, WHO HAEM4R) has been renamed nodal TFH cell lymphoma, NOS (WHO HAEM5) or follicular helper T cell lymphoma, NOS (ICC)
        • Chronic LPD of NK cells, a provisional entity in WHO HAEM4R, has been upgraded to NK large granular lymphocytic leukemia (WHO HAEM5) to reflect the clonal nature of the NK cells
          • Shares certain similarities with T large granular lymphocytic leukemia (e.g., cytologic features and recurrent STAT3 mutations) (Cancers (Basel) 2022;14:5236)
          • Nomenclature and provisional status remain the same under ICC
        • Primary cutaneous acral CD8 positive T cell lymphoma has been downgraded to an LPD in WHO HAEM5 and ICC due to its indolent nature
        • Extranodal NK / T cell lymphoma, nasal type is known simply as extranodal NK / T cell lymphoma under the WHO HAEM5
        • Hydroa vacciniforme-like LPD, an indolent EBV related lesion seen mostly in children, has been renamed hydroa vacciniforme LPD under both classifications
        • Chronic active EBV infection of T and NK cell type, systemic form has been upgraded to systemic chronic active EBV disease (WHO HAEM5) or chronic active EBV disease (ICC)
          • Premalignant, potentially life threatening condition with clonal proliferation of EBV infected T or NK cells that can cause systemic inflammation and organ failure (Immunol Med 2018;41:162)
      • While NK lymphoblastic leukemia / lymphoma, a provisional entity in WHO HAEM4R, remains provisional under ICC, it has been eliminated from WHO HAEM5 (Virchows Arch 2023;482:11)
        • Biologically heterogeneous; overlaps other CD56+ neoplastic entities (e.g., blastic plasmacytoid dendritic cell neoplasm, T lymphoblastic leukemia / lymphoma, acute myeloid leukemia and acute undifferentiated leukemia)
      Diagrams / tables

      Table 1: T / NK cell entities - comparison of WHO (2016), WHO (2022) and ICC (2022)
      WHO HAEM4R WHO HAEM5 ICC
      Precursor T cell neoplasms
      T lymphoblastic leukemia / lymphoma T lymphoblastic leukemia / lymphoma, NOS T lymphoblastic leukemia / lymphoma

      Early T cell precursor lymphoblastic leukemia / lymphoma

      Early T cell precursor lymphoblastic leukemia
      Early T cell precursor acute lymphoblastic leukemia, NOS
      Early T cell precursor acute lymphoblastic leukemia, BCL11B activated
      NK lymphoblastic leukemia / lymphoma* [Entity removed] NK cell acute lymphoblastic leukemia*
      Tumor-like lesions with T cell predominance
      [Not included] Kikuchi-Fujimoto disease [Not included]
      [Not included] Indolent T lymphoblastic proliferation [Not included]
      [Not included] Autoimmune lymphoproliferative syndrome [Not included]
      Mature T / NK cell leukemias
      T prolymphocytic leukemia T prolymphocytic leukemia T cell prolymphocytic leukemia
      T cell large granular lymphocytic leukemia T large granular lymphocytic leukemia T cell large granular lymphocytic leukemia
      Chronic lymphoproliferative disorder of NK cells* NK large granular lymphocytic leukemia Chronic lymphoproliferative disorder of NK cells*
      Adult T cell leukemia / lymphoma Adult T cell leukemia / lymphoma Adult T cell leukemia / lymphoma
      Sézary syndrome Sézary syndrome Sézary syndrome
      Aggressive NK cell leukemia Aggressive NK cell leukemia Aggressive NK cell leukemia
      Primary cutaneous T cell lymphomas
      Primary cutaneous CD4 positive small or medium T cell LPD* Primary cutaneous CD4 positive small or medium T cell LPD Primary cutaneous small or medium CD4 positive T cell LPD
      Primary cutaneous acral CD8 positive T cell lymphoma* Primary cutaneous acral CD8 positive lymphoproliferative disorder Primary cutaneous acral CD8 positive lymphoproliferative disorder
      Mycosis fungoides Mycosis fungoides Mycosis fungoides
      Primary cutaneous CD30 positive T cell LPD: lymphomatoid papulosis Primary cutaneous CD30 positive T cell LPD: lymphomatoid papulosis Primary cutaneous CD30 positive T cell LPD: lymphomatoid papulosis
      Primary cutaneous CD30 positive T cell LPD: primary cutaneous anaplastic large cell lymphoma Primary cutaneous CD30 positive T cell LPD: primary cutaneous anaplastic large cell lymphoma Primary cutaneous CD30 positive T cell LPD: primary cutaneous anaplastic large cell lymphoma
      Subcutaneous panniculitis-like T cell lymphoma Subcutaneous panniculitis-like T cell lymphoma Subcutaneous panniculitis-like T cell lymphoma
      Primary cutaneous gamma / delta T cell lymphoma Primary cutaneous gamma / delta T cell lymphoma Primary cutaneous gamma / delta T cell lymphoma
      Primary cutaneous CD8 positive aggressive epidermotropic cytotoxic T cell lymphoma* Primary cutaneous CD8 positive aggressive epidermotropic cytotoxic T cell lymphoma Primary cutaneous CD8 positive aggressive epidermotropic cytotoxic T cell lymphoma
      [Not included] Primary cutaneous peripheral T cell lymphoma, NOS [Not included]
      Intestinal T cell and NK cell lymphoid proliferations and lymphomas
      Indolent T cell lymphoproliferative disorder of the gastrointestinal tract* Indolent T cell lymphoma of the gastrointestinal tract Indolent clonal T cell LPD of the gastrointestinal tract
      [Not included] Indolent NK cell LPD of the gastrointestinal tract Indolent NK cell LPD of the gastrointestinal tract
      Enteropathy associated T cell lymphoma Enteropathy associated T cell lymphoma Enteropathy associated T cell lymphoma
      Type II refractory celiac disease
      Monomorphic epitheliotropic intestinal T cell lymphoma Monomorphic epitheliotropic intestinal T cell lymphoma Monomorphic epitheliotropic intestinal T cell lymphoma
      Intestinal T cell lymphoma, NOS Intestinal T cell lymphoma, NOS Intestinal T cell lymphoma, NOS
      Hepatosplenic T cell lymphoma
      Hepatosplenic T cell lymphoma Hepatosplenic T cell lymphoma Hepatosplenic T cell lymphoma
      Anaplastic large cell lymphoma
      Anaplastic large cell lymphoma, ALK positive ALK positive anaplastic large cell lymphoma Anaplastic large cell lymphoma, ALK positive
      Anaplastic large cell lymphoma, ALK negative ALK negative anaplastic large cell lymphoma Anaplastic large cell lymphoma, ALK negative
      Breast implant associated anaplastic large cell lymphoma* Breast implant associated anaplastic large cell lymphoma Breast implant associated anaplastic large cell lymphoma
      Nodal T follicular helper (TFH) cell lymphoma
      Angioimmunoblastic T cell lymphoma Nodal TFH cell lymphoma, angioimmunoblastic type Follicular helper T cell lymphoma, angioimmunoblastic type
      Follicular T cell lymphoma* Nodal TFH cell lymphoma, follicular type Follicular helper T cell lymphoma, follicular type
      Nodal peripheral T cell lymphoma (PTCL) with TFH phenotype* Nodal TFH cell lymphoma, NOS Follicular helper T cell lymphoma, NOS
      Other peripheral T cell lymphomas
      Peripheral T cell lymphoma, NOS Peripheral T cell lymphoma, NOS Peripheral T cell lymphoma, NOS
      EBV positive NK / T cell lymphomas
      [Not included] [variant of PTCL, NOS] EBV positive nodal T and NK cell lymphoma Primary nodal Epstein-Barr virus positive T / NK cell lymphoma*
      Extranodal NK / T cell lymphoma, nasal type Extranodal NK / T cell lymphoma Extranodal NK / T cell lymphoma, nasal type
      EBV positive T and NK cell lymphoid proliferations and lymphomas of childhood
      Severe mosquito bite allergy Severe mosquito bite allergy Severe mosquito bite allergy
      Hydroa vacciniforme-like lymphoproliferative disorder Hydroa vacciniforme lymphoproliferative disorder, classic or systemic Hydroa vacciniforme lymphoproliferative disorder, classic or systemic
      Chronic active EBV infection of T and NK cell type, systemic form Systemic chronic active EBV disease Chronic active EBV disease (T and NK cell phenotype)
      Systemic EBV positive T cell lymphoma of childhood Systemic EBV positive T cell lymphoma of childhood Systemic EBV positive T cell lymphoma of childhood
      Note: asterisk (*) denotes a provisional entity
      Microscopic (histologic) images

      Contributed by Tom Deng, M.D.
      Angiocentric destruction

      Angiocentric destruction

      Coagulative necrosis

      Coagulative necrosis

      CD3 positive large cells

      CD3

      CD56

      CD56

      EBV positivity by ISH

      EBER

      Board review style question #1

      Which of the following conditions is categorized by the WHO 5th edition (2022) as a tumor-like condition with T cell predominance?

      1. Castleman disease
      2. Cold agglutinin disease
      3. Erdheim-Chester disease
      4. Kikuchi-Fujimoto disease
      Board review style answer #1
      D. Kikuchi-Fujimoto disease is considered as a tumor-like condition with T cell predominance, which is a newly defined category in the WHO 5th edition (2022). Answers C and B are incorrect because Erdheim-Chester disease is a histiocytic lesion while cold agglutinin disease is predominated by plasma cells. Answer A is incorrect because Castleman disease is a proliferation of mostly B cells or plasma cells.

      Comment Here

      Reference: WHO 2022 & ICC-T / NK cell
      Board review style question #2
      Which of the following conditions is newly recognized by the WHO 5th edition (2022) and International Consensus Classification as neoplastic?

      1. Indolent NK cell lymphoproliferative disorder of the gastrointestinal tract
      2. Indolent T lymphoblastic proliferation
      3. NK lymphoblastic leukemia / lymphoma
      4. Primary cutaneous acral CD8 positive lymphoproliferative disorder
      Board review style answer #2
      A. Indolent NK cell lymphoproliferative disorder of the gastrointestinal tract, previously thought to be reactive, is newly recognized as neoplastic. Answer B is incorrect because indolent T lymphoblastic proliferation is not considered neoplastic under WHO 5th edition (2022) but a tumor-like lesion with T cell predominance. Answer C is incorrect because NK lymphoblastic leukemia / lymphoma, a provisional entity under the WHO 4th revised edition, is no longer considered a distinct entity. Answer D is incorrect because primary cutaneous acral CD8 positive lymphoproliferative disorder has been downgraded from lymphoma due to its indolent clinical course.

      Comment Here

      Reference: WHO 2022 & ICC-T / NK cell

      WHO classification T/NK cell
      Definition / general
      • Significant advances have occurred in the classification of T cell and natural killer (NK) cell neoplasms as a result of genomic studies and gene expression profiling (GEP)
      • New and provisional entities created
      • Novel molecular genetic data included for many of the entities
      • Several changes in the nomenclature have been incorporated
      Highlights of 2016 WHO classification of mature T / NK cell neoplasms
      • T cell prolymphocytic leukemia (T-PLL)
        • Whole exome and targeted sequencing have demonstrated recurrent mutations in genes of the JAK / STAT signaling pathway, including:
          • JAK3 (30 - 40% of cases)
          • STAT5B (21 - 36% of cases)
          • JAK1 (minority of cases)
        • Alterations of ATM, TCL1 and MTCP1 in the pathogenesis of T-PLL
        • Mutations in epigenetic modifiers, such as EZH2 and BCOR, identified in a small proportion of cases
      • Adult T cell leukemia / lymphoma
        • Derived from regulatory T cells (CD4+, CD25+, FOXP3+)
        • Etiologically linked to the human T cell leukemia virus (HTLV-1)
        • High frequency of TP53 and IRF4 mutations (aggressive)
        • High number of copy aberrations, including deletions of CDKN2A and PDL1 amplification (aggressive)
        • STAT3 mutations (indolent)
      • Intestinal T cell lymphomas
        • EATL: previously named enteropathy associated T cell lymphoma (EATL) type I (linked to celiac disease)
          • T cell receptor (TCR) alpha / beta+
          • No amplification or gains of MYC
          • 9p losses
          • Mutations in components of the JAK / STAT pathway
        • Monomorphic epitheliotropic intestinal T cell lymphoma (MEITL), previously named EATL, type II
          • T cell receptor (TCR) gamma / delta+ with nuclear expression of megakaryocyte-associated tyrosine kinase (MATK)
          • Amplification or gains of MYC
          • Activating mutations in STAT5B and JAK3
          • Most recurrently mutated gene in MEITL: SETD2
        • Intestinal T cell lymphoma, NOS
        • New provisional entity: indolent T cell lymphoproliferative disorder of the GI tract
          • Presents with nonspecific GI symptoms
          • May involve small intestine, oral cavity, stomach, colon or multiple sites
          • CD3+, CD8+ (rarely CD4+), TIA1+, granzyme B-
          • Low proliferative rate
          • Not associated with Epstein-Barr virus infection
          • No mutations in genes of the JAK / STAT pathway
      • Hepatosplenic T cell lymphoma
        • Aggressive, derives from nonactivated cytotoxic T cells
        • Affects liver, spleen, bone marrow
        • Express T cell receptor gamma / delta+, cytotoxic molecules and aberrantly multiple killer immunoglobulin receptors (Kirs) clustered under CD158
        • Few cases derived from T cell receptor alpha / beta+ cells
        • Mutations in:
          • Chromatin modifiers (SETD2) (67% of cases)
          • STAT5B (40% of cases)
          • STAT3 (very rare)
      • Subcutaneous panniculitis-like T cell lymphoma
        • Subcutaneous nodules where tumor cells infiltrate the subcutaneous fat
        • Cytotoxic CD8+ cell with expression of cytotoxic molecules
        • Cases with a T cell receptor gamma / delta phenotype are not included here but classified as cutaneous gamma delta T cell lymphoma
      • Cutaneous T cell lymphomas (An Bras Dermatol 2018;93:871)
        • Mycosis fungoides and Sézary syndrome
          • Derived from CD4+ cells
          • CD8 positivity more common in the pediatric population
          • Mutations affect interleukin 2 (IL2) signaling pathways
          • Mutations involve JAK / STAT pathway that leads to constitutive activation of STAT3
          • Inactivation of CDKN2A (9p21.3)
      • Primary cutaneous CD30+ T cell lymphoproliferative disorders include:
        • Lymphomatoid papulosis (LyP)
          • Papular / nodular lesions that resolve within weeks
          • Large CD30+ cells in the dermis
          • Histologic subtypes:
            • Type A, > 80%, sparse large and atypical neoplastic cells in mixed inflammatory infiltrate, CD4+, CD8-
            • Type B, < 5%, epidermotropic infiltrate of small lymphocytes CD4+, CD8-, can be CD30-
            • Type C, ~10%, cohesive infiltrate of large and atypical cells in sparse inflammatory cells, CD4+, CD8-
            • Type D, < 5%, infiltration of small and medium atypical epidermotropic cells, CD4-, CD8+
            • Type E, < 5%, angiocentric and angiodestructive pattern, CD4-, CD8+
          • Clonal rearrangement of the T cell receptor chain genes demonstrable in majority of LyP
          • DUSP22-IRF4, fusion in small subset of LyP (< 5%), small and epidermotropic lymphocytes associated with large neoplastic cells in the dermis (CD4- CD8+ or CD4- CD8-)
        • Primary cutaneous anaplastic large cell lymphoma (ALCL)
          • Solitary or multiple nodules with favorable prognosis
          • Distinct from systemic ALCL that affects the skin secondarily
          • Activated CD4+ phenotype with frequent expression of cytotoxic molecules
          • Lacks expression of EMA and ALK
          • T cell receptor rearrangement in most cases
          • Does not carry t(2;5) translocation, unlike systemic ALCL
          • DUSP22-IRF4 locus in 6p25.3 in 20 - 57% (J Invest Dermatol 2010;130:816)
          • NPM1-TYK2 in ~5% (Blood 2014;124:3768)
          • Overexpression of skin homing chemokine receptor genes CCR10 and CCR8 (An Bras Dermatol 2018;93:871)
      • Peripheral T cell lymphomas (PTCL), NOS
        • Cases remaining in this category show cytological and phenotypic heterogeneity
        • Expresses genes for cell adhesion, apoptosis, proliferation, transcription and signal transduction
        • At least 3 subtypes, characterized by overexpression of:
          • GATA3, regulators of the T cell differentiation into Th2 cells (poor survival)
          • TBX2 (also known as T-bet), regulators of the T cell differentiation into Th1 cells
          • Cytotoxic genes
      • Angioimmunoblastic T cell lymphoma (AITL) and other nodal lymphomas of T follicular helper (TFH) cell origin
        • Arise from TFH cells (TFH phenotype defined by the expression of at least 2, preferably 3, of the following markers: CD10, BCL6, PD1, CXCR5, ICOS and SAP)
        • Angioimmunoblastic T cell lymphoma (AITL)
          • Express whole T follicular helper (TFH) gene signature with expression of the 6 TFH associated genes in majority of cases
          • Mutations in a variety of genes; encoding epigenetic modifiers (i.e. IDH2, TET2), the small GTPase RHOA (60% of cases) and rarely, genes encoding components of the T cell receptor signaling pathway (i.e. FYN, CD28)
          • Mutations of IDH2 R172 specific for AITL
        • Follicular T cell lymphoma
          • Has T follicular helper (TFH) phenotype but lacks the characteristic histological features of AITL
          • Translocation t(5;9)(q32;q22) leading to fusion of ITK-SYK
        • Nodal peripheral T cell lymphoma (PTCL) with a T follicular helper phenotype
          • Previously classified as PTCL, NOS
          • Include cases that express CD4 and at least 2 T follicular helper markers and share certain pathological features of AITL
          • Mutations in TET2, RHOA and DNMT3A
      • Anaplastic large cell lymphoma (ALCL), ALK positive
        • Recurrent chromosomal translocation t(2;5)(p23;q35) involving ALK gene at 2p23 and the nucleophosmin NPM1 gene at 5q35
        • Nodal or extranodal involvement
        • Cytologic spectrum including cases with neoplastic small cells
        • Most have T cell phenotype and the remainder, null phenotype
        • > 90% of the cases shows rearrangement of the T cell receptor chain genes
        • Cells characteristically express ALK in both the nucleus and cytoplasm
        • Recurrent chromosomal translocation t(2;5)(p23;q35) involving ALK gene at 2p23 and the nucleophosmin NPM1 gene at 5q35
        • BCL6, PTPN12, SERPINA1 and CEBPB genes are overexpressed
        • Transcripts related to STAT3 regulators, cytotoxic molecules and T helper 17 cell associated molecules can be highly expressed
      • Anaplastic large cell lymphoma (ALCL), ALK negative
        • No longer a provisional entity in the 2016 WHO classification
        • Often displays a T cell phenotype with loss of several T cell antigens and strong expression of CD30 and cytotoxic molecules
        • Most cases have the T cell receptor chain genes rearranged
        • Constitutive activation of the JAK / STAT pathway through mutations of JAK1 or STAT3 or translocations involving tyrosine kinases other than ALK
        • Rearrangements of DUSP22 at 6p25.3 (33% of cases)
        • Rearrangements of TP63 (about 8%)
        • Expression of 3 genes (TNFRSF8, BATF and TMOD1) allows differentiation of ALK negative ALCL from PTCL, NOS (Blood 2012;120:1274)
      • Breast implant associated anaplastic large cell lymphoma
        • New provisional entity
        • Very rare form of ALCL (ALK negative) seen in female patients with breast implants
        • Usually localized disease that is confined to the serosal cavity and fibrous capsule surrounding the implant
        • Cytology and immunophenotype are similar to ALK negative ALCL
        • T cell receptor chain rearrangement in most cases
        • Recurrent activating JAK1 and STAT3 mutations
        • Typically excellent prognosis with surgery and removal of the implants
      Summary of changes in 2016 WHO classification of mature T / NK cell lymphoid neoplasms
      Entity Changes
      T cell large granular lymphocytic leukemia STAT3 and STAT5B mutations observed in a subset,
      the latter of which is less frequent and shows an association
      with more clinically aggressive disease
      Systemic Epstein-Barr virus+ T cell lymphoma of childhood Name changed from lymphoproliferative disorder to lymphoma
      due to its fulminant clinical course and desire to clearly
      distinguish it from chronic active Epstein-Barr virus infection
      Hydroa vacciniforme-like lymphoproliferative disorder Name changed from lymphoma to lymphoproliferative disorder
      due to its relationship with chronic active Epstein-Barr virus infection
      and a spectrum in terms of its clinical course
      Enteropathy associated T cell lymphoma (EATL) Diagnosis only to be used for cases formerly known as type I
      enteropathy associated T cell lymphoma (EATL), typically associated
      with celiac disease
      Monomorphic epitheliotropic intestinal T cell lymphoma Formerly type II EATL; segregated from type I EATL into a separate
      entity with distinct clinicopathologic features and lack of association
      with celiac disease
      Indolent T cell lymphoproliferative disorder of the GI tract New indolent provisional entity with superficial monoclonal
      intestinal T cell infiltrate; some cases show progression
      Lymphomatoid papulosis New subtypes described with similar clinical behavior but atypical
      histologic / immunophenotypic features
      Primary cutaneous gamma delta T cell lymphoma Important to exclude other cutaneous T cell lymphoid lesions that
      may also be derived from gamma delta T cells, such as mycosis
      fungoides or lymphomatoid papulosis
      Primary cutaneous acral CD8+ T cell lymphoma New indolent provisional entity, originally described as originating
      in the ear
      Primary cutaneous CD4+ small / medium T cell lymphoproliferative disorder No longer to be diagnosed as an overt lymphoma due to limited clinical
      risk, localized disease and similarity to clonal drug reactions
      Remains a provisional entity
      Peripheral T cell lymphoma (PTCL), NOS Subsets based on phenotype and molecular abnormalities being
      recognized that may have clinical implications but mostly not routinely
      evaluated in practice at this time
      Nodal T cell lymphomas with T follicular helper (TFH) phenotype An umbrella category created to highlight the spectrum of nodal
      lymphomas with a TFH phenotype, including angioimmunoblastic T cell
      lymphoma, follicular T cell lymphoma and other nodal PTCL with a TFH phenotype
      Overlapping recurrent molecular / cytogenetic abnormalities
      among subtypes, especially in epigenetic modifiers and RHOA
      GTPase, that could potentially impact therapy
      ALK negative anaplastic large cell lymphoma Now a distinct entity that includes cytogenetic subsets that
      appear to have prognostic implications (e.g. 6p25 rearrangements at
      IRF4-DUSP22 locus)
      Breast implant associated anaplastic large cell lymphoma New provisional entity distinguished from other ALK- ALCL; typically
      localized disease associated with excellent outcome

      2016 WHO classification of mature T / NK cell neoplasms
      • T cell prolymphocytic leukemia
      • T cell large granular lymphocytic leukemia
      • Chronic lymphoproliferative disorder of NK cells
      • Aggressive NK cell leukemia
      • Systemic Epstein-Barr virus+ T cell lymphoma of childhood*
      • Hydroa vacciniforme-like lymphoproliferative disorder*
      • Adult T cell leukemia / lymphoma
      • Extranodal NK / T cell lymphoma, nasal type
      • Enteropathy associated T cell lymphoma
      • Monomorphic epitheliotropic intestinal T cell lymphoma*
      • Indolent T cell lymphoproliferative disorder of the GI tract*
      • Hepatosplenic T cell lymphoma
      • Subcutaneous panniculitis-like T cell lymphoma
      • Mycosis fungoides
      • Sézary syndrome
      • Primary cutaneous CD30+ T cell lymphoproliferative disorders
        • Lymphomatoid papulosis
        • Primary cutaneous anaplastic large cell lymphoma
      • Primary cutaneous gamma delta cell lymphoma
      • Primary cutaneous CD8+ aggressive epidermotropic cytotoxic T cell lymphoma
      • Primary cutaneous acral CD8+ T cell lymphoma*
      • Primary cutaneous CD4+ small / medium T cell lymphoproliferative disorder*
      • Peripheral T cell lymphoma, NOS
      • Angioimmunoblastic T cell lymphoma
      • Follicular T cell lymphoma*
      • Nodal peripheral T cell lymphoma with T follicular helper phenotype*
      • Anaplastic large cell lymphoma, ALK+
      • Anaplastic large cell lymphoma, ALK-*
      • Breast implant associated anaplastic large cell lymphoma*

        *Asterisks denote current changes relative to the 2008 WHO classification
        Italics denote provisional entities
      Entities with name change (previous designations in the 2008 WHO classification are shown in parentheses)
      • Systemic Epstein-Barr virus+ T cell lymphoma in childhood (systemic Epstein-Barr virus+ T cell lymphoproliferative disorder)
      • Hydroa vacciniforme lymphoproliferative disorder (hydroa vacciniforme-like lymphoma)
      • Enteropathy associated T cell lymphoma (enteropathy associated T cell lymphoma, type 1)
      • Monomorphic epitheliotropic intestinal T cell lymphoma (enteropathy associated T cell lymphoma, type 2)
      • Primary cutaneous CD4+ small / medium T cell lymphoproliferative disorder (primary cutaneous CD4+ small / medium T cell lymphoma)
      Diagrams / tables

      Images hosted on other servers:

      Gene mutations in JAK / STAT pathway

      Board review style question #1
      Which of the following is true about angioimmunoblastic T cell lymphomas (AITL)?

      1. IDH2 R172 mutation is a recurrent genetic alteration seen in a subset of AITL
      2. Phenotypic variants include T cell and null cell types
      3. They are associated with Epstein-Barr virus (EBV) and ALK gene alterations
      4. They represent the extranodal manifestation of peripheral T cell lymphomas characterized by an absence of T follicular helper (TFH) markers
      Board review style answer #1
      A. IDH2 R172 mutation is a recurrent genetic alteration seen in a subset of AITL

      Comment Here

      Reference: WHO classification T/NK cell

      WHO classification-GI hematolymphoid tumors
      Table of Contents
      Definition / general | WHO (2019)
      Definition / general
      WHO (2019)

      Back to top
      Recent Lymphoma & related disorders Pathology books

      Auerbach: 2022

      Beck: 2021

      Cerroni: 2020

      Crane: 2021

      Dorfman: 2023

      Duffield: 2020

      Foucar: 2019

      Foucar: 2023

      Gru: 2023

      Hsi: 2017

      Hudnall: 2019

      IARC: 2017

      Jaffe: 2016

      Linden: 2015

      Medeiros: 2017

      Medeiros: 2021

      Medeiros: 2023

      Molina: 2019

      Naeim: 2018

      Subtil: 2019

      Vasef: 2019

      Wang: 2020

      Wang: 2018



      Find related Pathology books: dermatopathology, hematopathology, immunology / transplant, lab medicine, molecular
      Image 01 Image 02